180
1 PROF.MENDONÇA PARA APOSTILA COMPLETA VENHA FAZER UM DE MEUS CURSOS: Segunda feira 18:30 até 21:45 Curso e colégio integral (Específica semestral) Fone: (44) 3028-5280 Maringá Terça feira 17:30 até 19:30 Curso Prime (Física básica semestral) Fone: (43) 3025-1020 Londrina Quarta feira 14.00 até 17:00 Curso Sigma ( Específica modular) Fone: (43) 3321-2652 Londrina Quinta feira 19:15 até 22:30 Curso Saber (Específica aprofundada) Fone: (43) 3326-1212 Londrina Sexta feira 16:00 até 18:00 Curso e colégio integral (Física básica semestral) Fone: (44) 3028-5280 Maringá 2017

curso de específica de física

Embed Size (px)

Citation preview

Page 1: curso de específica de física

1

PROF.MENDONÇA PARA APOSTILA COMPLETA VENHA

FAZER UM DE MEUS CURSOS:

Segunda feira 18:30 até 21:45 Curso e colégio integral (Específica semestral) Fone: (44) 3028-5280 Maringá Terça feira 17:30 até 19:30 Curso Prime (Física básica semestral) Fone: (43) 3025-1020 Londrina Quarta feira 14.00 até 17:00 Curso Sigma ( Específica modular) Fone: (43) 3321-2652 Londrina Quinta feira 19:15 até 22:30 Curso Saber (Específica aprofundada) Fone: (43) 3326-1212 Londrina Sexta feira 16:00 até 18:00 Curso e colégio integral (Física básica semestral) Fone: (44) 3028-5280 Maringá

2017

Page 2: curso de específica de física

2

Sumário LISTA 1 - MOVIMENTO UNIFORME ................................................................................................................................... 3

LISTA 2 - MOVIMENTO UNIFORMEMENTE VARIADO ....................................................................................................... 7

LISTA 3 - LANÇAMENTO OBLÍQUO .................................................................................................................................. 12

LISTA 4 - MOVIMENTO CIRCULAR ................................................................................................................................... 17

LISTA 5 - VETORES E CINEMÁTICA VETORIAL .................................................................................................................. 21

LISTA 6 - LEIS DE NEWTON .............................................................................................................................................. 26

LISTA 7 - APLICAÇÕES DAS LEIS DE NEWTON ................................................................................................................. 31

LISTA 8 - TRABALHO E ENERGIA ...................................................................................................................................... 36

LISTA 9 - DINÂMICA IMPULSIVA E COLISÕES .................................................................................................................. 41

LISTA 10 - ESTÁTICA E HIDROSTÁTICA ............................................................................................................................ 45

LISTA 11 – LEIS DE KEPLER E GRAVITAÇÃO ..................................................................................................................... 51

LISTA 12 - TERMOMETRIA E DILATAÇÃO ........................................................................................................................ 57

LISTA 13 - CALORIMETRIA ............................................................................................................................................... 61

LISTA 14 - GASES E TERMODINÂMICA ............................................................................................................................ 66

LISTA 15 - TERMODINÂMICA .......................................................................................................................................... 72

LISTA 16 - ELETROSTÁTICA 1 – LEI DE COULOMB E CAMPO ELÉTRICO ........................................................................... 77

LISTA 17 – ELETROSTÁTICA 2 – POTENCIAL, ENERGIA E TRABALHO .............................................................................. 85

LISTA 18 – ELETRODINÂMICA 1 – LEIS DE ÔHM .............................................................................................................. 90

LISTA 19 - ELETRODINÂMICA 2 - GERADORES E RECEPTORES ........................................................................................ 97

LISTA 20 – ELETRODINÂMICA 3 – LEIS DE KIRCHHOFF ................................................................................................. 105

LISTA 21 – ÓPTICA 1 – FUNDAMENTOS E REFLEXÃO DA LUZ ....................................................................................... 111

LISTA 22 – REFLEXÃO DA LUZ ESPELHOS ESFÉRICOS .................................................................................................... 118

LISTA 23 – REFRAÇÃO DA LUZ LEI DE SNELL-DESCARTES .............................................................................................. 124

LISTA 24 – ESTUDO DAS LENTES ESFÉRICAS ................................................................................................................. 130

LISTA 25 – MOVIMENTO HARMÔNICO SIMPLES E ONDULATÓRIA .............................................................................. 136

LISTA 26 – INTERFERÊNCIA E ACÚSTICA........................................................................................................................ 145

LISTA 27 – MAGNETISMO E FONTES DE CAMPO MAGNÉTICO ..................................................................................... 150

LISTA 28 – FORÇA MAGNÉTICA E INDUÇÃO MAGNÉTICA ............................................................................................ 157

LISTA 29 – FÍSICA MODERNA ........................................................................................................................................ 164

PROVAS DE VESTIBULARES ......................................................................................................................... 176

UEL 1ª FASE 2017.............................................................................................................................................. 176

UEL 2ª FASE 2017.............................................................................................................................................. 177

UEM 1º DIA 2016 ............................................................................................................................................... 178

Page 3: curso de específica de física

3

LISTA 1 - MOVIMENTO UNIFORME RESUMO DE CONTEÚDO Características do Movimento Uniforme O movimento uniforme é o movimento que possui módulo da velocidade constante, ou seja, ela não varia com o passar do tempo. Entretanto, essa velocidade, apesar de ser constante, é diferente de zero, ou seja, ela pode assumir qualquer outro valor que não seja o zero. Sendo a aceleração definida da seguinte forma:

E sabendo que no movimento uniforme a variação do módulo da velocidade é igual a zero, pois a velocidade final é igual à velocidade inicial, concluímos que a aceleração escalar é constante e igual a zero. A função Horária do Movimento Uniforme No movimento uniforme temos que a velocidade escalar é constante e coincide com a velocidade escalar média em qualquer instante ou intervalo de tempo. Matematicamente, a velocidade escalar média pode ser expressa da seguinte forma:

Onde: • ΔS é a variação de posição do móvel, ΔS = S – So; • Δt é a variação do tempo, Δt = t – to. Substituído ΔS e Δt na equação da velocidade descrita acima, temos:

Fazendo tempo inicial igual a zero, to= 0, temos a função horária do movimento uniforme.

S = So + Vt EXERCÍCIOS Questão 01 - (UEG GO/2012)

A órbita do planeta Terra, em torno do Sol, possui uma distância aproximada de 930 milhões de quilômetros. Sabendo-se que o ano possui 365 dias e 5 horas, a velocidade média exercida pela Terra para executar essa órbita é, aproximadamente, de

a) 106.103 km/h b) 1.061 km/h c) 106 km/h d) 10,6 km/h

Questão 02 - (FMABC/2012)

Duas esferas de dimensões desprezíveis dirigem-se uma ao encontro da outra, executando movimentos retilíneos e uniformes (veja a figura). As esferas possuem velocidades cujos módulos valem 4m/s e 7m/s. A distância entre elas nos 4 segundos que antecedem a colisão é de

a) 50 b) 44 c) 28 d) 16 e) 12

Questão 03 - (UEL PR)

Sabe-se que o cabelo de uma pessoa cresce em média 3cm a cada dois meses. Supondo que o cabelo não seja cortado e nem caia, o comprimento total, após terem se passado 10 anos será: a) 800mm b) 1200mm c) 1000mm d) 1800mm e) 150mm

Questão 04 - (UFLA MG)

Considerando uma partícula em movimento retilíneo com velocidade constante, as seguintes afirmações são CORRETAS, exceto: a) O momento linear da partícula mantém-se

constante durante o movimento. b) A força resultante não-nula que atua na partícula é

constante em módulo, direção e sentido. c) A posição da partícula dependerá linearmente do

tempo. d) A energia cinética da partícula será conservada. e) A aceleração do movimento da partícula será nula.

Questão 05 - (FURG RS)

O gráfico representa o módulo das velocidades de dois automóveis como função do tempo. Com relação à área hachurada, podemos dizer que ela representa:

a) a diferença entre as acelerações dos dois

automóveis. b) a diferença entre as distâncias percorridas pelos

dois automóveis. c) a aceleração do automóvel A em relação ao

automóvel B. d) a diferença entre as velocidades dos dois

automóveis. e) uma grandeza sem qualquer significado físico.

Questão 06 - (FUVEST SP/2006)

Um automóvel e um ônibus trafegam em uma estrada plana, mantendo velocidades constantes em torno de 100 km/h e 75 km/h, respectivamente. Os dois veículos passam lado a lado em um posto de pedágio. Quarenta minutos (2/3 de hora) depois, nessa mesma estrada, o motorista do ônibus vê o automóvel ultrapassá-lo. Ele

Page 4: curso de específica de física

4

supõe, então, que o automóvel deve ter realizado, nesse período, uma parada com duração aproximada de a) 4 minutos b) 7 minutos c) 10 minutos d) 15 minutos e) 25 minutos

Questão 07 - (UFLA MG)

O gráfico abaixo representa a variação das posições de um móvel em função do tempo (S = f(t)).

10

0

-10

1 2 3 4 5 6 7 86

S(m)

t(s)

O gráfico de v x t que melhor representa o movimento acima é:

5

-5

10

0

V(m/s)

2 4 6 8 | | | |

t(s)

a.

5

-5

10

0

V(m/s)

2 4 6 8 | | | |

t(s)

b.

5

-5

10

0

V(m/s)

2 4 6 8 | | | |

t(s)

c.

5

-5

10

0

V(m/s)

2 4 6 8 | | | |

t(s)

d.

Questão 08 - (UFPE)

A equação horária para o movimento de uma partícula é x(t) = 15 – 2 t, onde x é dado em metros e t em segundos. Calcule o tempo, em s, para que a partícula percorra uma distância que é o dobro da distância da partícula à origem no instante t = 0 s.

Questão 09 - (UERJ/2011)

Uma partícula se afasta de um ponto de referência O, a partir de uma posição inicial A, no instante t = 0 s, deslocando-se em movimento retilíneo e uniforme, sempre no mesmo sentido. A distância da partícula em relação ao ponto O, no instante t = 3,0 s, é igual a 28,0 m e, no instante t = 8,0 s, é igual a 58,0 m. Determine a distância, em metros, da posição inicial A em relação ao ponto de referência O.

Questão 10 - (PUC MG/2006)

O gráfico mostra a velocidade (v) de um objeto em movimento retilíneo, em função do tempo t.

Sobre o movimento do objeto, é CORRETO afirmar: a) Analisando-se o gráfico como um todo, pode-se

afirmar que o objeto tende a parar. b) Entre os instantes 0 e 0,8 s , o objeto está em

movimento retilíneo uniformemente variado. c) Até 1,2 s , a distância percorrida pelo móvel foi de

2,4 m. d) A partir do instante 1,2 s , o objeto passa a se se

mover em movimento retilíneo uniforme. Questão 11 - (UEL PR)

Em grandes cidades, a rota das ambulâncias leva em consideração fatores como proximidade do local do chamado e rapidez no deslocamento. Considere um chamado proveniente da região central de uma cidade, às 19h, conforme ilustra a figura, e que para atendê-lo, estão disponíveis quatro bases de ambulâncias, X, Y, W e Z.

Para se definir a melhor rota, foram consideradas as velocidades médias desenvolvidas pelas ambulâncias em alguns intervalos de horários:

Page 5: curso de específica de física

5

Assim, o chamado comunicado às 19h será atendido mais rapidamente pela ambulância da base

a) X, seguindo pela rota 1. b) Z, seguindo pela rota 2. c) W, seguindo pela rota 3. d) Y, seguindo pela rota 4. e) Z, seguindo pela rota 5.

Questão 12 - (UEL PR)

Um pequeno animal desloca-se com velocidade média igual a 0,5 m/s. A velocidade desse animal em km/dia é: a) 13,8 b) 48,3 c) 43,2 d) 1,80 e) 4,30

Questão 13 - (UEL PR)

Nos edifícios, os números que identificam os apartamentos representam vetorialmente suas posições, isto é, esses números são compostos por dígitos que representam o andar (posição na vertical) e a localização do apartamento no andar (posição na

horizontal). Em um edifício de 10 andares, que tem um apartamento por andar, cada apartamento é identificado por um número que varia de 1 a 10. Se nesse edifício, cada andar tem altura de 5 metros, qual é a distância percorrida na direção vertical por alguém que sai do apartamento 3 e vai para o apartamento 9? a) 6 m b) 15 m c) 30 m d) 45 m e) 60 m

Questão 14 - (UEL PR)

O raio médio da órbita elíptica da Terra em torno do Sol é considerado para definir 1 Unidade Astronômica (U.A.): 1 U.A. 1,49108 km = 1,491011 m. A U.A. é

utilizada para medir os raios das órbitas dos planetas do sistema solar, entretanto é uma unidade muito pequena para ser utilizada como parâmetro de medida para as distâncias das estrelas. Para essas distâncias, é utilizado o Ano-Luz (A. L.) que é a distância percorrida pela luz em um ano. Por exemplo, a estrela -Centauri está a 4,3

A. L. de distância da Terra. Se a velocidade de propagação da luz é igual a c = 3 108 m/s, é correto

afirmar que a distância média entre o Sol e a Terra é de: a) 150 Segundos-Luz. b) 300 Segundos-Luz.

c) 430 Segundos-Luz. d) 500 Segundos-Luz. e) 600 Segundos-Luz.

Questão 15 - (UEM PR)

Um trem se move com velocidade constante. Dentro dele estão o observador A e um garoto. Na estação, parado sobre a plataforma, está o observador B. Quando o trem passa pela plataforma, o garoto joga uma bola verticalmente para cima. Desprezando-se a resistência do ar, podemos afirmar que: 01. o observador A vê a bola se mover

verticalmente para cima e cair nas mãos do garoto. 02. o observador B vê a bola descrever uma parábola e

cair nas mãos do garoto. 04. os dois observadores vêem a bola se mover

com a mesma aceleração. 08. o observador B vê a bola se mover

verticalmente para cima e cair atrás do garoto. 16. o observador A vê a bola descrever uma

parábola e cair atrás do garoto. Questão 16 - (UEM PR)

Com velocidade constante, um caminhão se move num trecho retilíneo horizontal, sem atrito. Ele transporta, sobre a carroceria, pedras e um garoto. Se o garoto

começa a arremessar pedras, pode–se concluir que a velocidade do caminhão, na direção inicial do movimento, 01. aumenta, se as pedras forem arremessadas para

trás. 02. diminui, se as pedras forem arremessadas para

frente. 04. diminui, se as pedras forem arremessadas

verticalmente para cima. 08. aumenta, se as pedras forem arremessadas

lateralmente, perpendicularmente à direção do movimento do caminhão.

16. permanece constante, qualquer que seja a direção em que o garoto arremessar as pedras.

Questão 17 - (UEM PR)

Os diagramas abaixo (a, b, c, d) mostram uma esfera movendo-se em quatro situações diferentes. Considerando que, em todas as situações, não existem forças dissipativas atuando, que, em a e em b, é dado um pequeno empurrão na esfera para que ela comece a se mover e que, em d, a colisão entre a esfera e a superfície é perfeitamente elástica, podese afirmar que,

01. em a, a esfera está em movimento retilíneo

uniforme (MRU). 02. em b, a esfera está em MRU. 04. em c, a esfera está em movimento harmônico

simples (MHS). 08. em d, a esfera está em MHS. 16. tanto em c como em d, decorrido algum tempo, a

esfera pára. 32. tanto em a como em b, a força que a superfície faz

sobre a esfera é constante. Questão 18 - (PUC MG)

Page 6: curso de específica de física

6

A tabela abaixo contém as velocidades, consideradas constantes, em metros por segundo, que quatro nadadoras apresentaram na ida e na volta nadando estilo livre em uma piscina de 50 metros de comprimento. nadadora A B C D E ida 1,00 1,25 0,50 0,60 0,80 volta 1,00 0,80 1,60 0,90 0,70

Qual delas fez a virada em primeiro lugar? a) A b) B c) C d) D e) E

Questão 19

Um candidato sai de sua residência para prestar

vestibular pretendendo percorrer a distância total até o

local da prova em uma hora, conduzindo seu automóvel

com velocidade média de 60 km/h. Após percorrer os

primeiros 10 km do percurso em 10 minutos, percebe

que esqueceu o documento de identificação e retorna

para apanhá-lo. Sua mãe o espera no portão com o

documento.

Desprezando-se o tempo para receber o documento e

manobrar o carro, para que esse candidato consiga

chegar ao local da prova no horário previsto

anteriormente, ele deverá desenvolver no percurso de

retorno à sua casa e ida até o local da prova uma

velocidade média, em km/h, igual a

a) 78.

b) 84.

c) 90.

d) 98.

e) 72.

Questão 20 - (UnB DF)

Qual é o tempo gasto para que um metrô de 200m a uma velocidade de 180km/h atravesse um túnel de 150m? Dê sua resposta em segundos.

GABARITO: 1) Gab: A 2) Gab: B 3) Gab: D 4) Gab: B 5) Gab: B 6) Gab: C 7) Gab: B 8) Gab: 15s 9) Gab: 10,0m 10) Gab: D 11) Gab: E 12) Gab: C 13) Gab: C 14) Gab: D 15) Gab: 01-02-04 16) Gab: 01-02 17) Gab: 02-04-08 18) Gab: B 19) Gab: B 20) Gab: 07 21) Gab: 04 litros 22) Gab: B 23) Gab: A 24) Gab: C 25) Gab: C 26) Gab: A 27) Gab: B 28) Gab: E 29) Gab: D

30) Gab: D 31) Gab: D 32) Gab: B 33) Gab:

a) vm 17,5 km/s

b) 't = 16667 s 4,63 h c) 2181. 34) Gab: C 35) Gab: D 36) Gab: D

37) Gab: C 38) Gab: B 39) Gab: C 40) Gab: A

Page 7: curso de específica de física

7

LISTA 2 - MOVIMENTO UNIFORMEMENTE VARIADO

RESUMO DE CONTEÚDO O movimento de uma partícula só é considerado uniformemente variado quando o módulo de sua velocidade varia, aumentando ou diminuindo ao longo do tempo. Quando observamos que a velocidade aumenta ou diminui de maneira uniforme, independentemente da trajetória descrita por essa partícula, quer dizer que a partícula possui aceleração constante. Exemplo: Se a aceleração é 10m/s2, a velocidade varia 10m/s a cada segundo, ou seja, significa que o valor da velocidade da partícula aumenta numa taxa constante.

Classificação do Movimento uniformemente variado

(MUV)

Para descrevermos o movimento de uma partícula devemos

adotar um referencial. Podemos classificar esse movimento como acelerado ou retardado: Se a variação do módulo da velocidade escalar é positivo, ou seja, o valor da velocidade escalar em módulo aumenta classificamos o movimento como acelerado. ( Movimento acelerado progressivo ou movimento acelerado regressivo). Se a variação do módulo da velocidade escalar é negativo, ou seja, o valor da velocidade escalar em módulo diminui, classificamos o movimento como retardado. ( Movimento retardado progressivo ou movimento retardado regressivo). Resumindo: Movimento acelerado - |v|=> aumenta. - a e v possuem o mesmo sinal. Movimento retardado - |v|=> diminui. - a e v possuem sinais contrários. Fonte: http://www.infoescola.com/fisica/movimento-uniformemente-variado-muv/ EXERCÍCIOS TEXTO: 1 -

Leia o texto e analise o gráfico.

Um objeto que não pode ser considerado uma partícula é solto de uma dada altura sobre um lago. O gráfico abaixo apresenta a velocidade desse objeto em função do tempo. No tempo t = 1, 0s, o objeto toca a superfície da água. Despreze somente a resistência no ar.

Questão 01 - (UEL PR/2011)

Qual a profundidade do lago?

a) 1 m b) 5 m c) 7 m d) 100 m e) 1000 m

Questão 02 - (ESCS DF/2009)

A velocidade de um corpo em função do tempo é dada pelo gráfico: O espaço percorrido pelo corpo entre 0 e 4s é:

a) 30m; b) 35m; c) 40m; d) 45m; e) 50m.

Questão 03 - (UFF RJ)

No gráfico, v2 indica o quadrado da velocidade e x, a posição. O movimento foi realizado em trajetória retilínea, partindo de posição inicial nula.

V

x0

2

O gráfico corresponde a um movimento: a) uniforme b) uniformemente acelerado com velocidade inicial não

nula c) uniformemente retardado com velocidade inicial

nula. d) uniformemente acelerado com velocidade inicial

nula.

e) uniformemente retardado com velocidade inicial não nula.

Questão 04 - (FEEVALE RS)

O gráfico abaixo representa a velocidade de um automóvel que se movimenta em uma avenida retilínea, partindo de um semáforo que abriu, até parar em um outro semáforo fechado.

Page 8: curso de específica de física

8

t(s)604020

20

v(m/s)

A distância entre os dois semáforos vale a) 200 m b) 400 m c) 600 m d) 800 m e) 1000 m

Questão 05 - (CESJF MG)

O gráfico abaixo representa a variação da velocidade de dois carros , A e B , em função do tempo . Os carros partem de uma mesma posição no mesmo instante , seguindo uma mesma trajetória retilínea e rumo a uma mesma cidade.

0 0,1 0,2 0,3 0,4 t(h)

V(km/h)

60

80 B

A

Após 6,0 minutos de movimento a distância entre os carros é de : a) 1,0 km b) 3,0 km c) 6,0 km d) 5,0 km e) N. R. A .

Questão 06 - (UNESP)

Um atleta de corridas de curto alcance, partindo do repouso, consegue imprimir a si próprio uma aceleração constante de 5,0m/s2 durante 2,0s e, depois, percorre o resto do percurso com a mesma velocidade adquirida no final do período de aceleração. a) Esboce o gráfico da velocidade do atleta em função

do tempo, numa corrida de 5s.

b) Qual é a distância total que ele percorre nessa corrida de 5s?

Questão 07 - (UNIRIO RJ)

Analisando o gráfico abaixo, que relaciona a posição dos móveis A e B com o tempo t, assinale a opção correta.

s

A

B

t

c a) VA = VB b) VA > VB c) VA < VB d) aA > aB e) aA < aB

Questão 08 - (UNIRIO RJ)

A velocidade de uma partícula varia com o passar do tempo conforme o gráfico abaixo.

v(m/s)

t(s)0 1 2 3 4 O seu deslocamento do instante 0 s até o instante 1 s foi de 1,5 m. Através da observação do gráfico podemos concluir que seu deslocamento entre os instantes 2 s e 3 s, em m, foi de: a) 2,0 b) 2,05 c) 3,0 d) 3,5 e) 4,0

Questão 09 - (UNIFOR CE)

O gráfico fornece a velocidade de um corpo, que se move em linha reta, em função do tempo. Sabe-se que, no instante t 0, o corpo se encontra na posição 20 m.

No instante t 8,0 s, o corpo estará na posição a) 40 m

b) 20 m

c) 20 m d) 40 m e) 60 m

Questão 10 - (UNIFOR CE)

O gráfico abaixo representa, em função do tempo, a velocidade escalar de uma partícula que está em movimento retilíneo.

Page 9: curso de específica de física

9

O movimento é acelerado SOMENTE no trecho a) I b) II c) III d) IV e) V

Questão 11 - (UNIOESTE PR/2007)

Em uma competição esportiva entre escolas, dois alunos, João e Pedro, disputam quem vai chegar primeiro à posição C. João está inicialmente em repouso na posição A, conforme a figura abaixo, e precisa se deslocar 144 m para alcançar o ponto C, seguindo a trajetória retilínea AC. No mesmo instante em que João inicia seu movimento com uma aceleração uniforme de 0,5 m/s2 mantida em todo o trajeto, Pedro está passando pela posição B com uma velocidade de 2 m/s. Pedro segue a trajetória retilínea BC de comprimento igual a 145 m e alcança o ponto C no mesmo instante que João. Assinale a alternativa que fornece o valor da aceleração de Pedro, em metros por segundo ao quadrado (com arredondamento na segunda casa decimal), no trecho BC.

a) 0,50

b) 0,01 c) 0,11 d) 0,17 e) 0,34.

Questão 12 - (UNIOESTE PR)

Um feixe de íons negativos é acelerado no interior de um tubo de raios catódicos. Cada íon é acelerado desde o repouso até a velocidade final de 2,0107 m/s em uma

distância de 10 mm. Determine, em fN (femto newtons), o módulo da força resultante que atua sobre cada íon nesse tubo de raios catódicos. Use, para a massa do íon, o valor de 10,01031 kg e lembre-se de que o

multiplicador f (femto) vale 1015.

Questão 13 - (UNIOESTE PR)

Uma pedra é atirada verticalmente para cima, a partir de uma janela de um edifício, situada a 5 m de altura em relação ao solo, com uma velocidade inicial igual a 20 m/s. No mesmo instante, de uma janela situada a 25 m de altura em relação ao solo, exatamente acima da primeira janela, é atirada verticalmente para cima uma outra pedra com uma velocidade inicial de 10 m/s. Desprezando a resistência do ar, determine após quantos segundos, a partir do instante do arremesso, as duas pedras estarão à mesma altura.

Questão 14 - (UNIOESTE PR/2006)

Considere a queda livre de um corpo a partir do repouso. Tome g = 10,0 m/s2. Com relação a esse movimento, pode-se afirmar que 00. a aceleração do corpo é constante e igual a

10,0m/s2, independente de considerar o atrito com o ar.

01. a distância que o corpo cai após 3,0s é de 50,0m, quando não há atrito.

02. se não há atrito com o ar, a sua velocidade, após ter caído 20,0m, é de 20,0m/s.

03. caso não haja atrito com o ar, o tempo necessário para o corpo atingir uma velocidade de 40,0m/s é de 6,0 s.

04. não havendo atrito com o ar, o tempo gasto para o corpo cair 500,0m é de 10,0s.

Questão 15 - (UNIOESTE PR/2009)

Em uma pista de testes um automóvel, partindo do repouso e com aceleração constante de 3 m/s2, percorre certa distância em 20 s. Para fazer o mesmo trajeto no mesmo intervalo de tempo, porém com aceleração nula, um segundo automóvel deve desenvolver velocidade de

a) 20 m/s

b) 25 m/s c) 80 km/h d) 100 km/h e) 108 km/h

Questão 16 - (UEL PR/2011)

No circuito automobilístico de Spa Francorchamps, na Bélgica, um carro de Fórmula 1 sai da curva Raidillion e, depois de uma longa reta, chega à curva Les Combes.

Figura: Circuito automobilístico de Spa Francorchamps

A telemetria da velocidade versus tempo do carro foi registrada e é apresentada no gráfico a seguir.

Qual das alternativas a seguir contém o gráfico que melhor representa a aceleração do carro de F-1 em função deste mesmo intervalo de tempo?

Page 10: curso de específica de física

10

a)

b)

c)

d)

e)

Questão 17

Analise as alternativas abaixo e assinale o que for correto.

01. O gráfico da velocidade em função do tempo, para

um móvel descrevendo um Movimento Retilíneo e Uniforme, é uma reta paralela ao eixo dos tempos.

02. O gráfico da posição em função do tempo, para um móvel descrevendo um movimento Retilíneo e Uniforme, é uma reta, e o coeficiente angular dessa reta fornece a velocidade do móvel.

04. O gráfico do espaço percorrido em função do tempo é uma reta para um móvel que realiza um Movimento Uniforme qualquer.

08. O espaço percorrido por um móvel, em um dado intervalo de tempo, pode ser obtido calculando-se a “área sob a curva” do gráfico da velocidade em função do tempo, para aquele dado intervalo de tempo.

16. O gráfico da velocidade em função do tempo, para um móvel descrevendo um Movimento Retilíneo Uniformemente Variado, é uma parábola.

Questão 18 - (UEL PR)

O que acontece com o movimento de dois corpos, de massas diferentes, ao serem lançados horizontalmente com a mesma velocidade, de uma mesma altura e ao mesmo tempo, quando a resistência do ar é desprezada? a) O objeto de maior massa atingirá o solo primeiro. b) O objeto de menor massa atingirá o solo primeiro. c) Os dois atingirão o solo simultaneamente. d) O objeto mais leve percorrerá distância maior. e) As acelerações de cada objeto serão diferentes.

Questão 19 - (UEL PR)

Um motorista dirige um automóvel a 72 km/h quando percebe que o semáforo a sua frente está fechado. Ele pisa, então, no pedal do freio e a velocidade do automóvel diminui como mostra o gráfico abaixo.

0 5,0 t(s)

72

v(km/h)

A menor distância que o automóvel deve estar do semáforo, no instante em que o motorista pisa no pedal do freio, para que não avance o semáforo é, em metros, a) 144 b) 72 c) 50 d) 30 e) 18

Questão 20 - (UEL PR/2008)

No departamento de Física da UEL, foi realizado um experimento de queda livre cuja equação de movimento foi obtida com auxílio de um computador. O experimento consistiu na aquisição de um sinal elétrico cada vez que um objeto, em queda, interrompia um feixe de luz laser que era direcionado por espelhos (separados por 2 cm) até ser coletado numa fotocélula (ver figura). O objeto de estudo foi uma pena, para qual a resistência do ar não pode ser desprezada. A fotocélula, quando recebia luz, produzia uma tensão elétrica, e o tempo entre as interrupções de luz eram registradas pelo computador. Ao final da queda, obteve-se um gráfico de espaço percorrido versus tempo (S x t) cujos dados são mostrados no gráfico abaixo. Pelo arranjo experimental, conseguiu-se simplificar a equação que descreve o movimento, uma vez que o espaço inicial, bem como a velocidade inicial, puderam ser considerados zero.

Page 11: curso de específica de física

11

Sabendo que a função polinomial que descreve o

movimento é do tipo 2CxBxAxF )( , e que

cm 0SA e cm/s 0vB , qual o valor aproximado da

aceleração “a” da pena em cm/s2? a) 9,8 cm/s2. b) 12,2 cm/s2. c) 5,3 cm/s2. d) 3,6 cm/s2. e) 7,1 cm/s2.

GABARITO: 1) Gab: C 2) Gab: C 3) Gab: B 4) Gab: D 5) Gab: A 6) Gab:

a)

V(m/s)

t(s)

10

5

0 1 2 3 4 5 ;

b) d = 40m 7) Gab: A 8) Gab: D 9) Gab: E 10) Gab: D 11) Gab: E 12) Gab: 20 13) Gab: 14) Gab: FFVFV 15) Gab: E 16) Gab: D 17) Gab: 15 18) Gab: C 19) Gab: C 20) Gab: E 21) Gab: 03 22) Gab: 07 23) Gab: 25 24) Gab: 30 25) Gab: 15 26) Gab: B 27) Gab: D 28) Gab: D 29) Gab: a) a = 6,0m/s2;

b) v = 11m/s; c) vH = 6,2m/s 30) Gab: 01-F; 02-V; 04-F; 08-V; 16-F; 32-F. 31) Gab: 05 32) Gab: B 33) Gab: 11

34) Gab: E 35) Gab: 29 36) Gab: B

37) Gab: A 38) Gab: 14 39) Gab: 20

40) Gab: 27

Page 12: curso de específica de física

12

LISTA 3 - LANÇAMENTO OBLÍQUO RESUMO TEÓRICO Quando uma bola é chutada em uma partida de futebol, podemos observar que ela realiza um movimento parabólico. Esse movimento é chamado de lançamento oblíquo. Considere um corpo sendo lançado a partir do solo, formando um ângulo α com a horizontal, com velocidade inicial v0.

Desprezando as forças dissipativas, o corpo fica sujeito apenas à ação da gravidade, descrevendo uma trajetória parabólica.

Movimento Horizontal Assim como no Lançamento Horizontal, o movimento na direção do eixo x, no lançamento oblíquo, é uniforme, pois a velocidade é constante. Portanto, a função horária do movimento horizontal é: x = vx.t

A distância horizontal percorrida pelo corpo desde o lançamento é chamada alcance máximo. Podemos determinar o alcance máximo pela equação:

Para determinar a posição do móvel em relação à horizontal temos que determinar a componente da velocidade inicial v0 na direção do eixo x. O módulo da velocidade na direção do eixo x é: vx = v0 . cosα Movimento Vertical O movimento vertical está sob a ação da gravidade, isso implica que o movimento é uniformemente variado e a velocidade vy diminui à medida que a altura em relação ao

solo aumenta. O componente da velocidade inicial na direção do eixo y é: v0y = v0 . senα As funções horárias do movimento vertical são: Função horária do espaço y = v0yt + gt2 2 Função horária da velocidade vy = v0y + gt Equação de Torricelli vy

2 = v0y2 + 2gy

A altura máxima pode ser calculada usando a equação:

Após atingir a altura máxima, o corpo move-se em queda livre sob ação da gravidade, e sua velocidade aumenta com o tempo.

EXERCÍCIOS TEXTO: 1 - Comum à questão: 1

Três bolas – X, Y e Z – são lançadas da borda de uma mesa, com velocidades iniciais paralelas ao solo e mesma direção e sentido.

A tabela abaixo mostra as magnitudes das massas e das velocidades iniciais das bolas.

810Z

105Y

205X

(m/s)

INICIAL VELOCIDADE

)g(

MASSABOLAS

Questão 01 - (UERJ/2012)

As relações entre os respectivos alcances horizontais Ax, Ay e Az das bolas X, Y e Z, com relação à borda da mesa, estão apresentadas em:

a) Ax < Ay < Az b) Ay = Ax = Az

c) Az < Ay < Ax d) Ay < Az < Ax

Questão 02 - (UEM PR)

Dois corpos idênticos A e B são lançados, simultaneamente, da mesma posição, com a mesma velocidade inicial, formando o mesmo ângulo a com a horizontal. Sobre o corpo A, atua apenas a força peso. Sobre o corpo B, além do próprio peso, atua, favoravelmente ao movimento, uma força horizontal constante. Pode-se afirmar que os corpos 01. chegam ao solo simultaneamente. 02. têm o mesmo alcance horizontal. 04. atingem a mesma altura máxima. 08. têm a mesma velocidade quando atingem o solo. 16. têm a mesma aceleração.

Questão 03 - (PUC RJ)

Três bolinhas são largadas da mesma altura h como mostra a figura. As bolinhas A e B parte do repouso e a bolinha C tem velocidade inicial vo horizontal.

A B C

S

h

Vo

Qual é a afirmativa correta? despreze a resistência do ar e o atrito da bolinha A com o solo. a) As três bolinhas chegam ao mesmo tempo na região

horizontal S do solo. b) A bolinha A chega primeiro em S e as bolinhas B e

C chegam juntas depois. c) bolinha B chega primeiro em S, em seguida a

bolinha A e por último a bolinha C.

Page 13: curso de específica de física

13

d) As bolinhas A e B chegam juntas em S e depois a bolinha C.

e) As boinhas B e C chegam juntas em S e depois chega a bolinha A.

Questão 04 - (UNESP/2007)

Em uma partida de futebol, a bola é chutada a partir do solo descrevendo uma trajetória parabólica cuja altura máxima e o alcance atingido são, respectivamente, h e s. Desprezando o efeito do atrito do ar, a rotação da bola e sabendo que o ângulo de lançamento foi de 45º em relação ao solo horizontal, calcule a razão s/h.

Dado: 2/245 cos45 sen 00

Questão 05 - (UFU MG)

Da superfície de uma mesa são lançadas horizontalmente duas pequenas esferas A e B, com velocidades iniciais v0 e 2v0, respectivamente. Se a resistência do ar for considerada irrelevante, como se relacionam os tempos de queda das esferas? a) tA = tB b) tA = tB/4 c) tA = tB/2 d) tA = 4tB e) tA = 2tB

Questão 06 - (FFFCMPA RS/2007)

Uma pedra pe arremessada horizontalmente, com uma velocidade de 20m/s, de uma ponte que está a 16m acima da superfície da água. Qual a velocidade da pedra, após atingir a água? (Considere g = 9,8m/s2) a) 26,7m/s. b) 13,5m/s. c) 713,6m/s. d) 42,5m/s. e) 246,4m/s.

Questão 07 - (UFF RJ/2011)

Após um ataque frustrado do time adversário, o goleiro se prepara para lançar a bola e armar um contraataque. Para dificultar a recuperação da defesa adversária, a bola deve chegar aos pés de um atacante no menor tempo possível. O goleiro vai chutar a bola, imprimindo sempre a mesma velocidade, e deve controlar apenas o ângulo de lançamento. A figura mostra as duas trajetórias possíveis da bola num certo momento da partida.

Assinale a alternativa que expressa se é possível ou não determinar qual destes dois jogadores receberia a bola no menor tempo. Despreze o efeito da resistência do ar.

a) Sim, é possível, e o jogador mais próximo

receberia a bola no menor tempo. b) Sim, é possível, e o jogador mais distante

receberia a bola no menor tempo. c) Os dois jogadores receberiam a bola em tempos

iguais. d) Não, pois é necessário conhecer os valores da

velocidade inicial e dos ângulos de lançamento.

e) Não, pois é necessário conhecer o valor da velocidade inicial.

Questão 08 - (UFC CE/2007)

Considere o lançamento de dois projéteis com o mesmo ângulo inicial. O projétil 1 tem a metade da massa do outro (denotado por 2). Qual a relação entre as velocidades iniciais dos projéteis para que tenham o mesmo alcance?

a) 21 v

2

1v

b) v1 = 2v2 c) v1 = v2

d) 21 v

3

1v

e) v1 = 3v2 TEXTO: 2 - Comuns às questões: 9, 10

Considere o enunciado abaixo.

Na figura que segue, estão apresentadas as trajetórias de dois projéteis, A e B, no campo gravitacional terrestre. O projétil A é solto da borda de uma mesa horizontal de altura H e cai verticalmente; o projétil B é lançado da borda dessa mesa com velocidade horizontal de 1,5 m/s. (O efeito do ar é desprezível no movimento desses projéteis.)

Questão 09 - (UFRGS/2007)

Se o projétil A leva 0,4 s para atingir o solo, quanto tempo levará o projétil B? a) 0,2 s b) 0,4 s c) 0,6 s d) 0,8 s e) 1,0 s

Questão 10 - (UFRGS/2007)

Qual será o valor do alcance horizontal X do projétil B? a) 0,2 m b) 0,4 m c) 0,6 m d) 0,8 m e) 1,0 m

TEXTO: 3 - Comum à questão: 11

Dados que podem ser necessários:

valor da aceleração da gravidade: g = 10,0 m/s2; velocidade da luz no vácuo: c = 3,0 x 108 m/s; equivalente mecânico da caloria: 1 cal = 4,186 joules; duração do intervalo de tempo de um ano na Terra: 3,0 x 107 s;

Questão 11 - (UNIOESTE PR/2008)

Considere a seguinte situação: uma bola de futebol foi chutada por um jogador com um ângulo de 30 graus acima da horizontal com uma velocidade inicial de módulo 20,0 m/s, passando a executar um movimento em duas dimensões, em um plano vertical. Suponha que

Page 14: curso de específica de física

14

a resistência do ar seja desprezível e que a aceleração da gravidade não varie, tendo sempre o valor de 10,0 m/s2. Com base no enunciado acima, considere as afirmativas a seguir: I. A força que acelera a bola de futebol tem um valor

que depende da altura na qual a bola estiver situada. II. O movimento da bola de futebol pode ser

decomposto nas direções horizontal e vertical e os movimentos nas duas direções podem ser considerados independentes entre si.

III. A quantidade de movimento da bola é uma grandeza escalar que se conserva durante o movimento da bola.

IV. A energia mecânica da bola é uma grandeza escalar que pode ser expressa em kW·h e que se conserva durante o movimento da bola.

Estão corretas apenas as afirmativas: a) I e II. b) II e IV. c) III e IV. d) I, II e III. e) I, III e IV.

Questão 12 - (UEL PR/2010)

Um sistema mecânico que consiste de um pequeno tubo

com uma mola consegue imprimir a uma esfera de massa m uma velocidade fixa v0. Tal sistema é posto para funcionar impulsionando a massa na direção vertical, a massa atingindo a altura máxima h e voltando a cair. Em seguida o procedimento é efetuado com o eixo do tubo formando um determinado ângulo com a direção horizontal de modo que o alcance R nesta direção seja maximizado. Tais situações estão representadas na figura a seguir.

Os experimentos ocorrem em um local onde a aceleração da gravidade g′ é um pouco menor que seu valor na superfície terrestre g = 9,8 m/s2. Baseado nesses dados e concordando com expressões cinemáticas para os movimentos de queda livre e lançamento oblíquo, é correto afirmar:

a) A razão R

h obedecerá a relação

'g2

g

R

h

b) A razão R

h obedecerá a relação

'g

g2

R

h

c) A razão R

h obedecerá a relação

g2

'g

R

h

d) A distância R a ser alcançada pela massa será a mesma que se obteria em um experimento na superfície terrestre porque tal quantidade só depende do valor da componente horizontal da velocidade v0 cos(θ).

e) R e h serão diferentes de seus valores obtidos em experimentos realizados na superfície mas a relação

2

1

R

h se manterá porque esta independe do valor

local da aceleração da gravidade. Questão 13 -

Um jogador de golf desfere uma tacada, imprimindo à bola uma velocidade inicial com módulo v0 = 20 m/s e ângulo = 45º em relação ao eixo-x horizontal, de

acordo com a figura abaixo. Desprezando a resistência aerodinâmica do ar e considerando que o módulo da aceleração da gravidade vale g = 10 m/s2, determine o alcance máximo A da bola de golf.

a) 4 metros b) 200 metros c) 100 metros d) 40 metros e) 2 metros

Questão 14 - (UEM PR/2013)

Uma pequena esfera é lançada do solo com velocidade inicial de módulo vo, em uma direção que forma um ângulo com a horizontal. Desprezando o atrito com o

ar, assinale o que for correto.

01. A função horária da posição da esfera no eixo horizontal é uma função de primeiro grau.

02. Tanto a função horária da posição da esfera no eixo vertical, quanto a função que descreve a trajetória da esfera (função da posição da esfera no eixo vertical em relação ao eixo horizontal) são funções de segundo grau.

04. A componente horizontal da velocidade inicial da esfera é dada por vo cos.

08. O intervalo de tempo que a esfera leva até atingir a altura máxima é igual ao que ela levaria se fosse lançada verticalmente para cima com a mesma velocidade inicial vo.

16. O módulo da velocidade com que a esfera atinge o solo é vo.

Questão 15 - (UEM PR/2006)

Em uma cena de filme, um policial em perseguição a um bandido salta com uma moto do topo de um prédio a outro. Considere que ambos os prédios têm o topo quadrado com uma área de 900 m2 e que o policial motorizado se lança horizontalmente com uma velocidade de 72 km/h. Considere ainda que a distância

Page 15: curso de específica de física

15

entre os prédios é de 20 m e que o topo do segundo prédio está 10 m abaixo do topo do primeiro. Nessas condições, pode-se afirmar que essa cena poderia ser real? (Considere a aceleração gravitacional igual a 10 m/s2. Despreze a resistência do ar.) a) Sim, pois o policial alcançaria o topo do segundo

prédio aproximadamente 8 m após a primeira borda do prédio.

b) Não, pois com essa velocidade inicial, o policial ultrapassaria o topo do segundo prédio.

c) Não, pois o policial cairia entre os prédios em queda livre.

d) Não, pois o policial atingiria a parede lateral do prédio em alguma altura do edifício.

e) Não, pois o policial alcançaria o topo do segundo prédio a aproximadamente 0,5 m da segunda borda do prédio, sem espaço suficiente para parar a moto.

Questão 16 - (UEM PR)

Uma pedra é lançada com um ângulo de 45º em relação ao eixo horizontal x e na direção positiva de x. Desprezando-se a resistência do ar, quais dos gráficos melhor representam a componente horizontal da velocidade (vx) versus tempo (t) e a componente vertical da velocidade (vy) versus tempo (t), respectivamente?

I.

II.

III.

IV.

V.

vx versus t vy versus t

a) I e IV b) II e I c) II e III

d) II e V e) IV e V

Questão 17 - (UEM PR/2008)

Um objeto ao nível do mar é lançado obliquamente com velocidade inicial de 100,0 m/s, com um ângulo de

lançamento tal que o 6,0)cos( (obs.: despreze a

resistência do ar). Considere g = 10,0 m/s2. Assinale o que for correto. 01. As componentes horizontal e vertical da velocidade

no instante de lançamento são vx = 60,0 m/s e vy = 80,0 m/s.

02. Desprezando a resistência do ar, o objeto não retorna ao nível de lançamento.

04. O alcance máximo do objeto é superior a 500 m. 08. O tempo necessário para o objeto atingir o alcance

máximo é 16,0 s. 16. O módulo da componente da velocidade no eixo

paralelo ao solo se mantém constante durante o percurso.

Questão 18 - (UFG GO)

50 ANOS DE BOMBA ATÔMICA As explosões das bombas atômicas, em agosto de 1945, sobre as cidades de Hiroshima e Nagasaki, fizeram, em 1995, cinqüenta anos. O fim da guerra e o início da era atômica se anunciavam e o clarão escurecia os olhares do mundo que oscilavam entre o pavor e o medo. O desconhecido explodia em solo japonês. A bomba detonava os horrores da guerra, levantava a poeira atômica das nações rivais e emitia sinais de que os tempos seriam outros para as nações. Por certo, nunca mais seriam os mesmos para os “filhos do clarão.”

Fontes: Superinteressante, ano 09,nº07, 1995; Veja,

02/08/95.

Considerando que, nas proposições a seguir, as informações técnicas (nomes, distâncias, massas, etc.), a respeito dos lançamentos e efeitos das bombas atômicas sobre Hiroshima e Nagasaki, são verdadeiras, analise as proposições, sob o aspecto da Física, e identifiq44ue as correta. 01. o avião B-29, chamado “Enola Gay”, soltou a bomba

apelidada de “Little Boy” sobre Hiroshima. A bomba foi solta a 9600m de altura e explodiu a aproximadamente 580m do solo. Se a resistência do ar fosse desprezada, a bomba levaria aproximadamente 32 segundos para explodir (adotar g = 10m/s2);

02. o avião B-29, chamado “Bock’s Car”, soltou a bomba apelidada de “Fat Man” acima de Nagasaki. A intenção era atingir o centro da cidade, mas por problemas de visibilidade atingiu a periferia. Ao soltar a bomba o avião estava justamente acima do ponto atingido;

04. pessoas que estavam distantes do ponto da explosão, escutaram primeiramente o som e depois viram o clarão da explosão;

08. “Little Boy”, de 4,0 toneladas, constituída de urânio, possuía poder de destruição equivalente a 15.000 toneladas de TNT e “Fat Man”, de 4,5 toneladas, constituída de plutônio, a 20.000 toneladas de TNT. Se a resistência do ar fosse desprezada e as duas bombas fossem soltas da mesma altura e no mesmo instante, “Fat Man” atingiria o solo antes de “Little

Boy ”. 16. Segundo o físico Naomi Shohno, o deslocamento de

ar provocado pelas bombas percorreu 740m no segundo posterior à explosão, 4 km em 10 segundos e 11 km em 30 segundos. Portanto, a velocidade média de deslocamento do ar no

Page 16: curso de específica de física

16

primeiro segundo foi de 2664 km/h e em 30 segundos foi de aproximadamente 1320km/h.

Questão 19 - (UFOP MG)

Uma partícula desloca-se em movimento retilíneo uniforme sobre uma plataforma horizontal lisa do ponto A ao ponto B, com velocidade v0 = 10m/s. A partir do ponto B, a partícula se movimenta sob a ação de seu peso até atingir o ponto D localizado em outra plataforma horizontal, como mostra a figura abaixo.

a) Calcule a distância do ponto C ao ponto D. b) Calcule o tempo que a partícula gasta para se

deslocar do ponto A ao ponto D. c) Determine a velocidade da partícula imediatamente

antes de atingir o ponto D. Dados: AB = 10m; BC = 20m; g = 10m/s2.

Questão 20 - (ITA SP)

Uma bola é lançada horizontalmente do alto de um edifício, tocando o solo decorridos aproximadamente 2s. Sendo de 2,5m a altura de cada andar, o número de andares do edifício é a) 5 b) 6 c) 8 d) 9 e) indeterminado pois a velocidade horizontal de

arremesso da bola não foi fornecida. GABARITO: 1) Gab: C 2) Gab: 01-04

3) Gab: E

4) Gab:

4h

s

5) Gab: A 6) Gab: A 7) Gab: B 8) Gab: C 9) Gab: B 10) Gab: C 11) Gab: B 12) Gab: E 13) Gab: D 14) Gab: 23 15) Gab: A 16) Gab: C 17) Gab: 29 18) Gab: 01-F; 02-F; 04-F; 08-F; 16-V. 19) Gab:

a) 20m; b) 3s; c) vR = 22,4m/s

20) Gab: C 21) Gab: D 22) Gab: D 23) Gab: D 24) Gab: A 25) Gab: 17 26) Gab: C 27) Gab: B 28) Gab: D 29) Gab: 04-08-16 30) Gab: B 31) Gab: D 32) Gab: A 33) Gab: E 34) Gab: B 35) Gab: 05 36) Gab: D 37) Gab: 19 38) Gab: 31 39) Gab: B 40) Gab: 21

Page 17: curso de específica de física

17

LISTA 4 - MOVIMENTO CIRCULAR

RESUMO TEÓRICO O movimento circular uniforme (MCU) é o movimento no qual o corpo descreve trajetória circular, podendo ser uma circunferência ou um arco de circunferência. A velocidade escalar permanece constante durante todo o trajeto e a velocidade vetorial apresenta módulo constante, no entanto sua direção é variável. A aceleração tangencial é nula (at = 0), porém, com a aceleração centrípeta não ocorre o mesmo, ou seja, a aceleração não é nula (ac ≠ 0). A direção da aceleração centrípeta, em cada ponto da trajetória, é perpendicular à velocidade vetorial e aponta para o centro da trajetória. O módulo da aceleração centrípeta é escrito da seguinte forma: ac = v2/r, onde r é o raio da circunferência descrita pelo móvel.

Um corpo que descreve um movimento circular uniforme passa de tempo em tempo no mesmo ponto da trajetória, sempre com a mesma velocidade. Assim, podemos dizer que esse movimento é repetitivo, e pode ser chamado de movimento periódico. Nos movimentos periódicos existem dois conceitos muito importantes que são: frequência e período. Frequência: é o número de voltas que o corpo efetua em um determinado tempo (f = 1/ T). Período: é o tempo gasto para se completar um ciclo (T = 1/ f). Ao observar a definição de período e de frequência podemos dizer que o período é o inverso da frequência. Equações do Movimento Circular As equações que determinam o movimento circular são as seguintes: Posição angular: S = φ .R, onde R é o raio da circunferência. Velocidade angular média: ωm = Δφ/Δt Aceleração centrípeta: ac = v2/R, onde R é o raio da circunferência. Força Centrípeta Para que um móvel possa descrever o movimento circular uniforme é necessário que esteja atuando uma força sobre ele, de modo que faça com que ele mude de posição, pois se tal fato não ocorrer o móvel passaria a descrever um movimento retilíneo uniforme. Essa força tem o nome de força centrípeta, e matematicamente é descrita da seguinte forma: Fc = m. ac Onde ac é a aceleração centrípeta, ac = v2/R. Substituindo na equação acima temos: Fc = m. v2/R A força centrípeta é sempre direcionada para o centro da circunferência. No cotidiano existem alguns exemplos de força centrípeta como a secadora de roupas e os satélites

que ficam em órbita circular em torno do centro da Terra. Fonte: http://www.brasilescola.com/fisica/movimento-circular.htm EXERCÍCIOS Questão 01 - (UNIMAR SP)

Um ciclista descreve um movimento circular uniforme no sentido anti-horário, conforme a trajetória abaixo. No ponto X, o vetor aceleração é melhor ilustrado por:

.X

a) b) c)

d) e)

Questão 02 - (PUC RS)

A velocidade angular do movimento de rotação da Terra é, aproximadamente, a) (/12) rad/h b) (/6) rad/h

c) (/4) rad/h

d) rad/h

e) 2 rad/h

Questão 03 - (UECE)

Em um relógio, o período de rotação do ponteiro dos segundos, o dos minutos e o das horas são, respectivamente: a) um segundo, um minuto e uma hora b) um minuto, uma hora e um dia c) um minuto, meia hora e um dia d) um minuto, uma hora e meio dia

Questão 04 - (UFSC/2008)

Um carro com velocidade de módulo constante de 20 m/s percorre a trajetória descrita na figura, sendo que

de A a C a trajetória é retilínea e de D a F é circular, no sentido indicado.

Assinale a(s) proposição(ões) CORRETA(S). 01. O carro tem movimento uniforme de A até C. 02. O carro tem movimento uniforme de A até F. 04. O carro tem aceleração de A até C. 08. O carro tem aceleração de D até F. 16. O carro tem movimento retilíneo uniformemente

variado de D até F. Questão 05 - (UFU MG/2007)

Três rodas de raios Ra, Rb e Rc possuem velocidades angulares wa, wb e wc, respectivamente, e estão ligadas entre si por meio de uma correia, como ilustra figura abaixo.

Page 18: curso de específica de física

18

Ao mesmo tempo que a roda de raio Rb realiza duas voltas, a roda de raio Rc realiza uma volta. Não há

deslizamento entre as rodas e a correia. Sendo ac R 3R

, é correto afirmar que:

a) caab w

3

4 we R

3

4R .

b) caab w3 we R

3

4R .

c) caab w

3

4 we R

2

3R .

d) caab w3 we R

2

3R .

Questão 06 - (UNINOVE SP/2009)

As rodas de um automóvel têm diâmetro de 60cm. Quando o veículo transita a 36 km/h e suas rodas não derrapam sobre o piso, a frequência com que elas giram é, em Hz, de, aproximadamente,

a) 16,7. b) 10,6. c) 5,3. d) 2,7. e) 1,4.

Questão 07 - (UFT TO/2008)

Em uma aula de física, os alunos observam um objeto

descrevendo um movimento circular uniforme. Seja v a

velocidade e v a aceleração do objeto. Após observarem

o fenômeno, fazem os seguintes comentários:

I. No movimento circular uniforme a soma das forças que agem no objeto não é nula, portanto existe aceleração não nula.

II. No movimento circular uniforme v muda

constantemente, enquanto a velocidade angular é constante.

a) Os comentários I e II estão corretos. b) Os comentários I e II estão errados. c) Apenas o comentário I está correto. d) Apenas o comentário II está correto.

Questão 08 - (ESPCEX/2009)

Uma máquina industrial é movida por um motor elétrico que utiliza um conjunto de duas polias, acopladas por uma correia, conforme figura abaixo. A polia de raio R1 = 15 cm está acoplada ao eixo do motor e executa 3000 rotações por minuto. Não ocorre escorregamento no

contato da correia com as polias. O número de rotações por minuto, que a polia de raio R2 = 60 cm executa, é de

Desenho Ilustrativo

a) 250 b) 500 c) 750 d) 1000 e) 1200

Questão 09 - (FAMECA SP/2010)

A relação entre as velocidades angulares de duas pessoas paradas, em relação à Terra, uma sobre o equador terrestre e outra, no polo norte, é

a) zero. b) 1/24. c) 1/12. d) 1. e) 6.

Questão 10 - (UNIMONTES MG/2010)

Na figura, estão representadas duas polias, A e B, com raios RA < RB, acopladas por um eixo.

É CORRETO afirmar:

a) As velocidades angulares dos pontos periféricos da

polia A são iguais às dos pontos periféricos da polia B.

b) As velocidades angulares dos pontos periféricos da polia A são maiores do que as dos pontos periféricos da polia B.

c) As velocidades lineares dos pontos periféricos da polia A são iguais às dos pontos periféricos da polia B.

d) As velocidades lineares dos pontos periféricos da polia A são maiores do que as dos pontos periféricos da polia B.

Questão 11 - (UEL)

Considere as seguintes afirmativas: I. No movimento circular uniforme, os vetores

velocidade e aceleração são perpendiculares entre si.

II. Objetos de mesma forma e dimensões, mas com massas diferentes, quando soltos de uma mesma altura, por estarem sob a influência da mesma aceleração gravitacional, chegam ao solo no mesmo instante.

III. Do ponto de vista microscópico, as forças responsáveis pelo atrito entre duas superfícies são as forças gravitacionais que atuam nas regiões em que as duas superfícies estão em contato.

Assinale a alternativa correta.

Page 19: curso de específica de física

19

a) Apenas as afirmativas I e III são verdadeiras. b) Apenas as afirmativas II e III são verdadeiras. c) Apenas as afirmativas I e II são verdadeiras. d) Apenas a afirmativa III é verdadeira. e) Todas as afirmativas são verdadeiras.

Questão 12 - (UEL PR)

A bicicleta tem o pedal preso a um disco denominado “coroa”. A corrente liga a coroa à catraca, que é o disco preso à roda traseira. A cada pedalada, a catraca gira várias vezes, pois seu diâmetro é menor que o diâmetro da coroa. Qual é a distância percorrida por uma bicicleta de aro 33 (raio da roda igual a 33cm), cuja coroa tem raio três vezes maior que o raio da catraca, no período igual a uma pedalada? a) 5,3 m b) 5,7 m c) 6,2 m d) 6,8 m e) 7,1 m

Questão 13 - (UEL PR/2006)

Os primeiros relógios baseavam-se no aparente movimento do Sol na abóboda celeste e no deslocamento da sombra projetada sobre a superfície de um corpo iluminado pelo astro. Considere que: a Terra

é esférica e seu período de rotação é de 24 horas no sentido oeste-leste; o tempo gasto a cada 15º de rotação é de 1 hora; o triângulo Brasília/Centro da Terra/Luzaka (Zâmbia) forma, em seu vértice central, um ângulo de 75°.

A hora marcada em Luzaka, num relógio solar, quando o sol está a pino em Brasília é: a) 5 horas. b) 9 horas. c) 12 horas. d) 17 horas. e) 21 horas.

Questão 14 - (UEL PR/2010)

Considere uma esfera sólida de raio r e momento de

inércia inicial 2ii mr

5

2I que gira com período T ao redor

de um eixo vertical que passa por seu centro. Essa esfera possui matéria uniformemente distribuída através de seu volume. Devido a um desequilíbrio de forças, essa matéria rearranja-se em uma nova configuração de equilíbrio cuja geometria é a de uma casca com formato

esférico e momento de inércia final 2ff mr

3

2I .

Sob que condições o período de rotação da esfera permanecerá inalterado?

a) Aumento na velocidade de rotação .

b) Esta condição será satisfeita se os raios iniciais e finais forem iguais: ri = rf já que neste caso o momento angular será conservado.

c) A conservação do momento angular implica em uma diminuição da velocidade angular e ao mesmo tempo um aumento no raio da esfera de forma que

if r3

5r .

d) A conservação do momento angular implica em uma

diminuição do raio da esfera de forma que if r3

2r

. e) A conservação do momento angular implica em uma

diminuição do raio da esfera de forma que if r5

3r

. Questão 15 - (UEL PR/2010)

Um ciclista descreve uma volta completa em uma pista que se compõe de duas retas de comprimento L e duas semicircunferências de raio R conforme representado na figura a seguir.

A volta dá-se de forma que a velocidade escalar média

nos trechos retos é v e nos trechos curvos é v3

2 . O

ciclista completa a volta com uma velocidade escalar

média em todo o percurso igual a v5

4 .

A partir dessas informações, é correto afirmar que o raio dos semicírculos é dado pela expressão:

a) L = πR

b) 2

RL

c) 3

RL

d) 4

RL

e) 2

R3L

Questão 16 - (UEM PR)

Um carro se move com velocidade constante em uma estrada curva num plano horizontal. Desprezando-se a

resistência do ar, pode-se afirmar corretamente que sobre o carro atua;

01. uma força na mesma direção e em sentido contrário ao centro da curva.

02. uma força de atrito na mesma direção e no mesmo sentido do centro da curva.

04. uma força perpendicular à trajetória e dirigida para cima.

Page 20: curso de específica de física

20

08. uma força perpendicular à trajetória e dirigida para baixo.

16. uma força na mesma direção e no mesmo sentido do movimento do carro.

Questão 17 - (UEM PR)

Uma barra constituída de material isolante tem, em cada extremidade, uma carga de 1 C. Se a barra girar em torno de seu ponto médio com velocidade angular w = 8

rad/s, podemos afirmar que

01. a barra completará quinze voltas em cada segundo.

02. a corrente elétrica proporcionada pelo movimento da barra será de 8 A.

04. o período das oscilações da barra será de 0,13 segundos.

08. a barra não executará um M.H.S.. 16. a corrente elétrica terá dimensão de

segundo por Coulomb. Questão 18 - (UEM PR)

Das afirmativas a seguir, assinale o que for correto. 01. Quando um móvel executa um movimento circular

uniforme, sua aceleração é nula. 02. No movimento circular uniforme, a frequência é

constante.

04. No movimento circular uniforme, o vetor velocidade tangencial é variável.

08. A forma angular da equação horária do movimento

circular uniforme é t0 , onde é a posição

angular do móvel no instante t, 0 é a posição

angular do móvel no instante 0t 0 e é a

velocidade angular do móvel. 16. A frequência é inversamente proporcional ao

quadrado do período. 32. Quando um ponto material percorre uma

circunferência em movimento circular uniforme, a projeção do ponto material sobre um diâmetro da circunferência realiza um movimento harmônico simples.

Questão 19 - (UEM PR)

Imagine que você esteja em um carrossel de parque de diversões que gira em um movimento circular uniforme. A figura abaixo representa o carrossel visto de cima. O brinquedo gira sempre paralelo ao chão, sem movimentos verticais. Imagine agora que você lança, do ponto P, uma chave para um amigo parado a uma certa distância do brinquedo. Em que posição deveria estar esse amigo para apanhar a chave? (Despreze a resistência do ar.)

a) I b) II c) III d) IV e) V

Questão 20 - (UEM PR/2009)

Duas polias, A e B, de raios R1 = 10cm e R2 = 20cm, giram acopladas por uma correia de massa desprezível que não desliza, e a polia A gira com uma frequência de

rotação de 20 rpm. Assinale a(s) alternativa(s) correta(s).

01. A velocidade de qualquer ponto P da correia é

aproximadamente 0,21 m/s. 02. A frequência angular de rotação da polia B é 2,0

rad/s. 04. A razão entre as frequências de rotação das polias

A e B é 2. 08. O período de rotação da polia A é 3,0 s. 16. A aceleração centrípeta experimentada por uma

partícula de massa m, colocada na extremidade da polia A (borda mais externa), é maior do que se a mesma partícula fosse colocada na extremidade da polia B.

GABARITO: 1) Gab: D 2) Gab: A 3) Gab: D 4) Gab: 11 5) Gab: D 6) Gab: C 7) Gab: A 8) Gab: C 9) Gab: D 10) Gab: A 11) Gab: C 12) Gab: C 13) Gab: D 14) Gab: E 15) Gab: A 16) Gab: 02-04-08 17) Gab: 10

18) Gab: 46 19) Gab: C 20) Gab: 29 21) Gab: E 22) Gab: B 23) Gab: C 24) Gab: C 25) Gab: A 26) Gab: E 27) Gab: 02 28) Gab: 51 29) Gab: A

30) Gab: D 31)

3

1

B

A

I

I 32) Gab: E

33) Gab: B 34) Gab: B 35) Gab: 29 36) Gab: E 37) Gab:

a) f = 3,3 rad/s

b) tf = 3s c) c = 3 rad/s d) = 4,1 rad

38) Gab: 14 39) Gab: C 40) Gab: FFVF 41) Gab: A

Page 21: curso de específica de física

21

LISTA 5 - VETORES E CINEMÁTICA VETORIAL

RESUMO TEÓRICO Vetores Determinado por um segmento orientado AB, é o conjunto de todos os segmentos orientados equipolentes a AB.

Se indicarmos com este conjunto, simbolicamente

poderemos escrever:

onde XY é um segmento qualquer do conjunto.

O vetor determinado por AB é indicado por ou B - A

ou .

Um mesmo vetor é determinado por uma infinidade de segmentos orientados, chamados representantes desse vetor, os quais são todos equipolentes entre si. Assim, um segmento determina um conjunto que é o vetor, e qualquer um destes representantes determina o mesmo vetor. Usando um pouco mais nossa capacidade de abstração, se considerarmos todos os infinitos segmentos orientados de origem comum, estaremos caracterizando, através de

representantes, a totalidade dos vetores do espaço. Ora, cada um destes segmentos é um representante de um só vetor. Consequentemente, todos os vetores se acham representados naquele conjunto que imaginamos.

As características de um vetor são as mesmas de qualquer um de seus representantes, isto é: o módulo, a direção e o sentido do vetor são o módulo, a direção e o sentido de qualquer um de seus representantes.

O módulo de se indica por | | .

Soma de vetores Se v=(a,b) e w=(c,d), definimos a soma de v e w, por:

v + w = (a+c,b+d) Propriedades da Soma de vetores

Diferença de vetores Se v=(a,b) e w=(c,d), definimos a diferença entre v e w, por:

v - w = (a-c,b-d) Produto de um número escalar por um vetor

Se v=(a,b) é um vetor e c é um número real, definimos a multiplicação de c por v como:

c.v = (ca,cb) Propriedades do produto de escalar por vetor Quaisquer que sejam k e c escalares, v e w vetores:

Módulo de um vetor O módulo ou comprimento do vetor v=(a,b) é um número real não negativo, definido por:

Vetor unitário Vetor unitário é o que tem o módulo igual a 1. Existem dois vetores unitários que formam a base canônica para o espaço R², que são dados por: i = (1,0) j = (0,1) Para construir um vetor unitário u que tenha a mesma direção e sentido que um outro vetor v, basta dividir o vetor v pelo seu módulo, isto é:

Observação: Para construir um vetor u paralelo a um vetor v, basta tomar u=cv, onde c é um escalar não nulo. Nesse caso, u e v serão paralelos: Se c = 0, então u será o vetor nulo. Se 0 < c < 1, então u terá comprimento menor do que v. Se c > 1, então u terá comprimento maior do que v.

Se c < 0, então u terá sentido oposto ao de v. Decomposição de vetores em Vetores Unitários Para fazer cálculos de vetores em apenas um dos planos em que ele se apresenta, pode-se decompor este vetor em vetores unitários em cada um dos planos apresentados. Sendo simbolizados, por convenção, î como vetor unitário

do plano x e como vetor unitário do plano y. Caso o problema a ser resolvido seja dado em três dimensões, o

vetor utilizado para o plano z é o vetor unitário .

Page 22: curso de específica de física

22

Então, a projeção do vetor no eixo x do plano cartesiano

será dado por , e sua projeção no eixo ydo plano

será: . Este vetor pode ser escrito como:

=( , ), respeitando que sempre o primeiro componente entre parênteses é a projeção em x e o segundo é a projeção no eixo y. Caso apareça um terceiro componente, será o componente do eixo z. No caso onde o vetor não se encontra na origem, é possível redesenhá-lo, para que esteja na origem, ou então descontar a parte do plano onde o vetor não é projetado.

Fonte: http://www.sofisica.com.br/conteudos/Mecanica/Cinematica/Vetores.php EXERCÍCOS Questão 01 - (UDESC/2012)

Observando o movimento de um carrossel no parque de diversões, conclui-se que seu movimento é do tipo circular uniforme.

Assinale a alternativa correta em relação ao movimento.

a) Não é acelerado porque o módulo da velocidade

permanece constante. b) É acelerado porque o vetor velocidade muda de

direção, embora mantenha o mesmo módulo. c) É acelerado porque o módulo da velocidade varia. d) Não é acelerado porque a trajetória não é retilínea. e) Não é acelerado porque a direção da velocidade

não varia. Questão 02 - (MACK SP/2012)

Um avião, após deslocar-se 120 km para nordeste (NE), desloca-se 160 km para sudeste (SE). Sendo um quarto de hora, o tempo total dessa viagem, o módulo da velocidade vetorial média do avião, nesse tempo, foi de

a) 320 km/h b) 480 km/h c) 540 km/h d) 640 km/h

e) 800 km/h Questão 03 - (FMTM MG/2006)

A figura apresenta uma “árvore vetorial” cuja resultante da soma de todos os vetores representados tem módulo, em cm, igual a

a) 8. b) 26. c) 34. d) 40. e) 52.

Questão 04 - (UNIFICADO RJ)

Considere um helicóptero movimentando-se no ar em três situações diferentes: I. Subindo verticalmente com velocidade constante; II. Descendo verticalmente com velocidade constante; III. Deslocando-se horizontalmente para a direita, com velocidade constante. A resultante das forças exercidas pelo ar sobre o helicóptero, em cada uma dessas situações, é corretamente representada por:

a.

b.

c.

d.

e.

I II III

Questão 05 - (MACK SP/2007)

O movimento de uma partícula é caracterizado por ter vetor velocidade e vetor aceleração não nulo de mesma direção. Nessas condições, podemos afirmar que esse movimento é a) uniforme. b) uniformemente variado. c) harmônico simples. d) circular uniforme. e) retilíneo.

Questão 06 - (PUC MG)

Marque a alternativa INCORRETA. a) Quando o módulo da velocidade é constante, a

aceleração pode não ser nula. b) Quando a aceleração é nula, o módulo da velocidade

é constante. c) A componente de um vetor é um vetor. d) Quando um vetor é nulo, ele pode ter componentes

não nulas. Questão 07 - (UNIUBE MG)

Em uma pista de Fórmula Indy, um piloto faz o trajeto CD e FA com movimentos retardado, os trajetos AB e DE com movimento acelerado e os trechos BC e EF com movimento uniforme. A alternativa que mostra o vetor aceleração resultante corretamente, supondo que o movimento se realize na ordem alfabética, é:

Page 23: curso de específica de física

23

F E

B C

A D

a.

F E

B C

A D

b.

F E

B C

A D

c.

F E

B C

A D

d.

A

F E

B C

D

e.

Questão 08 - (PUC MG)

Você e um amigo resolvem ir ao último andar de um edifício. Vocês partem juntos do primeiro andar. Entretanto, você vai pelas escadas e seu amigo, pelo elevador. Depois de se encontrarem na porta do elevador, descem juntos pelo elevador até o primeiro andar. É CORRETO afirmar que: a) o seu deslocamento foi maior que o de seu amigo. b) o deslocamento foi igual para você e seu amigo. c) o deslocamento de seu amigo foi maior que o seu. d) a distância que seu amigo percorreu foi maior que

a sua. Questão 09 - (UFSC)

Assinale a(s) afirmativa correta, some os valores respectivos e marque o resultado no cartão-resposta.

A representação correta da velocidade tangencial “ v”,

da quantidade de movimento “ q”, da aceleração

centrípeta “ a” e da força normal “ F

”, atuantes sobre

um ponto material de massa m que descreve um movimento circular uniforme (MCU), está(ão)

corretamente representada(s) no(s) esquema(s)

01.F

a

0

qv

02.F

a

0

q

v

04.

F

a

0

q v

08.

Fa

0

q

v

16.

F

a

0

q

v

32.

F

a

0

q

v

64. Fa

0

q

v

Questão 10 - (CESGRANRIO RJ)

A figura a seguir mostra a fotografia estroboscópica do movimento de uma partícula.

V

P

IV III

II

I

A aceleração da partícula, no ponto P da trajetória, é melhor representada pelo vetor: a) I; b) II; c) III; d) IV; e) V.

Questão 11 - (UEL PR/2008)

Um ciclista percorre as rotas 1 e 2 para se deslocar do ponto A ao ponto B, como mostrado no mapa a seguir, e registra em cada uma a distância percorrida. Assinale a alternativa que apresenta os valores aproximados da distância percorrida na rota 1 e na rota 2. Considere como aproximação todos os quarteirões quadrados com 100 m de lado. As rotas 1 e 2 encontram-se pontilhadas.

a) rota 1 800 m;

rota 2 800 m.

b) rota 1 700 m;

rota 2 700 m.

c) rota 1 800 m;

rota 2 900 m.

d) rota 1 900 m;

rota 2 700 m.

e) rota 1 900 m;

Page 24: curso de específica de física

24

rota 2 600 m

Questão 12 - (UEL PR/2013)

Uma família viaja para Belém (PA) em seu automóvel. Em um dado instante, o GPS do veículo indica que ele se localiza nas seguintes coordenadas: latitude 21°20’ Sul e longitude 48°30’ Oeste. O motorista solicita a um dos passageiros que acesse a Internet em seu celular e obtenha o raio médio da Terra, que é de 6730 km, e as coordenadas geográficas de Belém, que são latitude 1°20’ Sul e longitude 48°30’ Oeste. A partir desses dados, supondo que a superfície da Terra é esférica, o motorista calcula a distância D, do veículo a Belém, sobre o meridiano 48°30’Oeste.

Assinale a alternativa que apresenta, corretamente, o valor da distância D, emkm.

a) 67309

D

b) 2)6730(18

D

c) 67309

D

d) 673036

D

e) 67303

D

2

Questão 13 - (PUCCAMP SP/2011)

Analise o esquema abaixo.

O vetor resultante ou soma vetorial das três medidas acima representadas tem módulo

a) 11 b) 13 c) 15 d) 17 e) 19

Questão 14 - (UNESP/2012)

No dia 11 de março de 2011, o Japão foi sacudido por terremoto com intensidade de 8,9 na Escala Richter, com o epicentro no Oceano Pacífico, a 360 km de Tóquio, seguido de tsunami. A cidade de Sendai, a 320 km a nordeste de Tóquio, foi atingida pela primeira onda do tsunami após 13 minutos.

(O Estado de S.Paulo, 13.03.2011. Adaptado.)

Baseando-se nos dados fornecidos e sabendo que cos

0,934, onde é o ângulo Epicentro-Tóquio-Sendai, e que 28 32 93,4 215 100, a velocidade média, em

km/h, com que a 1.ª onda do tsunami atingiu até a cidade de Sendai foi de:

a) 10. b) 50. c) 100. d) 250. e) 600.

Questão 15 - (UFSCar SP)

Nos esquemas estão representadas a velocidade V e a

aceleração a do ponto material P. Assinale a

alternativa em que o módulo da velocidade desse ponto material permanece constante. a.

Pv

a

b.

P

a

v

c.

P

a

v

.

d.

P

a v.

e.

P v

a

Questão 16 - (UNIFESP SP/2002)

Na figura, são dados os vetores a , b e c

Sendo u a unidade de medida do módulo desses vetores,

pode-se afirmar que o vetor cbad :tem módulo

a) 2u, e sua orientação é vertical, para cima. b) 2u, e sua orientação é vertical, para baixo. c) 4u, e sua orientação é horizontal, para a direita. d) 2 u, e sua orientação forma 45º com a horizontal,

no sentido horário. e) 2 u, e sua orientação forma 45º com a horizontal,

no sentido anti-horário. Questão 17 - (UNIFOR CE)

Numa sala cúbica, de aresta a, uma mosca voa numa diagonal (segmento que une dois vértices, passando

Page 25: curso de específica de física

25

pelo centro a sala). O deslocamento da mosca tem módulo a) a

b) 2a

c) 3a

d) a49

e) 3a Questão 18 - (FUVEST SP/2002)

Em uma estrada, dois carros, A e B, entram simultaneamente em curvas paralelas, com raios RA e RB. Os velocímetros de ambos os carros indicam, ao longo de todo o trecho curvo, valores constantes VA e VB. Se os carros saem das curvas ao mesmo tempo, a relação entreVA e VB é:

a) VA = VB b) VA/VB = RA/ RB c) VA/VB = (RA/ RB )2

d) VA/VB = RB/ RA e) VA/VB = (RB/ RA)2

Questão 19 - (UFOP MG)

A velocidade do centro de massa de um disco que rola sem deslizar, em um plano horizontal é v. As velocidades dos pontos A e B, mostrados na figura são, respectivamente.

O

V

A

B

X

Y

a) – v e v b) 0 e v c) 0 e 2v d) v e v e) v e 2v

Questão 20 - (MACK SP)

Um corpo é atirado verticalmente para cima a partir do solo com velocidade inicial de módulo 50 m/s. O módulo de sua velocidade vetorial média entre o instante de lançamento e o instante em que retorna ao solo é: a) 50 m/s; b) 25 m/s; c) 5,0 m/s; d) 2,5 m/s; e) zero.

GABARITO: 1) Gab: B 2) Gab: E 3) Gab: C 4) Gab: A 5) Gab: E 6) Gab: D 7) Gab: B 8) Gab: B 9) Gab: 04 10) Gab: B 11) Gab: D 12) Gab: A 13) Gab: B 14) Gab: E 15) Gab: C 16) Gab: B 17) Gab: C 18) Gab: B 19) Gab: C 20) Gab: E 21) Gab: C 22) Gab: D 23) Gab: B 24) Gab: B 25) Gab: E 26) Gab: D 27) Gab: E 28) Gab: E 29) Gab: E 30) Gab: A 31) Gab: C 32) Gab: D 33) Gab: E 34) Gab: D 35) Gab: E 36) Gab: 25 37) Gab: A 38) Gab: 07 39) Gab: 03 40) Gab: 25

Page 26: curso de específica de física

26

LISTA 6 - LEIS DE NEWTON

RESUMO TEÓRICO

DINÂMICA

Quando se fala em dinâmica de corpos, a imagem que vem à cabeça é a clássica e mitológica de Isaac Newton, lendo seu livro sob uma macieira. Repentinamente, uma maçã cai sobre a sua cabeça. Segundo consta, este foi o primeiro passo para o entendimento da gravidade, que atraia a maçã. Com o entendimento da gravidade, vieram o entendimento de Força, e as três Leis de Newton. Na cinemática, estuda-se o movimento sem compreender sua causa. Na dinâmica, estudamos a relação entre a força e movimento. Força: É uma interação entre dois corpos. O conceito de força é algo intuitivo, mas para compreendê-lo, pode-se basear em efeitos causados por ela, como: Aceleração: faz com que o corpo altere a sua velocidade, quando uma força é aplicada. Deformação: faz com que o corpo mude seu formato, quando sofre a ação de uma força. Força Resultante: É a força que produz o mesmo efeito que todas as outras aplicadas a um corpo. Dadas várias forças aplicadas a um corpo qualquer:

A força resultante será igual a soma vetorial de todas as forças aplicadas:

Leis de Newton As leis de Newton constituem os três pilares fundamentais do que chamamos Mecânica Clássica, que justamente por isso também é conhecida por Mecânica Newtoniana. 1ª Lei de Newton - Princípio da Inércia

Quando estamos dentro de um carro, e este

contorna uma curva, nosso corpo tende a permanecer com a mesma velocidade vetorial a que estava submetido antes da curva, isto dá a impressão que se está sendo "jogado" para o lado contrário à curva. Isso porque a velocidade vetorial é tangente a trajetória.

Quando estamos em um carro em movimento e

este freia repentinamente, nos sentimos como se fôssemos atirados para frente, pois nosso corpo tende a continuar em movimento.

estes e vários outros efeitos semelhantes são explicados pelo princípio da inércia, cujo enunciado é:

"Um corpo em repouso tende a permanecer em repouso, e um corpo em movimento tende a permanecer em movimento." Então, conclui-se que um corpo só altera seu estado de inércia, se alguém, ou alguma coisa aplicar nele uma força resultante diferente se zero. 2ª Lei de Newton - Princípio Fundamental da Dinâmica Quando aplicamos uma mesma força em dois corpos de massas diferentes observamos que elas não produzem aceleração igual. A 2ª lei de Newton diz que a Força é sempre diretamente proporcional ao produto da aceleração de um corpo pela sua massa, ou seja:

ou em módulo: F=ma Onde: F é a resultante de todas as forças que agem sobre o corpo (em N);

m é a massa do corpo a qual as forças atuam (em kg); a é a aceleração adquirida (em m/s²). A unidade de força, no sistema internacional, é o N (Newton), que equivale a kg m/s² (quilograma metro por segundo ao quadrado). 3ª Lei de Newton - Princípio da Ação e Reação Quando uma pessoa empurra um caixa com um força F, podemos dizer que esta é uma força de ação. mas conforme a 3ª lei de Newton, sempre que isso ocorre, há uma outra força com módulo e direção iguais, e sentido oposto a força de ação, esta é chamada força de reação. Esta é o princípio da ação e reação, cujo enunciado é: "As forças atuam sempre em pares, para toda força de ação, existe uma força de reação." EXERCÍCIOS TEXTO: 1 - Comum à questão: 1

Considere as Leis de Newton e as informações a seguir.

Uma pessoa empurra uma caixa sobre o piso de uma sala. As forças aplicadas sobre a caixa na direção do movimento são: - Fp: força paralela ao solo exercida pela pessoa; - Fa: força de atrito exercida pelo piso. A caixa se desloca na mesma direção e sentido de Fp . A força que a caixa exerce sobre a pessoa é Fc .

Questão 01 - (UERJ/2012)

Se o deslocamento da caixa ocorre com velocidade constante, as magnitudes das forças citadas apresentam a seguinte relação:

a) Fp = Fc = Fa b) Fp > Fc = Fa c) Fp = Fc > Fa d) Fp = Fc < Fa

Questão 02 - (UPE/2012)

Um corpo de massa m está suspenso por duas molas

ideais, paralelas, com constantes elásticas k e

Page 27: curso de específica de física

27

deformadas de d. Sabendo que o sistema se encontra em equilíbrio, assinale a alternativa que expressa k.

Dado: Considere a aceleração da gravidade g.

a) d

mg2

b) d

mg

c) d

mg

2

d) mg

d2

e) mg

d

Questão 03 - (UFJF MG)

Colocando-se sobre as bordas de uma mesa dois lápis L1 e L2 de modo que uma parte deles sobressaia da

mesa, tal que possam sustentar um palito de churrasco c1 como mostra a figura, golpeando-se rapidamente o

palito de churrasco, ele se quebra e os lápis não. Este fenômeno está relacionado com:

L1

L2

C1

a) O princípio da inércia.

b) A conservação do momento. c) A conservação da energia. d) A conservação do momento e energia. e) A lei de conservação dos choques.

Questão 04 - (UFOP MG)

Quando um ônibus é freado bruscamente, todos os passageiros são lançados” para a frente. Explique fisicamente este fenômeno.

Questão 05 - (UFSC)

Uma mola comprimida no interior de um tubo cilíndrico impulsiona uma bola, projetando–a horizontalmente para fora do tubo. Desprezando–se a resistência do ar, o esquema que representa corretamente a(s) força(s) atuante(s) sobre a bola fora do tubo é:

01.

02.

04.

08.

16.

Questão 06 - (UFU MG/2011)

Um objeto é lançado verticalmente na atmosfera terrestre. A velocidade do objeto, a aceleração gravitacional e a resistência do ar estão representadas

pelos vetores v, g

e atritoF

, respectivamente.

Considerando apenas estas três grandezas físicas no movimento vertical do objeto, assinale a alternativa correta.

a)

b)

c)

d)

Questão 07 - (UFTM/2007)

As confusões entre os conceitos de peso e massa são muitas, mas sabemos que em um corpo qualquer, essas grandezas são distintas. Analise as afirmações: I. A massa de um corpo é uma medida da inércia

desse corpo. II. Peso está relacionado à força com que a Terra atrai

um corpo. III. O peso é uma grandeza vetorial. Já a massa é uma

grandeza escalar.

É correto o contido em a) I, apenas. b) I e II, apenas. c) I e III, apenas. d) II e III, apenas. e) I, II e III.

Questão 08 - (MACK SP/2008)

No sistema a seguir, o fio e a polia são considerados ideais e o atrito entre as superfícies em contato é desprezível. Abandonando-se o corpo B a partir do repouso, no ponto M, verifica-se que, após 2 s, ele passa pelo ponto N com velocidade de 8 m/s. Sabendo-se que a massa do corpo A é de 5 kg, a massa do corpo B é

Page 28: curso de específica de física

28

a) 1 kg b) 2 kg c) 3 kg d) 4 kg e) 5 kg

Questão 09 - (UFLA MG)

Dois corpos têm massas m1 e m2. Aplicando-se as forças

F1

e F2

em m1 e m2, estas passam a ter acelerações

a1

e a2

, respectivamente. Como resultados possíveis

para as acelerações, teremos, EXCETO: a) Se

a2 a1 , entãom2 m1 e F 2 F 1

b) Se

a a , entãom m e F F 21212 1

c) Se

a2 a1 , então m2 m1 e F 2 F 1

d) Se

a a , entãom m e F F 21212 1

e) Se a a , então m m e F F 21212 1

Questão 10 - (UNIFOR)

Num anel atuam simultaneamente três forças

coplanares, 1F , 2F e 3F , representadas abaixo.

A resultante 321 FFFR tem módulo, em newtons,

a) 11 b) 9,0 c) 8,0 d) 7,0 e) 5,0

Questão 11 - (UNIOESTE PR)

O equilíbrio é uma situação física comum no nosso cotidiano. Os engenheiros, por exemplo, ao elaborarem muitos de seus projetos, estão constantemente atentos para atender adequadamente às condições necessárias e suficientes para que o equilíbrio ocorra. Assinale, entre

as alternativas a seguir, aquela que apresenta um corpo em equilíbrio. a) Um brinquedo em movimento circular uniforme,

preso a uma corda. b) Um satélite em órbita em torno da Terra. c) Um livro no ponto mais alto da trajetória, quando

lançado verticalmente para cima por um aluno. d) Uma bola que se movimenta em uma trajetória

parabólica, após ter sido chutada pelo goleiro em um jogo de futebol.

e) Um elevador em movimento vertical com velocidade constante.

Questão 12 - (UNIOESTE PR)

São fornecidas abaixo diversas afirmativas, envolvendo conceitos de mecânica. Assinale a(s) alternativa(s) correta(s).

01. Um objeto, inicialmente em repouso, recebe um impulso e adquire velocidade. Após um curto intervalo de tempo, o impulso deixa de atuar. Pode-se afirmar que, a partir deste momento, como consequência do impulso, o objeto passa a ter uma força, a qual, aos poucos, vai cessando, até que o objeto volte ao repouso. Considere que o objeto não sofre nenhum outro tipo de interação. 02. Um objeto não pode realizar uma trajetória curva

com velocidade escalar constante, caso a soma

vetorial de todas as forças que sobre ele atuam seja nula.

04. A velocidade de um projétil, sujeito apenas ao campo gravitacional da Terra, lançado obliquamente, aumenta até atingir um valor máximo, após o que o projétil se desloca verticalmente, de volta para o chão.

08. É possível exercer uma força sobre um objeto sem que haja realização de trabalho.

16. Um objeto que está em repouso não pode estar sendo submetido à ação da gravidade.

32. A atração gravitacional entre dois objetos leva um certo tempo para ocorrer, não sendo instantânea.

64. Para que um objeto esteja em movimento, não é necessário haver uma força aplicada sobre ele.

Questão 13 - (UNIOESTE PR)

No teto de um elevador, está pendurado um dinamômetro que tem, na sua outra extremidade, um pequeno corpo de peso 1,6 N. O dinamômetro, no entanto, acusa 2,0 N. O elevador está a) subindo com velocidade constante. b) em repouso. c) descendo com velocidade constante. d) subindo com velocidade crescente. e) descendo com velocidade crescente.

Questão 14 - (UNIOESTE PR)

O bloco A sobre uma mesa tem massa 15,0kg, e o bloco B suspenso tem massa 5,0kg. O coeficiente de atrito entre o bloco A e a mesa vale 0,20. Considere que o fio utilizado para unir os blocos é ideal (sem peso e inextensível) e que a aceleração gravitacional vale 10,0m/s2.

Analise as afirmações e assinale-as devidamente.

Page 29: curso de específica de física

29

00. A aceleração adquirida pelos corpos, durante seus movimentos, é de 1,0 m/s2.

01. No fio ideal, a tração é de 45,0N. 02. A velocidade do bloco A, após 3,0s de iniciado o

movimento, é de 30,0 m/s. 03. A distância percorrida pelo bloco A, após 3,0 s de

iniciado o movimento, é de 4,5 m. 04. Os blocos A e B têm acelerações diferentes.

Questão 15 - (UEL PR)

“Em média, as grandes usinas geradoras brasileiras precisam de 2180 litros de água para gerar 1MW. A usina hidrelétrica de Três Marias tem 396 MW de potência instalada. Para gerar 1MW ela precisa que 2325 litros passem por suas turbinas. Quando 920 mil litros passam pelas suas turbinas, ela gera 396 MW, que é sua potência instalada. Qualquer quantidade superior de água não gerará mais energia, pois o máximo que a hidrelétrica pode produzir é 396MW.”

Matéria publicada na Folha de São Paulo do dia 22 de abril de 2001, página B4.

A água que desce da represa para a usina é usada com o objetivo de acionar as turbinas dos geradores de eletricidade. Na usina de Três Marias, quando 920 mil litros de água chegam na entrada das turbinas, é correto afirmar:

a) A velocidade será muito maior que a velocidade de 2325 litros de água.

b) A velocidade será igual à velocidade de 2325 litros de água.

c) A velocidade será menor que a velocidade de 2325 litros de água.

d) Não se pode comparar 920 mil litros de água com 2325 litros de água, porque as suas acelerações serão diferentes.

e) É necessário calcular a aceleração de cada uma das quantidades de água envolvidas, para se proceder à comparação.

Questão 16 - (UEL PR)

Uma pessoa apoia-se em um bastão sobre uma balança, conforme a figura abaixo. A balança assinala 70kg. Se a pessoa pressiona a bengala, progressivamente, contra a balança, a nova leitura:

a) Indicará um valor maior que 70kg. b) Indicará um valor menor que 70kg. c) Indicará os mesmos 70kg. d) Dependerá da força exercida sobre o bastão. e) Dependerá do ponto em que o bastão é apoiado na

balança. Questão 17 - (UEL PR)

O cabo de um reboque arrebenta se nele for aplicada uma força que exceda 1800N. Suponha que o cabo seja usado para rebocar um carro de 900kg ao longo de uma rua plana e retilínea. Nesse caso, que aceleração máxima o cabo suporta? a) 0,5m/s2. b) 1,0m/s2. c) 2,0m/s2. d) 4,0m/s2. e) 9,0m/s2.

Questão 18 - (UEL PR)

Um jogador de tênis, ao acertar a bola com a raquete, devolve-a para o campo do adversário. Sobre isso, é correto afirmar: a) De acordo com a Segunda Lei de Newton, a força

que a bola exerce sobre a raquete é igual, em módulo, à força que a raquete exerce sobre a bola.

b) De acordo com a Primeira Lei de Newton, após o impacto com a raquete, a aceleração da bola é grande porque a sua massa é pequena.

c) A força que a raquete exerce sobre a bola é maior que a força que a bola exerce sobre a raquete, porque a massa da bola é menor que a massa da raquete.

d) A bola teve o seu movimento alterado pela raquete. A Primeira Lei de Newton explica esse comportamento.

e) Conforme a Segunda Lei de Newton, a raquete adquire, em módulo, a mesma aceleração que a bola.

Questão 19 - (UEM PR)

Uma massa m é presa a uma associação de duas molas ideais 1 e 2, de constantes elásticas, respectivamente, k1 e k2, como ilustradas na figura abaixo.

Considerando-se x1 e x2, respectivamente, os deslocamentos das posições de equilíbrio das molas 1 e 2, e kE a constante elástica resultante da associação das molas, podemos afirmar que: 01. k2 x2 = k1 x1. 02. k2 x2 = mg. 04. kE = k1 + k2. 08. kE x = mg, sendo x = x1 + x2. 16. – kE x = resultante das forças que atua na massa

m, sendo x = x1 + x2. Questão 20 - (UEM PR)

Através de uma polia ideal, passa uma corda C1, que sustenta duas massas, m1 e m2. Outra corda, C2, presa a m2, sustenta uma massa m3, conforme a figura. Considere as cordas idênticas e ideais. Considere também que a tração em C1 é T1, e em C2 é T2. Se m1

m2 m3, pode-se afirmar corretamente que:

Page 30: curso de específica de física

30

01. quando m3 + m2 = m1, a aceleração do sistema é

nula. 02. quando m3 + m2 = m1, T1 = T2. 04. quando m3 + m2 = 2m1, a aceleração do sistema é

g/2. 08. invertendo–se a posição das massas m2 e m3, a

aceleração do sistema não se altera. 16. quando m3 > m2, T2 > T1.

GABARITO: 1) Gab: A 2) Gab: C 3) Gab: A 4) Gab: inércia 5) Gab: 08 6) Gab: A 7) Gab: E 8) Gab: C 9) Gab: D 10) Gab: E 11) Gab: E 12) Gab: 02+08+64 13) Gab: D 14) Gab: VVFVF 15) Gab: B 16) Gab: C 17) Gab: C 18) Gab: D 19) Gab: 01-02-08 20) Gab: 01-08 21) Gab: E 22) Gab: 17 23) Gab: A 24) Gab: 02+04+08+16 25) Gab: 01-02-04-08 26) Gab: 4,5 27) Gab: C 28) Gab: A 29) Gab: C 30) Gab: B 31) Gab: B 32) Gab: 17 33) Gab: A 34) Gab: B 35) Gab: A 36) Gab: C 37) Gab: A 38) Gab: D 39) Gab: C 40) Gab: B

Page 31: curso de específica de física

31

LISTA 7 - APLICAÇÕES DAS LEIS DE NEWTON

RESUMO TEÓRICO A maior dificuldade de estudantes de física do ensino médio é com relação à solução de problemas, nas mais diversas frentes da disciplina. Apresentarei aqui a solução de um exercício sobre interação entre blocos, relacionado a aplicação das Leis de Newton. Exemplo 1) (UF-PE) A figura abaixo mostra três blocos de massas mA = 1,0 kg, mB = 2,0 kg e mc = 3,0 kg. Os blocos se movem em conjunto, sob a ação de uma força F constante e horizontal, de módulo 4,2 N.

Desprezando o atrito, qual o módulo da força resultante sobre o bloco B? a) 1,0 N b) 1,4 N c) 1,8 N d) 2,2 N e) 2,6 N Solução. Primeiramente devemos saber que F = m.a (Segunda lei de Newton). Devemos saber também, que os três corpos se movem com a mesma aceleração, e que essa aceleração tem a mesma direção e sentido da força F. Assim podemos calcular a aceleração dos blocos pelo Sistema de Corpos Isolados (SCI) Na figura abaixo representamos as forças que agem em A, B e C

Em que F é a força aplicada.

Fa no primeiro bloco é a reação de b em a devido a F. Fa no segundo bloco é a ação de a em b devido a F Fc no segundo bloco é a reação de c em b devido a F Fc no terceiro bloco é a ação de de b em c devido a F n é a força normal e p é a força peso nos três casos

Simplificando-se os pesos com as forças de reação normal em cada caso temos que:

Como F = 4,2 N, temos: 4,2 = (1,0 + 2,0 + 3,0).a a = 4,2/6 a = 0,7 m/s2 Encontrada a aceleração devemos encontrar o valor da resultante em B. A resultante em B é: FB = FA – FC Subtraído as equações B e C temos:

FA = (mB + mC) . a FA = (2,0 + 3,0). 0,7 FA = 5,0 . 0,7 FA = 3,5 N Encontrando o valor de FC FC = mC . a FC = 3,0 . 0,7 FC = 2,1 N Assim: FB = 3,5 – 2,1

FB = 1,4 N Concluímos então que a força resultante em B é igual a 1,4 N. EXERCÍCIOS Questão 01 - (UFT TO/2011)

Um piloto de teste pisa no acelerador de uma Ferrari, para aumentar sua velocidade escalar, em uma pista plana horizontal. Considere que o coeficiente de atrito estático entre os pneus da Ferrari e a pista vale 0,5. Nesta situação, qual valor melhor representa o módulo da aceleração máxima que esta Ferrari pode atingir nesta pista? Desconsidere o atrito com o ar, e considere o módulo da aceleração da gravidade igual a 9,8 m/s2.

a) 3,8 m/s2 b) 4,9 m/s2 c) 16,2 m/s2 d) 11,1 m/s2 e) 9,8 m/s2

Questão 02 - (ACAFE SC/2011)

Um garoto obeso tem um peso de módulo 1200 N (120kgf) e tenta mover uma caixa pesada conforme a

figura. O coeficiente de atrito estático entre os sapatos do garoto e o piso é 0,5.

Assinale a alternativa correta que apresenta o módulo da máxima força horizontal, em newtons, que o garoto pode aplicar na caixa.

a) 1200 b) 600 c) 1800 d) 300

Questão 03 - (FEI SP)

Um caminhão, partindo do repouso, carrega uma bobina de aço de massa m sobre a sua carroceria sem que haja escorregamento. Quanto à força de atrito de

Page 32: curso de específica de física

32

escorregamento na bobina, quando o caminhão estiver acelerando podemos afirmar que: a) não há força de atrito entre a bobina e a carroceria b) tem direção normal à carroceria e sentido igual ao

de deslocamento do caminhão c) tem direção paralela à carroceria e sentido igual ao

de deslocamento do caminhão d) tem direção paralela à carroceria e sentido contrário

ao de deslocamento do caminhão e) tem direção normal à carroceria e sentido contrário

ao de deslocamento do caminhão Questão 04 - (UDESC/2011)

A figura abaixo mostra uma caixa de madeira que desliza para baixo com velocidade constante sobre o plano inclinado, sob a ação das seguintes forças: peso, normal e de atrito. Assinale a alternativa que representa corretamente o esquema das forças exercidas sobre a caixa de madeira.

a)

b)

c)

d)

e)

Questão 05 - (UFOP MG)

Em um bloco em movimento sobre uma superfície horizontal com velocidade constante atuam as quatro

forças mostradas na figura abaixo. Das forças indicadas, assinar quais representam um par ação-reação.

Fa FV

N

P

a) (N, P) b) (F, Fa) c) (N, Fa) d) (N, P) e (F, Fa) e) nenhum par construído com as forças da figura

constitui um par ação-reação. Questão 06 - (Unifacs BA/2011)

Considerando-se um pequeno bloco, preso na extremidade de um fio ideal, que descreve um movimento circular uniforme sobre uma superfície horizontal, sem atrito, pode-se afirmar:

01. A força resultante que atua sobre o bloco é zero. 02. O período do movimento independe do

comprimento do fio. 03. A tração no fio produz variação na direção da

velocidade do bloco. 04. O bloco afastará radialmente, sob a ação exclusiva

da força centrífuga, se o fio sofrer rompimento. 05. O módulo da aceleração centrípeta do bloco é

diretamente proporcional à sua velocidade angular.

Questão 07 - (UERJ)

Um balão, de peso igual a 0,1 N, está preso a um fio. Além da força de empuxo E, o ar exerce uma forço horizontal F que empurro e inclina o fio em relação à vertical, conforme mostra afigura.

A tração no fio tem módulo igual o 0,2 N. Calcule, em newtons, os módulos de: a) E; b) F.

Questão 08 - (FATEC SP)

Um corpo em movimento, num plano horizontal, descreve uma trajetória curva. É correto afirmar que: a) o movimento é necessariamente circular uniforme. b) a força resultante é necessariamente centrípeta. c) a trajetória é necessariamente parabólica. d) a força centrípeta existe apenas quando a trajetória

é circular. Questão 09 - (UFF RJ)

A figura mostra, em vista lateral, o exato instante em que uma pipa paira no ar, em equilíbrio, sob a ação do vento que sopra com uma velocidade horizontal constante.

Page 33: curso de específica de física

33

A força que o vento faz sobre a pipa nesta situação está mais bem representada, na figura, pelo vetor:

1F a.

2F b.

3F c.

4F d.

5F e.

Questão 10 - (FURG RS)

Um carro faz uma curva de 80 m de raio, com velocidade de módulo constante igual a 72 km/h. Podemos afirmar que sua aceleração é: a) Zero m/s2 b) 0,5 m/s2 c) 0,9 m/s2 d) 4 m/s2 e) 5 m/s2

Questão 11 - (UEL PR/2008)

Com relação a um corpo em movimento circular uniforme e sem atrito, considere as afirmativas seguintes: I. O vetor velocidade linear é constante. II. A aceleração centrípeta é nula.

III. O módulo do vetor velocidade é constante. IV. A força atua sempre perpendicularmente ao

deslocamento.

Assinale a alternativa que contém todas as afirmativas corretas. a) I e IV. b) II e III. c) III e IV. d) I, II e III. e) I, II e IV.

Questão 12 - (UEM PR)

O goleiro de um time de futebol bate um tiro de meta e a bola percorre a trajetória esquematizada abaixo. Despreze a resistência do ar e assinale o que for correto (o ponto B corresponde ao instante em que a bola atinge o solo).

01. No ponto A, a resultante das forças que atua sobre a bola é para a direita e para cima.

02. No ponto B, a resultante das forças que atua sobre a bola é nula.

04. No ponto A, a velocidade resultante da bola é para a direita e para cima.

08. No ponto B, a velocidade resultante da bola é nula. 16. No ponto A, a energia total da bola é maior que no

ponto B. Questão 13 - (UEM PR)

Um motociclista descreve uma circunferência num “globo da morte" de raio 4 m, em movimento circular uniforme, no sentido indicado pela seta curva, na figura abaixo.

A massa total (motorista + moto) é de 150 kg. Considere g = 10 m/s e assinale o que for correto. 01. A velocidade do motociclista em B é tangente à

circunferência e dirigida para baixo ().

02. A aceleração do motociclista no ponto C é dirigida para o centro da circunferência.

04. A força resultante sobre o motociclista no ponto A é dirigida para fora da circunferência e perpendicular à mesma ().

08. Se a velocidade do motociclista no ponto mais alto da circunferência for 12 m/s, a força exercida sobre o globo nesse ponto será 3900 N.

16. No ponto mais baixo da circunferência, a força exercida sobre o globo é a mesma que a da parte mais alta.

32. A velocidade mínima que o motociclista deve ter no ponto mais alto da circunferência para que ele consiga fazer a volta completa sem cair é 6,3 m/s.

Questão 14 - (UEM PR/2008)

Sobre aviões de acrobacia que executam loopings (movimento circular cuja trajetória ocorre em um plano vertical em relação ao horizonte) no céu, é correto afirmar que, durante a execução do looping, a) a aceleração centrífuga é duas vezes maior que a

centrípeta. b) a energia potencial gravitacional se anula durante o

looping. c) existe somente energia rotacional na execução da

manobra. d) a aceleração centrípeta torna-se mais intensa que a

aceleração da gravidade. e) a aceleração centrífuga reduz-se a um quarto da

centrípeta. Questão 15 - (UNIOESTE PR)

Um carro de massa 1800 kg está em movimento circular sobre uma rodovia circular plana de raio igual a 200 m. O módulo de sua velocidade é constante. O coeficiente de atrito estático entre os pneus do carro e a rodovia é

de 20,0e . Considere a aceleração da gravidade g =

10 m/s2 e desconsidere o atrito com o ar. Para as condições de movimento descritas, assinale a alternativa correta. a) O módulo da força de atrito que atua sobre o carro,

resultante da interação dos pneus do carro e a rodovia, tem valor constante de 3600 N,

Page 34: curso de específica de física

34

independentemente do módulo da velocidade de movimento do carro.

b) A força de atrito sobre o carro, resultante da interação entre os pneus do carro e a rodovia, é a força centrípeta que permite o movimento do carro em círculo.

c) Os vetores força peso e força centrípeta sobre o carro possuem o mesmo sentido.

d) A força de atrito sobre o carro, resultante da interação entre os pneus do carro e a rodovia, atua em sentido contrário à força centrípeta sobre o carro, que o mantém em movimento sobre a rodovia.

e) A força resultante sobre o carro é nula. Questão 16 - (UNIOESTE PR/2008)

Um carrinho de brinquedo é solto a partir do repouso para percorrer uma pista sinuosa como mostra a figura abaixo. Depois de descer a rampa de altura h, o carrinho encontra uma lombada, cuja elevação acompanha a forma de um semicírculo de raio r. Supondo que não exista nenhum atrito agindo no brinquedo, qual o valor máximo da razão h/r, para que o carrinho permaneça em contato com a pista na parte superior da lombada?

a) 1/2. b) 3/2. c) 4/3. d) 5/3. e) 5/2.

Questão 17 - (UNIOESTE PR/2010)

Uma criança pendura-se na extremidade livre de uma corda que tem a outra extremidade presa ao teto de uma sala de ginástica. Ela, então, impulsiona-se e faz uma trajetória circular cujo diâmetro é 0,8 m. Se a velocidade tangencial da criança, cuja massa é 40 kg, for 2,0 m/s, qual será o ângulo que a corda faz com uma linha vertical perpendicular ao solo? Considere a criança como uma partícula, a massa da corda desprezível e a aceleração gravitacional igual 10 m/s2.

a) 30º. b) 35º. c) 45º. d) 20º. e) 60º.

Questão 18 - (UEL PR)

A força de atrito é sempre dissipativa e resiste ao movimento, mas há situações em que, embora esta força seja resistente ao movimento, ela possibilita que o movimento seja favorecido. Assinale a alternativa que

apresenta a situação física em que a força de atrito com a superfície, ou de resistência de um fluido, favorece o movimento. a) Força de resistência do ar que atua em um

automóvel em movimento. b) Força de resistência do ar que atua em um

paraquedas. c) Força de atrito entre duas placas de vidro bem

polido molhadas. d) Força de atrito que o chão aplica nos pneus de um

carro em movimento. e) Força de atrito entre o pistão e o cilindro no motor

do automóvel. Questão 19 - (UEL PR/2012)

Uma pessoa, de massa 80,0 kg, consegue aplicar uma força de tração máxima de 800,0 N. Um corpo de massa M necessita ser levantado como indicado na figura a seguir. O coeficiente de atrito estático entre a sola do sapato da pessoa e o chão de concreto é e = 1,0.

Faça um esboço de todas as forças que atuam em todo o sistema e determine qual a maior massa M que pode ser levantada pela pessoa sem que esta deslize, para um ângulo = 45º.

Questão 20 - (UEM PR/2006)

Uma caixa contendo ferramentas está em repouso sobre uma superfície horizontal áspera. Uma pessoa está tentando colocá-la em movimento, empurrandoa com uma força paralela à superfície, mas não está conseguindo. Qual a razão para isso? a) A força que a mão da pessoa faz sobre a caixa é a

mesma que a caixa faz sobre a mão. b) A força que o solo faz sobre a caixa devido ao atrito

cinético é muito maior do que a força que a pessoa faz sobre a caixa.

c) A força de atrito estático que o solo exerce sobre a caixa é de mesma intensidade da força que a mão faz sobre a caixa.

d) A quantidade de momento que a caixa possui. e) O fato de o torque que a mão imprime à caixa ser

menor do que a força de atrito estático que o solo exerce sobre a caixa.

GABARITO: 1) Gab: B 2) Gab: B 3) Gab: C 4) Gab: E 5) Gab: E 6) Gab: 03 7) Gab: a) 0,27N b) 0,1N 8) Gab: C 9) Gab: B 10) Gab: E

11) Gab: C 12) Gab: 04 13) Gab: 43 14) Gab: D 15) Gab: B 16) Gab: B 17) Gab: C 18) Gab: D 19) Gab:

Page 35: curso de específica de física

35

M = 40 2 Kg. 20) Gab: C

21) Gab: 11 22) Gab: 03 23) Gab: C 24) Gab: B 25) Gab: C 26) Gab: D 27) Gab: a)v = 4m/s b) aR = 0,8m/s2

c) NN = 552N e NJ = 756N 28) Gab: 04 29) Gab: E 30) Gab: 42 31) Gab: C 32) Gab: D 33) Gab: 29 34) Gab: 23 35) Gab: C 36) Gab: 09 37) Gab: C 38) Gab: C 39) Gab: D 40) Gab: 50

Page 36: curso de específica de física

36

LISTA 8 - TRABALHO E ENERGIA

RESUMO TEÓRICO Não existe uma definição do que é energia, mas sabemos que a sua existência possibilita a execução de trabalho. A energia armazenada nos alimentos, por exemplo, faz com que os órgãos do corpo de uma pessoa funcionem corretamente. Os combustíveis fazem com que os veículos automotores se locomovam. Da mesma forma, a energia elétrica produzida pela bateria faz com que os elétrons dos fios condutores de energia se locomovam. Ao falar de energia é de extrema importância ressaltar o Princípio de Conservação da Energia. Princípio este que, segundo Lavoisier, diz: “Na natureza nada se perde, nada se cria, tudo se transforma”. De forma a exemplificar conversões de energia de um modo geral, consideremos uma mola relaxada (figura 1), ou seja, uma mola que não está esticada. Veja:

Para comprimir a mola é necessário um gasto de energia. Assim, aplica-se uma força em uma de suas extremidades, de forma que a mesma se contraia. Dizemos que ao se aplicar a força sobre a mola há a realização de um trabalho. Este trabalho corresponde à energia transferida da pessoa para a mola. A figura 2 representa a mola já comprimida e com uma trava no carrinho, impedindo que o mesmo se liberte. A mola comprimida armazena energia. Essa energia, porém, só pode ser manifestada ao se retirar a trava do

carrinho. A energia armazenada na mola é denominada de Energia Potencial Elástica. Potencial porque pode se manifestar e elástica porque está em um corpo elástico deformado. Agora, observando a figura 3, percebemos que o carrinho se libertou. Ao ser retirada a trava, a energia potencial que estava armazenada na mola se manifestou, fazendo com que o carrinho adquirisse movimento. Novamente temos a realização de trabalho. Agora esse trabalho corresponde à energia transferida da mola para o carrinho. A energia que o carrinho adquiriu é denominada de Energia Cinética. Energia Cinética: é a energia que está relacionada ao movimento dos corpos. Energia Potencial (gravitacional, elástica, elétrica, etc.): é a energia que um corpo possui em relação à

posição particular que ele ocupa. Na ausência de atrito, a energia mecânica total de um sistema se conserva, havendo apenas a transformação de energia potencial em energia cinética e vice-versa. Veja: Emec= Ec + Ep É de grande importância deixar bem claro que o trabalho e as formas de energia são grandezas escalares. Trabalho de uma força Trabalho é a medida da energia que é transferida para um corpo, em razão da aplicação de uma força ao longo de um deslocamento. Em Física, trabalho é normalmente representado por W(que vem do inglês work) ou mais

usadamente a letra grega tau .

Para calcular o trabalho de uma força é importante ressaltar que ele pode ser: Trabalho de uma força constante e paralela ao deslocamento: é calculado quando se tem a força sendo aplicada no mesmo sentido do deslocamento. Pode ser calculado da seguinte forma:

Como o ângulo entre a força e o deslocamento é zero faz com que o cosseno deste ângulo seja igual a 1, tornando a expressão equivalente à:

Onde D é o deslocamento sofrido pelo corpo.

Trabalho de uma força constante e não paralela ao deslocamento:

Quando temos a aplicação da força constante e não paralela, como no esquema acima, calculamos o trabalho da seguinte forma:

Onde ? é o ângulo formado entre a força e o deslocamento sofrido pelo corpo. No SI (Sistema Internacional de Unidades) o trabalho é dado em joule, que é representado pela letra (J) e a força é dada em newton (N). Essa unidade é uma homenagem ao físico britânico James Prescott Joule. No sistema CGS, a unidade de trabalho é o erg= dina x centímetro. Fonte: http://www.brasilescola.com/fisica/trabalho.htm EXERCÍCIOS Questão 01 - (UEL PR)

Page 37: curso de específica de física

37

Um motociclista resolve ir para a praia e pretende levar a sua motocicleta em uma caminhonete. Para colocar a motociclista na caminhonete ele pode ergue-la verticalmente ou empurra-la por uma rampa. Considerando desprezíveis as perdas por atrito, assinale a alternativa correta: a) O trabalho realizado para elevar a motocicleta

verticalmente é maior. b) O trabalho realizado pelo motociclista, em ambas as

situações, é o mesmo. c) A potência aplicada pelo motociclista, em ambas as

situações, é a mesma. d) O trabalho realizado para elevar a motocicleta ao

longo da rampa é menor. e) A força aplicada para elevar a motocicleta ao longo

da rampa é maior. Questão 02 - (UFV MG)

Um corpo de massa m se move com velocidade constante v sobre uma superfície plana horizontal e sem atrito. Após um certo instante de tempo, uma força constante de módulo F, com sentido contrário ao movimento, age sobre o corpo durante um intervalo de tempo t, fazendo-o parar.

Das opções abaixo, aquela que corresponde ao valor do trabalho realizado pela força F, durante o intervalo de tempo t, é:

a) vt

b) Fv

c) 2

2

1mv

d) Ft

e) t

Fv

Questão 03 - (UFF RJ)

Um homem de massa 70 kg sobe uma escada, do ponto A ao ponto B, e depois desce, do ponto B ao ponto C, conforme indica a figura.

Dado: g = 10 m/s2.

C

50cm30cm

A

B

O trabalho realizado pelo peso do homem desde o ponto A até no ponto C foi de:

a) 5,6 x 102 J

b) 1,4 x 103 J

c) 3,5 x 103 J

d) 1,4 x 102 J e) zero

Questão 04 - (UNIRIO RJ)

Três corpos idênticos de massa M deslocam-se entre

dois níveis como mostra a figura: A - caindo livremente; B - deslizando a longo de um tobogã e C - descendo uma rampa, sendo, em todos os movimentos, desprezíveis as

forças dissipativas. Com relação ao trabalho (w) realizado pela força-peso dos corpos, pode-se afirmar que:

C A B

a) WC > WB > WA

b) WC > WB = WA

c) WC = WB > WA

d) WC = WB = WA

e) WC < WB > WA

Questão 05 - (FURG RS)

Um bloco de 5 kg move-se em linha reta sobre uma superfície horizontal, sem atrito, sob a influência de uma força que varia conforme a posição, como mostrado na figura. Se a velocidade da partícula, ao passar pela origem, era 4,0 m/s, com que velocidade ela passará pelo ponto x = 8,0 m?

a) s/m28

b) s/m27

c) s/m26

d) s/m25

e) s/m24

Questão 06 - (FATEC SP/2011)

Uma bola de basquete é solta de uma altura de 1,0 metro e, a cada colisão com o chão, ela dissipa 10% de sua energia mecânica. Após 3 toques no chão, a bola atingirá uma altura de, aproximadamente,

a) 54 cm. b) 63 cm. c) 69 cm. d) 73 cm. e) 81 cm.

TEXTO: 1 - Comum à questão: 7

Considere o enunciado abaixo.

A figura que segue representa uma mola, de massa desprezível, comprimida entre dois blocos, de massas M1 = 1kg e M2 = 2kg, que podem deslizar sem atrito sobre uma superfície horizontal. O sistema é mantido inicialmente em repouso.

Page 38: curso de específica de física

38

Num determinado instante, a mola é liberada e se expande, impulsionando os blocos. Depois de terem perdido contato com a mola, as massas M1 e M2 passam a deslizar com velocidades de módulos v1 = 4 m/s e v2

= 2 m/s, respectivamente. Questão 07 - (UFRGS/2007)

Qual é o valor da energia potencial elástica da mola, em J, antes de ela ser liberada? a) 0 b) 4 c) 8 d) 12 e) 24

Questão 08 - (UERJ)

Um chaveiro, largado de uma varanda de altura h, atinge a calçada com velocidade . Para que o velocidade de

impacto dobrasse de valor, seria necessário largar esse chaveiro de uma altura maior, igual a: a) 2 h b) 3 h c) 4 h d) 6 h

Questão 09 - (UERJ)

Duas goiabas de mesma massa, G1 e G2, desprendem-se, num mesmo instante, de galhos diferentes. A goiaba G1 cai de uma altura que corresponde ao dobro daquela

de que cai G2. Ao atingirem o solo, a razão1

2

Ec

Ec, entre

as energias cinéticas de G2 e G1, terá o seguinte valor: a) 1/4 b) 1/2 c) 2 d) 4

Questão 10 - (FURG RS)

Associe as grandezas da coluna 1 com as características apontadas na coluna 2. Coluna 1 (1) Energia (2) Força Coluna 2 ( ) grandeza escalar ( ) medida em Joules ( ) possui módulo, direção e sentido ( ) medida com dinamômetro A alternativa que contém a associação correta da coluna

2, quando lida de cima para baixo, é: a) 1 - 1 - 2 - 2 b) 1 - 2 - 1 - 2 c) 1 - 2 - 2 - 1 d) 2 - 1 - 1 - 2 e) 2 - 2 - 1 - 1

Questão 11 - (UESC BA/2011)

Muitas vezes, uma pessoa se surpreende com o aumento de consumo de combustível apresentado por um veículo que faz uma viagem em alta velocidade.

Considere uma situação em que a intensidade da força total de resistência ao movimento, Fr, seja proporcional ao quadrado da intensidade da velocidade v do veículo. Se o veículo descrever movimento retilíneo e uniforme e duplicar o módulo da sua velocidade, então a potência desenvolvida pelo motor será multiplicada por

01. 4 02. 6 03. 8 04. 10 05. 12

Questão 12 - (FUVEST SP)

Nos manuais de automóveis, a caracterização dos motores é feita em CV (cavalo-vapor). Essa unidade, proposta no tempo das primeiras máquinas a vapor, correspondia à capacidade de um cavalo típico, que conseguia erguer, na vertical, com auxílio de uma roldana, um bloco de 75 kg, à velocidade de 1 m/s. Para subir uma ladeira, inclinada como na figura, um carro de 1000 kg, mantendo uma velocidade constante de 15 m/s (54 km/h), desenvolve uma potência útil que, em CV, é, aproximadamente, de:

a) 20 CV b) 40 CV c) 50 CV d) 100 CV e) 150 CV

Questão 13 - (FATEC SP)

No Sistema Internacional, a unidade de potência é watt (W). Usando apenas unidades das grandezas fundamentais, o watt equivale a: a) kg · m/s b) kg ·m2/s c) kg · m/s2 d) kg ·m2/s2 e) kg ·m2/s3

Questão 14 - (UFPE/2008)

Raios solares incidem verticalmente sobre um canavial

com 600 hectares de área plantada. Considerando que a energia solar incide a uma taxa de 1340 W/m2, podemos estimar a ordem de grandeza da energia solar que atinge a área do canavial, em uma hora. Esta energia, expressa em quilowatt.hora, tem ordem de grandeza 10n. Determine o valor de n. Dado: 1 hectare = 104 m2.

Questão 15 - (UEL PR)

Uma força constante age sobre um objeto de 5,0kg e eleva a sua velocidade de 3,0 m/s para 7,0 m/s em um intervalo de tempo de 4,0s. Qual a potência devido á força? a) 29,8W b) 11,1W c) 25,0W d) 36,1W

Page 39: curso de específica de física

39

e) 40,0W Questão 16 - (UEM PR)

Para levantar um corpo de 4 kg, inicialmente em repouso, a uma altura de 4 m, uma máquina aplicou uma força constante, que realizou um trabalho de 224 joules. A partir daí, considerando g = 10 m/s2 e desprezando a resistência do ar, assinale o que for correto. 01. Na posição inicial e na final, a energia

cinética do corpo é máxima. 02. Na posição inicial, na intermediária e na

final, a energia potencial será mínima e igual à energia cinética.

04. A energia do sistema será conservada, se, e somente se, considerarmos a existência do atrito entre o corpo em repouso e o seu movimento retilíneo e uniforme.

08. Ao atingir a altura de 4 m, a quantidade de

movimento do corpo será de 16 2 kg m/s.

16. A equação dimensional da energia cinética do corpo é dada por [L]2 [M][T]-2 .

32. A intensidade da força aplicada pela máquina ao corpo será de 56 N.

Questão 17 - (UEM PR)

Os diagramas abaixo mostram pequenas bolas que se movem sobre duas superfícies circulares.

No diagrama 1, na posição A, a bola tem energia cinética suficiente para atingir a posição C, sem perder o contato com a superfície. No diagrama 2, a bola é solta da posição A. Despreze a força de atrito e assinale o que for correto. 01. Na posição B, no diagrama 1, a força normal à bola

é menor, em módulo, que seu próprio peso. 02. Na posição B, no diagrama 2, a força normal à bola

é maior, em módulo, que seu próprio peso.

04. O trabalho total realizado sobre a bola, para ir de A até C, no diagrama 1, é maior que o trabalho total, para ir de A até C, no diagrama 2.

08. No diagrama 1, a bola se move com movimento circular uniforme.

16. No diagrama 2, a bola se move com movimento harmônico simples.

Questão 18 - (UEM PR)

Uma esfera é solta da posição A, a partir do repouso, sobre uma superfície circular correspondente a 1/3 de uma circunferência, conforme ilustra a figura. Despreze todas as forças dissipativas.

Durante o período em que a esfera permanece em contato com a superfície, é correto afirmar que: 01. a única força que realiza trabalho sobre a esfera é o

seu próprio peso. 02. o trabalho total realizado sobre a esfera, para atingir

o ponto C, é maior que o trabalho total realizado para atingir o ponto B.

04. a variação da energia cinética da esfera é sempre igual à variação de sua energia potencial.

08. a força normal à esfera aumenta à medida que sua velocidade aumenta.

16. a força peso da esfera aumenta à medida que sua velocidade aumenta.

Questão 19 - (UEL PR/2013)

Suponha que o conjunto formado pelo satélite e pelo foguete lançador possua massa de 1,0103 toneladas e

seja impulsionado por uma força propulsora de aproximadamente 5,0107 N, sendo o sentido de

lançamento desse foguete perpendicular ao solo. Desconsiderando a resistência do ar e a perda de massa devido à queima de combustível, assinale a alternativa que apresenta, corretamente, o trabalho realizado, em joules, pela força resultante aplicada ao conjunto nos primeiros 2, 0 km de sua decolagem.

Considere a aceleração da gravidade g = 10, 0 m/s2 em todo o percurso descrito.

a) 4,0107 J

b) 8,0107 J c) 4,01010 J

d) 8,01010 J

e) 10,01010 J

Questão 20 - (UNIOESTE PR/2010)

Usa-se uma corda para baixar verticalmente, de uma altura de 3,0 m, um bloco de massa 10 kg com uma aceleração constante de 3,0 m/s2. Considerando a aceleração da gravidade igual a 10 m/s2, o trabalho realizado pela corda sobre o bloco é

a) – 90 J. b) – 180 J. c) – 210 J. d) – 270 J.

e) – 300 J.

GABARITO: 1) Gab: B 2) Gab: C 3) Gab: D 4) Gab: D 5) Gab: A 6) Gab: D 7) Gab: D 8) Gab: C 9) Gab: B

Page 40: curso de específica de física

40

10) Gab: A 11) Gab: 03 12) Gab: A 13) Gab: E 14) Gab: 07 15) Gab: C 16) Gab: 08-16-32 17) Gab: 01-02-16 18) Gab: 01-08 19) Gab: D 20) Gab: C 21) Gab: D 22) Gab: B 23) Gab: D 24) Gab: 01-08-32 25) Gab: 07 26) Gab: 01-04-08 27) Gab: 09 28) Gab: A 29) Gab: A 30) Gab: C 31) Gab: B 32) Gab: C 33) Gab: A 34) Gab: 15 35) Gab: B 36) Gab: D 37) Gab:

Utilizando-se a massa transportada em um segundo, temos:

38) Gab: E 39) Gab: E 40) Gab: C 41) Gab: E 42) Gab: C 43) Gab: C 44) Gab: A

45) Gab: B

46) Gab: E 47) Gab: Ee = 0,50 J e a = 0,10 m/s2 48) Gab: 17 49) Gab:

a) s/m108,15s5,9103

m105,4

t

SV 3

7

12

2mv2

1

b) s/m106,1106

107682

m

2v 4

5

11

50) Gab: E

Page 41: curso de específica de física

41

LISTA 9 - DINÂMICA IMPULSIVA E COLISÕES RESUMO TEÓRICO Impulso é a grandeza física que relaciona a força que atua sobre um corpo e o intervalo de tempo que ela atua sobre o mesmo. Imagine a situação ilustrada abaixo, onde se tem a atuação de uma força constante durante um determinado intervalo de tempo, Δt = tf – ti, sobre um bloco de massa m.

Força sobre um bloco de massa m

O produto dessa força constante pelo intervalo de tempo de aplicação da mesma é chamado de Impulso, e é representado pela letra I. O impulso é uma grandeza vetorial, possui módulo, direção e sentido. Em módulo, a equação que determina o impulso pode ser escrita da seguinte forma:

I = F. Δt No Sistema Internacional de Unidades (SI), a unidade do impulso é o newton vezes segundo N.s. Quantidade de Movimento Imagine um corpo de massa m, que num determinado instante t possua velocidade V, por definição a quantidade de movimento é o produto entre essas duas grandezas, massa e velocidade. Como a velocidade é uma grandeza vetorial, por consequência a quantidade de movimento também é, e em módulo ela pode ser vista da seguinte forma:

Q = m. V A unidade de quantidade de movimento no Sistema Internacional de Unidades é o kg. m/s. Teorema Impulso – Quantidade de Movimento O teorema do impulso – quantidade de movimento diz que o impulso da resultante das forças que atuam sobre um corpo, num determinado intervalo de tempo, é igual à variação da quantidade de movimento do corpo no mesmo intervalo de tempo, matematicamente fica:

I = Qf - Qi Onde Qf é a quantidade de movimento final e Qi é a quantidade de movimento inicial. EXERCÍCIOS TEXTO: 1 - Comum à questão: 1

Considere o enunciado abaixo.

A figura que segue representa uma mola, de massa desprezível, comprimida entre dois blocos, de massas M1 = 1kg e M2 = 2kg, que podem deslizar sem atrito sobre uma superfície horizontal.

O sistema é mantido inicialmente em repouso.

Num determinado instante, a mola é liberada e se expande, impulsionando os blocos. Depois de terem perdido contato com a mola, as massas M1 e M2 passam a deslizar com velocidades de módulos v1 = 4 m/s e v2 = 2 m/s, respectivamente.

Questão 01 - (UFRGS/2007)

Quanto vale, em kg.m/s, o módulo da quantidade de movimento total dos dois blocos, depois de perderem contato com a mola? a) 0 b) 4 c) 8 d) 12 e) 24

Questão 02 - (UERJ)

Um peixe de 4 kg, nadando com velocidade de 1,0 m/s, no sentido indicado pela figura, engole um peixe de 1 kg, que estava em repouso, e continua nadando no mesmo sentido.

A velocidade, em m/s, do peixe imediatamente após a ingestão, é igual a: a) 1,0 b) 0,8 c) 0,6

d) 0,4 Questão 03 - (UFAC/2007)

Uma patinadora de 50 Kg, e um patinador de 75 Kg, estão em repouso sobre a pista de patinação, na qual o atrito é desprezível. O patinador empurra a patinadora e desloca-se para trás com velocidade de 0.3 m/s em relação ao gelo. Após 5 segundos, qual será a separação entre eles, supondo que suas velocidades permaneçam praticamente constantes? a) 3.0 m b) 4.0 m c) 1.5 m d) 4.5 m e) 3.75 m

Questão 04 - (UFRRJ /2007)

Eduardo, de massa igual a 30 kg, está parado, sentado em seu carrinho de 10 kg, quando seu cachorro Zidane, de 20 kg de massa, vem correndo e pula em seu colo. Sabendo que o carrinho com Eduardo e Zidane passa a ter uma velocidade de 0,5 m/s, determine a velocidade do cachorro antes de ser apanhado pelo dono, considerando-a na direção horizontal.

Questão 05 - (FGV/2009)

Num sistema isolado de forças externas, em repouso, a resultante das forças internas e a quantidade de movimento total, são, ao longo do tempo, respectivamente,

a) crescente e decrescente.

Page 42: curso de específica de física

42

b) decrescente e crescente. c) decrescente e nula. d) nula e constante. e) nula e crescente.

Questão 06 - (UNIFICADO RJ)

Em uma partida de futebol, a bola é lançada em linha reta na grande área e desviada por um jogador da defesa. Nesse desvio, a bola passa a se mover perpendicularmente à trajetória na qual foi lançada. Sabe-se que as quantidades de movimento imediatamente antes e imediatamente depois do desvio têm o mesmo módulo p. O impulso exercido sobre a bola durante o desvio referido no enunciado será igual a: a) zero b) p

c) 2p

d) 3p

e) 2p Questão 07 - (UNESP)

A intensidade (módulo) da resultante das forças que atuam num corpo, inicialmente em repouso, varia como mostra o gráfico.

F(N)

2

1

0

0 2 4 6 8 t(s)

Durante todo o intervalo de tempo considerado, o sentido e a direção dessa resultante permanecem inalterados. Nestas condições, a quantidade de movimento, em kg.m/s (ou Ns) adquirida pelo corpo é a) 8 b) 15 c) 16 d) 20 e) 24

Questão 08 - (UNAERP SP)

A figura mostra uma bola de bilhar de massa 200 g chocando-se contra a proteção lateral da mesa com velocidade 1 m/s, segundo um ângulo de 30° com a normal. Na sequência, retorna com o mesmo ângulo, e com o mesmo módulo da velocidade inicial. O módulo do impulso exercido pela mesa sobre a bola é, em N.s:

a) 3200,0

b) 3140,0

c) nulo

d) 3100,0

e) 3175,0

Questão 09 - (UFG GO/2010)

Um jogador de hockey no gelo consegue imprimir uma velocidade de 162 km/h ao puck (disco), cuja massa é de 170 g. Considerando-se que o tempo de contato entre o puck e o stick (o taco) é da ordem de um centésimo de segundo, a força impulsiva média, em newton, é de:

a) 7,65 b) 7,65102 c) 2,75103

d) 7,65103

e) 2,75104

Questão 10 - (UFTM/2013)

Em uma colisão frontal entre duas esferas, A e B, a

velocidade de A varia com o tempo, como mostra o gráfico.

Sabendo que a massa da esfera A é de 100 g, o módulo da força média que ela exerce sobre a esfera B durante essa colisão, em newtons, é igual a

a) 2,5. b) 1,5. c) 3,5. d) 0,5. e) 4,5.

Questão 11 - (UEM PR)

Um corpo de massa m, inicialmente em repouso, cai verticalmente de uma altura h = 20 m. Ao atingir a altura de 10 m, o corpo explode e se fragmenta em dois pedaços de massas m1 = m/3 e m2 = 2m/3. No instante da explosão, o pedaço de massa m1 é lançado horizontalmente para a direita com velocidade igual a 2 m/s. Considere que a aceleração da gravidade local vale

10 m/s2, despreze as ações de quaisquer forças de atrito e assinale o que for correto. 01. Se não tivesse explodido, o corpo de massa m

atingiria o solo com velocidade igual a 20 m/s. 02. Decorridos 2 s do início da queda do corpo de massa

m, o centro de massa do sistema constituído pelos corpos de massas m1 e m2 atingirá o solo.

04. O centro de massa do sistema constituído pelos corpos de massas m1 e m2 atingirá o solo à esquerda da posição que o corpo de massa m atingiria se não tivesse explodido.

08. No instante da explosão, o corpo de massa m2 é lançado horizontalmente para a esquerda com velocidade igual a 4 m/s.

Page 43: curso de específica de física

43

16. No instante da explosão, a velocidade vertical do corpo de massa m era 10 m/s e já havia decorrido

s 2 do início da sua queda.

32. O corpo de massa m2 atingirá o solo antes do corpo de massa m1.

64. A distância entre os pontos em que os corpos de

massas m1 e m2 atingirão o solo vale m236 .

Questão 12 - (UEM PR/2010)

Recentemente, no treino classificatório para o grande prêmio da Hungria de fórmula I, uma mola soltou-se do carro de Rubens Barrichello e colidiu violentamente com o capacete de outro piloto brasileiro, que vinha logo atrás, Felipe Massa. Considere que a massa da mola é muito menor que as massas somadas do carro, piloto e capacete, e que o capacete ficou parcialmente destruído. Considerando o exposto, assinale a(s) alternativa(s) correta(s).

01. Depois da colisão, os módulos do impulso dado à

mola e ao capacete são iguais. 02. As quantidades de movimento da mola, antes e

depois da colisão, são iguais. 04. Houve conservação de energia cinética do sistema

mola e capacete. 08. Depois da colisão, os módulos da aceleração da

mola e do capacete são iguais. 16. Houve conservação do momento linear total do

sistema. Questão 13 - (UEM PR/2012)

Durante o treino classificatório para o Grande Prêmio da Hungria de Fórmula 1, em 2009, o piloto brasileiro Felipe Massa foi atingido na cabeça por uma mola que se soltou do carro que estava logo à sua frente. A colisão com a mola causou fratura craniana, uma vez que a mola ficou ali alojada, e um corte de 8 cm no supercílio esquerdo do piloto. O piloto brasileiro ficou inconsciente e seu carro colidiu com a proteção de pneus. A mola que atingiu o piloto era de aço, media 12 cm de diâmetro e tinha, aproximadamente, 800 g. Considerando que a velocidade do carro de Felipe era de 270 km/h, no instante em que ele foi atingido pela mola, e desprezando a velocidade da mola e a resistência do ar, assinale o que for correto.

01. A quantidade de movimento (momento linear)

transferida do piloto para a mola foi de, aproximadamente, 75 kg.m.s–1.

02. Pode-se dizer que esse tipo de colisão é uma colisão perfeitamente inelástica.

04. Tomando-se o referencial do piloto Felipe Massa, pode-se dizer que a velocidade da mola era de -270 km/h.

08. Considerando que o intervalo de tempo do impacto (a duração do impacto) foi de 0,5 s, a aceleração média da mola foi de 150 m/s2.

16. Considerando que, após o final da colisão, a velocidade da mola em relação ao piloto é nula, e tomando o referencial do piloto Felipe Massa, pode-se afirmar que a função horária da posição da mola, após o final da colisão, foi de segundo grau.

Questão 14 - (UNIFICADO RJ)

Um revólver de brinquedo dispara bolas de plástico de encontro a um bloco de madeira colocado sobre uma

mesa. São feitos dois disparos, vistos de cima, conforme as figuras (1) e (2):

b l o c o

m e s a.

( 1 )b l o c o

m e s a

.( 2 )

Observa-se que na situação (1) o bloco permanece como estava, enquanto que na (2) ele tomba. Considere as três alternativas dadas a seguir: “A razão pela qual o bloco tomba na situação (2) e não tomba na situação (1) está ligada à (ao): I. massa da bola. II. variação da velocidade da bola. III. módulo da velocidade da bola. É (são) correta(s): a) apenas a I. b) apenas a II. c) apenas a III. d) apenas a I e II. e) a I, a II e a III.

Questão 15 - (UFC CE)

A figura abaixo mostra uma calha circular, de raio R, completamente lisa, em posição horizontal. Dentro dela há duas bolas, 1 e 2, idênticas e em repouso no ponto A. Ambas as bolas são disparadas, simultaneamente, desse ponto: a bola 1, para a direita, com velocidade v1 = 6 m/s e a bola 2, para a esquerda, com velocidade v2 = 2 m/s. As colisões entre as bolas são perfeitamente

elásticas. Indique onde ocorrerá a quarta colisão entre as bolas, após o disparo delas.

Av2

v1

D B

C a) Entre os pontos A e B b) Exatamente no ponto A c) Entre os pontos C e D d) Exatamente no ponto C e) Exatamente no ponto D

Questão 16 - (ACAFE SC)

Um rapaz de patins está parado no centro de uma pista, onde o atrito é desprezível, quando uma jovem de massa 50kg vem de encontro a ele, com velocidade de módulo 6,0m/s. O rapaz abraça-a e, após a interação, ambos estão se movimentando juntos, na mesma direção da velocidade inicial da moça. Se a massa do rapaz é de 70kg, qual é o módulo da velocidade resultante final do sistema, em m/s?

Page 44: curso de específica de física

44

jovemq

a) 5,0 b) 3,0 c) 6,0 d) 3,5 e) 2,5

Questão 17 - (UEPG PR)

Um bloco de massa 500 g e energia cinética igual a 100 J colide inelasticamente com um outro bloco, de massa 2 kg, inicialmente em repouso. Após a colisão, os blocos seguem juntos, na mesma direção e sentido do primeiro bloco. Calcule, em metros por segundo, a velocidade dos blocos após a colisão.

Questão 18 - (UFBA/2011)

Uma esfera rígida de massa m1 = 0,5kg, presa por um fio de comprimento L = 45,0cm e massa desprezível, é suspensa em uma posição tal que, como mostra a figura, o fio suporte faz um ângulo de 90º com a direção vertical. Em um dado momento, a esfera é solta, indo se chocar com outra esfera de massa m2 = 0,5kg, posicionada em repouso no solo.

Considerando o diâmetro das esferas desprezível e o choque entre elas perfeitamente elástico, determine a velocidade das esferas após o choque, supondo todas as forças dissipativas desprezíveis, o módulo da aceleração da gravidade local igual a 10m/s2 e o

coeficiente de restituição 21

'1

'2

vv

vv

, em que '

1v e '2v

são as velocidades finais das esferas e v1 e v2 as velocidades iniciais.

Questão 19 - (FATEC SP)

Num certo instante, um corpo em movimento tem energia cinética de 100 joules, enquanto o módulo de sua quantidade de movimento é 40kg m/s. A massa do corpo, em kg, é: a) 5,0 b) 8,0 c) 10 d) 16 e) 20

Questão 20 - (UNICAMP SP)

Dois patinadores inicialmente em repouso, um de 36kg e outro de 48kg, se empurram mutuamente par trás. O patinador de 48kg sai com velocidade de 18km/h. despreze o atrito.

a) Qual a velocidade com que sai o patinador de 36kg? b) Qual o trabalho total realizado por esses dois

patinadores? GABARITO: 1) Gab: A 2) Gab: B 3) Gab: E 4) Gab:

1,5m/s 5) Gab: D 6) Gab: C 7) Gab: A 8) Gab: A 9) Gab: B 10) Gab: A 11) Gab: 67 12) Gab: 17 13) Gab: 14 14) Gab: B 15) Gab: B 16) Gab: E 17) Gab: 04 18) Gab: 3,0m/s 19) Gab: B 20) Gab:

a) |VA| = 24km/h

b) 1400J 21) Gab: 2 N 22) Gab: C 23) Gab: 00. E 01. C 02. E 03. C 04. E 24) Gab: 01-02-04 25) Gab: 02-08-16 26) Gab: 01-F; 02-V; 04-V; 08-V; 16-V. 27) Gab:

a) gH2

b) VA= 0,3 gH2 ; VB= 0,7 gH2

c) 0,49H 28) Gab:

a) O choque com o piso faz com que surja uma força muito grande, pois o tempo de interação é muito baixo. O choque com o tapete faz este tempo aumentar, diminuindo, assim, a força sobre a xícara e a energia cinética se perde mais lentamente.

b) para t = 0,5s F = 1,4N

para t = 0,010s F = 21N

29) Gab: B 30) Gab: a) r = 2 10–4 m

b) |I| = 50 kg m/s 31) Gab: E 32) Gab: 07 33) Gab: C 34) Gab: B

35) Gab: C 36) Gab: Na colisão: quantidade de movimento se conserva e a energia mecânica diminui. h = 0,8 m 37) Gab: C 38) Gab: C 39) Gab: 23 40) Gab: C 41) Gab: C 42) Gab: B 43) Gab: a) E = 375 J Q = 150 kg m/s b) N = 270 N c) |a| = 5 m/s2 44) Gab: B 45) Gab: C

Page 45: curso de específica de física

45

LISTA 10 - ESTÁTICA E HIDROSTÁTICA

RESUMO DE CONTEÚDO

Estática é a parte da Mecânica que estuda as condições de equilíbrio de objetos. Há essencialmente dois tipos de equilíbrio: o equilíbrio estático, quando o objeto está em equilíbrio, parado (velocidade nula); e o equilíbrio dinâmico, quando o objeto em equilíbrio está em movimento. Dentro dessas duas possibilidades existem três condições de equilíbrio: O equilíbrio estável onde as forças do sistema em questão tendem a retornar o corpo ao estado de equilíbrio mesmo havendo uma perturbação nele. No equilíbrio estável, o corpo sempre retorna às condições de equilíbrio originais. O equilíbrio instável onde qualquer perturbação no sistema acarreta o fim do estado de equilíbrio por fazer com que alguma das forças do sistema, anteriormente equilibrada por outra de igual intensidade, passe a se sobrepor às outras,

crescendo demasiadamente e deixando assim de ser equiparada por outra, consequentemente, desequilibrando todo o sistema. No equilíbrio instável, o corpo pode sair das condições de equilíbrio sob qualquer perturbação. E, o equilíbrio indiferente onde o corpo tende a permanecer em sua condição atual de inércia, seja qual for ela. Se ele estiver parado, assim tenderá a ficar, se for colocado em movimento, permanecerá da mesma forma. No equilíbrio indiferente, qualquer estado de inércia assumido pelo corpo é mantido indiferentemente.

A Hidrostática é a parte da Física que estuda os fluídos (tanto líquidos como os gasosos) em repouso, ou seja, que não estejam em escoamento (movimento). 1. Massa Específica; Densidade Ao se afirmar que a massa específica da água é de 1000 kg/m³ estamos informando que 1 m³ de água possui uma massa de 1000 kg. Isto nos permite deduzir a definição de massa específica, que é a relação entre a massa e o volume ocupado por essa massa:

A massa específica é definida para corpos homogêneos. Já para os corpos não homogêneos essa relação é denominada densidade:

2. Pressão A pressão é definida como a aplicação de uma força distribuída sobre uma área:

A unidade de medida da pressão é newton por metro

quadrado (N/m²). A pressão pode também ser exercida entre

Page 46: curso de específica de física

46

dois sólidos. No caso dos fluídos o newton por metro quadrado é também denominado pascal (Pa). 3. Princípio de Stevin O princípio de Stevin nos permite calcular a pressão em um líquido em repouso, estando com sua superfície livre em contato com a atmosfera:

Uma das consequências do princípio de Stevin é: “Em um líquido em equilíbrio, a pressões são iguais em todos os pontos da mesma horizontal”. 4. Pressão atmosférica No planeta Terra em qualquer parte de sua superfície os corpos estão envoltos em um fluído gasoso, o ar. Como todo fluído ele causa uma pressão nos corpos nele imersos. A pressão atmosférica deve ser expressa em Pa (N/m²). Mas outras unidades podem ser encontradas: – atmosfera (atm) – milímetros de mercúrio (mmHg) ou centímetros de mercúrio (cmHg) – metros de coluna de água (mca). Então, é possível relacionar as várias medidas comparando-se os valores da pressão atmosférica ao nível do mar: 1 atm = 101325 Pa = 10,2 mca = 760 mmHg 5. Princípio de Pascal O Princípio de Pascal afirma que: “Um acréscimo de pressão exercido em qualquer ponto de um fluído é transmitido para

todo o fluído”. Com esse princípio é possível construir e dimensionar macacos hidráulicos, prensas hidráulicas, etc.

Como a pressão é igual em todos os pontos do fluído e supondo a área do pistão da direita sendo 5 vezes maior que o da esquerda tem-se:

Dessa maneira uma força F_1 será, no exemplo, amplificada (F_2 ) cinco vezes. Esse seria a versão hidráulica da alavanca mecânica concebida por Arquimedes. 6. Princípio de Arquimedes Deve-se também a Arquimedes a definição da força de Empuxo gerada por um corpo imerso em um fluído. “A força de empuxo de um corpo imerso em um fluído é igual ao peso do fluído deslocado”.

Se o empuxo for maior que a força peso do corpo, a tendência do corpo é de subir com aceleração. No caso de o peso ser menor que o empuxo, a tendência é de o corpo descer com aceleração. No caso do empuxo ser igual à força peso o corpo terá a tendência de permanecer parado. Nota: ao contrário do que se vê em muitos filmes e da crença geral, um submarino ao tentar emergir não solta subitamente toda a água armazenada em seus tanques de lastro. Isso provocaria uma subida acelerada difícil de ser controlada. A liberação do lastro de seus tanques é feita de forma controlada, de modo a manter a força de empuxo igual à força peso, e com isto conseguir uma subida gradual, com velocidade constante. O controle é feito pela hélice de propulsão em conjunto com as aletas controladoras de movimento vertical. EXERCÍCIOS Questão 01 - (UFU MG/2011)

No decorrer da história, o homem tem empregado princípios físicos para facilitar suas atividades cotidianas como, por exemplo, o uso de alavancas para reduzir seu esforço, o que implicou a construção de guindastes e outros tipos de máquinas. Considere o esquema abaixo, no qual uma pessoa exerce uma força de 50 N perpendicular à barra. Na outra extremidade da barra,

há um bloco de 10 Kg cujo centro de massa encontra-se a 1m do ponto de apoio da barra.

Desprezando o peso da barra, e com base na situação descrita, assinale a alternativa correta.

a) A pessoa consegue levantar a caixa na outra

extremidade. b) A pessoa só conseguirá levantar a caixa se aplicar

uma força maior que 50 N. c) Se a caixa tivesse 5 kg de massa e a mesma força

fosse aplicada, porém a 1,5m do ponto de apoio da barra, a pessoa não conseguiria levantá-la.

d) Só é possível levantar a caixa se a barra estiver com ponto de apoio localizado em seu ponto médio.

Questão 02 - (UEL PR/2012)

Uma das condições de equilíbrio é que a soma dos momentos das forças que atuam sobre um ponto de

apoio seja igual a zero.

Page 47: curso de específica de física

47

Figura 30: Modelo simplificado de um móbile

Considerando o modelo simplificado de um móbile (Fig.

30), onde AC representa a distância entre o fio que

sustenta m1 e o fio que sustenta m2, e ACAB8

1 , qual

a relação entre as massas m1 e m2?

a) m1 = 8

1 m2

b) m1 = 7 m2

c) m1 = 8 m2 d) m1 = 21 m2

e) m1 = 15 m2

Questão 03 - (UNIFOR CE/2012)

Num espetáculo circense, dois palhaços seguram pelas extremidades uma barra homogênea de 3m de comprimento que pesa 200N. Um terceiro palhaço com massa total de 50 kg pode deslizar sobre a barra com seu monociclo. O palhaço na extremidade A da barra só pode suportar uma força até 400 N. Até que distância “x” da extremidade B o palhaço poderá deslizar em seu monociclo? (Considere g = 10m /s2)

a) x = 1,5 m b) x = 1,8 m c) x = 2 m d) x = 2,4 m e) x = 2,5 m

Questão 04 - (UFTM/2012)

Em um dia de calmaria, um barco reboca um paraquedista preso a um paraglider. O barco e o paraquedista deslocam-se com velocidade vetorial e alturas constantes.

(www.gettyimages.pt)

Nessas condições,

a) o peso do paraquedista é a força resultante sobre

ele. b) a resultante das forças sobre o paraquedista é

nula. c) a força resultante exercida no barco é maior que a

resultante no paraquedista. d) a força peso do paraquedista depende da força

exercida pelo barco sobre ele. e) o módulo da tensão na corda que une o

paraquedista ao paraglider será menor que o peso do paraquedista.

Questão 05 - (UFPE/2012)

Uma trave, de massa M = 4,6 kg, é mantida na posição horizontal apoiada lateralmente em uma parede e por meio de um cabo de massa desprezível e inextensível, como mostrado na figura. Considerando que não haja atrito entre a trave e a parede, calcule a tração sobre o cabo, em newtons.

Questão 06 - (MACK SP/2013)

Em uma experiência de laboratório, um estudante utilizou os dados do gráfico da figura 1, que se referiam

à intensidade da força aplicada a uma mola helicoidal,

em função de sua deformação

XK

F. Com

esses dados e uma montagem semelhante à da figura 2, determinou a massa (m) do corpo suspenso. Considerando que as massas da mola e dos fios

(inextensíveis) são desprezíveis, que 2s/m10g e

que, na posição de equilíbrio, a mola está deformada de 6,4 cm, a massa (m) do corpo suspenso é

a) 12 kg b) 8,0 kg c) 4,0 kg d) 3,2 kg e) 2,0 kg

Questão 07 - (PUC RJ/2013)

Um pêndulo é formado por uma bola de 4,0 kg e um fio ideal de 0,2 m de comprimento. No ponto mais alto

Page 48: curso de específica de física

48

de sua trajetória, o cabo que sustenta o pêndulo forma um ângulo de 30º com a vertical.

Indique o módulo do torque realizado pelo peso da bola em Nm neste ponto. Considere g = 10,0 m/s2

a) 0,4 b) 4,0 c) 6,8 d) 10,0 e) 100

Questão 08 - (UEPG PR/2013)

Analisando a ação do sistema de forças que age no corpo, conforme figura abaixo, assinale o que for correto.

01. A resultante das forças sobre o eixo X será conhecida por Rx = F1cos – N – F2sen.

02. A resultante de todo o sistema é conhecida pela soma dos vetores Rx e Ry.

04. Se a resultante do sistema for igual a zero, então o corpo está em repouso absoluto.

08. A resultante das forças sobre o eixo Y será conhecida por Ry = F1cos + F2cos – P.

16. A força de atrito mostrada na figura está fora do eixo X e não pode ser computada como força atuante no sistema.

Questão 09 - (UFPE/2013)

A figura a seguir mostra um conjunto de objetos pontuais com massas iguais, dispostos ao longo de uma reta. A distância entre os objetos 1 e 2 é 4L, enquanto

que a distância entre os objetos 2 e 3 é igual a 16L. Calcule a posição do centro de massa do conjunto, medida a partir do objeto 2, em unidades de L.

Questão 10 - (UNIMONTES MG/2013)

Veja o desenho abaixo. As forças nos cabos A e B, em Newtons, são, respectivamente,

a) )tan()cos()(sen

100

,

)tan()cos()(sen

100

b) )tan()cos()(sen

100

,

)tan()cos()(sen

100

c) )tan()cos()(sen

100

,

)tan()cos()(sen

100

d) )tan()cos()(sen

100

,

)tan()cos()(gtan

100

Questão 11 - (ESCS DF/2012)

Uma barra rígida homogênea, de peso de módulo 900N e 4,0m de comprimento, está na horizontal, apoiada em dois suportes que exercem forças verticais para cima N1 e N2 sobre a barra, como mostra a figura a seguir:

Levando em conta que o primeiro suporte está na extremidade esquerda da barra e o segundo está a 1,0 m da extremidade direita, concluímos que os módulos dessas força são dados respectivamente por:

a) N1 = 450N ; N2 = 450N; b) N1 = 600N ; N2 = 300N; c) N1 = 300N ; N2 = 600N; d) N1 = 100N ; N2 = 800N; e) N1 = 800N ; N2 = 100N.

Questão 12 - (UEL PR)

Uma prancha de madeira AB, homogênea, de comprimento 8,0 m e peso 1 200 N, está apoiada em uma pedra, a 5,0 m da extremidade A que se apóia no solo, como indica a figura.

A

B

Um homem, de peso 600 N, caminha sobre a prancha, a partir de A. Em metros, a máxima distância que ele pode caminhar sobre a prancha sem levantar a extremidade A é a) 4,5 b) 5,5 c) 6,0 d) 6,5 e) 7,0

Questão 13 - (UEL PR)

Uma tesoura é uma ferramenta construída para ampliar a força exercida pela mão que a utiliza para cortar os objetos. A essa ampliação de força dá-se o nome de

“vantagem mecânica” dada por 2

1

1

2

d

d

F

F , onde o índice

1 é relativo ao cabo, e o índice 2 está relacionado à lâmina de corte. Sobre a vantagem mecânica da tesoura, é correto afirmar:

a) Se d1 for menor que d2, F2 é maior que F1. b) Se d1 for menor que d2, F1 é igual a F2.

Page 49: curso de específica de física

49

c) Se d1 for maior que d2, F2 é maior que F1. d) Se d1 for maior que d2, F1 é maior que F2. e) Se d1 for igual a d2, F1 é maior que F2.

Questão 14 - (UEL PR/2008)

Na figura seguinte, está ilustrada uma engenhoca utilizada para retirar água de poços. Quando acionada a manivela, que possui um braço de 30 cm, a corda é enrolada em um cilindro de 20 cm de diâmetro, após passar, dando uma volta completa, por um cilindro maior de 60 cm de diâmetro, o qual possui um entalhe para conduzir a corda sem atrito. De acordo com os conhecimentos de mecânica, qual é, aproximadamente, a força mínima que deve ser aplicada à manivela para manter o sistema em equilíbrio? Considere que a força peso do balde cheio de água é 100 N.

a) 33 N. b) 50 N. c) 66 N. d) 100 N. e) 133 N.

Questão 15 - (UEM PR)

Duas pessoas usam uma haste reta de 3,00 m e massa desprezível, com extremidades, A e B, mantida

horizontalmente, para carregar um objeto C, cuja distância de A é igual a d. A primeira pessoa segura no

extremo A da haste, exercendo uma força vertical AP F

; por sua vez, a segunda pessoa segura no extremo B,

exercendo uma força vertical BPF (ver a figura).

A partir disso, e sabendo-se que a soma das forças que atuam sobre a haste e a soma dos torques aplicados à haste, relativo ao ponto A, são nulas, assinale o que for correto.

01. As forças AP F e B

PF podem ser diferentes, se d

= 1,50 m.

02. O módulo de AP F é diferente de zero newton, se

d = 3,00 m.

04. A soma dos módulos de AP F e B

PF depende do

valor de d.

08. O módulo de AP F é igual ao dobro do módulo de

BPF , se d = 1,00 m.

16. O módulo de BPF é quatro vezes maior que o

módulo de AP F , se d = 2,40 m.

Questão 16 - (UEM PR)

Uma barra homogênea, cujo peso é P1, está fixa nos pontos B e C conforme a figura a seguir. Na extremidade D da barra, é pendurado um peso P2. Considerando-se que as forças exercidas pelos apoios B e C são, respectivamente, NB e NC, e estão na direção vertical, e que AB = 1m, BC = 2m e CD = 1m, é correto afirmar que:

01. |NC| > |NB|, quaisquer que sejam P1 e P2 não nulos. 02. NC e NB estão dirigidas para cima, quaisquer que

sejam P1 e P2 não nulos. 04. |NC| + |NB| = P1 + P2, se P1 = P2. 08. NB = 0, se P1 = P2. 16. |NC| = |NB|, se P2 = 0 e P1 0. 32. |NC| = 3 |NB|, se P1 = 0 e P2 0.

Questão 17 - (UEM PR)

A figura a seguir representa um bloco de massa igual a

kg 22,0 , apoiado sobre um plano inclinado. O ângulo q,

entre o plano inclinado e o plano horizontal, é igual a 45º. Sabe-se que o coeficiente de atrito estático entre o bloco e o plano inclinado é igual a 0,5 e que a aceleração da gravidade local é 10,0 m/s2. Qual deverá ser o menor valor da força F (em N) para que o bloco fique em repouso sobre o plano inclinado?

Questão 18 - (UEL PR/2011)

A figura a seguir apresenta um vaso preenchido com dois fluidos diferentes não miscíveis. O fluido 1 apresenta densidade de 1g/cm3 e o fluido 2, densidade de 0, 7g/cm3.

Sendo h1 = h + h2, qual a razão h/h3?

Page 50: curso de específica de física

50

a) 0,7 b) 1 c) 5 d) 3,2

e) 100 Questão 19 - (UEL PR)

Deseja-se que um corpo formado de madeira e aço fique flutuando em equilíbrio quando completamente imerso em água. Sabendo-se que as massas específicas da madeira, água e aço são, respectivamente, 0,25 g/cm3, 1g/cm3 e 8 g/cm3, calcule a relação entre o volume de madeira V1 e o volume de aço V2 do corpo, de modo que ocorra o equilíbrio.

a) 321

V

V

2

1

b) 34

V

V

2

1

c) 3

32V

V

2

1

d) 328

V

V

2

1

e) 3

16V

V

2

1

Questão 20 - (UEL PR/2010)

Numa residência, o reservatório de água está situado a 10 metros de altura em relação a uma torneira. Assinale a alternativa que apresenta a pressão exercida na válvula da torneira quando a torneira é mantida fechada. Dados: densidade específica da água de 1 × 103 kg/m3, aceleração da gravidade 10 m/s2 e pressão atmosférica

1 atm = 1,05 × 105 N/m2.

a) 1 atm b) 10 atm c) 15 N/m2 d) 2500 N/m2 e) 2,05 × 105 N/m2

GABARITO: 1) Gab: A 2) Gab: B 3) Gab: B 4) Gab: B 5) Gab: 92 N 6) Gab: C 7) Gab: B 8) Gab: 03 9) Gab: 04 10) Gab: B 11) Gab: C 12) Gab: E 13) Gab: C 14) Gab: A 15) Gab: EEECC 16) Gab: CECCCC 17) Gab: 01 18) Gab: A 19) Gab: D 20) Gab: E 21) Gab: 77 22) Gab: EECCE 23) Gab: CCCCCE 24) Gab: B 25) Gab: E 26) Gab: C 27) Gab: 07 28) Gab: C 29) Gab: A 30) Gab:

31) Gab: A 32) Gab: A 33) Gab: B 34) Gab: D 35) Gab: B 36) Gab: D 37) Gab: A 38) Gab: E 39) Gab: C 40) Gab: D 41) Gab: A 42) Gab: E 43) Gab: D 44) Gab: A 45) Gab: B 46) Gab: E 47) Gab: E 48) Gab: E 49) Gab: D 50) Gab: C

Page 51: curso de específica de física

51

LISTA 11 – LEIS DE KEPLER E GRAVITAÇÃO

RESUMO DE CONTEÚDO As Leis de Kepler A observação de corpos celestes é um fato que, de acordo com registros, vem de milhares de anos. Vários povos desde a Antiguidade observavam os corpos e desde então falavam de fenômenos astronômicos, trabalhavam a cultura da lavoura ou até colocavam os seus deuses no céu e atribuíam a eles as manifestações divinas. O estudo dos astros teve início com os gregos antigos. Foram eles os primeiros a tentarem explicar o movimento dos corpos celestes. O mais importante deles foi Cláudio Ptolomeu, que propôs o sistema planetário geocêntrico (Terra como centro do universo). Segundo esse sistema, a Terra é o centro de todo o Universo. O Sol e a Lua descreviam órbitas circulares ao redor da Terra. Quanto aos outros planetas, cada um deles descreveria órbitas circulares em torno de um centro que por sua vez descreveriam órbitas circulares ao redor da Terra. O sistema geocêntrico prevaleceu por muitos anos, somente séculos mais tarde é que foram feitas contestações e levantadas novas hipóteses sobre o movimento dos corpos celestes e todo o universo. Nicolau Copérnico, em seus estudos, propôs o Sol como centro do Universo, heliocentrismo, segundo o qual os planetas, então conhecidos na época, descreveriam órbitas circulares ao redor do Sol.

Esse sistema permaneceu durante um bom tempo, até que anos mais tarde Johannes Kepler, discípulo de Tycho Brahe, determinou as leis do Universo assim como as conhecemos hoje. Kepler herdou de seu mestre todas as suas anotações e com seus estudos determinou três leis: Lei das Órbitas: os planetas descrevem órbitas elípticas ao redor do Sol, que ocupa um dos focos da elipse descrita.

Lei das Áreas: o segmento imaginário que une o centro do Sol e o centro do planeta varre áreas proporcionais aos intervalos de tempo dos percursos.

O ponto p é chamado periélio e o ponto a, afélio. Em p a distância Terra-Sol é mínima e a velocidade é máxima e em a a distância Terra-Sol é máxima e a velocidade é mínima. Lei dos Períodos: o quadrado do período de revolução de

cada planeta é proporcional ao cubo do raio médio da respectiva órbita. T2= Kr3

Sendo T o tempo gasto para um planeta dar uma volta completa ao redor do Sol, e r a medida do semieixo maior de sua órbita (denominado raio médio), K é uma constante de proporcionalidade que só depende da massa do Sol. As leis de Kepler dão uma visão cinemática do Universo, mas não basta só entender os movimentos dos planetas, é também necessário entender como eles conseguem permanecer sempre na mesma trajetória, descrevendo as mesmas órbitas elípticas e não caem, como é o caso da Lua sobre a Terra. A lei da Gravitação Universal explica como isso é possível. A Lei da Gravitação Universal A lei da Gravitação foi proposta por Sir Isaac Newton, cientista inglês famoso por seus estudos e contribuições na Física e na Matemática, além de também ser alquimista e astrônomo. Autor de célebres livros como o Philosophiae Naturalis Principia Mathematica no qual ele descreve a Lei da Gravitação Universal e As Leis de Newton. Diz a história que Newton estava sob uma macieira quando dela caiu uma maçã sobre a sua cabeça. Não sabemos se isso realmente é verdade ou não, o que é muito importante é que isso fez com que se explorassem mais os mistérios do universo e a Gravitação Universal. Newton explicou a razão pela qual a Lua não cai sobre a Terra descrevendo a seguinte equação, equação esta que

determina a Lei da Gravitação Universal:

G é uma constante gravitacional e seu valor é igual a 6,67.10-11 N.m2/Kg2 m1 e m2 são as massas dos corpos que se atraem, medida em Kg. r é a distância entre os dois corpos, medida em metros(m).

F é a força gravitacional, e é medida em N. Com tal equação matemática Newton descobriu que os corpos se atraem mutuamente, fazendo com que eles não caiam uns sobre os outros e sempre mantenham a mesma trajetória, ou seja, a sua órbita elíptica ao redor do Sol, como descobriu Johannes Kepler em uma de suas três leis do movimento dos planetas. EXERCÍCIOS Questão 01 - (UFSC/2012)

"Eu medi os céus, agora estou medindo as sombras. A mente rumo ao céu, o corpo descansa na terra."

Com esta inscrição, Johannes Kepler encerra sua passagem pela vida, escrevendo seu próprio epitáfio. Kepler, juntamente com outros grandes nomes, foi responsável por grandes avanços no que se refere à mecânica celeste.

No que se refere à história e à ciência por trás da mecânica celeste, assinale a(s) proposição(ões) CORRETA(S).

01. O astrônomo Cláudio Ptolomeu defendia o sistema

geocêntrico, com a Terra no centro do sistema planetário. Já Nicolau Copérnico defendia o sistema heliocêntrico, com o Sol no centro do

Page 52: curso de específica de física

52

sistema planetário. Tycho Brahe elaborou um sistema no qual os planetas giravam em torno do Sol e o Sol girava em torno da Terra.

02. Galileu Galilei foi acusado de herege, processado pela Igreja Católica e julgado em um tribunal por afirmar e defender que a Terra era fixa e centralizada no sistema planetário.

04. Kepler resolveu o problema das órbitas dos planetas quando percebeu que elas eram elípticas, e isso só foi possível quando ele parou de confiar nas observações feitas por Tycho Brahe.

08. O movimento de translação de um planeta não é uniforme; ele é acelerado entre o periélio e o afélio, e retardado do afélio para o periélio.

16. A teoria da gravitação universal, de Newton, é válida para situações nas quais as velocidades envolvidas sejam muito grandes (próximas à velocidade da luz) e o movimento não ocorra em campos gravitacionais muito intensos.

32. A teoria da relatividade geral de Einstein propõe que a presença de uma massa deforma o espaço e o tempo nas suas proximidades, sendo que, quanto maior a massa e menor a distância, mais intensos são seus efeitos. Por isso a órbita de Mercúrio não pode ser explicada pela gravitação de Newton.

TEXTO: 1 - Comum à questão: 2

Em setembro de 2010, Júpiter atingiu a menor distância da Terra em muitos anos. As figuras abaixo ilustram a situação de maior afastamento e a de maior aproximação dos planetas, considerando que suas órbitas são circulares, que o raio da órbita terrestre (RT) mede 1,51011 m e que o raio da órbita de Júpiter (RJ) equivale a 7,51011 m.

Questão 02 - (UNICAMP SP/2012)

De acordo com a terceira lei de Kepler, o período de revolução e o raio da órbita desses planetas em torno

do Sol obedecem à relação

3

T

J

2

T

J

R

R

T

T

, em que TJ

e TT são os períodos de Júpiter e da Terra, respectivamente. Considerando as órbitas circulares representadas na figura, o valor de TJ em anos terrestres é mais próximo de

a) 0,1. b) 5. c) 12. d) 125.

Questão 03 - (UERJ)

Um asteroide A é atraído gravitacionalmente por um planeta P Sabe-se que a massa de P é maior do que a massa de A. Considerando apenas a interação entre A e P, conclui-se

que:

a) o módulo da aceleração de P é maior do que o módulo da aceleração de A

b) o modulo da aceleração de P é menor do que o módulo da aceleração de A

c) a intensidade da força que P exerce sobre A é maior do que a intensidade da força que A exerce sobre P

d) a intensidade da força que P exerce sobre A é menor do que a intensidade da força que A exerce sobre P

Questão 04 - (UNIMEP SP)

Assinale a alternativa correta: a) Dentro de um mesmo campo gravitacional, e no

vácuo, todos corpos caem com a mesma aceleração. b) Em qualquer lugar no universo os corpos mais

pesados caem mais rápidos que os corpos mais leves.

c) Na Terra, na Lua ou em Marte, a aceleração da gravidade é igual a 9,8 m/s2.

d) A Terra é o centro do sistema solar, sendo a Lua um dos planetas.

e) Nenhuma das anteriores. Questão 05 - (UFMG/2007)

Três satélites – I, II e III – movem-se em órbitas circulares ao redor da Terra. O satélite I tem massa m e os satélites II e III têm,

cada um, massa 2m . Os satélites I e II estão em uma mesma órbita de raio

r e o raio da órbita do satélite III é 2r .

Nesta figura (fora de escala), está representada a posição de cada um desses três satélites:

Sejam FI, FII e FIII os módulos das forças gravitacionais da Terra sobre, respectivamente, os satélites I, II e III. Considerando-se essas informações, é CORRETO afirmar que a) FI = FII < FIII. b) FI = FII > FIII. c) FI < FII < FIII. d) FI < FII = FIII.

Questão 06 - (UESC BA/2006)

Admitindo-se que o raio da órbita de Júpiter é 5 vezes maior que o da Terra, o período de revolução do Júpiter, em anos terrestre, é igual a 01. 5

02. 2

1

5

03. 125

04. 2

1

125

Page 53: curso de específica de física

53

05. 3

1

125

Questão 07 - (ITA SP)

Uma estrela mantém presos, por meio de sua atração gravitacional, os planetas Alfa, Beta e Gama. Todos descrevem órbitas elípticas, em cujo foco comum se encontra a estrela, conforme a primeira lei de Kepler. Sabe-se que o semieixo maior da órbita de Beta é o dobro daquele da órbita de Gama. Sabe-se também que

o período de Alfa é 2 vezes maior que o período de

Beta. Nestas condições, pode-se afirmar que a razão entre o período de Alfa e o de Gama é

a) 2 .

b) 2. c) 4.

d) 4 2 .

e) 6 2 .

Questão 08 - (UFPE/2007)

A figura ilustra um satélite artificial de massa m que circunda a Terra (massa M e raio R), em movimento circular e uniforme. Sabe-se que o satélite está a uma

distância 2/R3d da superfície terrestre. Efeitos de

resistência do ar e atrito são desprezados. A constante de gravitação universal é denotada por G. Para tal situação, qual é o módulo da aceleração do satélite?

a) 2GM/(3R2) b) 4Gm/(9R2) c) 4GM/(25R2) d) 2Gm/(5R2) e) 16Gm/(9R2)

Questão 09 - (ITA SP/2008)

Numa dada balança, a leitura é baseada na deformação de uma mola quando um objeto é colocado sobre sua plataforma. Considerando a Terra como uma esfera homogênea, assinale a opção que indica uma posição da balança sobre a superfície terrestre onde o objeto terá a maior leitura. a) Latitude de 45º. b) Latitude de 60º. c) Latitude de 90º. d) Em qualquer ponto do Equador. e) A leitura independe da localização da balança já que

a massa do objeto é invariável. Questão 10 - (UFPel RS/2008)

Costuma-se dizer que a Lua está sempre caindo sobre a Terra. Por que a Lua não cai sobre a Terra, afinal? a) Porque a Lua gira em torno da Terra. b) Porque a aceleração da gravidade da Lua é menor

que a da Terra. c) Porque ambas, Terra e Lua, se atraem com forças

de mesmo módulo, mesma direção e sentidos opostos.

d) Porque a massa da Terra é maior que a massa da Lua.

e) Porque o raio da Lua é menor que o raio da Terra.

f) I.R. Questão 11 - (UEL PR/2010)

Um dos grandes e fundamentais problema da física atual é explicar as curvas de rotação das galáxias. Este nome é dado ao gráfico onde se esboça a velocidade de rotação das estrelas que compõe uma dada galáxia, em função da distância ao núcleo galáctico, como mostrado na figura anterior. Nessa figura a curva superior representa os resultados experimentais das medidadas astronômicas das velocidades das estrelas orbitando o núcleo galáctico, enquanto a curva inferior representa o resultado esperado para essas velocidades, quando se utiliza a Lei da Gravitação Universal de Newton. Numa primeira aproximação, pode-se estimar a massa da galáxia com um modelo simplificado que considera o equilíbrio entre as forças gravitacional e centripeta que atuam numa dada estrela que interage com o material galáctico no interior de sua órbita. A compreensão destas curvas proporciona informações sobre a distribuição da massa das galáxias. Com a utilização de radiotelescópios, constatou-se que frequentemente as curvas de rotação ficam constantes a grandes distâncias do núcleo galáctico, contrariando a previsão de uma lei do inverso do quadrado da distância para a força e o

esgotamento do material das galáxias a tais distâncias. Várias idéias foram propostas para explicar a discrepância entre as velocidades previstas teoricamente e as medidas experimentalmente; citamos as duas mais aceitas atualmente: proposta de modificação da lei de gravitação de Newton e a existência de Matéria Escura. Valores fornecidos:

Com base na figura, no texto, nos conhecimentos sobre o assunto, e considerando que as estrelas, inclusive o sol, movem-se em orbitas circulares ao redor do núcleo galático e utilizando os valores fornecidos, considere as afirmativas a seguir:

I. A massa da galáxia, no interior da obtida do sol,

utilizando-se a velocidade observada do sol é 1,9 vezes maior que a massa obtida utilizando-se a velocidade prevista pela utilização da teoria da gravitação de Newton. A explicação dessa diferença é um dos principais problemas da física atual.

II. A dificuldade na estimativa do número de estrelas

nas galáxias é o fator que gera a discrepância entre os valores dessas duas velocidades.

III. A diferença entre a velocidade medida e a calculada do modelo teórico é devido às imprecisões nos

Page 54: curso de específica de física

54

cálculos computacionais dado que o modelo teórico é exato.

IV. O equilíbrio entre a força centrífuga e a gravitacional fornece uma estimativa aproximada para a massa da galáxia no interior da órbita solar de 1011

massas solares.

Assinale a alternativa correta.

a) Somente as afirmativas I e II são corretas. b) Somente as afirmativas I e IV são corretas. c) Somente as afirmativas III e IV são corretas. d) Somente as afirmativas I, II e III são corretas. e) Somente as afirmativas II, III e IV são corretas.

Questão 12 - (UEL PR)

Um satélite artificial é colocado em órbita ao redor da Terra. Seja RT o raio da Terra (distância do nível do mar até o centro da Terra) e P o peso do satélite artificial ao nível do mar, onde a aceleração da gravidade tem módulo g. Este satélite, ao se encontrar numa altura h acima do nível do mar, estará sujeito a uma aceleração da gravidade g’. Determine a razão entre g’ e g.

a) hR

R

g

'g

T

T

b) T

T

R

hR

g

'g

c) hR

hg

'g

T

d) 22

T

2

hR

hg

'g

e) 2

T

2T

)hR(

R

g

'g

Questão 13 -

O volume e a massa do planeta Marte são menores

que os da Terra. Lá, a pressão atmosférica é muito

pequena, da ordem de 0,007 atm, e a temperatura pode

variar de –12 ºC a 62 ºC. A tabela a seguir apresenta

alguns parâmetros de Marte e da Terra, considerando-

se que as órbitas desses planetas sejam circulares.

Tendo como referência as informações acima, assinale

a opção correta.

a) A gravidade em Marte é duas vezes maior que na

Terra.

b) Caso exista água em Marte, ela ferverá a uma

temperatura menor que na Terra.

c) Se, na Terra, um corpo tem massa de 70 kg,

então, em Marte, ele terá massa menor.

d) O ano em Marte é menor que o ano na Terra.

Questão 14 - (UEL PR)

Observe a figura abaixo.

Se o satélite americano for estacionário, isto é, se seu período de rotação for igual a 24 horas (86 400 s), qual é a sua altitude? Dados: G = 6,7.10–11 N.m2 / kg2; massa da Terra: MT = 6,0.1024 kg; raio da Terra: rT = 6,38.106 m a) 36.104 m b) 36.106 m c) 36.108 m d) 36.109 m e) 36.1010 m

Questão 15 - (UFPR/2013)

Dois satélites, denominados de SA e SB, estão orbitando um planeta P. Os dois satélites são esféricos e possuem tamanhos e massas iguais. O satélite SB possui uma órbita perfeitamente circular e o satélite SA uma órbita elíptica, conforme mostra a figura abaixo.

Em relação ao movimento desses dois satélites, ao longo de suas respectivas órbitas, considere as

seguintes afirmativas:

1. Os módulos da força gravitacional entre o satélite SA e o planeta P e entre o satélite SB e o planeta P são constantes.

2. A energia potencial gravitacional entre o satélite SA e o satélite SB é variável.

3. A energia cinética e a velocidade angular são constantes para ambos os satélites.

Assinale a alternativa correta.

a) Somente a afirmativa 1 é verdadeira. b) Somente a afirmativa 2 é verdadeira. c) Somente a afirmativa 3 é verdadeira. d) Somente as afirmativas 1 e 2 são verdadeiras. e) Somente as afirmativas 2 e 3 são verdadeiras.

Questão 16 - (UEM PR/2009)

Page 55: curso de específica de física

55

Um corpo, solto próximo à superfície da Terra, está sujeito à ação da força gravitacional oriunda da interação Terra-corpo. Com base nessa afirmação, assinale o que for correto.

01. A força de reação, devido à atração gravitacional

que Terra exerce sobre o corpo, é a força de natureza gravitacional com que o corpo atrai a Terra.

02. O campo gravitacional da Terra é representado pelo

vetor campo gravitacional g , que pode ser

considerado constante quando medido ao nível do mar.

04. A terceira lei de Kepler estabelece que o quadrado do período de revolução de um planeta em órbita do Sol é diretamente proporcional ao cubo do raio médio de sua órbita.

08. Se colocarmos o corpo sobre uma mesa, ao nível do mar, a força de reação à força peso do corpo será a força normal originada do contato entre o corpo e a mesa.

16. Um corpo colocado ora na superfície da Terra ora na superfície da Lua apresentará o mesmo peso e a mesma massa.

Questão 17 - (UEM PR/2009)

No seu movimento de translação, a Terra descreve uma trajetória elíptica ao redor do Sol. Considerando que a única força que atua sobre ela, em toda a trajetória, deve–se à atração gravitacional entre a Terra e o Sol, podemos afirmar corretamente que

01. a velocidade da Terra é máxima, no periélio. 02. a energia potencial gravitacional da Terra em

relação ao Sol é máxima, no afélio. 04. a força que o Sol faz sobre a terra é máxima, no

afélio. 08. a energia mecânica total do sistema Terra-Sol é a

mesma, no afélio e no periélio. 16. o trabalho realizado pela força atrativa que o Sol faz

para levar a Terra do periélio ao afélio é negativo. Questão 18 - (UNIOESTE PR)

Sobre a aceleração da gravidade, é correto afirmar 01. que, em um mesmo local, a aceleração da gravidade é uma grandeza vetorial com direção e sentido iguais à direção e sentido da força da gravidade. 02. que as diferentes massas da Terra e da Lua definem

valores diferenciados para a aceleração da gravidade em suas superfícies.

04. que, em queda livre, todos os objetos apresentam aceleração igual a 9,8 m/s2, independentemente da distância em relação à superfície da Terra.

08. que a aceleração de pacotes de energia como os fótons, na ausência da resistência do ar, é determinada pela aceleração da gravidade local.

16. que a aceleração da gravidade próxima à superfície terrestre independe de características próprias dos objetos sujeitos a esta aceleração.

32. que o valor da aceleração da gravidade diminui à medida que nos afastamos do nível do mar para maiores altitudes, devido à redução da densidade do ar.

64. que o valor da aceleração da gravidade na Terra, ao nível do mar, é aproximadamente 9,8 N/kg.

Questão 19 - (UNIOESTE PR)

No mês de Outubro de 2004 ocorreu um eclipse lunar. Sobre fenômenos relacionados à Astronomia é correto afirmar que:

01. o eclipse lunar ocorre quando há um alinhamento entre o Sol, a Lua e a Terra, sendo que a sombra da Lua é projetada sobre a Terra. 02. o eclipse lunar é um fenômeno observado apenas

em algumas regiões do planeta Terra, diferentemente do eclipse solar, que atinge igualmente todas as regiões do planeta.

04. a força que mantém a Lua em órbita em torno da Terra é da mesma natureza que a força com a qual a Terra atrai uma pedra em sua superfície.

08. o eclipse solar não deve ser observado diretamente, pois a luz emitida pelo Sol pode causar danos irreversíveis aos olhos.

16. o eclipse lunar não deve ser observado diretamente, pois a luz emitida pela Lua pode causar danos irreversíveis aos olhos.

32. as forças que atuam a partir de uma interação a distância, ou seja, sem o contato entre os corpos, podem ocorrer na superfície da Terra e entre os astros.

64. a força que mantém os planetas em órbita em torno do Sol é de natureza magnética.

Questão 20 - (UNIOESTE PR/2006)

Astrônomos da Nasa anunciaram a descoberta, este ano, de um novo planeta, Gliese 876d, parecido com a Terra e fora do sistema solar. O novo planeta gira ao redor da

estrela Gliese 876, na constelação de Aquário, a 15 anos-luz da Terra. Apesar de ser o menor planeta já encontrado fora do sistema solar, tem o dobro do tamanho e da gravidade terrestres. O dia em Gliese, 876d dura 46,56 horas, e a temperatura diurna, na superfície, varia entre 200 e 400°C. Analise as afirmações e assinale-as devidamente. Tome c = 3 x 108m/s. 00. A ordem de grandeza, em metros, da distância entre

Gliese 876d e a Terra é de 108m. 01. Um pulso de luz, partindo de Gliese 876d, chega à

Terra num intervalo de tempo igual a 15 anos. 02. Se um de nossos atletas, recordistas de 100 metros

rasos, corresse em Gliese 876d, facilmente ele melhoraria seu tempo.

03. O ano em Gliese 876d é maior do que o ano terrestre.

GABARITO: 1) Gab: 33 2) Gab: C 3) Gab: B 4) Gab: A 5) Gab: C 6) Gab: 04 7) Gab: C 8) Gab: C 9) Gab: C 10) Gab: A 11) Gab: B 12) Gab: E 13) Gab: B 14) Gab: B 15) Gab: B 16) Gab: 07 17) Gab: 27 18) Gab: 01+02+16+64 19) Gab: 04+08+32 20) Gab: FVFF 21) Gab: E 22) Gab: 05 23) Gab: E 24) Gab: B 25) Gab:

vA(3) = 2,0 x 10–6 m/s

RA = 2

2

11

)0,3()01,0(

10x67,6

2

1

26) Gab: C 27) Gab: B 28) Gab: E 29) Gab: VFVF 30) Gab: D 31) Gab: D 32) Gab: D 33) Gab: D 34) Gab: 84 35) Gab:

a) 2

2

22

24

P2T

2PT

gP

gT105,1

30

15,0

10

106

md

dm

F

F

b) 2P

P

P

2

r

GM

r

v

Page 56: curso de específica de física

56

s/m102r

GMv 2

P

P

36) Gab: C 37) Gab: A 38) Gab: 09 39) Gab: C 40) Gab: B

Page 57: curso de específica de física

57

LISTA 12 - TERMOMETRIA E DILATAÇÃO

RESUMO DE CONTEÚDO

Graduação de um Termômetro

A graduação de um termômetro consiste basicamente na determinação dos chamados pontos fixos .Os pontos fixos servem como referência para medida de todos os outros valores de temperatura. Essa graduação é feita a partir de uma substância pura, no caso a água a uma pressão de 1atm(atmosfera). O primeiro ponto fixo é obtido mergulhando-se o termômetro em um recipiente que contenha gelo em fusão. Já o segundo ponto fixo é

determinado mergulhando-se o termômetro em um recipiente que contenha água em ebulição.

1º Ponto fixo =Ponto de Fusão do Gelo (corresponde a 0 ºC a um 1 atm)

2º Ponto Fixo = Ponto de Ebulição da água

(corresponde a 100 ºC a um 1 atm) Lembrarmos que a pressão varia proporcionalmente a temperatura, somente, podemos dizer que a água pura satura (vaporiza) a 100ºC a uma dada pressão correspondente a 1 atm. Definidos os pontos fixos da água na escala Celsius, destacamos 2 escalas importantes de temperatura; Fahrenheit e Kelvin. Escala Fahrenheit ( Utilizada em países de língua inglesa)

1º ponto Fixo: 32

2º ponto Fixo: 212

Escala Kelvin (é a escala de temperatura do SI)

1º ponto Fixo: 273

2º ponto Fixo: 373 A partir dos respectivos valores dos pontos fixos de cada escala podemos relacionar essas escalas de modo a obter

uma equação que converta uma temperatura em uma dada escala para outra escala. Vejamos:

Por Interpolação obtemos a equação C/5 = F-32/9 = K-273/5 Vale destacar que o zero absoluto é um valor teórico obtido experimentalmente que indica a “paralisação /morte da matéria” cujo valor é 0K. DILATAÇÃO TÉRMICA Em termos físicos, dizemos que a temperatura de um corpo (ou objeto) é uma medida da agitação dos átomos e moléculas que o constituem. Sendo assim, podemos dizer que quanto maior for a temperatura de um corpo, maior será

a agitação de seus átomos ou moléculas. O aumento da temperatura de um corpo não provoca somente o aumento da agitação de átomos e moléculas, mas provoca também outros efeitos no corpo. Dessa forma, todos os corpos que estão na natureza são passíveis de sofrer algum efeito por conta do aumento de sua temperatura. Em alguns corpos, esses efeitos podem ser visíveis, já em outros, não. Quando há aumento de temperatura de um objeto, ele sofre também aumento em suas dimensões, portanto, os objetos dilatam-se. Definimos a dilatação como sendo a variação nas dimensões de um objeto devido à variação da temperatura. Portanto, podemos dizer que a dilatação é uma manifestação macroscópica da variação da energia cinética das moléculas e átomos. Quase todos os materiais existentes na natureza sofrem aumento em seu volume quando há elevação de sua temperatura, em consequência disso, a densidade desses objetos diminui. Pode-se dizer que essa regra é válida para quase todas as substâncias. Uma exceção é a água que apresenta comportamento diferente quando sua temperatura varia de 0ºC a 4ºC. Esse fenômeno é conhecido como estado anômalo da água.

O aumento de temperatura gera um aumento na separação entre os átomos ou moléculas de uma substância que causa

uma variação de volume A figura acima nos permite entender melhor como acontece a dilatação nos materiais com o aumento da temperatura. Podemos ver que na primeira situação, a certa temperatura, os átomos e moléculas da substância vibram ao redor de uma posição de equilíbrio. Para esse valor de temperatura, a distância média entre as moléculas é praticamente constante. Porém, quando há aumento na temperatura da substância, as moléculas ou átomos começam a vibrar mais rapidamente, isto é, vibram com maior amplitude, consequentemente, ficam um pouco mais distantes umas das outras. Essa maior separação, isto é, essa maior distância entre os átomos e moléculas, reflete-se no aumento das dimensões do objeto. Podemos concluir dizendo que os efeitos provocados pela dilatação térmica são de suma importância no momento em que se projetam máquinas que usam diferentes tipos de peças de diferentes materiais, que ficam em contato entre si, pois com a variação da temperatura, essas peças irão se dilatar de forma diferente. EXERCÍCIOS Questão 01 - (UNIFOR CE)

Uma certa massa de gás perfeito sofre uma

transformação isobárica e sua temperatura varia de 293K para 543K. A variação da temperatura do gás,

Page 58: curso de específica de física

58

nessa transformação, medida na escala Fahrenheit, foi de a) 250° b) 273° c) 300° d) 385° e) 450°

Questão 02 - (UNIFOR CE)

A temperatura de determinada substância é 50°F. A temperatura absoluta dessa substância, em kelvins, é a) 343 b) 323 c) 310 d) 283 e) 273

Questão 03 - (UNIFOR CE)

Uma escala termométrica arbitrária X atribui o valor 20°X para a temperatura de fusão do gelo e 80°X para a temperatura de ebulição da água, sob pressão normal. Quando a temperatura de um ambiente sofre uma variação de 30°X, a correspondente variação na escala Celsius é de: a) 20°C

b) 30°C c) 40°C d) 50°C e) 60°C

Questão 04 - (UFF RJ)

Um turista brasileiro, ao desembarcar no aeroporto de Chicago, observou que o valor da temperatura lá indicado, em °F, era um quinto do valor correspondente em °C. O valor observado foi: a) - 2 °F b) 2 °F c) 4 °F d) 0 °F e) - 4 °F

Questão 05 - (MACK SP)

Uma pessoa mediu a temperatura de seu corpo, utilizando-se de um termômetro graduado na escala Fahrenheit, e encontrou o valor97,7oF. Essa temperatura, na escala Celsius, corresponde a: a) 36,5oC b) 37,0oC c) 37,5oC d) 38,0oC e) 38,5oC

Questão 06 - (UEL PR)

Uma régua de aço, de forma retangular, tem 80 cm de comprimento e 5,0cm de largura à temperatura de 20ºC. Suponha que a régua tenha sido colocada em um local cuja temperatura é 120ºC. Considerando o coeficiente de dilatação térmica linear do aço 1,1 x 10-5 ºC-1, a variação do comprimento da régua é: a) 0,088cm b) 0,0055cm c) 0,0075cm d) 0,0935cm e) 0,123cm

Questão 07 - (UEG GO/2007)

Em uma experiência de dilatação térmica, dois anéis têm um mesmo raio a 25 ºC. Quando aquecidos a +273,25 ºC, o anel A se encaixa dentro do anel B. Tendo em vista essa experiência, é CORRETO afirmar: a) Nesta temperatura, cessa a atividade molecular e os

anéis se encaixam. b) Para a experiência ser verdadeira deve haver uma

mínima diferença entre os raios a 25 ºC. c) O fato se explica só se o anel A estiver próximo do

seu ponto de fusão, tornando-se maleável. d) O coeficiente de dilatação do anel A é menor do que

o do anel B. Questão 08 - (UNIRIO RJ)

A figura abaixo representa uma lâmina bimetálica. O coeficiente de dilatação linear do metal A é a metade no coeficiente de dilatação linear do metal B. À temperatura ambiente, a lâmina está vertical. Se a temperatura for aumentada em 200°C, a lâmina:

A B

a) continuará na vertical. b) curvará para a frente. c) curvará para a trás. d) curvará para a direita. e) curvará para a esquerda.

Questão 09 - (MACK SP/2007)

Num ensaio em laboratório, dispõe-se de um disco de espessura desprezível e de uma haste, ambos constituídos de um mesmo material. Numa certa

temperatura o , o diâmetro do disco e o comprimento

da haste são iguais a do. Dobrando-se a temperatura desses corpos, a haste passa a ter um comprimento d e o disco terá um diâmetro aproximadamente igual a: a) d

b) d4

5

c) d2

3

d) 2 d

e) d2

5

Questão 10 - (UNIFOR CE)

O comprimento de uma barra de alumínio a 20,0°C é 100,0cm. Quando é aquecida a 100°C, seu comprimento passa a ser 100,2cm. Nessas condições, o coeficiente de dilatação linear médio do alumínio, em

°C1, vale

a) 1,7 106

b) 2,0 106

c) 1,7 105

d) 2,0 105

e) 2,5 105

Questão 11 - (UEL PR)

O gráfico abaixo representa a relação entre as escalas termométricas Y e X. Existe uma temperatura na qual as duas escalas indicam o mesmo valor.

Page 59: curso de específica de física

59

-30

-20

-10

10

20

30

00-60 -40 -20 20 40 60 80

ºX

ºY

a) –30 b) –15 c) 15 d) 100 e) 450

Questão 12 - (UEM PR)

Um pesquisador dispunha de dois termômetros: um, calibrado na escala Celsius e outro, calibrado na escala Fahrenheit. Resolveu, então, construir um terceiro termômetro, sobre o qual o ponto de fusão do gelo foi marcado com 40 graus Xis (40oX) e o ponto de ebulição da água com 240 graus Xis (240oX). Representando por tC, tF e tX as respectivas leituras das temperaturas nas escalas Celsius, Fahrenheit e Xis, o pesquisador fez algumas observações. Assinale o que for correto. 01. A temperatura lida na escala Celsius se relaciona

com a lida na escala Fahrenheit segundo a equação tC = (5/9)(tF –32)

02. A temperatura lida na escala Celsius se relaciona com a lida na escala Xis segundo a equação tC = tX – 20.

04. A temperatura lida na escala Fahrenheit se relaciona com a lida na escala Xis segundo a equação tF = 0,9tX – 4.

08. Quando tC = –40oC, os outros dois termômetros indicam tF = –40oF e tX = –40oX.

16. Uma variação de temperatura de 10 graus na escala Xis corresponde a uma variação de 10 graus na escala Celsius.

32. A temperatura em que a água tem densidade máxima é 24oX.

Questão 13 - (UEM PR)

O coeficiente linear de expansão térmica (coeficiente de dilatação) de um material é dado pela equação

T

L

L

1

0

na qual L0 é o comprimento do material,

à temperatura T0; L = L – L0; T = T – T0;

L é o comprimento do material, à temperatura T. Pode–se afirmar corretamente que: 01. a é uma constante adimensional. 02. L é diretamente proporcional a T.

04. o gráfico L x T é uma reta cujo coeficiente linear

é nulo. 08. o gráfico L x T é uma reta cujo coeficiente angular

é L0.

16. o gráfico L x T é uma reta cujo coeficiente linear é L0 (1 – T0).

32. o gráfico L x T é uma reta cujo coeficiente angular é L0.

Questão 14 - (UEM PR)

Aquecendo-se, à pressão constante, uma certa massa de água a partir de 0oC, observa-se que o volume

ocupado por ela, em função da temperatura, é dado pelo gráfico abaixo. Considerando que, durante esse processo, não houve perda de massa, assinale o que for correto.

01. Para 0oC < T < 4oC, o coeficiente de dilatação térmico da água é positivo.

02. Para T > 4oC, o coeficiente de dilatação térmico da água é positivo.

04. O peso dessa massa de água é máximo em T = 4oC. 08. A densidade da água é máxima em T = 4oC. 16. A densidade da água em 0°C é menor que em 2oC. 32. Ao colocarmos um recipiente aberto com água, à

temperatura ambiente, em um freezer, esta começa a resfriar-se uniformemente por convecção, ou seja, a água da superfície, mais fria, desce, pois tem maior densidade que a água do fundo, que sobe à superfície. No entanto, ao atingir 4oC, a movimentação deixa de ocorrer e a água da superfície continua a esfriar, de modo que a solidificação ocorre primeiramente na superfície.

Questão 15 - (UEM PR)

A figura a seguir representa uma haste metálica com comprimento inicial l0 = 3,0 m, um cubo metálico com volume inicial V0 = 1,0 m3 e uma placa quadrada plana e fina, também metálica, com área inicial A0 = 1,0 m2. A extremidade A da haste é fixa em uma parede vertical e tem a direção de uma normal à parede. A extremidade B da haste toca a face esquerda do cubo cuja base se apóia em um piso horizontal sem atrito que faz um ângulo de 90º com a parede. A placa quadrada está disposta paralelamente ao plano da parede e encontra-se equilibrada sobre a face superior do cubo, seccionando-a em duas metades. Considere que todo o sistema sofre um acréscimo de temperatura de 300ºC, que o coeficiente de dilatação linear do material da haste vale 2,0105/ºC, que o coeficiente de dilatação

superficial do material da placa vale 4,0105/ºC e que o

coeficiente de dilatação volumétrica do material do cubo vale 6,0105/ºC. Observe que um ponto P se encontra

no meio do lado superior da placa quadrada. Denomine de lx o deslocamento que o ponto P sofre na horizontal,

na direção paralela à da haste e de ly o deslocamento

que o ponto P sofre na vertical, na direção perpendicular à da haste. Calcule, em mm, a soma algébrica de lx com ly.

Page 60: curso de específica de física

60

Questão 16 - (UNIOESTE PR/2008)

Uma conhecida dica para se abrir um vidro de conserva (palmito, azeitonas) bem fechado é despejar água fervendo sobre a tampa metálica com o objetivo de dilatá-la. Para abrir um desses vidros, suponha que você

precise dilatar em 2mm 2,0 uma tampa de aço circular

com área inicial de 2cm 50 e massa de 100 g.

Assinale a alternativa que representa o valor correto do calor que teria que ser fornecido à tampa para promover essa dilatação, considerando que não há perdas de calor para o ambiente (dados: Coeficiente de dilatação linear

do aço: -1-5 Cº 10 x 2,1 , Calor específico do aço:

Cº J/kg 450c ).

a) 100 Joules. b) 0,75 Joules. c) 75 Joules. d) 0,50 Joules. e) 50 Joules.

Questão 17 - (ESCS DF/2013)

A sensação de um paciente ao ter a campânula de um estetoscópio encostada nele é de que a peça está fria; aos poucos, essa sensação diminui. Considere que essa campânula tenha a forma cilíndrica e seja constituída de uma liga metálica isotrópica com coeficiente de dilatação volumétrico igual 60 10–6 ºC–1. Considere,

ainda, que a peça esteja à temperatura ambiente de 20 ºC, tenha diâmetro de 6 cm e espessura de 1 cm, conforme ilustrado na figura abaixo.

Sabendo que o paciente se encontra à temperatura de 38 ºC, assinale a opção que apresenta corretamente o acréscimo sofrido pelo raio da peça, em micrômetros, ao entrar em equilíbrio térmico com o paciente.

a) 10,8 b) 16,2 c) 21,6 d) 5,4 e) 8,1

Questão 18 - (UEPG PR/2013)

Aquecendo uma determinada substância, suas dimensões sofrem alterações conhecidas por dilatação térmica. Sobre esse fenômeno, assinale o que for correto.

01. Quando se aquece uma placa que contém um

orifício, as dimensões do orifício se contraem. 02. Um recipiente tem sua capacidade volumétrica

diminuída quando a sua temperatura aumenta. 04. Uma lâmina bimetálica, constituída por dois

materiais de coeficientes de dilatação diferentes e sendo um o dobro do outro, se curva para o lado daquela que tem maior coeficiente de dilatação; e se esfriada, se curva para o lado daquela de menor coeficiente de dilatação.

08. A variação de volume de uma substância é proporcional ao produto do seu volume inicial e à variação de temperatura que é submetida.

16. É impossível determinar o coeficiente de dilatação real de um líquido sem levar em conta o

coeficiente de dilatação do recipiente que o contém.

Questão 19 - (MACK SP/2013)

Uma pequena placa de certa liga metálica de coeficiente de dilatação linear médio igual a 2010–6 ºC–1 possui um

orifício de diâmetro 5,0 mm. Essa placa deve ser presa sobre uma superfície por meio de um pino de diâmetro 5,1 mm, inserido nesse orifício. Para que seja possível prender essa placa com esse pino, nós a aquecemos sem que ocorra a mudança do estado de agregação de seu material. A variação de temperatura mínima, que deve sofrer essa placa, para conseguirmos fixá-la é de

a) 1 000 ºC b) 700 ºC c) 500 ºC d) 300 ºC e) 200 ºC

Questão 20 - (PUC SP/2013)

Considere um recipiente ideal, no interior do qual são colocados 2,4 litros de água e uma fina haste metálica de espessura e massa desprezíveis, comprimento inicial igual a 10cm e coeficiente de dilatação volumétrico igual a 3,610–5 ºC–1, que estão em equilíbrio térmico a uma

temperatura de 20ºC. O conjunto é colocado no interior de um forno de potência constante e igual a 4000W, que é ligado durante 3 minutos. Considerando que toda energia térmica liberada pelo forno foi integralmente absorvida pelo conjunto (água+haste), determine a dilatação linear sofrida pela haste metálica após o tempo de aquecimento.

Adote: calor específico da água = 1,0cal/gºC densidade da água = 1g/cm3 1cal = 4J

a) 9,0 x 10–3 cm b) 1,14 x 10–2 cm c) 3,42 x 10–2 cm d) 2,6 x 10–3 cm e) 7,8 x 10–3 cm

GABARITO: 1) Gab: E 2) Gab: D 3) Gab: D 4) Gab: E 5) Gab: A 6) Gab: A 7) Gab: D 8) Gab: E 9) Gab: A 10) Gab: E 11) Gab: A 12) Gab: 13 13) Gab: ECCCCC 14) Gab: ECECCC 15) Gab: 33 16) Gab: C 17) Gab: A 18) Gab: 24 19) Gab: A 20) Gab: A 21) Gab: A 22) Gab: A 23) Gab: C 24) Gab: E 25) Gab: B 26) Gab: 14 27) Gab: 18 28) Gab: E 29) Gab: D

30) Gab: A 31) Gab: A 32) Gab: C 33) Gab: 77 K 34) Gab: A 35) Gab: C 36) Gab: B 37) Gab: E 38) Gab: A 39) Gab: D 40) Gab: 13

Page 61: curso de específica de física

61

LISTA 13 - CALORIMETRIA

RESUMO DE CONTEÚDO A calorimetria é a ciência que estuda o calor. Calor é uma forma de energia em trânsito, ou seja, é a energia transferida de um corpo com maior temperatura para um corpo de menor temperatura. Em um sistema isolado, o calor é transferido do corpo de maior temperatura para o corpo de menor temperatura até que o equilíbrio térmico seja atingido.

Trocas de calor A definição de calor é usada apenas para indicar a energia que está sendo transferida, e não a energia que o corpo possui. A unidade de medida da quantidade de calor Q no Sistema Internacional (SI) é o joule (J): Calor sensível Quando há variação de temperatura sem que haja variação do estado físico da matéria, dizemos que o calor é sensível. Podemos calcular o calor sensível pela equação:

Q = m.c.T Onde: Q = quantidade de calor m = massa do corpo c = calor específico * T = variação da temperatura

* Calor específico é a quantidade de calor necessária para a variação unitária da temperatura na unidade de massa. O Calor específico é uma grandeza que depende da composição de cada substância.

Substância Calor específico (Cal/gºC)

Chumbo 0,031

Prata 0,056

Ferro 0,11

Água 1,0

Capacidade Térmica É a quantidade de calor que produz no corpo uma variação unitária de temperatura. Matematicamente podemos escrever a capacidade térmica pela equação:

Q = C. T Onde: C = Capacidade Térmica Q = Quantidade de Calor cedida ou recebida pelo corpo T = Variação da Temperatura

No SI, a unidade de medida da capacidade térmica é J/K (joule por Kelvin). Calor Latente Calor latente é a grandeza física que está relacionada à quantidade de calor que um corpo precisa receber ou ceder para mudar de estado físico. Matematicamente, essa definição fica da seguinte forma:

Q = m.L

Onde L é o calor latente da substância e tem como unidade a cal/g. O calor latente pode assumir tanto valores positivos quanto negativos. Se for positivo, quer dizer que o corpo está recebendo calor; se negativo, ele está cedendo calor. Transferências de Calor Como sabemos, o calor é energia térmica em trânsito, e assim sendo, ele pode ser transmitido de um corpo para outro das seguintes maneiras: Condução: é quando o calor se transfere mediante a agitação das partículas que compõem o material, sem que ocorra o transporte de matéria nesse processo. Esse tipo de transferência ocorre, por exemplo, quando alguém segura

uma barra de ferro no fogo. Logo, a pessoa que está segurando a barra notará o aumento de temperatura.

Convecção: esse tipo de transferência ocorre em razão das diferenças de densidades das partes quentes e frias das substâncias envolvidas. Exemplos de convecção são as geladeiras e as brisas do mar.

Radiação: tipo de transferência de calor ocorre através da radiação que é feita por ondas eletromagnéticas (raios infravermelhos), os quais podem se propagar mesmo na ausência de matéria (vácuo). Exemplo desse tipo são os raios solares que aquecem a Terra.

Estufa

Page 62: curso de específica de física

62

EXERCÍCIOS Questão 01 - (UDESC/2011)

Um estudante deseja medir o calor específico de um anel de ouro. O anel é aquecido em um forno e em seguida é colocado em um reservatório com água.

Assinale a alternativa que contém a grandeza física que não é necessária para realizar essa medida:

a) calor específico da água b) massa da água c) temperaturas iniciais da água e do anel d) tempo necessário para atingir o equilíbrio térmico e) temperaturas finais da água e do anel

Questão 02 - (UNESP/2011)

A quantidade de energia informada na embalagem de uma barra de chocolate é igual a 200 kcal. Após o consumo dessa barra, uma pessoa decide eliminar a energia adquirida praticando uma corrida, em percurso plano e retilíneo, com velocidade constante de 1,5 m/s, o que resulta em uma taxa de dissipação de energia de 500 W. Considerando 1 kcal 4 200 J, quantos

quilômetros, aproximadamente, a pessoa precisará correr para dissipar a mesma quantidade de calorias ingeridas ao comer o chocolate?

Questão 03 - (UFLA MG)

Dizemos que o calor latente de fusão da água é 80 cal/g, e sua temperatura de fusão é de 0ºC. Isto significa que a) se fornecermos menos de 80 cal a 1 g de gelo, todo

o gelo continua sólido. b) 1 g de H2O a 0ºC estará necessariamente na fase

sólida. c) fornecendo 80 cal a 1 g de gelo a 0ºC, sua

temperatura aumenta de 1ºC. d) são necessárias 80 cal para derreter totalmente 1 g

de gelo a 0ºC. e) 1 g de gelo a 0ºC possui 80 cal.

Questão 04 - (FMTM MG)

Duas peças metálicas de mesma massa, uma de alumínio (cAl = 0,22 cal/g.ºC) e outra de ferro (cFe = 0,11 cal/g.ºC), recebem iguais quantidades de calor Q e não há trocas de calor com o meio externo. A relação entre as variações da temperatura do alumínio e do ferro Al/Fe será igual a:

a) 0,5. b) 1,0. c) 2,0. d) 3,0.

e) 4,0. Questão 05 - (MACK SP)

Uma fonte térmica fornece calor, à razão constante, a 200 g de uma substância A (calor específico = 0,3 cal/go.C ) e em 3 minutos eleva sua temperatura em 5o C. Essa mesma fonte, ao fornecer calor a um corpo B, eleva sua temperatura em10oC , após 15 minutos. A capacidade térmica do corpo B é: a) 150 cal /oC c) 100 cal /oC e) 50 cal /oC b) 130 cal /oC d) 80 cal /oC

Questão 06 - (Fac. Santa Marcelina SP/2013)

Nos ambientes climatizados artificialmente, os aparelhos de ar condicionado são colocados na parte superior do ambiente, assim como os aquecedores são colocados na parte inferior. Esses posicionamentos são explicados porque, dessa forma,

a) a condução do calor é mais rápida. b) há formação das correntes de convecção. c) a condução do calor é facilitada para todos os

lados. d) as correntes de convecção ficam mais rápidas. e) a radiação do calor se faz uniformemente.

Questão 07 - (FAMECA SP/2013)

Uma das formas mais utilizadas para o aproveitamento da energia solar é o aquecimento da água em edificações residenciais, industriais, comerciais e, principalmente, em hospitais, por meio de aquecedores solares. A figura ilustra o esquema de funcionamento de um aquecedor solar de água.

É correto afirmar que a energia vinda do Sol, por

a) radiação, aquece a água que circula na tubulação

também por radiação. b) radiação, aquece a água que será armazenada

num reservatório adiabático. c) condução, aquece a água que circula na tubulação

por radiação. d) radiação, aquece a água que será armazenada

num reservatório bom condutor térmico. e) condução, aquece a água que circula na tubulação

por convecção. Questão 08 - (IFSP/2013)

Leia o texto a seguir.

Durante o dia, parte da energia solar é captada pela superfície da Terra e absorvida, enquanto a outra parte é irradiada para a atmosfera, de volta para o espaço. Os gases naturais que existem na atmosfera funcionam como uma espécie de capa protetora que impede a dispersão total do calor, o que ajuda a manter o planeta quente. Se esse processo, denominado efeito estufa, não existisse, a temperatura da superfície terrestre seria, em média, cerca de 34ºC mais fria do que é hoje. Portanto, pode-se afirmar que o efeito estufa é imprescindível para a manutenção da vida sobre a Terra.

(www.rudzerhost.com/ambiente/estufa.htm#topo

Acesso em: 22.10.2012. Adaptado.)

Nos últimos séculos, a ação do homem vem promovendo, na atmosfera, um aumento considerável na taxa de dióxido de carbono (CO2), gás importante na produção do efeito estufa. A ação antropogênica, ou seja, a interferência humana sobre o meio ambiente é

Page 63: curso de específica de física

63

apontada como uma das responsáveis pelo aumento, acima do normal, da temperatura no planeta.

Sobre esse assunto assinale a alternativa correta.

a) A destruição da camada de ozônio pelo aumento

de dióxido de carbono na atmosfera é um dos fatores responsáveis pelo efeito estufa.

b) O aumento da concentração de gases, como o gás carbônico, por ação antropogênica se deve principalmente pela queima de combustíveis fósseis e destruição de florestas naturais.

c) Uma das consequências do efeito estufa é a diminuição de absorção de raios solares, o que interfere na fotossíntese, provocando alterações na cadeia alimentar dos diferentes ecossistemas.

d) A ação antropogênica não pode ser considerada um fator de alteração ambiental, uma vez que o efeito estufa existe independentemente da supressão de gases como o CO2 pelo homem.

e) As chuvas ácidas são consequência do efeito estufa e provocam a acidificação de oceanos, interferindo na sobrevivência do fitoplancton marinho.

Questão 09 - (UCS RS/2013)

A operação de uma lâmpada incandescente baseia-se

no aquecimento, por corrente elétrica, de um filamento metálico, até que esse atinja uma determinada temperatura e, por transmissão de energia, seja capaz de sensibilizar sistemas óticos, como um olho ou uma câmera fotográfica. Essa transmissão de energia acontece por

a) convecção. b) radiação. c) condução. d) compressão adiabática. e) fusão.

Questão 10 - (ACAFE SC)

É comum encontrarmos, nos estádios de futebol ou nas praias, vendedores carregando caixas de isopor contendo latinhas com bebidas geladas. Para que elas permaneçam geladas por mais tempo, deve-se colocar: a) gelo ao lado das latinhas com bebida. b) gelo sob as latinhas com bebida. c) as latinhas com bebida em água à temperatura de

0C.

d) somente as latinhas com bebida no isopor. e) gelo sobre as latinhas com bebida.

Questão 11 - (UNIUBE MG)

Quando, numa noite de baixa temperatura, vamos para a cama, n´s a encontramos fria, mesmo que sobre ela estejam vários cobertores de lã. Passado algum tempo nos aquecemos porque a) o cobertor de lã impede a entrada de frio. b) o cobertor de lã não é aquecedor, mas sim um bom

isolante térmico. c) o cobertor de lã só produz calor quando em contato

com o nosso corpo. d) o cobertor de lã não é um bom absorvedor de frio. e) o corpo humano é um bom absorvedor de frio.

Questão 12 - (UFMG)

Existem várias propriedades físicas que variam com a temperatura. Assinale a alternativa que apresenta uma propriedade física que NÃO varia com a temperatura.

a) A massa de mercúrio dentro de um termômetro b) A pressão dentro de um botijão de gás c) A resistência elétrica de um material condutor d) O comprimento de uma barra metálica

Questão 13 - (PUC MG)

Uma garrafa térmica tem paredes prateadas e duplas com vácuo no espaço intermediário. A vantagem de se fabricarem garrafas térmicas assim é porque as paredes prateadas:

a) absorvem o calor e o vácuo é um ótimo isolante térmico. b) são altamente refletoras e o vácuo, um ótimo

isolante térmico. c) absorvem o calor e o vácuo é um excelente

condutor. d) são altamente refletoras e o vácuo é um excelente

condutor. Questão 14 - (ACAFE SC)

É melhor vestir uma roupa na cor _______________ quando caminhar ao longo de uma estrada, à noite, para ter mais segurança, porque ela _______________ melhor a luz dos veículos. A alternativa VERDADEIRA que completa o enunciado

acima, em sequência, é: a) preta - reflete b) branca - absorve c) preta - absorve d) branca - reflete e) neutra - neutraliza

Questão 15 - (MACK SP/2013)

Uma das características meteorológicas da cidade de São Paulo é a grande diferença de temperatura registrada em um mesmo instante entre diversos pontos do município. Segundo dados do Instituto Nacional de Meteorologia, a menor temperatura registrada nessa cidade foi –2 ºC, no dia 2 de agosto de 1955, embora haja algumas indicações, não oficiais, de que, no dia 24 de agosto de 1898, registrou-se a temperatura de –4 ºC. Em contrapartida, a maior temperatura teria sido 37 ºC, medida em 20 de janeiro de 1999. Considerando-se 100 g de água, sob pressão atmosférica normal, incialmente a –4 ºC, para chegar a 37 ºC, a quantidade de Energia Térmica que esta massa deverá receber é

DADOS:

a) 11,3 kcal b) 11,5 kcal c) 11,7 kcal d) 11,9 kcal e) 12,1 kcal

Questão 16 - (UNIOESTE PR/2009)

Um experimento simples para estimar a potência de um forno de micro-ondas é medir a elevação da temperatura de uma certa quantidade de água colocada em seu interior, de forma a permitir o cálculo do calor absorvido durante um certo intervalo de tempo. Suponha que 500 g de água, a temperatura inicial de 20ºC, foram

Page 64: curso de específica de física

64

colocadas no forno e este mantido em funcionamento durante um minuto. Verifica-se que a água atingiu a temperatura de 40ºC. Pode-se afirmar que neste processo a taxa com que a água absorveu energia foi de

(Dados: calor específico da água: -1-1 Cº g cal 1c e

J 4,2cal 1 )

a) 167 W b) 540 W c) 700 W d) 1000 W e) 1200 W

Questão 17 - (UNIOESTE PR/2009)

Deseja-se resfriar 20 litros de chá, inicialmente a 90ºC, até atingir a temperatura de 20ºC. Para atingir este objetivo é colocado gelo, a 0ºC, juntamente com o chá num recipiente termicamente isolado. Considerando para o chá a mesma densidade e o mesmo calor específico da água, a quantidade de gelo que deve ser misturada é

a) 14 kg b) 15,4 kg c) 17,5 kg d) 140 g e) 17,5 g

Questão 18 - (UEL PR/2011)

Um martelo de massa M = 1,2 kg, com velocidade de módulo 6, 5 m/s, golpeia um prego de massa m = 14 g e para, após cada impacto. Considerando que o prego absorve toda a energia das marteladas, uma estimativa do aumento da temperatura do prego, gerado pelo impacto de dez marteladas sucessivas, fornecerá o valor aproximado de: Dado: Calor específico do ferro c = 450J/kgºC

a) 40 ºC b) 57 ºC c) 15 ºK d) 57 ºK e) 15 ºF

Questão 19 - (UEM PR)

Dois corpos de materiais diferentes A e B com mesma massa (mA = mB = m) são colocados em recipientes idênticos, de modo que a mesma quantidade de calor por unidade de tempo (PA = PB = P) é fornecida a ambos. A temperatura (T) em função do tempo (t) para esses corpos é representada na figura a seguir. Considerando esse resultado, assinale o que for correto.

01. Para T < TC, o calor específico de A é maior que o de B.

02. Para T > TC, o calor específico de A é menor que o de B.

04. Em T = TC, o corpo B sofre uma transição de fase, cujo calor latente é P(t3 – t1) / m.

08. O calor específico de B é maior para T < TC que para T > TC.

16. O calor específico de A é cA = 0F

4

TTm

Pt

.

32. Se o processo é realizado a volume constante, então a variação da energia interna de A entre 0 e t4 é

)T(TmcU of

Questão 20 - (UEM PR/2009)

O gráfico abaixo ilustra a variação da quantidade de calor em função da variação da temperatura para duas substâncias diferentes. Analise o gráfico e assinale a(s) alternativa(s) correta(s).

01. Se A e B tiverem massas idênticas, a capacidade térmica de A é maior que a capacidade térmica de B.

02. A capacidade térmica das substâncias depende da

massa das mesmas. 04. A capacidade térmica de B é 3,0 cal/ºC. 08. Se os calores específicos das substâncias forem os

mesmos, a massa de B é maior que a de A. 16. Para aquecer a substância A de 10,0 ºC para 20,0

ºC em 10 minutos, a fonte de calor fornece potência a uma taxa constante de 10,0 cal/min.

GABARITO: 1) Gab: D 2) Gab: 2,52 km 3) Gab: D 4) Gab: A 5) Gab: A 6) Gab: B 7) Gab: B 8) Gab: B 9) Gab: B 10) Gab: E 11) Gab: B 12) Gab: A 13) Gab: B 14) Gab: D 15) Gab: D 16) Gab: C 17) Gab: A 18) Gab: A 19) Gab: CCCECC 20) Gab: 23 21) Gab: 03 22) Gab: A 23) Gab: C 24) Gab: A 25) Gab: A 26) Gab: C 27) Gab: E 28) Gab: 30 29) Gab: B 30) Gab: B 31) Gab: E 32) Gab: C 33) Gab: A 34) Gab: C 35) Gab: B

Page 65: curso de específica de física

65

36) Gab: A 37) Gab: B 38) Gab: E 39) Gab: D 40) Gab: A 41) Gab: A 42) Gab: A 43) Gab: 23 44) Gab: 13 45) Gab:

a) 6600 cal b) 11 g

46) Gab: B 47) Gab: D 48) Gab: A 49) Gab: A 50) Gab: C

Page 66: curso de específica de física

66

LISTA 14 - GASES E TERMODINÂMICA

RESUMO DE CONTEÚDO Os gases representam o estado físico da matéria cujos valores de volume, densidade ou forma própria não são definidos. Apresentam alto grau de desordem causado pelo deslocamento livre das partículas que os constituem (átomos, moléculas ou íons – geralmente moléculas) e são objetos de estudos por possuírem grande aplicabilidade no cotidiano, e por ser a camada material na qual mais mantemos contato, afinal, normalmente todo o nosso corpo fica em contato com gases (ar atmosférico). As propriedades dos gases são variáveis, ou seja, por haver determinados e específicos espaços entre seus constituintes (que podem aumentar ou diminuir) o volume, a densidade, a pressão, a viscosidade podem ser alterados. E, é dessa grande inconstância dos gases, que se deriva o estudo dos gases.

Estudo dos Gases

Por possuírem grande mobilidade, os gases são altamente difusos: tendem a preencher rapidamente todo e qualquer recipiente no qual está contido. O estudo dos gases, em nível acadêmico de ensino médio, restringe-se aos gases ideais ou perfeitos, que são aqueles que apresentam proporção direta entre molaridade, volume, temperatura e pressão de um modo homogêneo e previsível. Dentre todas as propriedades que os gases podem apresentar, seguem as mais usuais:

Pressão: Somatória das forças que cada

constituinte de um gás exerce sobre as paredes de um corpo, ou recipiente, em uma determinada área.

Volume: Espaço ocupado por um gás em um determinado recipiente.

Temperatura: Estado térmico de agitação das partículas de um gás.

E, a essas variáveis (sobre gases ideais) são apresentadas as seguintes fórmulas:

1. Lei de Boyle-Mariotte -> PV = K 2. Lei de Charles -> VT-¹ = K 3. Lei de Gay-Lussac -> PT-¹ = K

Essas leis significam a constância dos gases perfeitos nas variáveis: pressão (P), volume (V) e temperatura (T); opondo-se aos gases reais, onde essas leis não se aplicam. Obs.: T-1 = 1/T

Lei de Boyle-Mariotte

A primeira lei dos gases informa que o produto pressão-volume de um gás ideal é constante para certa temperatura e molaridade. Ou seja, mantendo-se a massa de gás e temperatura constantes, aumentando ou diminuindo-se a pressão (ou volume), diminui-se ou aumenta-se o volume (ou pressão), respectivamente, em uma relação inversamente proporcional.

Ex.: Se, a 1 atm de pressão, um gás apresenta 2 l de volume. A 2 atm de pressão, o mesmo gás terá 1 l de volume, de fato que: 1.2 = 2.1 = K

Lei de Charles

A segunda lei dos gases mostra que o produto entre o volume e o inverso da temperatura é constante para a mesma massa de gás e pressão. De modo que, se a uma temperatura de

298 K (ou 25°C), determinado gás possui 2l de volume, a 320 K o mesmo gás terá volume proporcional, de modo que: 2.298-1 = V.320-1 V = 320.2.298-1 V = 2,15 l Ou seja, após aumentar a temperatura em 22K, o volume aumenta em 0,15 l.

Lei de Gay-Lussac

A última lei dos gases determina que o produto entre a pressão e o inverso da temperatura de um gás é constante para um dada massa e volume constantes. Ex.: Se determinado gás a 298 K possui pressão igual a 3 atm, à 100 K essa pressão será igual a: 3.298-1 = P2.100-1 P2 = 3.100.298-1 P2 = 1,006 atm

Lei dos Gases Perfeitos e Equação de Clapeyron

Unificando-se as três leis dos gases ideais, tem-se a lei dos

gases perfeitos: PV / T = K E, ainda, adicionando a relação de Avogadro, onde a massa de um gás é proporcional à sua quantidade de matéria, tem-se: PV / nT = R E, a esse R foi dado a conotação de constante dos gases perfeitos, donde deriva a equação de Clapeyron:

PV = nRT Sendo: P = pressão que o gás se encontra, em atm; V = volume do recipiente onde o gás está contido, em l; n = quantidade de matéria do gás, em mol; R = constante dos gases perfeitos, em atm.l.mol-1.K-1; T = temperatura do gás, em K. EXERCÍCIOS Questão 01 - (UNIOESTE PR/2008)

Um sistema termodinâmico percorre o caminho

CBA representado no diagrama PV abaixo.

Assinale a alternativa correta.

a) A transformação CB é isocórica.

b) O trabalho realizado pelo sistema no percurso

BA é de J10 x 0,5 4 .

c) Se a temperatura do sistema no ponto A for de 300 K, no ponto B será de 150 K.

Page 67: curso de específica de física

67

d) Se a transformação CB for adiabática, o sistema

não trocará calor com o meio externo nessa transformação.

e) O ciclo ACBA pode ser fechado com uma

transformação isotérmica. Questão 02 - (UNIOESTE PR/2006)

O gráfico abaixo mostra o comportamento da temperatura de uma amostra de óleo em função do tempo de aquecimento. O volume da amostra de óleo é de 1,0 litro e a pressão é mantida constante durante todo o período de aquecimento. Ao final de 10 minutos de aquecimento verifica-se que ainda restavam 500 g de óleo no estado líquido. Sabe-se que a densidade do óleo vale 0,60 g/cm3 e que a amostra recebe calor a uma taxa constante e de valor igual a 60 cal/s. Assinale a(s) alternativa(s) correta(s).

01. O calor específico do óleo vale 0,4 cal/(g ºC). 02. A taxa de aquecimento da amostra de óleo pode ser

expressa como 3,6 x 103 cal/min. 04. A capacidade térmica da amostra tem o valor de 240

cal/g. 08. O calor latente de vaporização do óleo testado vale

216 cal/g. 16. Caso seja sempre mantida a mesma taxa de

aquecimento da amostra, após 40 minutos do início do aquecimento, todo o óleo terá sido vaporizado.

32. Caso seja duplicada a taxa de aquecimento da amostra, após 10 minutos do início do aquecimento, todo o óleo terá sido vaporizado.

64. Durante o intervalo de tempo em que houve variação de temperatura da amostra, a mesma recebeu uma quantidade total de calor de 1,44 x 104 calorias.

Questão 03 - (UNIOESTE PR/2008)

Um cilindro hermeticamente fechado de volume V contém um gás ideal à pressão P e temperatura T. Nessas condições iniciais, a velocidade média das moléculas do gás é vm. Por meio de dispositivos conectados ao cilindro, pode-se modificar os valores do volume, da pressão e da temperatura. Assinale a alternativa que mostra novos valores desses parâmetros: V1, P1, T1, com os quais a velocidade média das moléculas do gás dobra.

a) T/2 T P/2; P V/2; V 111

b) T T P; P V/4; V 111

c) 4T T 4P; P V; V 111

d) 2T T 2P; P V; V 111

e) 2T T 2P; P 2V; V 111

TEXTO: 2 - Comum à questão: 4

Considere a montagem a seguir, que mostra um gás ideal em equilíbrio em um recipiente cilíndrico. Uma mola de constante elástica k tem uma de suas extremidades presa a um suporte rígido, e a outra extremidade está presa ao êmbolo do recipiente. A massa do êmbolo é desprezível e, na situação descrita, a mola não está comprimida nem alongada.

Questão 04 - (PUC MG/2013)

Considerando-se o recipiente com 128g de oxigênio à pressão de 1 atm e temperatura de 27ºC, o volume ocupado pelo gás será de aproximadamente:

a) 1,0 m3 b) 400 litros c) 8,3 m3 d) 100 litros

Questão 05 - (UEL PR)

Um freezer é programado para manter a temperatura em seu interior a –19ºC. Ao ser instalado, suponha que a temperatura ambiente seja de 27ºC. Considerando que o sistema de fechamento da porta a mantém

hermeticamente fechada, qual será a pressão no interior do freezer quando ele tiver atingido a temperatura para a qual foi programado? a) 0,72atm b) 0,78atm c) 0,85atm d) 0,89atm e) 0,94atm

Questão 06 - (UEL PR)

Certa quantidade de gás perfeito sofre uma transformação cíclica, passando pelos estados 1, 2, 3, 4, 5 e 1, conforme está indicado no diagrama pV.

Page 68: curso de específica de física

68

p(atm)

V(litros)

1 2

3

45

7

6

5

4

3

2

1

1 2 3 4 5 6 7 80

O trabalho realizado pelo gás nessa transformação cíclica, medido em atm.L, é um valor mais próximo de a) 50 b) 40 c) 30 d) 20 e) 10

Questão 07 - (UEL PR/2010)

Os diagramas PV a seguir representam o comportamento de um gás:

É correto afirmar:

a) O diagrama (a) representa um processo isotérmico

com a temperatura inicial maior que a temperatura final.

b) Os diagramas (a) e (b) resultam no mesmo trabalho realizado pelo sistema após a expansão.

c) O diagrama (b) representa um processo adiabático. d) O diagrama (c) representa um processo isobárico. e) O diagrama (c) representa um processo de

expansão. Questão 08 - (UEL PR)

O gráfico abaixo, que relaciona a pressão com o volume, apresenta três evoluções de gases, conforme as curvas I, II e III. Sobre essas evoluções, é correto afirmar:

p

v

II

II

I

a) A evolução III é isotérmica. b) Na evolução I o gás cedeu calor. c) O trabalho realizado pelo gás na evolução I é maior

que o trabalho realizado na evolução II. d) Na evolução I a temperatura diminuiu. e) O trabalho na evolução II é negativo.

Questão 09 - (UEM PR/2012)

Um recipiente de volume igual a 1,2 m3 contém uma amostra de gás ideal à temperatura de 27ºC e à pressão de 4,98.104 N/m2. Considerando R = 8,3Jmol-1K-1 e k = 1,4.10-23JK-1 e o número de Avogrado igual a 6.1023, assinale o que for correto.

01. A quantidade de mols dessa amostra gasosa é de

24mols. 02. O número total de moléculas dessa amostra

gasosa é de 1,44.1025 moléculas.

04. A energia cinética média de cada uma das moléculas da amostra gasosa é de 6,3.10-21J.

08. Se a temperatura da amostra gasosa for aumentada de 27ºC para 54ºC, a pressão terá seu valor aumentado em 100%, mantendo-se inalterados o volume e o número de mols.

16. Se o número de mols for duplicado, a pressão terá seu valor duplicado, se se mantiverem inalterados o volume e a temperatura.

Questão 10 - (UEM PR)

Um êmbolo de espessura desprezível, que pode mover-se livremente, divide, inicialmente, um cilindro de 240 cm de comprimento em duas partes iguais A e B (figura). O compartimento A contém um gás sob pressão de 2 atm, e o B contém o mesmo gás, à mesma temperatura, sob pressão de 1atm. Considerando que, ao soltar o êmbolo, ocorre um processo isotérmico, a distância, em centímetros, entre a posição inicial do êmbolo e a posição final é...

Questão 11 - (UEM PR)

Considere, sobre a superfície da Terra, uma coluna vertical com paredes adiabáticas preenchida por uma gás ideal. Baseando-se nessas informações e considerando que o gás está em equilíbrio térmico sob a ação do campo gravitacional terrestre, assinale o que for correto.

Page 69: curso de específica de física

69

01. As partes extremas superior e inferior do gás estão sujeitas a pressões diferentes.

02. A pressão na parte extrema superior não é maior que a da parte extrema inferior.

04. A temperatura numa região superior do gás é menor que a de outra inferior.

08. A temperatura T, a pressão P, o volume V e o número de moles n do gás estão relacionados pela equação PV = nRT, onde R é a constante universal dos gases perfeitos.

16. As partes extremas superior e inferior do gás têm a mesma densidade.

Questão 12 - (UEM PR)

São realizados três experimentos com um gás ideal. No primeiro, o volume do gás é duplicado, enquanto sua pressão permanece constante. No segundo, a partir das mesmas condições iniciais, a pressão é duplicada, enquanto o volume permanece constante e, no terceiro experimento, a partir das mesmas condições iniciais, o volume é duplicado, enquanto a temperatura permanece constante. Assinale o que for correto. 01. Nos três experimentos, foi fornecido calor ao gás. 02. Nos três experimentos, foi realizado trabalho pelo

gás. 04. Nos três experimentos, a energia interna do gás

aumentou.

08. Nos experimentos 1 e 2, a variação de energia interna do gás é a mesma.

16. Nos experimentos 1 e 3, o trabalho realizado pelo gás é o mesmo.

Questão 13 - (UFPR/2013)

Segundo o documento atual da FIFA “Regras do Jogo”, no qual estão estabelecidos os parâmetros oficiais aos quais devem atender o campo, os equipamentos e os acessórios para a prática do futebol, a bola oficial deve ter pressão entre 0,6 e 1,1 atm ao nível do mar, peso entre 410 e 450 g e circunferência entre 68 e 70 cm. Um dia antes de uma partida oficial de futebol, quando a temperatura era de 32 ºC, cinco bolas, identificadas pelas letras A, B, C, D e E, de mesma marca e novas foram calibradas conforme mostrado na tabela ao lado:

00,1E

90,0D

80,0C

70,0B

60,0A

(atm)

PressãoBola

No dia seguinte e na hora do jogo, as cinco bolas foram levadas para o campo. Considerando que a temperatura ambiente na hora do jogo era de 13 ºC e supondo que o volume e a circunferência das bolas tenham se mantido constantes, assinale a alternativa que apresenta corretamente as bolas que atendem ao documento da FIFA para a realização do jogo.

a) A e E apenas. b) B e D apenas. c) A, D e E apenas. d) B, C, D e E apenas. e) A, B, C, D e E.

Questão 14 - (ESCS DF/2013)

Abaixo, está representado um ciclo de ventilação pulmonar idealizado. Durante o início da inspiração, a

pressão do gás dentro dos pulmões é mínima, havendo acréscimo da pressão interna e do volume com a entrada de ar. Assim que o pulmão está em sua capacidade máxima, há um período curto de pausa, em que trocas gasosas são realizadas a um volume constante, o que reduz a pressão. A pressão interna começa a decrescer à medida que há esvaziamento parcial da câmara, e atinge pressão mínima.

Internet: <www.livemedical.net/respiratory-mechanics>

(com adaptações).

Entre as opções a seguir, assinale a que melhor representa o ciclo de ventilação descrito acima.

a)

b)

c)

d)

e)

Questão 15 - (PUC RJ/2013)

Um sistema termodinâmico recebe certa quantidade de calor de uma fonte quente e sofre uma expansão isotérmica indo do estado 1 ao estado 2, indicados na figura. Imediatamente após a expansão inicial, o sistema sofre uma segunda expansão térmica, adiabática, indo de um estado 2 para o estado 3 com coeficiente de Poisson = 1,5.

Page 70: curso de específica de física

70

a) Determine o volume ocupado pelo gás após a primeira expansão, indo do estado 1 ao estado 2.

b) Determine a pressão no gás quando o estado 3 é atingido.

Questão 16 - (UCS RS/2013)

Os motores a combustão, como o dos automóveis movidos a gasolina ou a álcool, são classificados como máquinas térmicas que, operando em ciclos, entre fontes de calor quentes e frias, e recebendo e liberando fluídos operantes, produzem trabalho. Suponha um motor a combustão hipotético que possua um gás ideal como fluído operante e que nunca o troque. As transformações de estado desse gás ideal, durante um ciclo de operação do motor, estão representadas no gráfico pressão X Volume abaixo. Conclui-se que o gás ideal

a) tem, durante todo o ciclo, a mesma temperatura. b) tem, durante todo o ciclo, o mesmo volume. c) gera quantidade de calor liberado mais trabalho

executado maior do que a quantidade de calor recebido.

d) tem, durante todo o ciclo, sua pressão variando. e) mantém o produto da sua pressão pelo seu volume

constante durante todo o ciclo, de modo que sua temperatura sempre varie.

Questão 17 - (UEM PR/2013)

Com relação ao comportamento térmico dos gases ideais, assinale o que for correto.

01. Mantendo-se a temperatura de uma amostra de

gás ideal constante, a variação da pressão dessa amostra é inversamente proporcional à variação de seu volume.

02. Mantendo-se a pressão de uma amostra de gás ideal constante, a temperatura da amostra é inversamente proporcional ao volume dessa amostra.

04. Em condições idênticas de temperatura e pressão, gases que ocupam volumes idênticos possuem o mesmo número de partículas.

08. A energia cinética média das partículas de uma amostra de gás ideal é diretamente proporcional à temperatura da amostra.

16. Mantendo-se o volume constante, a pressão de uma amostra de gás ideal é diretamente proporcional à velocidade média das partículas desse gás elevada ao quadrado.

Questão 18 - (UNICAMP SP/2013)

A boa ventilação em ambientes fechados é um fator importante para o conforto térmico em regiões de clima quente. Uma chaminé solar pode ser usada para aumentar a ventilação de um edifício. Ela faz uso da energia solar para aquecer o ar de sua parte superior, tornando-o menos denso e fazendo com que ele suba, aspirando assim o ar dos ambientes e substituindo-o por ar vindo do exterior.

a) A intensidade da radiação solar absorvida por uma

placa usada para aquecer o ar é igual a 400 W/m2. A energia absorvida durante 1,0 min por uma placa de 2 m2 é usada para aquecer 6,0 kg de ar.

O calor específico do ar é Cºkg

J1000c . Qual é a

variação de temperatura do ar nesse período? b) A densidade do ar a 290 K é = 1,2 kg/m3.

Adotando-se um número fixo de moles de ar mantido a pressão constante, calcule a sua densidade para a temperatura de 300 K. Considere o ar como um gás ideal.

Questão 19 - (UEG GO/2013)

Dentro de um cilindro com pistão móvel está confinado um gás monoatômico. Entre a parte superior, fixa, do cilindro e o pistão existe uma barra extremamente fina de metal, de comprimento l0, com coeficiente de dilatação linear , ligada por um fio condutor de calor a

uma fonte térmica. A barra é aquecida por uma temperatura que provoca uma dilatação linear l,

empurrando o pistão que comprime o gás. Como a área da base do cilindro é A e o sistema sofre uma transformação isobárica a uma pressão , o trabalho

realizado é igual a:

a) Al0 b) A22l02

c) 2Al0

d) (Al0)/2

Questão 20 - (UEPG PR/2013)

Certa massa de gás é confinada por um êmbolo no interior de um recipiente a uma temperatura ,

conforme é mostrado abaixo. Nesse contexto, assinale o que for correto.

01. Se os coeficientes de dilatação do êmbolo e do cilindro forem iguais e todo sistema receber uma quantidade de calor, o gás se expande e realizará trabalho sobre o êmbolo.

02. Sendo o êmbolo e o cilindro metálicos e seus coeficientes de dilatação o dobro um do outro, respectivamente, e estando perfeitamente ajustados, e ocorrendo o aquecimento de todo sistema, o gás não se dilata, porém haverá aumento da pressão.

04. O gás cedendo calor quando o sistema receber trabalho, sua energia interna diminui.

08. O estudo dos gases é realizado com base em três grandezas: o volume, a temperatura e a pressão.

Page 71: curso de específica de física

71

Na sua dilatação apresentam o mesmo valor para o coeficiente de dilatação.

16. Ao aplicar uma força no êmbolo, puxando-o para cima, o volume do gás aumenta permanecendo constante as demais variáveis.

GABARITO: 1) Gab: D 2) Gab: 91 3) Gab: C 4) Gab: D 5) Gab: C 6) Gab: E 7) Gab: E 8) Gab: C 9) Gab: 23 10) Gab: não fornecido pela Universidade 11) Gab: CCEEE 12) Gab: CEECE 13) Gab: D 14) Gab: E 15) Gab:

a) V2 = 5 10–5 m3

b) P3 = 2,3 105 Pa

16) Gab: D 17) Gab: 29 18) Gab:

a) = 8 ºC b) 1 = 1,16 kg/m3

19) Gab: A 20) Gab: 15 21) Gab:

AB = 2,8 105 J

UDA = 2 104 J

22) Gab: C 23) Gab: U = 971,8 J.

24) Gab: 08 25) Gab: 14 26) Gab: E 27) Gab: C 28) Gab: 31 29) Gab: D 30) Gab:

a) A partir das informações contidas no gráfico:

Comparando-se os estados termodinâmicos a e c, tem-se:

Procedendo às devidas substituições numéricas:

b) Como a energia interna do gás (U) é dada por

nRT2

3U , e sendo Ta = Tc, conclui-se que Ua =

Uc.

31) Gab: D 32) Gab: A 33) Gab: D 34) Gab: C 35) Gab:

a) 81012 partículas/m3

b) Psub/Pnave = 10 36) Gab: B 37) Gab: D 38) Gab: B 39) Gab: 16 40) Gab: D

Page 72: curso de específica de física

72

LISTA 15 - TERMODINÂMICA

RESUMO DE CONTEÚDO A termodinâmica é o ramo da física que estuda as relações entre o calor trocado, representado pela letra Q, e o trabalho realizado, representado pela letra τ, num determinado processo físico que envolve a presença de um corpo e/ou sistema e o meio exterior. É através das variações de temperatura, pressão e volume, que a física busca compreender o comportamento e as transformações que ocorrem na natureza. Calor é energia térmica em trânsito, que ocorre em razão das diferenças de temperatura existentes entre os corpos ou sistemas envolvidos. Energia é a capacidade que um corpo tem de realizar trabalho. A termodinâmica tem como principais pontos o estudo de duas leis, que são - Primeira Lei da Termodinâmica: essa lei diz que a variação da energia interna de um sistema pode ser expressa através da diferença entre o calor trocado com o meio externo e o trabalho realizado por ele durante uma determinada transformação.

As transformações que são estudadas na primeira lei da

termodinâmica são: Transformação isobárica: ocorre à pressão constante, podendo variar somente o volume e a temperatura; Transformação isotérmica: ocorre à temperatura constante, variando somente as grandezas de pressão e volume; Transformação isocórica ou isovolumétrica: ocorre à volume constante, variando somente as grandezas de pressão e temperatura Transformação adiabática: é a transformação gasosa na qual o gás não troca calor com o meio externo, seja porque ele está termicamente isolado ou porque o processo ocorre de forma tão rápida que o calor trocado é desprezível. - Segunda Lei da Termodinâmica: enunciada pelo físico francês Sadi Carnot, essa lei faz restrições para as transformações realizadas pelas máquinas térmicas como, por exemplo, o motor de uma geladeira. Seu enunciado, segundo Carnot, diz que: Para que um sistema realize conversões de calor em trabalho, ele deve realizar ciclos entre uma fonte quente e fria, isso de forma contínua. A cada ciclo é retirada uma quantidade de calor da fonte quente, que é parcialmente

convertida em trabalho e a quantidade de calor restante é rejeitada para a fonte fria.

EXERCÍCIOS

Questão 01 - (UNIOESTE PR/2009)

A termodinâmica sistematiza as leis empíricas sobre o comportamento térmico dos corpos macroscópicos e obtém seus conceitos diretamente dos experimentos. Tendo como base as leis da termodinâmica, analise as seguintes proposições:

I. Uma transformação adiabática é aquela em que o

sistema não troca calor com a vizinhança. Então o trabalho realizado pelo sistema é feito à custa da diminuição da energia interna do sistema.

II. Em uma máquina térmica a energia térmica é integralmente transformada em trabalho.

III. É impossível a energia térmica fluir espontaneamente de um sistema mais frio para um sistema mais quente.

IV. O ciclo de Carnot é um ciclo reversível ideal com o qual seria possível obter o máximo rendimento.

Estão corretas

a) I e II b) I e IV c) III e IV d) I, III e IV e) Todas

Questão 02 - (UFPR)

Considere um cilindro de paredes termicamente isoladas, com exceção da base inferior, que é condutora de calor. O cilindro está munido de um êmbolo de área 0,01 m2 e peso 25 N, que pode mover-se sem atrito. O êmbolo separa o cilindro em uma parte superior, onde existe vácuo, e uma parte inferior, onde há um gás ideal, com 0,01 mol e volume inicial de 10 litros. À medida que o gás é aquecido, o êmbolo sobe até uma altura máxima de 0,1 m, onde um limitador de curso o impede de subir mais. Em seguida, o aquecimento prossegue até que a pressão do gás duplique. Com base nessas informações, é correto afirmar:

Page 73: curso de específica de física

73

01. Enquanto o êmbolo estiver subindo, o processo é isobárico.

02. Após o êmbolo ter atingido o limitador, o processo é adiabático.

04. O trabalho realizado no trecho de expansão do gás é de 2,5 J.

08. A temperatura no instante inicial é igual a 402 K. 16. O calor fornecido ao gás, na etapa de expansão, é

utilizado para realizar trabalho e para aumentar a temperatura do gás.

32. O trabalho realizado pelo gás durante a etapa de expansão é igual ao trabalho total realizado pelo gás desde o início do aquecimento até o momento em que o gás atinge o dobro da pressão inicial.

Questão 03 - (UFPR)

No século XVII, uma das interpretações para a natureza do calor considerava-o um fluido ponderável que preenchia os espaços entre os átomos dos corpos quentes. Essa interpretação explicava corretamente alguns fenômenos, porém falhava em outros. Isso motivou a proposição de uma outra interpretação, que teve origem em trabalhos de Mayer, Rumford e Joule, entre outros pesquisadores. Com relação aos conceitos de temperatura, calor e trabalho atualmente aceitos pela Física, avalie as seguintes afirmativas:

I. Temperatura e calor representam o mesmo conceito

físico. II. Calor e trabalho estão relacionados com

transferência de energia. III. A temperatura de um gás está relacionada com a

energia cinética de agitação de suas moléculas.

Assinale a alternativa correta. a) Somente as afirmativas II e III são verdadeiras. b) Somente a afirmativa I é verdadeira. c) Somente a afirmativa II é verdadeira. d) Somente a afirmativa III é verdadeira. e) Somente as afirmativas I e II são verdadeiras.

Questão 04 - (UFPR/2008)

Os estudos científicos desenvolvidos pelo engenheiro francês Nicolas Sadi Carnot (1796–1832) na tentativa de melhorar o rendimento de máquinas térmicas serviram de base para a formulação da segunda lei da termodinâmica. Acerca do tema, considere as seguintes afirmativas: 1. O rendimento de uma máquina térmica é a razão

entre o trabalho realizado pela máquina num ciclo e o calor retirado do reservatório quente nesse ciclo.

2. Os refrigeradores são máquinas térmicas que transferem calor de um sistema de menor temperatura para outro a uma temperatura mais elevada.

3. É possível construir uma máquina, que opera em ciclos, cujo único efeito seja retirar calor de uma fonte e transformá-lo integralmente em trabalho.

Assinale a alternativa correta. a) Somente as afirmativas 1 e 3 são verdadeiras.

b) Somente a afirmativa 1 é verdadeira. c) Somente a afirmativa 2 é verdadeira. d) Somente as afirmativas 1 e 2 são verdadeiras. e) Somente as afirmativas 2 e 3 são verdadeiras.

Questão 05 - (UEL PR/2008)

Considere um sistema termodinâmico e analise as seguintes afirmativas. I. Para que a entropia decresça quando um gás ideal

sofre uma expansão adiabática livre, indo de um volume v1 para um volume v2, v2 deve ser maior que v1.

II. No nível molecular, a temperatura é a grandeza que mede a energia cinética média de translação das moléculas de um gás monoatômico e a primeira lei da Termodinâmica nos permite definir a energia interna U do sistema.

III. Um processo é irreversível, em termos termodinâmicos, graças à dissipação de sua energia e à variação positiva de sua entropia.

IV. A segunda lei da Termodinâmica pode ser enunciada da seguinte forma: a entropia do universo sempre cresce (ou permanece constante, em um processo reversível).

Assinale a alternativa que contém todas as afirmativas corretas.

a) I e II. b) I e III. c) II e IV. d) I, III e IV. e) II, III e IV.

Questão 06 - (UEL PR/2008)

Na parte traseira das geladeiras é onde, em geral, os fabricantes colocam uma grade preta sustentando uma serpentina da mesma cor. Qual é o estado do fluido de refrigeração neste setor da geladeira? a) Líquido, alta pressão, alta temperatura. b) Líquido, baixa pressão, alta temperatura. c) Líquido, pressão atmosférica, baixa temperatura. d) Gás, alta pressão, baixa temperatura. e) Gás, pressão atmosférica, alta temperatura.

Questão 07 - (UEM PR/2012)

Sobre as transformações termodinâmicas que podem ocorrer com um gás ideal confinado em um cilindro com pistão, assinale o que for correto.

01. Um gás ideal realiza trabalho ao se expandir,

empurrando o pistão contra uma pressão externa. 02. Em uma transformação adiabática ocorre troca de

calor com a vizinhança. 04. A energia interna de uma amostra de gás ideal não

varia, quando este sofre uma transformação isovolumétrica.

08. Quando o gás ideal sofre uma compressão, o trabalho é realizado por um agente externo sobre o gás ideal.

16. O gás ideal não realiza trabalho em uma transformação isovolumétrica.

Questão 08 - (UEM PR)

Com relação aos conceitos de calor e temperatura, pode-se afirmar que: 01. é possível transferir calor para um corpo, sem

provocar aumento de sua temperatura. 02. é possível variar a temperatura de um corpo, sem

que haja transferência de calor para ele.

isolantetérmico

vácuo

limitadorde curso

êmbolo

gás

chama

h

Page 74: curso de específica de física

74

04. calor e temperatura são quantidades equivalentes de energia.

08. a temperatura de um corpo pode ser medida pelo grau de agitação das moléculas que o constituem.

16. um corpo a uma alta temperatura, quando colocado em contato com outro, a uma baixa temperatura, faz com que haja um fluxo de calor do mais quente para o mais frio.

Questão 09 - (UEM PR)

Tendo por base a primeira lei da termodinâmica e as transformações abaixo relacionadas, assinale a(s) alternativa(s) em que a energia do sistema aumenta. 01. Um sistema recebe trabalho sem fornecer calor. 02. Um sistema recebe calor sem fornecer trabalho. 04. Um sistema recebe calor e trabalho ao mesmo

tempo. 08. Um sistema fornece trabalho sem receber calor. 16. Um sistema fornece calor sem receber trabalho.

Questão 10 - (UEG GO/2013)

Um refrigerador é, em essência, um tipo de máquina térmica que retira calor dos alimentos e envia-o para o meio ambiente. Para que consiga realizar esta tarefa, porém, ele precisa realizar um trabalho através de um gás. Esta máquina térmica se contrapõe ao fato de que

o calor

a) sempre flui espontaneamente do corpo mais quente para o corpo mais frio.

b) pode ser utilizado para realizar trabalho em líquidos e gases.

c) nunca é trocado entre corpos com temperaturas diferentes.

d) é energia térmica em trânsito e, por isso, os gases conseguem realizar trabalho.

Questão 11 - (UEM PR/2013)

Sobre os conceitos de termodinâmica, assinale o que for correto.

01. Estando em um sistema isolado, dois corpos A e

B, um com maior temperatura do que o outro, quando colocados em contato, após certo intervalo de tempo, os dois entrarão em equilíbrio térmico, isto é, estarão a uma mesma temperatura.

02. Em um sistema isolado, a energia total desse sistema permanece inalterada.

04. Em um sistema isolado, a entropia desse só pode aumentar ou manter-se constante.

08. Não é possível realizar um processo em que o único efeito seja retirar certa quantidade de calor de um corpo com temperatura menor e transferir para um corpo com temperatura maior.

16. A quantidade de calor retirada de uma fonte de calor por uma máquina térmica que opera em ciclos pode ser convertida totalmente em trabalho.

Questão 12 - (UNIUBE MG/2013)

Dentre as duas leis da termodinâmica, a segunda é a que tem maior aplicação na construção de máquinas e utilização na indústria, pois trata diretamente do rendimento das máquinas térmicas. Dois enunciados, aparentemente diferentes, ilustram a 2ª Lei da Termodinâmica, os enunciados de Clausius e KelvinPlanck:

• Enunciado de Clausius: O calor não pode fluir, de forma espontânea, de um

corpo de temperatura menor, para um outro corpo de temperatura mais alta.

• Enunciado de KelvinPlanck: É impossível a construção de uma máquina que, operando em um ciclo termodinâmico, converta toda a quantidade de calor recebido em trabalho.

Fonte: http://www.sofi sica.com.br/conteudos/Termologia/ Termodinamica/2leidatermodinamica.php

Sobre as definições acima e sobre a segunda lei da termodinâmica são feitas algumas afirmações:

I. O enunciado de Clausius determina o sentido

natural do fluxo de calor que vai da temperatura mais alta para a mais baixa, e, para que o fluxo seja inverso, é necessário que um agente externo realize um trabalho sobre esse sistema, tal como o que ocorre nos refrigeradores.

II. O enunciado de Kelvin Planck implica que é possível existir um dispositivo térmico que tenha um rendimento de 100%.

III. Em certa situação, um motor a vapor realiza um trabalho de 24kJ. Quando lhe foi fornecida uma quantidade de calor igual a 46kJ, o rendimento esperado dessa máquina foi de 100%, pois ela transformou integralmente calor em trabalho útil.

IV. No ciclo de Carnot, o rendimento máximo teórico de uma máquina a vapor, cujo fluido entra a 500ºC e abandona o ciclo a 300ºC, foi de,

aproximadamente, 26%.

Estão CORRETAS as afirmações contidas em:

a) I e IV, apenas b) I e III, apenas c) I e II, apenas d) II e IV, apenas e) III e IV, apenas

Questão 13 - (UFPR/2012)

Segundo a teoria cinética, um gás é constituído por moléculas que se movimentam desordenadamente no espaço do reservatório onde o gás está armazenado. As colisões das moléculas entre si e com as paredes do reservatório são perfeitamente elásticas. Entre duas colisões sucessivas, as moléculas descrevem um MRU. A energia cinética de translação das moléculas é diretamente proporcional à temperatura do gás. Com base nessas informações, considere as seguintes afirmativas:

1. As moléculas se deslocam todas em trajetórias

paralelas entre si. 2. Ao colidir com as paredes do reservatório, a

energia cinética das moléculas é conservada. 3. A velocidade de deslocamento das moléculas

aumenta se a temperatura do gás for aumentada.

Assinale a alternativa correta.

a) Somente a afirmativa 1 é verdadeira. b) Somente a afirmativa 2 é verdadeira. c) Somente a afirmativa 3 é verdadeira. d) Somente as afirmativas 1 e 2 são verdadeiras. e) Somente as afirmativas 2 e 3 são verdadeiras.

Questão 14 - (UEFS BA/2012)

Tomar chá preto, a 80°C, com uma quantidade de leite é hábito bastante comum entre os londrinos. O valor dessa temperatura em ºF (Fahrenheit), que é o sistema utilizado na Inglaterra, é, aproximadamente,

a) 165 b) 169

Page 75: curso de específica de física

75

c) 172 d) 176 e) 180

Questão 15 - (UFMS/2010)

A figura da esquerda mostra um êmbolo no interior de um cilindro que está contido no interior de uma câmara. O cilindro está imerso em água com gelo, e a câmara isola termicamente todo o sistema das vizinhanças. O ar contido no interior do cilindro está em equilíbrio térmico com todo o sistema a 0° C e sua pressão é igual à pressão atmosférica externa. O cilindro pode trocar calor apenas com a água, o ar e o gelo. Em seguida, é colocado um tijolo bruscamente sobre o êmbolo, comprimindo rapidamente o ar no interior do cilindro. Após um certo tempo, todo o sistema água e gelo volta novamente ao equilíbrio térmico de 0° C, mas a pressão do ar, no interior do cilindro, fica maior que a pressão atmosférica. Com fundamentos na termodinâmica e considerando que o ar é um gás ideal e que não há vazamentos, é correto afirmar:

01. O produto da pressão do ar pelo volume que ele ocupa é igual nas duas situações de equilíbrio.

02. Na situação representada pela figura da direita, existe menos massa de gelo que na situação representada pela figura da esquerda.

04. A partir da situação representada pela figura da esquerda, até a situação representada pela figura da direita, a transformação sofrida pelo ar pode ser compreendida por dois processos termodinâmicos, o primeiro adiabático e o segundo isobárico.

08. A partir da situação representada pela figura da esquerda até a situação representada pela figura da direita, a temperatura do ar permaneceu sempre constante.

16. Não haverá troca de calor entre o cilindro e a água, mesmo depois de jogar o tijolo e esperar atingir o novo equilíbrio.

Questão 16 - (FMABC/2013)

Determinada máquina térmica foi projetada para operar realizando o ciclo de Carnot. Quando em operação, o trabalho útil fornecido pela máquina, a cada ciclo, é de 3200J. As temperaturas das fontes térmicas são 427ºC e 77ºC, respectivamente. Nestas condições, a quantidade de calor retirada da fonte quente, a quantidade de calor rejeitada para a fonte fria e o

rendimento da máquina térmica são, respectivamente, iguais a: Adote 4J = 1cal

a) 3900J, 700J, 82% b) 6400J, 3200J, 50% c) 3200J, 6400J, 50% d) 700J, 3900J, 82% e) 1600J, 3200J, 50%

Questão 17 - (IFSP/2013)

A Lei da Conservação da Energia assegura que não é possível criar energia nem a fazer desaparecer. No funcionamento de determinados aparelhos, a energia é conservada por meio da transformação de um tipo de

energia em outro. Em se considerando um telefone celular com a bateria carregada e em funcionamento, durante uma conversa entre duas pessoas, assinale a alternativa que corresponde à sequência correta das possíveis transformações de energias envolvidas no celular em uso.

a) Térmica – cinética – sonora. b) Química – elétrica – sonora. c) Cinética – térmica – elétrica. d) Luminosa – elétrica – térmica. e) Química – sonora – cinética.

TEXTO: 1 - Comum à questão: 18

Equipe de cientistas descobre o primeiro exoplaneta habitável

O primeiro exoplaneta habitável foi encontrado depois de observações que duraram 11 anos, utilizando uma mistura de técnicas avançadas e telescópios convencionais. A equipe descobriu mais dois exoplanetas orbitando em volta da estrela Gliese 581. O mais interessante dos dois exoplanetas descobertos é o Gliese 581g, com uma massa três vezes superior à da Terra e um período orbital (tempo que o planeta leva para dar uma volta completa em torno de sua estrela) inferior a 37 dias. O raio da órbita do Gliese 581g é igual

à 20% do raio da órbita da Terra, enquanto sua velocidade orbital é 50% maior que a velocidade orbital da Terra. O Gliese 581g está “preso” à estrela, o que significa que um lado do planeta recebe luz constantemente, enquanto o outro é de perpétua escuridão. A zona mais habitável na superfície do exoplaneta seria a linha entre a sombra e a luz, com temperaturas caindo em direção à sombra e subindo em direção à luz. A temperatura média varia entre –31°C e –12°C, mas as temperaturas reais podem ser muito maiores na região de frente para a estrela (até 70 ºC) e muito menores na região contrária (até –40°C). A gravidade no Gleise 581g é semelhante à da Terra, o que significa que um ser humano conseguiria andar sem dificuldades. Os cientistas acreditam que o número de exoplanetas potencialmente habitáveis na Via Láctea pode chegar a 20%, dada a facilidade com que Gliese 581g foi descoberto. Se fossem raros, dizem os astrônomos, eles não teriam encontrado um tão rápido e tão próximo. No entanto, ainda vai demorar muito até que o homem consiga sair da Terra e comece a colonizar outros planetas fora do sistema solar.

Texto adaptado de artigo da Revista VEJA, Edição 2185,

ano 43, n 40 de 06 de outubro de 2010. Questão 18 - (UFT TO/2011)

Suponha que uma máquina de Carnot seja construída utilizando como fonte fria o lado do planeta Gliese 581g que nunca recebe luz e como fonte quente o lado que sempre recebe luz. A temperatura da fonte fria Tf= – 40°C e da fonte quente Tq=70°C. A cada ciclo a máquina retira da fonte quente 1000J de calor. Considerando que a máquina trabalha com um gás ideal, leia os itens abaixo:

I. A máquina pode ser representada por um ciclo

com duas transformações adiabáticas reversíveis e duas transformações isotérmicas reversíveis.

II. Se o ciclo desta máquina consiste de uma expansão isotérmica, uma expansão adiabática, uma compressão isotérmica e uma compressão adiabática, respectivamente, então ocorre

transformação de calor em trabalho útil. III. O rendimento da máquina é maior do que 40%.

Page 76: curso de específica de física

76

IV. A cada ciclo uma quantidade de calor maior que 700J é rejeitada para a fonte fria.

Marque a opção CORRETA:

a) I e III são verdadeiras b) I e II são verdadeiras c) I e IV são verdadeiras d) III e IV são verdadeiras e) II e IV são verdadeiras

Questão 19 - (UFRN/2013)

A biomassa é uma das principais fontes de energia renovável e, portanto, máquinas que a utilizam como combustível para geração de energia são importantes do ponto de vista ambiental. Um exemplo bastante comum é o uso da biomassa com o objetivo de acionar uma turbina a vapor para gerar trabalho. A figura abaixo mostra, esquematicamente, uma usina termoelétrica simplificada.

Nessa termoelétrica, a queima da biomassa na fornalha produz calor, que aquece a água da caldeira e gera vapor a alta pressão. O vapor, por sua vez, é conduzido por tubulações até a turbina que, sob a ação deste, passa a girar suas pás.

Considere desprezíveis as perdas de calor devido às diferenças de temperatura entre as partes dessa máquina térmica e o ambiente. Nesse contexto, a variação da energia interna da água da caldeira

a) é maior que a soma do calor a ela fornecido pela

queima da biomassa com o trabalho realizado sobre a turbina.

b) é igual à soma do calor a ela fornecido pela queima da biomassa com o trabalho realizado sobre a turbina.

c) é igual à diferença entre o calor a ela fornecido pela queima da biomassa e o trabalho realizado sobre a turbina.

d) é maior que a diferença entre calor a ela fornecido pela queima da biomassa e o trabalho realizado sobre a turbina.

Questão 20 - (UFRN/2013)

O Sol irradia energia para o espaço sideral. Essa energia tem origem na sua autocontração gravitacional. Nesse processo, os íons de hidrogênio (prótons) contidos no seu interior adquirem velocidades muito altas, o que os leva a atingirem temperaturas da ordem de milhões de graus. Com isso, têm início reações exotérmicas de fusão nuclear, nas quais núcleos de hidrogênio são fundidos, gerando núcleos de He (Hélio) e propiciando a produção da radiação, que é emitida para o espaço. Parte dessa radiação atinge a Terra e é a principal fonte de toda a energia que utilizamos.

Nesse contexto, a sequência de formas de energias que culmina com a emissão da radiação solar que atinge a terra é

a) Térmica Potencial Gravitacional Energia de

Massa Cinética Eletromagnética. b) Cinética Térmica Energia de Massa

Potencial Gravitacional Eletromagnética.

c) Energia de Massa Potencial Gravitacional Cinética Térmica Eletromagnética.

d) Potencial Gravitacional Cinética Térmica

Energia de Massa Eletromagnética.

GABARITO: 1) Gab: D 2) Gab: VFVFVV 3) Gab: A 4) Gab: D

5) Gab: E 6) Gab: A 7) Gab: 25 8) Gab: CCECC 9) Gab: CCCEE 10) Gab: A 11) Gab: 15 12) Gab: A 13) Gab: E 14) Gab: D 15) Gab: 07 16) Gab: B 17) Gab: B 18) Gab: B 19) Gab: C 20) Gab: D 21) Gab: 82 22) Gab: 05 23) Gab: C 24) Gab: E 25) Gab: B 26) Gab: 02 27) Gab:

TRABALHO WB > 0 (POSITIVO). ENERGIA INTERNA UA<0 (NEGATIVO).

UB<0 (NEGATIVO).

CALOR QA<0 (NEGATIVO).

QB > 0 (POSITIVO). 28) Gab: 02 29) Gab: B 30) Gab: D 31) Gab: C 32) Gab: B 33) Gab: C

34) Gab: Fgás = 2,0 103 N e J 3

400

35) Gab: E 36) Gab: 29 37) Gab: 25 38) Gab: A 39) Gab: 25 40) Gab: 09

Page 77: curso de específica de física

77

LISTA 16 - ELETROSTÁTICA 1 – LEI DE COULOMB E CAMPO ELÉTRICO

TÓPICOS: Carga elétrica, eletrização, lei de Coulomb

e campo elétrico

Resumo de conteúdo

Assim como ocorre com a massa, a carga elétrica também

não é criada ou destruída, mas apenas transferida de um

corpo para outro. As formas pelas quais podemos ter essa

transferência resumem-se basicamente em três: o atrito, o

contato e a indução.

Eletrização por atrito

Quando dois corpos são atritados, pode ocorrer a passagem

de elétrons de um corpo para outro. Nesse caso diz-se que

houve uma eletrização por atrito.

Consideremos um bastão de plástico sendo atritado com um

pedaço de lã, ambos inicialmente neutros, conforme a

Figura 1. A experiência mostra que, após o atrito, os corpos

passam a manifestar propriedades elétricas, pois há

transferência de elétrons do bastão para a lã, o que ocorre

devido às condições inerentes aos materiais envolvidos.

Figura 1. Eletrização por atrito.

No exemplo acima descrito, houve transferência de elétrons

do vidro para a lã, sendo que o contrário também

representaria uma eletrização por atrito.

Na eletrização por atrito, os dois corpos envolvidos ficam

carregados com cargas iguais, em intensidade, porém de

sinais contrários.

Eletrização por contato

Quando colocamos dois corpos condutores em contato, um

eletrizado e o outro neutro, pode ocorrer a passagem de

elétrons de um para o outro, fazendo com que o corpo neutro

se eletrize.

Conforme Figura 2, o corpo eletrizado transfere cargas

elétricas ao corpo neutro, o que ocorre devido à força natural

da distribuição de cargas elétricas por dois ou mais materiais

condutores.

Figura 2. Eletrização por contato.

As cargas em excesso do condutor eletrizado negativamente

se repelem e alguns elétrons passam para o corpo neutro,

fazendo com que ele fique também com elétrons em excesso

e, portanto, eletrizado negativamente.

Na eletrização por contato, os corpos condutores ficam

eletrizados com cargas de mesmo sinal, e não

necessariamente em mesma intensidade.

Na eletrização por contato, a soma das cargas dos corpos é

igual antes e após o contato, se o sistema for eletricamente

isolado.

Eletrização por indução

A eletrização de um condutor neutro pode ocorrer por

simples aproximação de um outro corpo eletrizado, sem que

haja o contato entre eles.

Consideremos, conforme a Figura 3, um condutor

inicialmente neutro (B) e um corpo eletrizado negativamente

(A). Quando aproximamos A de B, as suas cargas negativas

repelem os elétrons livres do corpo neutro para posições

mais distantes possíveis.

Figura 3. Eletrização por indução.

Dessa forma, o corpo fica com falta de elétrons numa

extremidade e com excesso de elétrons em outra. O

fenômeno da separação de cargas num condutor, provocado

pela aproximação de um corpo eletrizado, pode também ser

denominado indução eletrostática.

Na indução eletrostática ocorre apenas uma separação entre

algumas cargas positivas e negativas já existentes no corpo

condutor. Fonte: http://www.infoescola.com/eletrostatica/eletrizacao/

Lei de Coulomb

Esta lei, formulada por Charles Augustin Coulomb, refere-se às forças de interação (atração e repulsão) entre duas cargas elétricas puntiformes, ou seja, com dimensão e massa desprezível. Lembrando que, pelo princípio de atração e repulsão, cargas com sinais opostos são atraídas e com sinais iguais são repelidas, mas estas forças de interação têm intensidade igual, independente do sentido para onde o vetor que as descreve aponta. O que a Lei de Coulomb enuncia é que a intensidade da força elétrica de interação entre cargas puntiformes é diretamente proporcional ao produto dos módulos de cada carga e inversamente proporcional ao quadrado da distância que as separa. Ou seja:

Onde a equação pode ser expressa por uma igualdade se considerarmos uma constante k, que depende do meio onde as cargas são encontradas. O valor mais usual de k é

Page 78: curso de específica de física

78

considerado quando esta interação acontece no vácuo, e seu valor é igual a:

Então podemos escrever a equação da lei de Coulomb como:

Para se determinar se estas forças são de atração ou de repulsão utiliza-se o produto de suas cargas, ou seja:

Fonte:

http://www.sofisica.com.br/conteudos/Eletromagnetismo/El

etrostatica/leidecoulomb.php

Campo Elétrico

Assim como a Terra tem um campo gravitacional, uma

carga Q também tem um campo que pode influenciar as

cargas de prova q nele colocadas. E usando esta analogia,

podemos encontrar:

Desta forma, assim como para a intensidade do campo

gravitacional, a intensidade do campo elétrico (E) é definido

como o quociente entre as forças de interação das cargas

geradora do campo (Q) e de prova (q) e a própria carga de

prova (q), ou seja:

Chama-se Campo Elétrico o campo estabelecido em todos os

pontos do espaço sob a influência de uma carga geradora de

intensidade Q, de forma que qualquer carga de prova de

intensidade q fica sujeita a uma força de interação (atração

ou repulsão) exercida por Q.

Já uma carga de prova, para os fins que nos interessam, é

definida como um corpo puntual de carga elétrica conhecida,

utilizado para detectar a existência de um campo elétrico,

também possibilitando o cálculo de sua intensidade.

Vetor Campo Elétrico

Voltando à analogia com o campo gravitacional da Terra, o

campo elétrico é definido como um vetor com mesma direção

do vetor da força de interação entre a carga geradora Q e a

carga de prova q e com mesmo sentido se q>0 e sentido

oposto se q<0. Ou seja:

A unidade adotada pelo SI para o campo elétrico é o N/C

(Newton por coulomb).

Interpretando esta unidade podemos concluir que o campo

elétrico descreve o valor da força elétrica que atua por

unidade de carga, para as cargas colocadas no seu espaço

de atuação.

EXERCÍCIOS

Questão 01 - (UNIOESTE PR/2010)

Quando se fricciona uma régua de plástico em um

casaco de lã ou um pente de plástico nos cabelos secos,

consegue-se atrair para a régua ou para o pente

pedacinhos de papel, palha, fiapos de tecidos etc. Este

fenômeno é denominado eletrização por atrito ou

triboeletrização. Em relação à triboeletrização considere

as afirmações abaixo:

I. O casaco de lã e a régua de plástico ficam

eletrizados com cargas elétricas de mesmo sinal.

II. Para que os pedacinhos de papel sejam atraídos

para a régua de plástico eles devem estar

eletrizados também.

III. Os pedacinhos de papel são atraídos somente

quando a régua ou pente de plástico forem

carregados com cargas positivas.

IV. Os pedacinhos de papel exercem uma força elétrica

de menor intensidade sobre a régua de plástico. É

por isso que a régua não é atraída pelos pedacinhos

de papel.

Em relação às afirmações, assinale a alternativa correta.

a) Apenas as afirmativas I e II estão corretas.

b) Apenas a afirmativa III está correta.

c) Todas as afirmativas estão corretas.

d) Apenas as afirmativas II, III e IV estão corretas.

e) Todas as afirmativas são incorretas.

Questão 02 - (UFSC/2013)

A eletricidade estática gerada por atrito é fenômeno

comum no cotidiano. Pode ser observada ao

pentearmos o cabelo em um dia seco, ao retirarmos um

casaco de lã ou até mesmo ao caminharmos sobre um

tapete. Ela ocorre porque o atrito entre materiais gera

desequilíbrio entre o número de prótons e elétrons de

cada material, tornando-os carregados positivamente

ou negativamente. Uma maneira de identificar qual tipo

de carga um material adquire quando atritado com

outro é consultando uma lista elaborada

experimentalmente, chamada série triboelétrica, como

a mostrada abaixo. A lista está ordenada de tal forma

que qualquer material adquire carga positiva quando

atritado com os materiais que o seguem.

Page 79: curso de específica de física

79

Com base na lista triboelétrica, assinale a(s)

proposição(ões) CORRETA(S).

01. A pele de coelho atritada com teflon ficará

carregada positivamente, pois receberá prótons do

teflon.

02. Uma vez eletrizados por atrito, vidro e seda

quando aproximados irão se atrair.

04. Em processo de eletrização por atrito entre vidro

e papel, o vidro adquire carga de + 5 unidades de

carga, então o papel adquire carga de – 5 unidades

de carga.

08. Atritar couro e teflon irá produzir mais eletricidade

estática do que atritar couro e pele de coelho.

16. Dois bastões de vidro aproximados depois de

atritados com pele de gato irão se atrair.

32. Um bastão de madeira atritado com outro bastão

de madeira ficará eletrizado.

Questão 03 - (UFU MG/2013)

Entre os meses de junho e setembro, o clima na cidade

de Uberlândia fica muito seco. A umidade relativa do ar

atinge valores inferiores a 10%, muito abaixo do ideal

para seres humanos, que é de 70 a 80%. Além das

consequências indesejadas para a saúde, nesta época

do ano é muito comum as pessoas levarem choques ao

saírem dos carros. Em geral, os bancos dos carros são

feitos de couro ou outro tecido, enquanto o painel, o

contato da alavanca do câmbio e o volante são

constituídos por plástico e borracha, não havendo,

portanto, contato com metal no interior do veículo.

Considerando as informações dadas, marque, para as

afirmativas abaixo, (V) Verdadeira, (F) Falsa ou (SO)

Sem Opção.

1. Os materiais com os quais o motorista tem contato

dentro do carro são condutores.

2. Quando o motorista está utilizando um calçado

com solado de borracha ou de madeira, é comum

levar um choque ao descer do carro e encostar-se

à sua lataria. Esse fenômeno ocorre porque há

uma diferença de potencial entre a lataria do

veículo e o corpo do motorista, que é eletrizado ao

atritar-se com o banco do carro.

3. Para medir a quantidade de carga dos objetos

eletrizados por atrito, nesta época do ano, basta

utilizar um eletroscópio.

4. Para diminuir a intensidade dos choques que

ocorrem nesta época do ano, devem-se tocar os

objetos com a palma da mão em vez da ponta dos

dedos, pois a palma da mão tem uma área de

contato maior, diminuindo a intensidade da

descarga elétrica.

Questão 04 - (UEPG PR/2011)

Considere quatro esferas metálicas idênticas e isoladas

uma da outra. Três esferas (a,b,c) estão, inicialmente,

descarregadas e a quarta esfera (d) está eletrizada com

carga igual a Q. A seguir a esfera d é posta

sucessivamente em contato com as esferas a, b e c. No

final todas as esferas estão eletrizadas. Sobre as cargas

adquiridas pelas esferas, ao final do processo, assinale

o que for correto.

01. As quatro esferas estarão igualmente eletrizadas.

02. A esfera a estará eletrizada com carga igual a Q/2.

04. As esferas c e d estarão eletrizadas com cargas

iguais a Q/8.

08. As esferas a, b e c estarão eletrizadas com cargas

iguais a Q/3.

16. A esfera b estará eletrizada com carga igual a Q/4.

Questão 05 - (FGV/2008)

Sendo k a constante eletrostática e G a constante de

gravitação universal, um sistema de dois corpos

idênticos, de mesma massa M e cargas de mesma

intensidade +Q, estarão sujeitos a uma força resultante

nula quando a relação Q

M for igual a

a) G

k

b) k

G

c) G

k

d) k

G

e) 2

G

k

Questão 06 - (UEM PR/2008)

O diagrama abaixo ilustra três esferas neutras de

metal, x, y e z, em contato entre si e sobre uma

superfície isolada.

Assinale a alternativa cujo diagrama melhor representa

a distribuição de cargas das esferas quando um bastão

carregado positivamente é aproximado da esfera x,

mas não a toca.

Page 80: curso de específica de física

80

Questão 07 - (UNIOESTE PR/2008)

Em 1913, o físico dinamarquês Niels Bohr propôs um

modelo para a estrutura do átomo de hidrogênio, o qual

foi chamado de Modelo de Bohr. De acordo com o Modelo

de Bohr, num átomo de hidrogênio, o elétron move-se

sob a ação de uma força de origem elétrica, descrita pela

lei de Coulomb, a qual equaciona a atração que o núcleo

do átomo exerce sobre o elétron.

Com relação ao Modelo de Bohr, assinale a alternativa

correta.

a) O movimento do elétron do átomo em torno do

núcleo é caracterizado por um vetor velocidade

constante.

b) A força coulombiana que age sobre o elétron atua

como força centrípeta, em reação à força centrífuga

que o elétron exerce sobre o núcleo.

c) Como o elétron está em equilíbrio, seu vetor de

velocidade é constante.

d) A energia total de cada elétron pode ser expressa

como a soma de uma energia cinética e uma energia

potencial nuclear.

e) Num átomo de hidrogênio, o elétron descreve uma

órbita circular de raio m10 x 0,5 -11 , a uma

velocidade igual a m/s 10 x 0,2 6 . Neste caso, o

elétron está sujeito a uma aceleração radial igual a 219 km/s 10 x 0,8 .

Questão 08 - (UEM PR/2008)

No diagrama abaixo, duas esferas carregadas

positivamente, A e B, de massas mA e mB, estão

localizadas a uma distância d uma da outra.

Assinale a alternativa cujo diagrama melhor representa

a direção e o sentido da força gravitacional Fg e da

força eletrostática Fe agindo sobre a esfera A devido à

massa e à carga da esfera B.

Questão 09 - (UEM PR/1998)

Considere uma carga puntiforme de 10-3N de peso e 10-

9C de carga, suspensa por um fio isolante (inextensível

e massa desprezível), entre as placas verticais de um

capacitor plano, carregado como mostra a figura abaixo.

Desprezando-se os efeitos de borda e sabendo que na

posição de equilíbrio o fio forma um ângulo de /4rad

com a vertical, assinale o que for correto.

Dados:

P = força peso;

T = tensão no fio;

EF = força elétrica;

q = carga puntiforme.

01. A velocidade e a aceleração da carga elétrica q são

proporcionais a tg /2, na posição de equilíbrio.

02. A carga puntiforme q só estará em equilíbrio, se

Fx = 0,5 N e = 1,6 N.m (Fx são os

componentes das forças, na direção x, e são os

torques).

04. A carga elétrica puntiforme q é negativa.

08. O módulo da força elétrica é numericamente igual

ao módulo da força peso.

Page 81: curso de específica de física

81

16. As forças que atuam na carga puntiforme q são

forças newtonianas, formando um sistema de forças

não coplanares.

32. A carga puntiforme q é positiva.

Questão 10 - (UEM PR/2001)

As cargas puntiformes (q1 e q2) da figura, ambas

positivas, estão em equilíbrio, separadas por uma

distância d, sendo q1 = 4q2. Nessas condições, assinale

o que for correto.

01. As cargas q1 e q2 interagem com forças de mesma

direção e de mesmo sentido.

02. O módulo da força de q1 sobre q2 é quatro vezes

maior que o módulo da força de q2 sobre q1.

04. A carga q1 cria, na posição ocupada por q2, um

campo elétrico cujo módulo é quatro vezes maior

que o campo elétrico criado por q2, na posição

ocupada por q1.

08. Na linha entre as cargas, existirá um ponto onde o

campo elétrico é nulo, porém o potencial é diferente

de zero.

16. Na linha entre as cargas, existirá um ponto à

distância (4/5)d da carga q2 onde os potenciais,

gerados por essas cargas, são iguais.

32. Trazendo uma outra carga (Q) positiva, do infinito

até um ponto da linha que passa pelas cargas, a

energia potencial do novo sistema, formado pelas

três cargas, aumenta se o ponto estiver entre as

cargas e diminui se o ponto estiver fora das cargas.

Questão 11 - (UEM PR/2003)

Três pequenos objetos, com cargas elétricas idênticas,

estão fixos no espaço e alinhados como mostra a figura

a seguir. O objeto C exerce sobre B uma força igual a 3

dinas. Qual o módulo da força elétrica resultante que

atua sobre B, em virtude das ações de A e de C?

Questão 12 - (UEL PR/1999)

Três partículas carregadas positivamente, cada uma com

carga q, ocupam os vértices de um triângulo retângulo

cujos catetos são iguais e medem d. Sabendo-se que as

cargas estão num meio cuja constante dielétrica é k, a

força elétrica resultante sobre a carga do ângulo reto é

dada pela expressão:

a) 2

2

2d

q k

b) 2

2

2d

q k 2

c) 2

2

d

q k

d) 2

2

d

q k 2

e) 2

2

d

q k 2

Questão 13 - (UEL PR/2011)

O caráter hidrofóbico do poliuretano está associado à

força de repulsão eletrostática entre as moléculas do

material e as moléculas de água, fenômeno físico que

ocorre entre corpos com cargas elétricas de mesmo

sinal.

É correto afirmar que a força de repulsão eletrostática

a) tem sentido contrário à força de atração

eletrostática entre corpos eletricamente neutros.

b) é maior entre dois corpos com mesma carga

elétrica +Q do que entre dois corpos com mesma

carga elétrica –Q.

c) será duas vezes maior se a distância entre os

corpos carregados for reduzida à metade.

d) aumenta com o quadrado da distância entre

corpos eletricamente carregados.

e) é diretamente proporcional à quantidade de carga

para corpos eletricamente carregados.

Questão 14 - (UEL PR/2011)

Devido ao balanceamento entre cargas elétricas

positivas e negativas nos objetos e seres vivos, não se

observam forças elétricas atrativas ou repulsivas entre

eles, em distâncias macroscópicas. Para se ter,

entretanto, uma ideia da intensidade da força gerada

pelo desbalanceamento de cargas, considere duas

pessoas com mesma altura e peso separadas pela

distância de 0,8 m. Supondo que cada uma possui um

excesso de prótons correspondente a 1% de sua massa,

a estimativa da intensidade da força elétrica resultante

desse desbalanceamento de cargas e da massa que

resultará numa força-peso de igual intensidade são

respectivamente:

Dado: Massa de uma pessoa: m = 70 kg

a) 91017 N e 6103 kg

b) 601024 N e 61024 kg

c) 91023 N e 61023 kg

d) 41017 N e 41016 kg

e) 601020 N e 41019 kg

Questão 15 - (UEL PR/2012)

Figura 23: Robert Smithson. Molhe Espiral, 1970. Rocha

negra,

cristais de sal, terra, água vermelha (algas). 457,2 m de

comprimento e

aproximadamente 4,57 m de largura. Grande Lago Salgado,

Utah (EUA).

Page 82: curso de específica de física

82

A obra Molhe Espiral (Fig. 23) faz lembrar o modelo

atômico “planetário”, proposto por Ernest Rutherford

(Fig. 24). Esse modelo satisfaz as observações

experimentais de desvio de partículas alfa ao

bombardearem folhas de ouro. Entretanto, ele falha

quando se leva em conta a teoria do eletromagnetismo,

segundo a qual cargas aceleradas emitem radiação

eletromagnética. Assim, o elétron perde energia

executando uma trajetória em espiral e colapsando no

núcleo (Fig. 25).

Figura 24: Modelo atômico “planetário”

Figura 25: Colapso do elétron no núcleo

Com base no enunciado, nas figuras 24 e 25 e nos

conhecimentos sobre mecânica e eletromagnetismo,

considere as afirmativas a seguir.

I. A variação do vetor velocidade do elétron

evidencia que seu movimento é acelerado.

II. Se o módulo da velocidade linear do elétron é

constante em toda a trajetória da figura 25, a sua

velocidade angular aumentará até o colapso com

o núcleo.

III. O átomo de Rutherford poderia ser estável se o

elétron possuísse carga positiva.

IV. Na figura 25, o elétron está desacelerando, uma

vez que a força de repulsão eletrostática diminui

com o decréscimo do raio da órbita.

Assinale a alternativa correta.

a) Somente as afirmativas I e II são corretas.

b) Somente as afirmativas II e IV são corretas.

c) Somente as afirmativas III e IV são corretas.

d) Somente as afirmativas I, II e III são corretas.

e) Somente as afirmativas I, III e IV são corretas.

Questão 16 - (UEM PR/2000)

Uma esfera metálica de raio R, isolada, está carregada

com uma carga elétrica Q. Seja r a distância do centro

da esfera a qualquer ponto dentro (r < R) ou fora (r >

R) da esfera. Nessas condições, assinale o que for

correto.

01. A carga elétrica se distribui uniformemente em toda

a massa da esfera.

02. O campo elétrico e o potencial elétrico são

constantes no interior da esfera.

04. Para r > R, o campo elétrico é inversamente

proporcional ao quadrado da distância e tem direção

perpendicular à superfície da esfera.

08. As equipotenciais associadas ao campo elétrico da

esfera, para r > R, são superfícies esféricas

concêntricas com a esfera e igualmente espaçadas.

16. O potencial elétrico é uma grandeza escalar,

enquanto o campo elétrico é uma grandeza vetorial.

Questão 17 - (UEM PR/2000)

Considerando que, em uma dada região do espaço,

existe um campo elétrico, assinale o que for correto.

01. Em cada ponto dessa região, o vetor campo elétrico

(E) é perpendicular à linha de força que passa por

esse ponto.

02. Em quaisquer pontos dessa região, as linhas de

força são sempre perpendiculares às equipotenciais.

04. Um elétron, colocado nessa região, sofre ação de

uma força na mesma direção e sentido do campo.

08. Na ausência de qualquer força externa, uma carga

elétrica puntiforme, colocada nesse campo, sempre

percorre uma linha de força.

16. É nulo o trabalho realizado para deslocar uma carga

de prova sobre uma equipotencial.

32. Em se tratando de um campo elétrico uniforme,

uma carga de prova sempre se desloca com

velocidade constante.

Questão 18 - (UEM PR/2004)

O campo elétrico entre duas placas condutoras vale E =

2,0104 N/C e a distância entre elas é d = 7,0 mm.

Suponha que um elétron (qe = 1,610–19 C e me =

9,110–31 kg) seja liberado em repouso nas

proximidades da placa negativa. Com base na situação

descrita, assinale o que for correto.

01. A força F que atuará sobre o elétron terá a mesma

direção e sentido do campo elétrico.

02. O módulo da força F que atuará sobre o elétron é

igual a 3,210–15 N.

04. Sabendo-se que o peso do elétron é desprezível em

comparação com a força elétrica que atuará sobre

ele, pode-se afirmar que o movimento do elétron

será retilíneo uniformemente variado e que o

módulo da aceleração adquirida por ele é

3,51015m/s2.

08. O tempo que o elétron gastará para ir de uma placa

a outra será 4,010–9s.

16. A velocidade do elétron ao chegar à placa positiva é

14,0106 m/s.

32. A diferença de potencial entre as placas é 140 V.

64. O trabalho que o campo elétrico realiza sobre o

elétron, ao deslocá-lo da placa negativa para a placa

positiva, é 2,2410–18 Nm.

Questão 19 - (UEM PR/2005)

Considere uma carga elétrica, positiva, isolada no vácuo,

cujo módulo é q = 12 C. Assinale o que for correto.

01. Em qualquer ponto em torno da carga q, as linhas

de força têm a mesma direção e o mesmo sentido

do vetor campo elétrico gerado por ela.

02. A intensidade do campo elétrico gerado pela carga

q, em um ponto situado a 5 cm de distância, é igual

a 4,32107 N/C.

04. O potencial elétrico no ponto situado a 5 cm de

distância da carga é igual a 2,16106 V.

Page 83: curso de específica de física

83

08. A diferença de potencial elétrico entre dois pontos

situados em uma mesma superfície equipotencial é

diferente de zero.

16. O trabalho realizado pela força elétrica do campo

elétrico para deslocar uma carga q2 = 15 C desde

o infinito até o ponto situado a 5 cm da carga q é,

em módulo, igual a 32,40 J.

32. O trabalho realizado pela força elétrica do campo

elétrico para deslocar uma carga entre dois pontos

pertencentes à mesma linha de força é nulo.

64. Potencial elétrico e trabalho são grandezas

vetoriais.

Questão 20 - (UEL PR/1999)

Um condutor esférico de raio R, eletrizado e em

equilíbrio eletrostático, está imerso no ar. O gráfico que

representa o módulo do vetor campo elétrico E, em

função da distância r ao centro do condutor é E

0 R r

a.

E

0 R r

b.

E

0 R r

c.

E

0 R r

d.

E

0 r

e.

GABARITO:

1) Gab: E 2) Gab: 14

3) Gab: FVF 4) Gab: 22

5) Gab: C 6) Gab: D

7) Gab: E 8) Gab: C

9) Gab: EEECEC 10) Gab: EECCEE

11) Gab: 24 12) Gab: D

13) Gab: E 14) Gab: B

15) Gab: A 16) Gab: 02-04-16

17) Gab: 02-08-16

18) Gab: 38 19) Gab: 23 20) Gab: A

21) Gab: B 22) Gab: D

23) Gab: B

24) Gab:

1. Ao atritar o pente de plástico na flanela, ele é

eletrizado por atrito e estabelece ao seu redor um

campo elétrico não uniforme. A bolinha de isopor,

suposta inicialmente neutra, sofre um processo de

polarização (similar a um processo de indução, mas

em um isolante) e, estando em um campo não

uniforme, sofre ação de uma força resultante

atrativa.

2. Não, pois estando a bolinha envolvida por uma

gaiola metálica, não haverá campo elétrico atuando

sobre a mesma devido à blindagem eletrostática no

interior dessa gaiola. Portanto, a bolinha não será

polarizada, logo, não será atraída.

3. Sim, pois o pente eletrizado no interior da gaiola a

deixa induzida, possibilitando a existência de um

campo elétrico externo a ela, polarizando a bolinha

de isopor e atraindo-a.

25) Gab: D

26) Gab: A

27) Gab:

a) tTerra-Sol = 500 s

b) q = 2 10–9 C

28) Gab: C

29) Gab: 18

30) Gab:

a) .k

mgx0

b) .k

qEmg'x 00

c)

kmg2

qEmgh

20

.

d) .m

k

2

1f

Page 84: curso de específica de física

84

.k

qEA 0

31) Gab: E 32) Gab: 13 33) Gab: A 34) Gab: C 35) Gab: 06 36) Gab: 07 37) Gab: B 38) Gab: C 39) Gab: D 40) Gab: 07

Page 85: curso de específica de física

85

LISTA 17 – ELETROSTÁTICA 2 – POTENCIAL, ENERGIA E TRABALHO

RESUMO DE CONTEÚDO Potencial Elétrico Imagine um campo elétrico gerado por uma carga Q, ao ser colocada um carga de prova q em seu espaço de atuação podemos perceber que, conforme a combinação de sinais entre as duas cargas, esta carga q, será atraída ou repelida, adquirindo movimento, e consequentemente Energia Cinética. Lembrando da energia cinética estudada em mecânica, sabemos que para que um corpo adquira energia cinética é necessário que haja uma energia potencial armazenada de alguma forma. Quando esta energia está ligada à atuação de um campo elétrico, é chamada Energia Potencial

Elétrica ou Eletrostática, simbolizada por .

A unidade usada para a é o joule (J).

Pode-se dizer que a carga geradora produz um campo elétrico que pode ser descrito por uma grandeza chamada Potencial Elétrico (ou eletrostático). De forma análoga ao Campo Elétrico, o potencial pode ser descrito como o quociente entre a energia potencial elétrica e a carga de prova q. Ou seja:

A unidade adotada, no SI para o potencial elétrico é o volt (V), em homenagem ao físico italiano Alessandro Volta, e a unidade designa Joule por coulomb (J/C). Quando existe mais de uma partícula eletrizada gerando campos elétricos, em um ponto P que está sujeito a todas estes campos, o potencial elétrico é igual à soma de todos os potenciais criados por cada carga, ou seja:

Uma maneira muito utilizada para se representar potenciais é através de equipotenciais, que são linhas ou superfícies perpendiculares às linhas de força, ou seja, linhas que representam um mesmo potencial. Para o caso particular onde o campo é gerado por apenas uma carga, estas linhas equipotenciais serão circunferências, já que o valor do potencial diminui uniformemente em função do aumento da distância (levando-se em conta uma representação em duas dimensões, pois caso a representação fosse tridimensional, os equipotenciais seriam representados por esferas ocas, o que constitui o chamado efeito casca de cebola, onde quanto mais interna for a casca, maior seu potencial).

Fonte: http://www.sofisica.com.br/conteudos/Eletromagnetismo/Eletrostatica/potencial.php

Campo elétrico uniforme

Definição

Chama-se campo elétrico uniforme àquele em que o vetor campo tem mesma intensidade, mesma direção e mesmo sentido em todos os pontos. Como as linhas de força de um campo são sempre tangentes ao vetor campo, concluímos que num campo uniforme as linhas de força são retas e paralelas.

Exemplo – Suponhamos dois condutores planos, paralelos e próximos. Se eles forem carregados com cargas de mesmo valor absoluto e sinais opostos, o campo elétrico que se formará entre eles será uniforme. As linhas de força são paralelas entre si e perpendiculares aos planos; apenas nos bordos o campo deixa de ser uniforme: as linhas de força se curvam, como mostra a figura abaixo.

Fonte: http://efisica.if.usp.br/eletricidade/basico/campo/campo_eletrico_unif/ EXERCÍCIOS Questão 01 - (UNIOESTE PR/2009)

Considere duas cargas, de mesmo valor e sinal,

colocadas a uma distância d uma da outra. É INCORRETO afirmar que

a) a força eletrostática resultante sobre uma terceira

carga colocada no ponto médio da linha que une as cargas é nula, qualquer que seja o sinal e o valor da carga.

b) o campo elétrico é nulo no ponto médio da linha que une as cargas.

c) o potencial elétrico é nulo no ponto médio da linha que une as cargas.

d) se a carga elétrica das duas cargas for reduzida à metade e a distância entre elas duplicada, a nova força entre as cargas será 16 vezes menor que a força original.

e) se a distância entre as cargas for duplicada a energia potencial elétrica do sistema será reduzida à metade do seu valor original.

Questão 02 - (UEM PR)

Page 86: curso de específica de física

86

Considere uma partícula de massa m e carga Q+. Então, é correto afirmar que: 01. a massa carregada em repouso produzirá em seu

redor, respectivamente, um campo gravitacional e um elétrico.

02. a carga Q+ poderá ser escrita como sendo um número inteiro N de carga elementar e+ ou e- .

04. Q+, sendo colocada nas proximidades de um plano infinito carregado negativamente, sofrerá uma força repulsiva.

08. o potencial elétrico produzido pela carga puntiforme Q+, a uma distância finita r do seu centro, será nulo.

16. as superfícies eqüipotenciais geradas por Q+ podem ser representadas por uma família de esferas concêntricas, com Q+ no centro.

32. o potencial elétrico de Q+, a uma distância r de seu centro, poderá assim ser representado:

Questão 03 - (UEM PR)

Duas cargas puntiformes +q e –q são mantidas, em equilíbrio, nos vértices do retângulo de lados a = 3 m e b = 4 m, conforme a figura. Considere a constante de

Coulomb K = 04/1 e o potencial V = 0, no infinito.

Nessas condições, assinale o que for correto.

01. O potencial no ponto B é maior que o potencial no ponto A, ou seja, VB > VA.

02. No cruzamento das diagonais do retângulo, o potencial é nulo. Porém, o campo elétrico é diferente de zero.

04. VA – VB =( 6

1 Kq)Volts

08. O trabalho necessário para deslocar uma terceira carga q', em equilíbrio, de A até B, é igual à energia potencial do sistema formado pelas três cargas.

16. O campo elétrico resultante, no ponto A, é igual ao campo elétrico resultante, no ponto B.

32. (VA – VB ) (VB – VA).

Questão 04 - (UEM PR/2009)

Com relação à natureza do impulso nervoso que percorre as células do tecido nervoso, assinale o que for correto.

01. Nas sinapses, há transporte de carga elétrica e

formação de corrente elétrica ao longo do neurônio. 02. As sinapses elétricas permitem a contração

sincrônica, de frequência relativamente bem definida, como no caso do batimento cardíaco.

04. O impulso nervoso pode ser visto como a propagação do potencial de ação ao longo do neurônio.

08. A diferença de potencial entre o potencial de repouso do neurônio, que é -70 mV, e o potencial de despolarização, que é +40 mV, é 0,11 V.

16. Em um neurônio em repouso, o interior do axônio é eletricamente negativo, fazendo que o campo elétrico esteja direcionado para a superfície externa do axônio.

Questão 05 - (UEL PR)

A figura abaixo mostra duas cargas elétricas +q e –q, separadas pela distância 2a e simétrica em relação ao eixo x. É correto afirmar que:

a

a

OB

-q

+q

x

y

a) O campo elétrico no ponto O é nulo. b) O potencial elétrico no ponto O é diferente de zero. c) A força elétrica que atuaria em uma carga +q

colocada em B teria direção vertical com sentido para cima.

d) A força elétrica que atuaria em uma carga –q colocada em B teria sentido para cima.

e) Uma carga +q colocada em B apresentará trajetória retilínea, deslocando-se verticalmente para baixo.

Questão 06 - (UEL PR)

Considere o campo elétrico gerado por uma carga elétrica puntiforme +q1, localizada no centro de um circuito de raio R. Uma outra carga elétrica puntiforme q2 é levada da posição A para B, de B para C de C para D e finalmente de D para A, conforme mostra a figura abaixo. Sobre isso, considere as afirmativas.

R

B

C

D

A

+q1

I. O trabalho é menor na trajetória BC que na

trajetória DA. II. O trabalho na trajetória AB é positivo se a carga q2

for positiva. III. O trabalho na trajetória AB é igual ao trabalho no

trajeto BC + CD + DA. IV. O trabalho na trajetória AB + BC + CD + DA é nulo. Sobre as afirmativas acima, assinale a alternativa

correta. a) Apenas as afirmativas I e IV são verdadeiras. b) Apenas as afirmativas I, II e IV são verdadeiras. c) Apenas as afirmativas II e III são verdadeiras. d) Apenas as afirmativas II, III e IV são verdadeiras. e) Apenas as afirmativas III e IV são verdadeiras.

Questão 07 - (UEL PR)

Um elétron escapa da placa negativa de um capacitor, com velocidade inicial desprezível. Se a diferença de

Page 87: curso de específica de física

87

potencial entre as placas do capacitor é de 200 V e a carga elementar é de 1,6 . 10–19C, a energia cinética com que o elétron atinge a placa positiva é, em joules, a) 3,2 . 10–23 b) 8,0 . 10–22 c) 3,2 . 10–21 d) 8,0 . 10–18 e) 3,2 . 10–17

Questão 08 - (UFPR/2012)

A unidade de uma grandeza física pode ser escrita como

As

mKg3

2

.Considerando que essa unidade foi escrita em

termos das unidades fundamentais do SI, assinale a alternativa correta para o nome dessa grandeza.

a) Resistência elétrica. b) Potencial elétrico. c) Fluxo magnético. d) Campo elétrico. e) Energia elétrica.

Questão 09 - (UFPR/2008)

Atualmente, podem-se encontrar no mercado filtros de ar baseados nas interações eletrostáticas entre cargas. Um possível esquema para um desses filtros é apresentado na figura abaixo (à esquerda), na qual a placa circular 1 mantém-se carregada negativamente e a placa 2 positivamente. O ar contendo os poluentes é forçado a passar através dos furos nos centros das placas, no sentido indicado na figura. No funcionamento desses filtros, as partículas de poeira ou gordura contidas no ar são eletrizadas ao passar pela placa 1. Na região entre as duas placas existe um campo elétrico E, paralelo ao eixo x, de modo que, quando as partículas carregadas passam por essa região, ficam sujeitas a uma

força elétrica, que desvia seu movimento e faz com se depositem na superfície da placa 2. Investigando o campo elétrico produzido no interior de um desses filtros, obteve-se o gráfico mostrado abaixo (à direita), no qual está representado o módulo do campo E em função da distância x entre um ponto P e a placa 1.

Com base no gráfico, a força elétrica que age sobre uma partícula de carga q = 3,2 × 10-6 C situada dentro do filtro e a 3,0 mm da placa 1 é: a) 0,64 N b) 1,82 N c) 0,24 N d) 6,00 N

e) 0,48 N Questão 10 - (UFPR/2012)

Um próton movimenta-se em linha reta paralelamente às linhas de força de um campo elétrico uniforme, conforme mostrado na figura. Partindo do repouso no ponto 1 e somente sob ação da força elétrica, ele percorre uma distância de 0,6 m e passa pelo ponto 2. Entre os pontos 1 e 2 há uma diferença de potencial V

igual a 32 V.

Considerando a massa do próton igual a 1,6 x 10-27 kg e sua carga igual a 1,6 x 10-19 C, assinale a alternativa que apresenta corretamente a velocidade do próton ao passar pelo ponto 2.

a) 2,0 x 104 m/s. b) 4,0 x 104 m/s. c) 8,0 x 104 m/s.

d) 1,6 x 105 m/s. e) 3,2 x 105 m/s.

Questão 11 - (UEM PR/2012)

Considerando um condutor elétrico metálico maciço e esférico, de raio R, carregado positivamente e disposto no vácuo, assinale o que for correto.

01. Para pontos d < R, no interior do condutor, o

campo elétrico é nulo. 02. Para pontos D > R, o campo elétrico gerado em D

é proporcional a D2. 04. A densidade superficial de cargas no condutor é

2R4

Q

.

08. O potencial elétrico no interior do condutor sofre

variação, na forma Vd = d

kQ , com d < R.

16. A superfície desse condutor elétrico esférico é uma superfície equipotencial.

Questão 12 - (UEM PR/2009)

Com relação à capacitância e aos capacitores, é correto afirmar que

01. a diferença de potencial entre as placas de um

capacitor de placas paralelas corresponde ao trabalho, por unidade de carga, necessário para deslocar uma pequena carga de uma placa a outra do capacitor.

02. a capacitância de um capacitor de placas paralelas é a constante de proporcionalidade entre a carga acumulada no mesmo e a diferença de potencial entre suas placas.

04. a capacitância equivalente de N capacitores associados em série é o somatório das capacitâncias dos capacitores individuais.

08. capacitores com capacitâncias variáveis podem ser empregados em circuitos elétricos para a sintonia de receptores de rádio.

16. a capacitância equivalente de N capacitores associados em paralelo é menor que a menor capacitância individual empregada na associação.

Page 88: curso de específica de física

88

Questão 13 - (UEM PR/2013)

Em um experimento realizado no vácuo, uma das extremidades de um fio delgado, inextensível e isolante é presa a um suporte fixo, enquanto a outra extremidade do fio é presa a uma pequena esfera de peso 2 x 10–3 N, carregada com uma carga positiva de 5 x 10–6 C. Esse conjunto, que lembra um pêndulo, é colocado no interior de um capacitor de placas paralelas, no centro geométrico do mesmo. As placas do capacitor, que possuem lados muito maiores do que o comprimento do fio, estão dispostas na vertical, distanciadas uma da outra por 5 cm, e, na posição de equilíbrio, quando a esfera está em repouso no interior das placas, o fio que prende a esfera faz um ângulo de 30º com a vertical. Considerando que o conjunto suporte-fio-esfera não altera as características do campo elétrico no interior do capacitor, analise as alternativas abaixo e assinale o que for correto.

Dado: tg 30º = 3

3

01. O módulo do campo elétrico entre as placas do

capacitor é de C

N10x

15

32 3.

02. A diferença de potencial entre as placas do

capacitor é de V3

320.

04. A densidade superficial de cargas, em valor absoluto, em cada placa do capacitor, é de

2m

C

15

32.

08. O módulo da força elétrica experimentada pela esfera, na posição de equilíbrio, é de 3 x 10–6 N.

16. O módulo da tensão experimentada pelo fio, na

posição de equilíbrio, é de N10x3

34 3.

Questão 14 - (UFPR)

A invenção dos capacitores ocorreu há mais de dois séculos, conforme registrado na literatura especializada. Embora os princípios básicos de projeto e funcionamento dos capacitores tenham permanecido os mesmos, a utilização de novos materiais e tecnologias de fabricação permitiram melhorar a eficiência e reduzir as dimensões desses componentes. A miniaturização foi necessária para que eles pudessem se adequar à evolução de outros dispositivos da eletrônica, como os circuitos integrados. Com relação aos princípios básicos dos capacitores, assinale a alternativa correta. a) A capacitância de um capacitor aumenta quando é

inserido um material dielétrico entre suas placas. b) Num capacitor de placas paralelas, quanto maior a

área das placas, menor será a capacitância. c) A capacitância pode ser expressa no SI em V/C. d) Cargas elétricas de mesmo sinal são armazenadas

nas duas placas do capacitor. e) Os capacitores podem armazenar corrente elétrica.

Questão 15 - (ACAFE SC/2012)

Em uma cartilha fornecida pelos DETRANs do país é alertado sobre o risco em caso de acidente e cabos elétricos estarem em contato com os veículos. Nesta cartilha há um erro conceitual quando é afirmado que: “No interior dos veículos, as pessoas estão seguras, desde que os pneus estejam intactos e não haja nenhum contato com o chão. Se o cabo estiver sobre o

veículo, elas podem ser eletrocutadas ao tocar o solo. Isso já não ocorre se permanecerem no seu interior, pois o mesmo está isolado pelos pneus.”

Noções de Primeiros Socorros no Trânsito, p. 25/São Paulo: ABRAMET – 2005

Assinale a alternativa correta que proporciona uma justificativa cientificamente adequada para a situação descrita na cartilha.

a) As pessoas jamais estarão seguras, pois os pneus

não tem isolamento adequado. b) As pessoas devem permanecer no interior do carro

porque estão blindadas eletricamente, independente de estarem isoladas pelos pneus.

c) Os pneus devem estar cheios de ar, caso contrário não haverá isolamento.

d) Se as pessoas estiverem com calçados de borracha elas podem saltar do carro.

TEXTO: 2 - Comum à questão: 16

Quando um rolo de fita adesiva é desenrolado, ocorre uma transferência de cargas negativas da fita para o rolo, conforme ilustrado na figura ao lado. Quando o campo elétrico criado pela distribuição de cargas é maior que o campo elétrico de ruptura do meio, ocorre uma descarga elétrica. Foi demonstrado recentemente

que essa descarga pode ser utilizada como uma fonte econômica de raios-X.

Questão 16 - (UNICAMP SP/2011)

No ar, a ruptura dielétrica ocorre para campos elétricos a partir de E = 3,0106 V/m . Suponha que ocorra uma

descarga elétrica entre a fita e o rolo para uma diferença de potencial V = 9 kV. Nessa situação, pode-se afirmar que a distância máxima entre a fita e o rolo vale

a) 3 mm. b) 27 mm. c) 2 mm. d) 37 nm.

Questão 17 - (UNIFOR CE)

Uma esfera condutora de raio 30 cm, eletrizada com carga de 3,0 x 109 C, no vácuo, está em equilíbrio

eletrostático. O potencial elétrico, em volts, e o módulo do vetor campo elétrico, em volts por metro, num ponto a 10 cm do centro da esfera valem, respectivamente, Dado: Constante eletrostática do vácuo = 9,0 x 109 Nxm2/C2 a) 9,0 . 10 e 9,0 . 103 b) 9,0 . 10 e zero. c) 2,7 . 102 e 2,7 . 103 d) 2,7 . 102 e zero. e) zero e 2,7 . 103

Questão 18 - (UNIOESTE PR/2008)

Page 89: curso de específica de física

89

A figura abaixo representa a região central de duas placas paralelas idênticas (P1 e P2), de espessura desprezível e carregadas eletricamente com igual quantidade de carga, porém de sinais opostos. A carga em cada placa está uniformemente distribuída e, como consequência, existe, entre as placas, um campo elétrico

uniforme de intensidade m/V10 x 0,1E 4 , cuja

orientação está indicada na figura. Tendo por base os dados apresentados, assinale a alternativa correta:

a) Se a distância entre as placas é mm 0,3d , o valor

da diferença de potencial entre as placas P1 e P2 é 30000 volts.

b) A placa P1 está sujeita a uma força de atração exercida pela placa P2, cujo módulo pode ser calculado através da Lei de Ampère.

c) A força eletrostática sobre uma partícula

eletricamente carregada com carga C10 x 0,3q -6

é de 0,01 N e atua na mesma direção e sentido que o campo elétrico.

d) O trabalho realizado pela força eletrostática para deslocar uma partícula carregada eletricamente

com uma carga C10 x 0,3 q -6 , do ponto B ao

ponto A, é positivo. e) Uma partícula de massa m colocada na região entre

as placas P1 e P2 permanece em equilíbrio. Isto significa que a partícula é eletricamente carregada

com carga negativa de módulo E/mgq , onde g é

a aceleração da gravidade. Questão 19 - (UEL PR)

Milikan determinou o valor da carga elétrica elementar (carga elétrica do elétron, qe) com um experimento representado pelo desenho abaixo. Uma pequena gota de óleo de massa m, está em equilíbrio, sob a ação do campo gravitacional e do campo elétrico de módulo E, vertical, uniforme e orientado para baixo. O experimento é desenvolvido em uma região que pode ser considerada como vácuo. Qual das alternativas abaixo está correta?

m E

a) A carga total da gota é mg/E e é positiva. b) A diferença entre o número total de prótons e

elétrons, na gota, é dada por mg/(Eqe). c) A carga elétrica total da gota é E/(mg) e é positiva. d) O número total de elétrons na gota é Eqe/(mg). e) A força gravitacional sobre a gota é nula, porque ela

está no vácuo. Questão 20 - (UEL PR)

Uma partícula de massa 1,6 g permanece em repouso num ponto de espaço onde o vetor campo elétrico é vertical, para baixo, tendo intensidade E = 8,0 . 104 V/m. Pode-se, daí, concluir que a carga elétrica da partícula é

a) positiva, de 0,10 C

b) negativa, de 0,20 C c) positiva, de 0,20 C

d) negativa, de 0,50 C

e) positiva, de 0,50 C

GABARITO: 1) Gab: C 2) Gab: CCEECE 3) Gab: ECCECC 4) Gab: 14 5) Gab: C 6) Gab: E 7) Gab: E 8) Gab: B 9) Gab: E 10) Gab: C 11) Gab: 21 12) Gab: 11 13) Gab: 19 14) Gab: A 15) Gab: B 16) Gab: A 17) Gab: B 18) Gab: E

19) Gab: B 20) Gab: B 21) Gab:

a) m

E|q|a

b) t = 0v

L

c) y = m2

tE|q| 2

d) y = 0Vm

LE|q|

22) Gab: D 23) Gab: B 24) Gab: 28 25) Gab: a)1,5 C b)x = 0,4 m

26) Gab: 27 27) Gab: a) A gaiola de Faraday é uma blindagem eletrostática que não permite que haja campo elétrico no interior dos condutores ou de estruturas metálicas b) 1) Um carro de metal quando recebe uma descarga elétrica funciona como uma gaiola de Faraday; 2) o aparelho de micro-ondas, que não permite que as micro-ondas escapem durante o aquecimento dos alimentos, por causa de sua estrutura metálica, 3) cabos coaxiais de TV a cabo possuem uma malha de metal, intercalada com um isolante, que revestem o fio condutor interno que transporta o sinal recebido pela antena. Essa malha funciona como uma gaiola de Faraday. 28) Gab: a)A diferença de potencial mínima, VM, é calculada da seguinte forma: E = V/d V = E.d = 3 106 Volt/m 3.000 m = 9 109 Volt

b)Como a energia liberada por um raio (ER) é igual a 109J e ocorrem 3 raios a cada segundo, logo a potência gerada pelos raios que caem a cada segundo será de: PR = 3,0 109W

c)Uma vez que a potência dos raios que caem a cada segundo é PR = 3,0 109W

N(conjunto de três raios) = Potência de Itaipu/(Potência gerada pelos três raios) N(conjunto de três raios) = 15.000.000.000 W/ 3,0x109W N(conjunto de três raios) = 15.000.000.000 W/ 3,0x000.000.000W N(conjunto de três raios) = 5,0 Logo, o número de raios que deveriam cair a cada segundo para gerar uma potência equivalente a da Usina de Itaipu é de:

Número de raios = N(número do conjunto de três raios) 3 = 5 3 = 15 raios

29) Gab: D 30) Gab: B 31) Gab: E 32) Gab: A 33) Gab: B 34) Gab: 07 35) Gab: B 36) Gab: A 37) Gab: D 38) Gab: 11 39) Gab: 24 40) Gab: A

Page 90: curso de específica de física

90

LISTA 18 – ELETRODINÂMICA 1 – LEIS DE ÔHM

RESUMO DE CONTEÚDO

Introdução à Eletrodinâmica

Eletrodinâmica é a parte da física que estuda a energia elétrica em movimento. Como sabemos, a energia elétrica é muito importante para o mundo de hoje. Sem ela, você não poderia estar visualizando esta página na internet, nem acender lâmpadas ou televisores. Para começar a estudar essa matéria, é necessário saber algumas coisas estudadas em química. Os átomos são formados por prótons, nêutrons e elétrons. Os elétrons podem estar "presos" ao núcleo (prótons e nêutrons), ou seja, não estão livres e não podem se mover livremente. Os elétrons livres é que farão o transporte da energia elétrica (corrente elétrica). Por isso dividimos as substâncias químicas em condutores e não condutores. Condutores de eletricidade

Para que um material seja condutor de eletricidade, é preciso que ele tenha elétrons livres para que a energia seja transportada dentro da substância. Exemplos de condutores são os metais (cobre, ouro), etc. Quando os elétrons viajam entre os átomos de uma substância, eles "esbarram" em outros elétrons e no próprio núcleo, fazendo com que os átomos se agitem mais,

causando um aumento de temperatura do material. É por isso que qualquer material que transporte eletricidade irá esquentar. Essa é uma propriedade chamada resistência elétrica. Existem também os materiais chamados de Supercondutores, que são substâncias que oferecem pouquíssima resistência à passagem de elétrons. Exemplos deles são materiais cerâmicos, que em temperatura normal se comportam como isolantes, mas se a temperatura for abaixo de 196º negativos, se tornam condutores. Isolantes (não condutores)

São os materiais que não possuem elétrons livres, e portanto não conseguem transportar energia elétrica. Exemplos de isolantes são a borracha, plásticos, porcelana, água pura (a água de torneira e a água mineral conduzem eletricidade, mas por causa das impurezas (outros minérios) contidas no líquido). Carga elementar

Cada elétron possui uma carga (e), cuja unidade é o Coulomb (C), e vale 1,6 . 10-19C. Carga quantizada

É a quantidade de carga elétrica em um determinado número de elétrons. A equação é bastante simples: Q = n . e onde n é o número de elétrons e e é carga elementar (1,6 . 10-19 C). Leis de Ohm

O cientista Georg Simon Ohm (1787 – 1854), a partir de suas medidas experimentais, chegou a conclusão de que todos os materiais sujeitos a uma diferença de potencial

apresentam uma resistência de valor constante à passagem

da corrente elétrica.

Desta forma, sendo a resistência elétrica uma constante, a intensidade da corrente elétrica cresce proporcionalmente ao valor da tensão aplicada, obedecendo à seguinte expressão:

U = R.I Mas esta equação é satisfeita para resistores ôhmicos e não ôhmicos. Portanto não deve ser utilizada como uma declaração da Lei de Ohm, tendo como válido apenas a expressão verbal citada anteriormente. Graficamente, para resistores ôhmicos, a primeira lei de Ohm mostra:

Observa-se que o coeficiente angular deste gráfico, dado por U/i resulta na resistência elétrica, constante para qualquer diferença de potencial. Obviamente, há um limite de validade para esta, que é denominada a primeira lei de Ohm. Para tensões muito altas, a resistência acaba não tendo um comportamento linear. Dentro do limite em que a lei de Ohm é válida, ela tem a seguinte forma: “A resistência de um objeto é independente da intensidade ou do sinal da diferença de potencial aplicada” A segunda forma, conhecida como segunda lei de Ohm, relaciona a resistência elétrica com as dimensões do objeto e as características do material de que ele é composto. Para tanto, foi considerado um objeto de um material de resistividade ρ, dimensões cilíndricas de comprimento l e área de seção transversal reta S mostrado na figura abaixo.

Através de suas análises, este cientista concluiu que a resistividade de cada material é constante para

Page 91: curso de específica de física

91

qualquer campo elétrico aplicado, e desta forma, poderia obter uma expressão para determinar a resistência elétrica. Esta propriedade, segundo Ohm, é diretamente proporcional à resistividade do material, ao comprimento e inversamente proporcional à área de seção transversal reta do respectivo objeto, e é enunciada como segue: “A resistividade (ou condutividade) de um material é independente da intensidade, direção e sentido do campo elétrico”. Matematicamente, assume a forma:

R =ρ.l/S É válido lembrar que apenas esta última é verdadeiramente condizente com o enunciado da lei de Ohm. Esta lei é válida para certas faixas de temperaturas e de campo elétrico aplicados. Desta forma, os resistores são considerados ôhmicos porque obedecem à lei de Ohm dentro dos limites de tensão aplicados no local do circuito ao qual compõe. Alguns dispositivos à base de semicondutores, como diodos e transistores não são ôhmicos (HALLIDAY - 2007). Referências bibliográficas: HALLIDAY, David, Resnik Robert, Krane, Denneth S. Física 3, volume 2, 5 Ed. Rio de Janeiro: LTC, 2004. 384 p. BISQUOLO, Paulo Augusto Resistência elétrica, resistividade e leis de Ohm. Disponível em: (http://educacao.uol.com.br/fisica/ult1700u46.jhtm) acessado em 23/01/2010.

EXERCÍCIOS

Questão 01 - (FCM MG/2014)

A intensidade luminosa da lâmpada de filamento de

uma lanterna a pilhas diminui, quando esta for usada

por um longo tempo.

A melhor explicação para esse fato é:

a) As conexões das pilhas se oxidam.

b) A força eletromotriz das pilhas diminui.

c) A resistência interna das pilhas aumenta.

d) A resistência do filamento da lâmpada aumenta.

Questão 02 - (UNISA SP/2014)

A Eletroneuromiografia (ENMG) é um procedimento que

avalia a função do sistema nervoso periférico e muscular,

através do registro das respostas elétricas geradas por

estes sistemas, as quais são detectadas graficamente por

um eletroneuromiógrafo. Durante a primeira etapa, breves

choques elétricos, toleráveis e inofensivos, são aplicados no

braço ou na perna do paciente, para determinar como seus

nervos estão conduzindo a corrente elétrica. Considerando

um pulso de corrente elétrica de 200 miliampères aplicado

durante 300 microsegundos, a quantidade de carga que

atravessa o músculo, em microcoulombs, é igual a

a) 600.

b) 60.

c) 60 000.

d) 6.

e) 6 000.

Questão 03 - (UNEB/2014)

FISCHETTI, Mark. Energia expolosiva. Scientific American

Brasil – Aula Aberta, ano II, n. 15, 2013.

Grandes recalls de fabricantes de baterias de íons de

lítio para notebooks suscitaram questões sobre como

essas fontes de energia podem aquecer a ponto de

pegar fogo. Igualmente válida é a dúvida sobre por que

os acidentes não são mais frequentes: são poucos

proporcionalmente às centenas de milhões de baterias

vendidas anualmente.

As células eletroquímicas de íons de lítio empregam

vários materiais, mas quase todas são recarregáveis,

como as usadas em câmeras fotográficas e telefones

celulares, que utilizam óxido de lítio-cobalto no cátodo

e grafite no ânodo. Embora essa formulação seja “de

certo modo inerentemente insegura”, a fabricação

cuidadosa e os dispositivos de segurança embutidos

limitaram os acidentes a poucas ocorrências. Mesmo

assim, os fabricantes de baterias têm aumentado a

capacidade de carga em determinada célula devido à

demanda dos fabricantes de eletrônicos por maior

durabilidade. Portanto, agora a margem de erros é

ainda menor. Aumentando o número de íons na célula,

os fabricantes quadruplicaram a capacidade energética

desde seu lançamento comercial em 1991. (FISCHETTI.

2013. p. 10-11).

Considere uma pilha de íon-lítio utilizada em marca-

passos cardíaco, bastante leve, hermeticamente

fechada para não liberar gases, com durabilidade em

torno de 10 anos, e com alta densidade de carga de

0,8Ah/cm3. Sabendo-se que o valor da carga de um

Page 92: curso de específica de física

92

elétron, em módulo, é igual a 1,610–19C, é correto

afirmar que a ordem de grandeza do número de

elétrons existentes na pilha, com volume de 1,0cm3, é

igual a

01. 1014

02. 1015

03. 1018

04. 1020

05. 1022

Questão 04 - (UERJ/2014)

No circuito, uma bateria B está conectada a três

resistores de resistências R1, R2 e R3 :

Sabe-se que R2 = R3 = 2R1.

A relação entre as potências P1, P2, e P3,

respectivamente associadas a R1, R2, e R3, pode ser

expressa como:

a) P1 = P2 = P3

b) 2P1 = P2 = P3

c) 4P1 = P2 = P3

d) P1 = 2P2 = 2P3

Questão 05 - (ACAFE SC/2014)

Em uma situação cotidiana, uma pessoa liga duas

lâmpadas incandescentes em paralelo em uma rede de

220V. As lâmpadas apresentam certa intensidade

luminosa (brilho), sendo que a lâmpada 2 tem um

filamento de mesmo material, mesmo comprimento,

mas é mais grosso que o filamento da lâmpada 1.

Nessas condições, a alternativa correta é:

a) Desligando a lâmpada L1, a lâmpada L2 diminui o

seu brilho.

b) A lâmpada L1 brilha mais que a lâmpada L2.

c) As lâmpadas L1 e L2 tem o mesmo brilho.

d) A lâmpada L2 brilha mais que a lâmpada L1.

Questão 06 - (FCM MG/2014)

Um chuveiro elétrico possui uma resistência interna

(Rch) a qual fica incandescente quando ele é ligado,

transferindo o calor para a água. Porém, os fios de

ligação da rede elétrica que abastecem o chuveiro se

aquecem muito pouco em comparação com a

resistência do chuveiro. Os fios de ligação possuem uma

resistência (Rfio).

Isso se justifica devido ao fato de a potência dissipada

ser

a) inversamente proporcional à resistência quando a

tensão elétrica é comum e Rch < Rfio.

b) inversamente proporcional à resistência quando a

corrente elétrica é comum e Rch < Rfio.

c) proporcional à resistência quando a tensão elétrica

é comum e Rfio < Rch.

d) proporcional à resistência quando a corrente

elétrica é comum e Rfio < Rch.

Questão 07 - (FMJ SP/2014)

Uma empresa fabricante de focos cirúrgicos informa

que as lâmpadas de seus equipamentos são bivolt (110

V/220 V) e de potência 165 W. As correntes elétricas,

em Ampère, que percorrem essas lâmpadas quando

ligadas, respectivamente, em 110 V e 220 V, são

a) 1,22 e 0,87.

b) 0,67 e 1,33.

c) 0,0136 e 0,0034.

d) 1,5 e 0,75.

e) 2,25 e 0,56.

Questão 08 - (UFG GO/2014)

No clima tropical, que abrange a maior parte do Brasil,

há com frequência a ocorrência de relâmpagos com

maior ou menor sazonalidade. Tipicamente esta

descarga elétrica ocorre sob uma diferença de potencial

de 108 V com intensidade da ordem de 105 A e dura

cerca de 0,5 s. Considerando-se a região Centro-Sul do

Brasil, quais são os três fatores mais relevantes para a

ocorrência de relâmpagos e qual é a energia em joule

associada a um relâmpago para os dados

apresentados?

a) O relevo, o encontro de massas de ar frias e

quentes e o espaço urbano, e 5 x 1012.

b) Maritimidade, o deslocamento das massas de ar

frias e a proximidade de grandes rios, e 2 x 1013.

c) Maritimidade, a existência de uma serra e as

massas de ar frias, e 2 x 1012.

d) O relevo, o encontro de massas de ar frias e

quentes e o espaço urbano, e 2 x 1013.

e) O relevo, o espaço urbano e o estacionamento das

massas de ar frias, e 5 x 1012.

Questão 09 - (Unicastelo SP/2014)

O circuito da figura é formado por uma bateria, um

voltímetro e uma chave interruptora ideais, dois

resistores ôhmicos e fios de ligação de resistência

desprezível. Inicialmente a chave está aberta.

Page 93: curso de específica de física

93

A indicação do voltímetro, em volts, com a chave aberta

e a potência dissipada pelo resistor de 8 , em watts,

quando a chave for fechada são, respectivamente,

iguais a

a) 6 e 0.

b) 12 e 4.

c) 0 e 0.

d) 6 e 2.

e) 12 e 2.

Questão 10 - (UEA AM/2014)

Seja um resistor de resistência elétrica R representado

por

Uma associação de quatro resistores idênticos a este e

que fornece uma resistência equivalente igual a R está

corretamente representada por

a)

b)

c)

d)

e)

Questão 11 - (ESCS DF/2014)

A figura acima ilustra uma ponte de Wheatstone, um

circuito elétrico que permite a medição do valor de uma

resistência desconhecida. O circuito é composto por

uma fonte de tensão V, um galvanômetro G e uma rede

de quatro resistores, sendo três destes conhecidos e

ajustáveis. Para determinar a resistência do resistor

desconhecido, os outros três são ajustados e

balanceados até que a corrente elétrica medida no

galvanômetro — IG — seja nula.

Considerando essas informações, assinale a opção

correta.

a) Para R1R3 = R2R4, a corrente no galvanômetro será

nula.

b) Em uma ponte de Wheatstone equilibrada, é válida

a relação R1 + R2 = R3 + R4.

c) Se a diferença de potencial de A para B for igual à

diferença de potencial de A para C, a energia

dissipada em R1 será igual à energia dissipada em

R4.

d) Para IG = 0, a energia dissipada no ramo ABD será

igual àquela dissipada no trecho ACD.

Questão 12 - (FCM MG/2013)

Uma ligação elétrica residencial chamada de three-way

é usada quando se necessita acender ou apagar uma

mesma lâmpada a partir de dois interruptores, como no

exemplo da figura esquerda abaixo. Antes de subir a

escada, a pessoa acende a lâmpada por meio de um

primeiro interruptor e, quando chega ao andar de cima,

apaga a lâmpada pelo outro interruptor.

Os interruptores three-way (figura da direita) possuem

três saídas de fios. O three-way P pode estar ligado nos

contatos 1 ou 2; o three-way Q pode estar ligado nos

contatos 3 ou 4. A figura abaixo mostra o esquema da

ligação entre os dois three-ways, a lâmpada, a fase e o

neutro.

A lâmpada estará acesa se os interruptores P e Q

estiverem conectados, respectivamente, em:

a) 1 e 4 ou 2 e 4.

b) 1 e 4 ou 2 e 3.

c) 1 e 3 ou 2 e 3.

d) 1 e 3 ou 2 e 4.

Questão 13 - (UFG GO/2013)

É crescente o número de bicicletas elétricas circulando

nas ruas das cidades. Essas bicicletas possuem uma

bateria eletrolítica de 12,5 V semelhante às baterias

automotivas, que proporciona uma potência constante

para iniciar e manter o movimento do sistema ciclista-

bicicleta. Suponha uma situação sem qualquer atrito em

que um sistema de 100 kg sobre uma rua plana é

acelerado do repouso até a velocidade de 18 km/h.

Page 94: curso de específica de física

94

Nessas condições, qual é a quantidade de carga, em

Coulomb, fornecida pela bateria?

a) 100

b) 130

c) 200

d) 1000

e) 1250

Questão 14 - (UFPA/2013)

No rio Amazonas, um pescador inexperiente tenta

capturar um poraquê segurando a cabeça do peixe com

uma mão e a cauda com a outra. O poraquê é um peixe

elétrico, capaz de gerar, entre a cabeça e a cauda, uma

diferença de potencial de até 1500 V. Para esta

diferença de potencial, a resistência elétrica do corpo

humano, medida entre as duas mãos, é de

aproximadamente 1000 . Em geral, 500 mA de

corrente contínua, passando pelo tórax de uma pessoa,

são suficientes para provocar fibrilação ventricular e

morte por parada cardiorrespiratória. Usando os valores

mencionados acima, calculamos que a corrente que

passa pelo tórax do pescador, com relação à corrente

suficiente para provocar fibrilação ventricular, é:

a) um terço.

b) a metade.

c) igual.

d) o dobro.

e) o triplo.

Questão 15 - (UEG GO/2011)

O poraquê (Electrophorus electricus) é um peixe da

espécie actinopterígio, gimnotiforme, que pode chegar

a três metros de comprimento, e atinge cerca de trinta

quilogramas. É uma das conhecidas espécies de

peixeelétrico, com capacidade de geração elétrica que

varia de 300 até 1.500 volts, aproximadamente. Sobre

as interações elétricas no poraquê, é CORRETO afirmar:

a) uma pessoa com uma resistência de 100.000 Ω

poderá segurar, com as duas mãos,

tranquilamente, um poraquê de 300 volts, já que

através dela passará uma corrente menor que

0,070 ampères, valor que poderia causar

distúrbios sérios e provavelmente fatais.

b) uma corrente de 0,1 ampères passará pelo corpo

de uma pessoa com a pele totalmente molhada,

com resistência de apenas 1.000 Ω, quanto ela

tocar, com as duas mãos, um poraquê de 1.000

volts.

c) uma pessoa, com uma resistência elétrica de

100.000 Ω, ao tocar, com as duas mãos no

poraquê, cuja voltagem é de 300 volts, terá

produzida em seu corpo uma corrente de 30 mA

ampères.

d) qualquer pessoa pode tocar livremente o poraquê,

pois choques elétricos não superaquecem tecidos

nem lesam quaisquer funções normais do corpo

humano.

Questão 16 - (FCM MG/2014)

A figura abaixo mostra uma resistência de imersão

(ebulidor) mergulhada num recipiente com água,

interligada num amperímetro ideal; os terminais desse

circuito estão conectados a um gerador elétrico.

Deseja-se aquecer a água até a fervura, evaporando

metade da água.

Existem, inicialmente, 400 g de água a 20 ºC dentro do

recipiente. Desconsidera-se o calor absorvido pelo

recipiente e pelo ebulidor, que possui resistência de 420

. Considera-se que:

esse experimento é realizado ao nível do mar;

1 cal é igual a 4,2 J;

o calor específico da água vale 1,0 cal/g.ºC;

o calor de vaporização da água vale 540 cal/g.

O tempo necessário para atingir o objetivo desejado é

de, aproximadamente

a) 1,3 minutos.

b) 5,8 minutos.

c) 10,3 minutos.

d) 11,7 minutos.

Questão 17 - (UERJ/2014)

Cinco resistores de mesma resistência R estão

conectados à bateria ideal E de um automóvel,

conforme mostra o esquema:

Inicialmente, a bateria fornece ao circuito uma potência

PI. Ao estabelecer um curto-circuito entre os pontos M

e N, a potência fornecida é igual a PF.

A razão I

F

P

Pé dada por:

a) 4

7

b) 15

14

c) 1

d) 6

7

Questão 18 - (Unicastelo SP/2014)

O chuveiro elétrico de uma residência pode funcionar no

modo INVERNO ou no modo VERÃO. No primeiro caso

dissipa 8 000 W e no segundo, 6 000 W.

Page 95: curso de específica de física

95

(www.da-educa.com. Adaptado.)

Em pleno verão, alguém nota que a chave seletora está

ajustada para a posição inverno. Para que os banhos

sejam mais confortáveis e também para economizar

energia elétrica, duas mudanças são propostas:

(I) reposicionar a chave seletora na posição verão;

(II) reduzir o tempo de banho da família em 20%.

Se as duas medidas forem adotadas simultaneamente,

o consumo de energia elétrica, devido apenas aos

banhos com esse chuveiro elétrico, será reduzido em

a) 40%.

b) 45%.

c) 30%.

d) 25%.

e) 35%.

Questão 19 - (MACK SP/2014)

Duas pilhas elétricas de resistências internas nulas,

associadas em série, “alimentam” a lâmpada

incandescente ilustrada na figura ao lado. O

amperímetro ideal A indica a intensidade de corrente

elétrica 0,50 A, quando o voltímetro V, também ideal,

indica a d.d.p. de 3,00 V. Sabendo-se que a potência

dissipada por efeito Joule no filamento da lâmpada

corresponde a 1/4 do valor nominal, indicado pelo

fabricante, pode-se afirmar que os valores máximos

nominais, de potência e tensão elétrica especificados na

lâmpada (potência — d.d.p.), são

a) 1,50 W — 3,00 V

b) 3,00 W — 3,00 V

c) 3,00 W — 6,00 V

d) 6,00 W — 6,00 V

e) 6,00 W — 3,00 V

Questão 20 - (FUVEST SP/2014)

A curva característica de uma lâmpada do tipo led

(diodo emissor de luz) é mostrada no gráfico da página

de respostas. Essa lâmpada e um resistor de resistência

R estão ligados em série a uma bateria de 4,5 V, como

representado na figura abaixo. Nessa condição, a

tensão na lâmpada é 2,5 V.

a) Qual é o valor da corrente iR no resistor?

b) Determine o valor da resistência R.

c) A bateria de 4,5 V é substituída por outra de 3 V,

que fornece 60 mW de potência ao circuito, sem

que sejam trocados a lâmpada e o resistor. Nessas

condições, qual é a potência PR dissipada no

resistor?

Note e adote:

As resistências internas das baterias devem ser

ignoradas.

GABARITO:

1) Gab: C 2) Gab: B 3) Gab: 05

4) Gab: D 5) Gab: D 6) Gab: D

7) Gab: D 8) Gab: A 9) Gab: E

10) Gab: D 11) Gab: A 12) Gab: D

13) Gab: A 14) Gab: E 15) Gab: A

16) Gab: B 17) Gab: D 18) Gab: A

19) Gab: D

20) Gab:

a) A4.10ou A04,0i -2R

b) 50R

c) mW20Pou W10.2P R2

R

21) Gab: E 22) Gab: 40 23) Gab: A

24) Gab: B 25) Gab: 09 26) Gab: C

27) Gab:

a) 11 lâmpadas

b) Nada ocorrerá, pois como as lâmpadas estão

ligadas em paralelo, as demais continuarão sendo

alimentadas, mesmo com a interrupção de uma

das lâmpadas.

Page 96: curso de específica de física

96

c) Mudando a tensão para 220V, será possível

aumentar o número de lâmpadas.

28) Gab: C 29) Gab: 05 30) Gab: C

31) Gab: A 32) Gab: D 33) Gab:

a) R = 2

3

b) A membrana da célula pode ser estudada como um capacitor de placas planas e pararelas. Assim, podemos afirmar que no seu interior o campo elétrico é uniforme. Nessa circunstância: E d = U

9

3

107

1070

d

UE

E = 107 V/m c) Entendemos que cada proteína que compõe um

neurônio do cérebro humano transporta, por segundo, 210 Na+ para fora e 140 K+ para dentro da célula. Logo, a intensidade da corrente devido a uma molécula de proteína é dada pela diferença entre as cargas:

19106,170s 1

e140210

t

qi

i = 112 10–19 A

De acordo com o "Note e adote", cada neurônio é constituído por um milhão de moléculas de proteína. Então conluímos que: I = 106 i = 106 112 10–19

I = 112 10–13 A

34) Gab: A 35) Gab: D 36) Gab: D 37) Gab: D 38) Gab: C 39) Gab: A 40) Gab: 14

Page 97: curso de específica de física

97

LISTA 19 - ELETRODINÂMICA 2 - GERADORES E RECEPTORES

Gerador real É qualquer dispositivo que tenha a capacidade de transformar um tipo de energia em energia elétrica. Uma pilha ou uma bateria são geradores que convertem energia química em energia elétrica. Uma turbina eólica é um gerador que transforma a energia do vento em eletricidade. E, numa hidrelétrica, é um gerador que transforma a energia mecânica em elétrica. O valor máximo da tensão que pode ser fornecida por um gerador qualquer é chamado força eletromotriz (abreviadamente, fem). E, como é tensão, a fem também é medida em volts (V). Num mundo ideal, um gerador ideal seria aquele que consegue aproveitar toda sua fem – ou seja, sempre impor o máximo de tensão a um circuito. Porém, no mundo real, os geradores não são 100% eficientes – ou seja, a tensão efetiva que um gerador real consegue impor ao circuito é sempre menor que sua força eletromotriz. É que os geradores reais têm, eles mesmos, certa resistência elétrica, a chamada resistência interna (r). É assim que se representa um gerador real conectado a um circuito:

Pela primeira lei de Ohm, a tensão em qualquer ponto do circuito é dada pelo produto da resistência pela corrente elétrica. A relação vale também, é claro, para a resistência interna (r) : U = r . i

Matematicamente, o valor da tensão útil do circuito UAB (a diferença entre a tensão máxima e a tensão dissipada) é dada pela expressão:

A equação acima mostra que a tensão entre os terminais de um gerador depende da corrente elétrica que o atravessa,

numa função de 1º grau. (Repare que a equação poderia ser reescrita como U = – r . i +f). Portanto, o gráfico que relaciona a tensão fornecida ao circuito (U) e à corrente elétrica (i) é uma reta. E o coeficiente angular dessa reta é negativo. Veja como fica o gráfico que relaciona corrente e tensão entre os terminais do gerador, num gerador real:

Receptor Real A energia elétrica produzida por um gerador é recebida por algum equipamento que a transforma em outro tipo de energia. Qualquer dispositivo que faz a conversão de um tipo de energia em outro que não seja apenas energia térmica é chamado receptor. A tensão útil de um receptor – ou seja, a tensão efetivamente utilizada para o funcionamento do equipamento – é chamada força contraeletromotriz(abreviadamente, fcem) e seu símbolo é f‘. Esta é a representação gráfica de um receptor real num circuito elétrico:

Page 98: curso de específica de física

98

Num receptor, a tensão total fornecida entre os pontos A e B (U’AB) é a soma entre a tensão dissipada pela resistência interna do receptor (r’) e a tensão útil do receptor (f‘). Veja:

A função matemática que define a tensão entre os terminais de um receptor é: Essa equação define a tensão fornecida aos terminais do receptor em função da corrente elétrica que o atravessa. É uma função de 1º grau, que poderia ser reescrita como U’ = r’ . i +f‘.

Repare que o coeficiente de i (r’. i) é positivo. Então, o gráfico que relaciona tensão fornecida ao receptor (U’) e a corrente elétrica do circuito (i) é uma reta de coeficiente angular positivo

Circuito gerador-receptor

Para que um motor elétrico execute sua função – transforme energia elétrica em energia mecânica –, ele deve ser associado a um gerador que lhe forneça energia elétrica. Por exemplo, um microfone (receptor), associado a uma pilha (gerador) constitui um circuito elétrico.

Graficamente, esse circuito é representado assim, com todas as resistências internas e as forças eletromotriz e contraeletromotriz:

Page 99: curso de específica de física

99

Num circuito composto de geradores e receptores, a intensidade da corrente elétrica é dada pela primeira lei de Ohm. Matematicamente:

EXERCÍCIOS

Questão 01 - (FPS PE/2014)

No circuito mostrado na figura abaixo, um amperímetro

ideal (A) foi ligado na saída da fonte de tensão ideal de

9V. Considerando os resistores ligados ao circuito, a

leitura da corrente no amperímetro foi de

aproximadamente:

a) 0,11 ampère

b) 0,33 ampère

c) 1,11 ampères

d) 3,33 ampères

e) 6,33 ampères

Questão 02 - (UEG GO/2014)

Observe o circuito a seguir.

O circuito encontra-se aberto e o voltímetro V e o

amperímetro A são ideais. Esse fato garante que

a) a potência lançada será de 72,0 W.

b) o voltímetro marcará 12,0 V.

c) o amperímetro detectará 6,0 A.

d) o V e A registrarão valores nulos.

Questão 03 - (ACAFE SC/2013)

Denominam-se instrumentos de medidas elétricas a

todos os dispositivos que são utilizados para medir as

magnitudes elétricas e assegurar assim o bom

funcionamento das instalações e máquinas elétricas.

Sendo dois desses instrumentos um amperímetro e

voltímetro considerados ideais, significa considerar

que:

a) os instrumentos podem ser usados para qualquer

intensidade de corrente elétrica que nunca se

avariam.

b) o amperímetro tem resistência nula e o voltímetro

tem resistência infinita.

c) ambos os instrumentos são bem construídos e

nunca quebram.

d) o amperímetro só pode ser utilizado em corrente

contínua e o voltímetro em corrente alternada.

Questão 04 - (UFTM/2013)

O circuito da figura é constituído por dois resistores de

resistências constantes e iguais a R, um reostato, cuja

resistência pode variar de zero (com o cursor no ponto

I) a R (com o cursor no ponto II), um gerador ideal de

força eletromotriz constante E, um amperímetro

também ideal e fios de ligação com resistência

desprezível.

Quando o cursor do reostato é conectado no ponto I, o

amperímetro indica uma corrente elétrica de

intensidade 1,00 A. É correto afirmar que, se o cursor

for conectado no ponto II, o amperímetro indicará, em

ampères, uma corrente de intensidade

Page 100: curso de específica de física

100

a) 0,50.

b) 1,25.

c) 1,00.

d) 1,50.

e) 0,75.

Questão 05 - (Fac. Santa Marcelina SP/2013)

Assim como todos os corpos materiais, o corpo humano

também possui a característica de conduzir eletricidade.

Os efeitos nocivos da eletricidade ao corpo humano, que

podem, no limite, leva-lo à morte, são provocados pela

corrente elétrica. Sabendo que existe um limiar a partir

do qual a corrente elétrica torna-se ameaçadora e que

a resistência do corpo humano varia de 1 300 a 3 000

ohms, o intervalo que compreende as máximas

correntes elétricas, em mA, suportadas pelo corpo

humano, quando submetido a uma diferença de

potencial de 50 V, é, aproximadamente,

a) 38 e 17.

b) 48 e 25.

c) 58 e 35.

d) 28 e 10.

e) 18 e 5.

Questão 06 - (PUC RJ/2013)

O gráfico acima apresenta a medida da variação de

potencial em função da corrente que passa em um

circuito elétrico.

Podemos dizer que a resistência elétrica deste circuito

é de:

a) 2,0 m

b) 0,2

c) 0,5

d) 2,0 k

e) 0,5 k

Questão 07 - (UFPR/2013)

A indústria eletrônica busca produzir e aperfeiçoar

dispositivos com propriedades elétricas adequadas para

as mais diversas aplicações. O gráfico ao lado ilustra o

comportamento elétrico de três dispositivos eletrônicos

quando submetidos a uma tensão de operação V entre

seus terminais, de modo que por eles circula uma

corrente i.

Com base na figura acima, assinale a alternativa

correta.

a) O dispositivo D1 é não ôhmico na faixa de –30 a

+30 V e sua resistência vale 0,2 k.

b) O dispositivo D2 é ôhmico na faixa de –20 a +20 V

e sua resistência vale 6 k.

c) O dispositivo D3 é ôhmico na faixa de –10 a +10 V

e sua resistência vale 0,5 k.

d) O dispositivo D1 é ôhmico na faixa de –30 a +30 V

e sua resistência vale 6 k.

e) O dispositivo D3 é não ôhmico na faixa de –10 a

+10 V e sua resistência vale 0,5 k.

Questão 08 - (PUC MG/2013)

O principal componente de um chuveiro elétrico é a sua

resistência elétrica. Quando ela estraga, o equipamento

deixa de aquecer a água. Sobre a capacidade do

chuveiro de aquecer a água e sua resistência elétrica, é

CORRETO afirmar:

a) Se a resistência do chuveiro for trocada por outra

de maior valor, ele irá aquecer mais a água,

fornecendo mais calor a ela.

b) Quando fechamos um pouco a torneira,

aumentamos a potência do chuveiro e a água sai

com temperatura mais elevada.

c) Quando abrimos mais a torneira, diminuímos a

potência do chuveiro fazendo com que a água saia

com temperatura mais baixa.

d) Se diminuímos a resistência elétrica do chuveiro,

aumentamos sua potência fazendo com que a

água saia com uma maior temperatura.

Questão 09 - (FPS PE/2014)

A figura abaixo mostra o circuito utilizado por uma

lanterna elétrica que utiliza duas pilhas (AA), tendo

cada pilha uma força eletromotriz nominal = 1,5 V. As

duas pilhas são ligadas em série entre si e em série com

a lâmpada, cuja resistência elétrica do seu filamento

aquecido (quando a lâmpada está ligada) vale R = 60

. Considerando que cada pilha tem uma resistência

elétrica interna r = 0,5 , a corrente elétrica resultante

será aproximadamente:

Page 101: curso de específica de física

101

a) 4,9 mA

b) 49 mA

c) 4,9 A

d) 490 mA

e) 49 A

Questão 10 - (MACK SP/2014)

No circuito desenhado abaixo, a intensidade de corrente

elétrica contínua que passa pelo resistor de 50 é de

80 mA. A força eletromotriz do gerador ideal é igual a

a) 1,5 V

b) 3,0 V

c) 4,5 V

d) 5,0 V

e) 6,0 V

Questão 11 - (FMABC/2013)

Observe o circuito abaixo. Quando a chave seletora (ch)

está posicionada em X, o amperímetro ideal (A) registra

1,5A. Quando mudamos a chave seletora para a posição

Y, o amperímetro passa a registrar 1,2A. Determine a

fem (força eletromotriz) E da bateria.

a) 3V

b) 6V

c) 12V

d) 18V

e) 24V

Questão 12 - (PUC RJ/2013)

Um determinado circuito é composto de uma bateria de

12,0 V e mais quatro resistores, dispostos como mostra

a figura.

a) Determine a corrente elétrica no ponto A indicado

na figura.

b) Determine a diferença de potencial entre os pontos

B e C apresentados na figura.

Questão 13 - (UCS RS/2013)

No circuito abaixo R1 = 10 , R2 = R3 = 6 e V = 26 V.

Qual é o valor da corrente elétrica que passa pelo ponto

A?

a) 0,4 A

b) 1,6 A

c) 2,0 A

d) 2,6 A

e) 4,0 A

Questão 14 - (UEPG PR/2013)

Os circuitos elétricos podem ser constituídos por vários

dispositivos elétricos. Abaixo é esquematizado um

circuito composto por geradores, resistores e chaves

for-mando o que é conhecido por malha. Sobre o

circuito, assinale o que for correto.

01. No circuito tem-se dois nós, três ramos e duas ma-

lhas, e a corrente elétrica atravessará todo o

circuito somente quando as chaves estiverem

fechadas.

02. As chaves estando fechadas, em um dos nós, a

soma algébrica das intensidades de correntes é

nula.

04. Estando a chave 2 aberta e E1 sendo igual a E2 a

intensidade de corrente na malha correspondente

será nula.

08. Estando a chave 1 aberta, a tensão nos terminais

do resistor equivalente será igual a soma das

f.e.m. dos geradores E2 e E3.

16. Em qualquer malha de um circuito em rede, a so-

ma algébrica das f.e.m. é igual à soma algébrica

das quedas de tensão RiE .

Questão 15 - (FUVEST SP/2013)

Em uma aula de laboratório, os alunos determinaram a

força eletromotriz e a resistência interna r de uma

bateria. Para realizar a tarefa, montaram o circuito

representado na figura abaixo e, utilizando o voltímetro,

Page 102: curso de específica de física

102

mediram a diferença de potencial V para diferentes

valores da resistência R do reostato. A partir dos

resultados obtidos, calcularam a corrente I no reostato

e construíram a tabela apresentada na página de

respostas.

a) Complete a tabela abaixo com os valores da

corrente I.

b) Utilizando os eixos abaixo, faça o gráfico de V em

função de I.

c) Determine a força eletromotriz e a resistência

interna r da bateria.

Note e adote:

Um reostato é um resistor de resistência variável.

Ignore efeitos resistivos dos fios de ligação do circuito.

Questão 16 - (FATEC SP/2013)

Como funciona uma usina nuclear?

A fissão dos átomos de urânio dentro das varetas do

elemento combustível aquece a água que passa pelo

reator a uma temperatura de 320 graus Celsius. Para

que não entre em ebulição – o que ocorreria

normalmente aos 100 graus Celsius – esta água é

mantida sob uma pressão 157 vezes maior que a

pressão atmosférica.

O gerador de vapor realiza uma troca de calor entre as

águas de um primeiro circuito e as águas de um circuito

secundário, os quais são independentes entre si. Com

essa troca de calor, as águas do circuito secundário se

transformam em vapor e movimentam a turbina, que,

por sua vez, aciona o gerador elétrico.

(eletronuclear.gov.br/Saibamais/Espa%C3%A7odoCon

hecimento/Pesquisaescolar/

EnergiaNuclear.aspx Acesso em: 15.02.2013.

Adaptado)

Usando como base apenas o texto apresentado,

identificam-se, independentemente da ordem, além da

energia nuclear, três outros tipos de energia:

a) elétrica, luminosa e eólica.

b) elétrica, eólica e mecânica.

c) elétrica, mecânica e térmica.

d) térmica, mecânica e luminosa.

e) térmica, mecânica e eólica.

Questão 17 - (UFTM/2011)

A figura mostra um circuito formado por um gerador G,

uma chave interruptora Ch e fios de ligação, todos

considerados ideais, e por três lâmpadas iguais. Na

situação inicial, a chave Ch está fechada.

Quando a chave for aberta, o circuito dissipará uma

potência elétrica

a) 50% menor do que na situação inicial.

b) 50% maior do que na situação inicial.

c) igual à dissipada na situação inicial.

d) 25% maior do que na situação inicial.

e) 25% menor do que na situação inicial.

TEXTO: 1 - Comum à questão: 18

Esta prova aborda fenômenos físicos em situações do

cotidiano, em experimentos científicos e em avanços

tecnológicos da humanidade. Em algumas questões,

como as que tratam de Física Moderna, as fórmulas

necessárias para a resolução da questão foram

fornecidas no enunciado. Quando necessário use g = 10

m/s2 para a aceleração da gravidade na superfície da

Terra e = 3.

Questão 18 - (UNICAMP SP/2010)

Telas de visualização sensíveis ao toque são muito

práticas e cada vez mais utilizadas em aparelhos

celulares, computadores e caixas eletrônicos. Uma

tecnologia frequentemente usada é a das telas

resistivas, em que duas camadas condutoras

transparentes são separadas por pontos isolantes que

impedem o contato elétrico.

Page 103: curso de específica de física

103

a) O contato elétrico entre as camadas é estabelecido

quando o dedo exerce uma força F sobre a tela,

conforme mostra a figura ao lado. A área de contato

da ponta de um dedo é igual a A = 0,25 cm2.

Baseado na sua experiência cotidiana, estime o

módulo da força exercida por um dedo em uma tela

ou teclado convencional, e em seguida calcule a

pressão exercida pelo dedo. Caso julgue necessário,

use o peso de objetos conhecidos como guia para a

sua estimativa.

b) O circuito simplificado da figura no espaço de

resposta ilustra como é feita a detecção da posição

do toque em telas resistivas. Uma bateria fornece

uma diferença de potencial U = 6 V ao circuito de

resistores idênticos de R = 2 k. Se o contato

elétrico for estabelecido apenas na posição

representada pela chave A, calcule a diferença de

potencial entre C e D do circuito.

Questão 19 - (UEFS BA/2010)

O gerador elétrico é um dispositivo que fornece energia

às cargas elétricas elementares, para que essas se

mantenham circulando.

Considerando-se um gerador elétrico que possui fem

= 40,0V e resistência interna r = 5,0, é correto afirmar

que

a) a intensidade da corrente elétrica de curto circuito

é igual a 10,0A.

b) a leitura de um voltímetro ideal ligado entre os

terminais do gerador é igual a 35,0V.

c) a tensão nos seus terminais, quando atravessado

por uma corrente elétrica de intensidade i = 2,0A,

é U = 20,0V.

d) a intensidade da corrente elétrica que o atravessa

é de 5,6A, quando a tensão em seus terminais é

de 12,0V.

e) ele apresenta um rendimento de 45%, quando

atravessado por uma corrente elétrica de

intensidade i = 3,0A.

Questão 20 - (UESPI/2008)

O circuito indicado na figura é composto por uma bateria

ideal de força eletromotriz ε e cinco resistores ôhmicos

idênticos, cada um deles de resistência elétrica R. Em tal

situação, qual é a intensidade da corrente elétrica que

atravessa a bateria ideal?

a) )R7/(3

b) )R5/(

c) )R4/(3

d) )R5/(4

e) R/

GABARITO:

1) Gab: B 2) Gab: B 3) Gab: B

4) Gab: E 5) Gab: A 6) Gab: D

7) Gab: D 8) Gab: D 9) Gab: B

10) Gab: E 11) Gab: E 12) Gab:

a) 2,7 mA

b) VBC = 1,4 V

13) Gab: C 14) Gab: 31 15) Gab:

a)

b)

c) = 1,2 V e r = 0,4

16) Gab: C

17) Gab: A

18) Gab:

a) 2,0 × 104 N/m2

b) 2 V

19) Gab: D

20) Gab: A

21) Gab: B

Page 104: curso de específica de física

104

22) Gab: i1 = 6,0 A e i2 = 3,0 A

23) Gab: C

24) Gab:

1.

2. Considerando o amperímetro e o voltímetro como

quase ideais, a resistência do amperímetro é muito

menor que a do resistor e a do voltímetro é muito

maior que a do resto. Assim, a ddp medida para os

dois voltímetros é praticamente a mesma e a

corrente medida nos amperímetros também. Mas no

caso I, a corrente é ligeiramente maior que no caso

II. Como a resistência é a razão entre a ddp no

voltímetro e a corrente no amperímetro, a

resistência no circuito II é ligeiramente maior que

em I.

Considerando os dois equipamentos ideais, o valor

da resistência medido é o mesmo para os dois

casos.

25) Gab: C 26) Gab: A 27) Gab: B

28) Gab: D 29) Gab: E 30) Gab: C

31) Gab: A 32) Gab: 11 33) Gab: C

34) Gab: 14 35) Gab: 28 36) Gab: 26

37) Gab: 11 38) Gab: C 39) Gab: A

40) Gab: C

Page 105: curso de específica de física

105

LISTA 20 – ELETRODINÂMICA 3 – LEIS DE KIRCHHOFF As Leis de Kirchhoff são empregadas em circuitos elétricos mais complexos, como por exemplo circuitos com mais de uma fonte de resistores estando em série ou em paralelo. Para estuda-las vamos definir o que são Nós e Malhas:

Nó: é um ponto onde três (ou mais) condutores são ligados. Malha: é qualquer caminho condutor fechado.

Fig. 1: Circuito com várias malhas e nós

Analisando a figura 1, vemos que os pontos a e d são nós, mas b, c, e e f não são. Identificamos neste circuito 3 malhas definidas pelos pontos: afed, adcb e badc.

Primeira lei de Kirchhoff (lei dos nós)

Em qualquer nó, a soma das correntes que o deixam(aquelas cujas apontam para fora do nó) é igual a soma das correntes que chegam até ele. A Lei é uma consequência da conservação da carga total existente no circuito. Isto é uma confirmação de que não há acumulação de cargas nos nós.

Segunda lei de Kirchhoff (lei das malhas)

A soma algébrica das forças eletromotrizes (f.e.m) em qualquer malha é igual a soma algébrica das quedas de potencial ou dos produtos iR contidos na malha.

Aplicando as leis de Kirchhoff

Exemplo 1: A figura 1 mostra um circuito cujos elementos têm os seguintes valores: E1=2,1 V, E2=6,3 V, R1=1,7 Ώ, R2=3,5 Ώ. Ache as correntes nos três ramos do circuito.

Fig. 1: Circuito com várias malhas e nós

Solução: Os sentidos das correntes são escolhidos arbitrariamente. Aplicando a 1ª lei de Kirchhoff (Lei dos Nós) temos:

i1 + i2 = i3

Aplicando a 2ª Lei de Kirchhoff (Lei das Malhas): partindo

do ponto a percorrendo a malha abcd no sentido anti-

horário. Encontramos:

ou

Se percorrermos a malha adef no sentido horário temos:

ou

Ficamos então com um sistema de 3 equações e 3 incógnitas, que podemos resolver facilmente:

Resolvendo o sistema temos que:

i1 = 0,82A i2 = -0,4A i3 = 0,42A

Os sinais das correntes mostram que escolhemos corretamente os sentidos de i1 e i3, contudo o sentido de i2 está invertido, ela deveria apontar para cima no ramo central da figura 1.

Exemplo 2: Qual a diferença de potencial entre os pontos a e d da figura 1?

Solução: Pela Lei da Malhas temos:

Page 106: curso de específica de física

106

Observe que se não alterarmos o sentido da corrente i2,

teremos que utilizar o sinal negativo quando for feito algum cálculo com essa corrente.

EXERCÍCIOS

Questão 01 - (MACK SP/2013)

No circuito ilustrado abaixo, a d.d.p. entre os pontos A

e B é constante e igual a 1,5 V e o amperímetro A é

considerado ideal. As lâmpadas, L1, L2, L3 e L4, são

idênticas entre si e cada uma delas apresenta a

seguinte indicação nominal de fábrica: 1,5 V — 0,5 W.

Ao fechar-se a chave Ch,

a) as lâmpadas L3 e L4 “queimam” e o Amperímetro

indica 0,17 A.

b) as lâmpadas L3 e L4 “queimam” e o Amperímetro

indica 0,34 A.

c) todas as lâmpadas acendem e o Amperímetro

indica 0,25A.

d) todas as lâmpadas acendem e o Amperímetro

indica 0,125A.

e) todas as lâmpadas “queimam”.

Questão 02 - (UPE/2012)

Um motor elétrico sob tensão 220 V é alimentado por

uma corrente elétrica de 10 A. A potência elétrica útil

do motor é de 2000 W. Assinale a alternativa que

corresponde à força contraeletromotriz, em volts, à

resistência interna do motor, em ohms, e ao rendimento

elétrico do motor, respectivamente.

a) 200; 2; 0,80

b) 200; 2; 0,91

c) 400; 4; 1

d) 400; 4; 0,80

e) 400; 4; 1,5

Questão 03 - (UFU MG/2012)

O circuito representado na figura abaixo mostra um

gerador de força eletromotriz (E) igual a 12 V e

resistência interna (r) de 2 , ligado a um receptor, de

força contra-eletromotriz (E´) de 8V e resistência

interna (r´) de 4 .

a) Calcule o valor da intensidade da corrente que percorre o circuito.

b) Nas condições dadas, qual o rendimento obtido pelo gerador e pelo receptor?

Questão 04 - (UFPel RS/2010)

No circuito mostrado na figura abaixo, temos uma

associação de resistores ligados a duas baterias cujas

f.e.m. são ε1 = 6,0 V e ε2 = 24,0 V e cujas resistências

internas são, respectivamente, r1 = 1,0 Ω e r2 = 2,0 Ω.

De acordo com seus conhecimentos sobre

Eletrodinâmica e com o texto, analise cada uma das

seguintes afirmativas.

I. O sentido da corrente elétrica é determinado pela

f.e.m. de maior valor, portanto, no circuito, a

corrente tem sentido horário.

II. No circuito da bateria com ε1 a corrente está

passando do polo positivo para o negativo, desta

forma, essa bateria está funcionando como um

receptor (gerador de f.c.e.m.).

III. A intensidade da corrente elétrica no circuito é de

2,0A.

IV. O valor da diferença de potencial entre os pontos A

e B é de 12 V.

Dessas afirmativas, estão corretas apenas

a) III e IV.

b) I e II.

c) I, III e IV.

d) II e IV.

e) II e III.

f) I.R.

Questão 05 - (PUC MG/2012)

Page 107: curso de específica de física

107

Um pequeno motor elétrico utilizado em veículos

automotivos (por exemplo, o motor do limpador de

para-brisas) é ligado à bateria do veículo que lhe aplica

uma voltagem VAB = 12V, fornecendo-lhe uma corrente

de 5,0 A . O motor possui uma resistência interna r =

0,2 e, devido a essa resistência, parte da energia

fornecida ao motor pela bateria transforma-se em calor

(efeito Joule), fazendo com que o motor se aqueça. A

energia restante é convertida em energia mecânica de

rotação do motor. É CORRETO afirmar:

a) A potência útil desse motor é de 55 w.

b) O calor gerado por esse motor, em 1 minuto de

funcionamento, é de 60J.

c) A potência fornecida pela bateria ao motor é de 5

W.

d) A potência dissipada por efeito joule é de 50% da

potência fornecida pela bateria ao motor.

Questão 06 - (UFV MG/2007)

As figuras abaixo representam diferentes arranjos de

transformadores num sistema de transmissão de

energia elétrica. NA, NB, NC e ND representam o número

de voltas dos enrolamentos nos transformadores.

Supondo que NA < NB e que NC > ND, o arranjo CORRETO

de transformadores para a transmissão de energia

elétrica desde a usina até a casa, por uma rede muito

longa, é:

a)

b)

c)

d)

e)

Questão 07 - (UEM PR/2013)

Analise as alternativas abaixo e assinale o que for

correto.

01. Em um circuito elétrico, o trecho entre dois nós

consecutivos é denominado ramo, e o conjunto de

ramos, que formam um percurso fechado, é

denominado malha.

02. Em um circuito elétrico, a soma das intensidades

das forças eletromotrizes em um nó é nula.

04. Ao percorrer-se uma malha, em um percurso

fechado, a soma algébrica das intensidades de

corrente elétrica nessa malha é nula.

08. A 1ª. lei de Kirchhoff, ou lei dos nós, está baseada

no princípio da conservação da carga elétrica.

16. A 2ª. lei de Kirchhoff, ou lei das malhas, está

baseada no princípio da conservação da energia.

Questão 08 - (UNIFOR CE/2013)

Em qualquer circuito de malhas múltiplas, as Leis de

Kirchhoff dos Nós e das Malhas, estabelecem

respectivamente que a soma das correntes que chegam

a um nó, é igual a soma das correntes que dele saem e

a soma algébrica das forças eletromotrizes em uma

malha é igual a soma algébrica das quedas de potencial.

De acordo com estes enunciados, podemos concluir que

a) a Lei dos Nós representa a Lei da Conservação da

Energia e a Lei das Malhas, a Lei da Conservação

do Momento Linear.

b) a Lei dos Nós representa a Lei da Conservação da

Energia e a Lei das Malhas, a Lei da Conservação

da Carga Elétrica.

c) a Lei dos Nós representa a Lei da Conservação do

Momento Linear e a Lei das Malhas, a Lei da

Conservação da Energia.

d) a Lei dos Nós representa a Lei da Carga Elétrica e

a Lei das Malhas, a Lei da Conservação do

Momento Linear.

e) a Lei dos Nós representa a Lei da Conservação da

Carga Elétrica e a Lei das Malhas, a Lei da

Conservação da Energia.

Questão 09 - (UEMG/2013)

A figura mostra duas lâmpadas ligadas a uma pilha por

fios condutores. A força eletromotriz da pilha vale 1,5

V.

Sobre essa situação, é CORRETO afirmar que

a) a tensão aplicada na lâmpada 1 é menor do que

1,5 V.

b) as duas lâmpadas estão ligadas em paralelo.

c) a corrente elétrica na lâmpada 1 é maior que na

lâmpada 2.

d) a tensão aplicada na lâmpada 2 é maior que 0,75

V.

Questão 10 - (UFG GO/2012)

Page 108: curso de específica de física

108

A figura a seguir representa um dispositivo que permite

o controle automático de iluminação. Ele é constituído

por um resistor r, uma lâmpada incandescente de

resistência RL e um fotorresistor de resistência RF. A

resistência RF diminui com o aumento da intensidade da

luz incidente sobre o fotorresistor e, para que a

luminosidade da própria lâmpada não interfira no seu

funcionamento, é adicionada sobre ele uma capa

protetora.

A tensão Vf aplicada ao circuito é de 220 V, a lâmpada

possui potência nominal de 100 W e tensão nominal de

220 V.

Tendo em vista o exposto, calcule:

a) a resistência elétrica da lâmpada, RL;

b) a corrente elétrica na lâmpada, quando RF = 3r e

RL = 11r.

Questão 11 - (UPE/2012)

Um circuito com duas malhas contém duas fontes de

tensão constante E1 = E2 = 14 V e três resistores R1 =

1,0 ohm, R2 = 3,0 ohms e R = 1,0 ohm, conforme

mostrado na figura a seguir:

Analise as seguintes proposições:

I. A corrente que passa pelo resistor R1 vale 6 A.

II. O sentido da corrente que passa pelo resistor R2 é

da esquerda para a direita.

III. A potência dissipada no resistor R2 vale 12 W.

IV. O sentido da corrente que passa pelo resistor R é

de cima para baixo.

Estão CORRETAS

a) I, II, III e IV.

b) II, III e IV.

c) I, II e III.

d) II e IV.

e) I, III e IV.

Questão 12 - (UFRJ/2009)

Uma bateria ideal de força eletromotriz está ligada a

um circuito como ilustra a figura a seguir.

Calcule a diferença de potencial BA VV entre os

pontos terminais A e B em função de .

Questão 13 - (UEM PR/2010)

Um motor elétrico de corrente contínua, com seu rotor e

suas bobinas de campo ligados em série, possui

resistência interna de 5,0 Ω. Quando ligado a uma rede

elétrica de 220 V, e girando com carga total, ele recebe

uma corrente de 4,0 A. Analise as alternativas abaixo e

assinale o que for correto.

01. A força contraeletromotriz no rotor do motor é 200

V.

02. A potência fornecida ao motor, em plena carga, é

880 W.

04. A energia dissipada na resistência interna do motor

é 80 W.

08. A potência líquida do motor é 72% da potência de

entrada.

16. Se o motor, ligado à rede elétrica de 220 V,

repentinamente deixar de girar, a potência

dissipada na resistência interna do motor cai a zero.

Questão 14 - (PUCCAMP SP/2010)

Hoje, ninguém consegue imaginar uma residência sem

eletrodomésticos (aparelho de TV, aparelho de som,

geladeira, máquina de lavar roupa, máquina de lavar

louça, etc). Uma enceradeira possui força contra-

eletromotriz de 100 V. Quando ligada a uma tomada de

120 V ela dissipa uma potência total de 40 W. Nestas

condições, a resistência interna da enceradeira, em

ohms, vale

a) 2,0

b) 3,0

c) 5,0

d) 10

e) 20

Questão 15 - (ITA SP/2008)

Durante a realização de um teste, colocou-se 1 litro de

água a 20ºC no interior de um forno de microondas.

Após permanecer ligado por 20 minutos, restou meio

litro de água. Considere a tensão da rede de 127 V e de

12 A a corrente consumida pelo forno. Calcule o fator de

rendimento do forno.

Dados: calor de vaporização da água Lv =540 cal/g;

calor específico da água c = 1cal/gºC; 1 caloria = 4,2

joules.

Questão 16 - (ESCS DF/2003)

Duas lâmpadas incandescentes M e N são ligadas em

220V. A lâmpadas M tem potência P e a área da seção

reta de seu filamento é S. A lâmpada N tem potência P1,

seu filamento é do mesmo material e tem o mesmo

Page 109: curso de específica de física

109

comprimento da lâmpada M, e a área da seção reta é 4

S. A relação entre as potências P e P1 é:

a) 1/4

b) 1/2

c) 2

d) 4

e) 16

Questão 17 - (UFRJ/2010)

Um estudante dispunha de duas baterias comerciais de

mesma resistência interna de 0,10 , mas verificou, por

meio de um voltímetro ideal, que uma delas tinha força

eletromotriz de 12 Volts e a outra, de 11Volts. A fim de

avaliar se deveria conectar em paralelo as baterias para

montar uma fonte de tensão, ele desenhou o circuito

indicado na figura a seguir e calculou a corrente i que

passaria pelas baterias desse circuito.

a) Calcule o valor encontrado pelo estudante para a

corrente i.

b) Calcule a diferença de potencial VA − VB entre os

pontos A e B indicados no circuito.

TEXTO: 1 - Comuns às questões: 18, 19

A Revolução Industrial consistiu em um conjunto de

mudanças tecnológicas com profundo impacto no

processo produtivo em nível econômico e social. Iniciada

na Inglaterra em meados do século XVIII, expandiu-se

pelo mundo a partir do século XIX. James Hargreaves,

1764, na Grã-Bretanha, inventa a fiadora “spinning

Jenny”, uma máquina de fiar rotativa que permitia a um

único artesão fiar oito fios de uma só vez.; James Watt,

1768, inventa a máquina a vapor; Gottlieb Daimler,

1885, inventou um motor a explosão etc.

Questão 18 - (UEPB/2010)

Acerca do assunto tratado no texto I, em relação ao

motor elétrico, analise as proposições a seguir,

escrevendo V ou F conforme sejam verdadeiras ou

falsas, respectivamente:

( ) O motor elétrico é um elemento de trabalho que

converte energia elétrica em energia mecânica de

rotação.

( ) O motor elétrico é uma máquina que converte

energia mecânica de rotação em energia elétrica.

( ) Um motor elétrico é uma aplicação do princípio

fundamental do eletromagnetismo que afirma que

uma força magnética vai atuar sobre um condutor

elétrico se esse condutor estiver convenientemente

colocado num campo magnético e for percorrido por

uma corrente elétrica.

Após a análise feita, assinale a alternativa que

corresponde à sequência correta:

a) VVV

b) FVF

c) VVF

d) FVV

e) VFV

Questão 19 - (UEPB/2010)

Ainda acerca do assunto tratado no texto I, resolva a

seguinte situação-problema:

Um motor elétrico tem resistência interna de 2,0 e está

ligado a uma ddp de 100 V. Verifica-se que ele é

percorrido por uma corrente elétrica de intensidade igual

a 5,0 A. A força contra-eletromotriz do motor e a

potência total recebida pelo motor, respectivamente,

são

a) 80 V; 350 W

b) 90 V; 450 W

c) 90 V; 500 W

d) 70 V; 300 W

e) 100 V; 400 W

Questão 20 - (UEPG PR/2010)

Dispositivos que transformam outras formas de energia

em energia elétrica são conhecidos como geradores

elétricos. Dispositivos capazes de receber energia

elétrica e transformá-la em outras formas de energia

que não a térmica são denominados receptores

elétricos. Sobre geradores elétricos e receptores

elétricos, assinale o que for correto.

01. Quando um gerador encontra-se em circuito

aberto, a diferença entre seus terminais é igual a

sua força eletromotriz.

02. A potência útil fornecida por um gerador a um

circuito onde só existem resistores será máxima

se a resistência equivalente do circuito for igual à

resistência interna do gerador.

04. Geradores elétricos podem ser associados

somente em paralelo.

08. O rendimento elétrico de um receptor corresponde

ao produto entre a potência elétrica útil e a

potência elétrica fornecida ao receptor.

16. A força contra-eletromotriz pode, em termos

práticos, ser pensada como uma força eletromotriz

que se opõe à passagem da corrente elétrica.

Page 110: curso de específica de física

110

GABARITO:

1) Gab: C 2) Gab: B

3) Gab:

a) A3

2i ou i 0,67 A

b) Gerador: 9

8G ou 0,89

Receptor: 4

3R ou 0,75

4) Gab: E 5) Gab: A 6) Gab: D

7) Gab: 25 8) Gab: E

9) Gab: A 10) Gab:

a) RL = 484

b) 0,32 A.

11) Gab: E

12) Gab:

A corrente que sai da bateria se reparte em duas iguais,

de valor )R3/(i , pois segue dois caminhos com a

mesma resistência 3R e sob a mesma tensão ε .

Percorrendo o caminho de A até B que passa,

inicialmente, pelo resistor de resistência R e, depois,

pelo de resistência 2R, obtemos

Rii R2)1(RVV BA .

Substituindo o valor da corrente, obtemos

3/VV BA .

13) Gab: 03 14) Gab: D 15) Gab: %80

16) Gab: A 17) Gab:

a) i = 5,0 A

b) VA − VB=11,5 V

18) Gab: E 19) Gab: C 20) Gab: 19

21) Gab:

a) I = 2,0 A

b) P2 = 0,80 W

c) VM = 8,0 V

22) Gab: E 23) Gab: E 24) Gab: C

25) Gab:

i = A8,120

36

20

E

26) Gab: B

27) Gab: A

28) Gab:

a)

iRVab2

7

b)

R7

3i

2

112

29) Gab: A

30) Gab: E

31) Gab: A 32) Gab: D 33) Gab: E 34) Gab: A 35) Gab: D 36) Gab: A 37) Gab: 33 38) Gab: B 39) Gab: E 40) Gab: 30

Page 111: curso de específica de física

111

LISTA 21 – ÓPTICA 1 – FUNDAMENTOS E REFLEXÃO DA LUZ

A Óptica é a parte da Física responsável pelo estudo da luz e dos fenômenos associados a ela. Como a luz apresenta comportamento dual, podendo ser considerada como onda ou partícula, os estudos da Óptica dividem-se em duas partes:

Óptica física – quando se considera a natureza ondulatória

da luz;

Óptica geométrica – quando a luz é considerada uma

partícula e seus estudos são feitos a partir do conceito de raios de luz, conferindo um modelo geométrico para a luz. Definições importantes da Óptica Geométrica Como o foco deste texto é apenas a Óptica Geométrica, antes de conhecermos seus princípios, vejamos algumas definições importantes: Os raios de luz são segmentos de reta que representam a direção e o sentido de propagação da luz. Eles podem ser emitidos por dois tipos de fonte:

Fontes primárias: que emitem luz própria, como o sol, a

chama de uma vela ou uma lâmpada;

Fontes secundárias: que refletem a luz que recebem de

uma fonte primária, como a lua que reflete a luz que recebe do sol, ou um livro, que só pode ser visto se refletir a luz que recebe de uma lâmpada. As fontes luminosas também podem ser classificadas em relação à sua dimensão:

Fontes extensas: quando possuem dimensões consideráveis se comparadas às dimensões do objeto a ser

iluminado. Por exemplo: uma lâmpada acesa perto de um livro;

Fontes pontuais: se as dimensões da fonte de luz forem consideradas desprezíveis em relação ao objeto a ser iluminado. Um conjunto de raios de luz constitui um feixe de luz. A luz emitida por uma fonte pontual propaga-se em todas as direções, sendo assim, ele é denominado feixe divergente de raios de luz. Quando os raios são paralelos, como no caso da luz emitida por uma lanterna, dizemos que o feixe de luz é convergente.

O feixe de luz emitido pela lanterna possui raios de luz paralelos, ou seja, é convergente Princípios da Óptica Geométrica Existem três princípios adotados pela Óptica Geométrica para

explicar os fenômenos luminosos. O primeiro é denominado Princípio da Propagação Retilínea da Luz e afirma que: “Em meios homogêneos e transparentes, a luz propaga-se em linha reta.” Esse princípio explica vários fenômenos, como a semelhança geométrica entre a sombra e o objeto que a produz, além da formação de penumbra e dos eclipses. O segundo princípio da Óptica Geométrica é o da independência dos raios luminosos, que tem o seguinte enunciado:

“Quando dois ou mais feixes de luz se cruzam, um não altera a propagação do outro.”

Os dois feixes de luz interceptam-se e continuam propagando-se na mesma direção Por fim, o terceiro princípio, que é o da reversibilidade dos raios luminosos: “A trajetória seguida pela luz independe do seu sentido

de propagação.” A Óptica Geométrica é responsável pelo estudo de vários conceitos físicos, entre eles a formação de sombra, penumbra e eclipse; a reflexão e a refração da luz, bem como a formação da imagem em espelhos, nas lentes e nos instrumentos ópticos.

Espelho Plano

Pode-se considerar como um espelho plano, qualquer superfície plana que seja capaz de refletir a luz incidente. Assim, os espelhos planos podem ser encontrados em diversos formatos (circular, triangular, polígonos, etc...), em diferentes objetos (mesa, chapa de metal, superfície de um lago sem ondas, etc...), desde que a superfície tenha a característica de sempre ser plana e muito bem polida, para que exista o reflexo dos raios de luz. Entre os elementos ópticos o espelho plano é o considerado mais simples.

Figura 1 – Ilustra raios incidentes na superfície de um espelho plano. Vejamos como se formam as imagens nesse caso. Para isso temos a situação: um ponto (P) de um objeto que está a certa distancia (d) de um espelho plano, conforme indica a figura 2 abaixo:

Page 112: curso de específica de física

112

Figura 2 – Reflexão e prolongamento de dois raios de luz refletidos pelo espelho. Repare que na parte de trás do espelho encontramos uma imagem refletida, o ponto P’, que é fruto do prolongamento dos dois raios de luz emanados do ponto P ao incidirem o plano do espelho. A intersecção dos raios prolongados decorre então das leis de reflexão, que também envolvem o princípio de Fermat, que diz: “de todos os caminhos possíveis para ir de um ponto a outro, a luz segue aquele que é percorrido no tempo mínimo.” Pierre Fermat, entre seus estudos encontrou um método novo para época (1657) para determinar a trajetória dos raios luminosos, baseado na ideia de que a natureza sempre atua pelo caminho mais curto. Os conceitos de simetria significam que o ponto P e P’, permanecem na mesma reta normal ao espelho e estão equidistantes (d = d’) a superfície refletora. Podemos notar que o objeto (P) e a imagem (P’) possuem o mesmo tamanho, e, em caso de movimento relativo ao espelho, possuirão iguais velocidades. Uma outra característica das imagens formadas pelos espelhos planos é a de que elas são enantiomorfas, ou seja, a simetria de dois objetos que não podem ser sobrepostos. Na formação da imagem existe uma inversão da direita para a esquerda e não de baixo para cima. Por exemplo, uma imagem refletida da mão esquerda de uma pessoa será a mão direita, no entanto a imagem dos pés refletidos não significa que eles estão na cabeça. Isso nos leva a crer que nunca na vida, uma pessoa conseguiu

observar a própria face como ela realmente é (utilizando espelhos). São muitos os exemplos em nosso dia a dia dessa aplicação, um bem comum dessa característica está nas ambulâncias, pois se repararmos na frente delas está escrita a palavra ambulância invertida, isso porque o carro que estiver a frente ao observá-la pelo retrovisor verá a palavra escrita corretamente.

Figura 3 – Exemplo de imagem real que será invertida ao ser observado por um espelho plano. Pelas figuras que vimos anteriormente, percebemos que um objeto localizado na frente do espelho plano (objeto real)

após os raios de luz atingirem o espelho, os prolongamentos dos raios nos fornecem uma imagem que dá a impressão de estar situada atrás do espelho (virtual). Nesse sentido não podemos dizer que todo espelho plano fornece uma imagem virtual, isso somente ocorrerá se os raios incidentes forem de um objeto real, caso contrário, uma imagem virtual nos fornecerá uma o reflexo de uma imagem real. Contudo, o objeto e a imagem são constatados como de naturezas opostas. Ao observarmos por espelho plano podemos definir o seu campo visual que nada mais é do que a região do espaço que pode ser vista pelo observador através de um espelho. Para determinarmos a região do campo visual, basta tomar o ponto P’, simétrico de P, e prolongarmos as linhas das extremidades do espelho plano.

Figura 4 – Definição do campo visual para espelhos planos. Quando usamos apenas um espelho plano observamos uma única imagem de cada objeto. Porém se colocarmos o objeto entre dois espelhos que formam um ângulo entre si, poderemos notar mais de duas imagens. O número de imagens nada mais é do que o resultado de sucessivas reflexões nos dois espelhos, que aumenta a medida que o ângulo entre os espelhos diminui.

Figura 5 – Ilustra a associação de dois espelhos com um determinado ângulo. Imagem: http://www.dma.uem.br/matemativa/conteudo/exposicao/simetrias/ frisos_rosetas/2_espelhos_articulados/s08_15.JPG De maneira geral, podemos utilizar uma expressão matemática que relaciona o número de imagens n com o ângulo entre os espelhos a.

EXERCÍCIOS

Questão 01 - (FMJ SP/2014)

Fibra óptica é um filamento de vidro, ou de materiais

poliméricos, com capacidade de transmitir luz. O

filamento pode ter diâmetros variáveis, dependendo de

sua aplicação, indo desde diâmetros mais finos que um

fio de cabelo até alguns milímetros. A transmissão da

luz em seu interior se dá, basicamente,

a) por sucessivas reflexões.

Page 113: curso de específica de física

113

b) alternando reflexões com difrações.

c) por sucessivas refrações.

d) alternando refrações com reflexões.

e) alternando refrações com difrações.

Questão 02 - (UNESP/2013)

Cor da chama depende do elemento queimado

Por que a cor do fogo varia de um material para outro?

A cor depende basicamente do elemento químico em

maior abundância no material que está sendo

queimado. A mais comum, vista em incêndios e em

simples velas, é a chama amarelada, resultado da

combustão do sódio, que emite luz amarela quando

aquecido a altas temperaturas. Quando, durante a

combustão, são liberados átomos de cobre ou bário,

como em incêndio de fiação elétrica, a cor da chama

fica esverdeada.

(Superinteressante, março de 1996. Adaptado.)

A luz é uma onda eletromagnética. Dependendo da

frequência dessa onda, ela terá uma coloração

diferente. O valor do comprimento de onda da luz é

relacionado com a sua frequência e com a energia que

ela transporta: quanto mais energia, menor é o

comprimento de onda e mais quente é a chama que

emite a luz. Luz com coloração azulada tem menor

comprimento de onda do que luz com coloração

alaranjada.

(http://papofisico.tumblr.com. Adaptado.)

Baseando-se nas informações e analisando a imagem,

é correto afirmar que, na região I, em relação à região

II,

a) a luz emitida pela chama se propaga pelo ar com

maior velocidade.

b) a chama emite mais energia.

c) a chama é mais fria.

d) a luz emitida pela chama tem maior frequência.

e) a luz emitida pela chama tem menor comprimento

de onda.

Questão 03 - (UFPA/2012)

Em 29 de maio de 1919, em Sobral (CE), a teoria da

relatividade de Einstein foi testada medindo-se o desvio

que a luz das estrelas sofre ao passar perto do Sol. Essa

medição foi possível porque naquele dia, naquele local,

foi visível um eclipse total do Sol. Assim que o disco

lunar ocultou completamente o Sol foi possível observar

a posição aparente das estrelas. Sabendo-se que o

diâmetro do Sol é 400 vezes maior do que o da Lua e

que durante o eclipse total de 1919 o centro do Sol

estava a 151 600 000 km de Sobral, é correto afirmar

que a distância do centro da Lua até Sobral era de

a) no máximo 379 000 km

b) no máximo 279 000 km

c) no mínimo 379 000 km

d) no mínimo 479 000 km

e) exatamente 379 000 km

Questão 04 - (UNIFOR CE)

O esquema representa o alinhamento do Sol, d Terra e

da Lua no momento de um eclipse.

TERRA

LUA

SOL

A

Neste instante, uma pessoa situada no ponto A

observará um eclipse:

a) parcial da Lua

b) total da Lua

c) anular do Sol

d) parcial do Sol

e) total do Sol

Questão 05 - (FUVEST SP)

Uma câmera de segurança (C), instalada em uma sala,

representada em planta na figura, “visualiza” a região

clara indicada. Desejando aumentar o campo de visão

da câmera, foi colocado um espelho plano, retangular,

ocupando toda a região da parede entre os pontos A e

B.

Nessas condições, a figura que melhor representa a

região clara, que passa a ser visualizada pela câmera, é

Page 114: curso de específica de física

114

Questão 06 - (FM Petrópolis RJ/2014)

Um objeto é colocado entre dois espelhos planos cujas

superfícies refletoras formam um ângulo . Sabe-se que

a medida de é um divisor positivo de 24 e que o

número total de imagens que esse objeto produz é

maior que 17 e menor que 59.

Quantos são os possíveis valores de ?

a) 2

b) 3

c) 4

d) 5

e) 6

Questão 07 - (UFG GO/2014)

A figura a seguir representa um dispositivo óptico

constituído por um laser, um espelho fixo, um espelho

giratório e um detector. A distância entre o laser e o

detector é d = 1,0 m, entre o laser e o espelho fixo é

m 3h e entre os espelhos fixo e giratório é D = 2,0

m.

Sabendo-se que = 45º, o valor do ângulo para que

o feixe de laser chegue ao detector é:

a) 15º

b) 30º

c) 45º

d) 60º

e) 75º

Questão 08 - (UNESP/2014)

Uma pessoa está parada numa calçada plana e

horizontal diante de um espelho plano vertical E

pendurado na fachada de uma loja. A figura representa

a visão de cima da região.

Olhando para o espelho, a pessoa pode ver a imagem

de um motociclista e de sua motocicleta que passam

pela rua com velocidade constante V = 0,8 m/s, em

uma trajetória retilínea paralela à calçada, conforme

indica a linha tracejada. Considerando que o ponto O na

figura represente a posição dos olhos da pessoa parada

na calçada, é correto afirmar que ela poderá ver a

imagem por inteiro do motociclista e de sua motocicleta

refletida no espelho durante um intervalo de tempo, em

segundos, igual a

a) 2.

b) 3.

c) 4.

d) 5.

e) 1.

Questão 09 - (UNIRG TO/2014)

Para realizar um experimento, um estudante fixou uma

ponteira laser na parede, formando um ângulo θ=30°

com a vertical, a qual aponta diretamente para um

espelho móvel, conforme a figura a seguir.

Considerando-se que o espelho se aproxima da parede

com uma velocidade constante de 1,0 m/s, conclui-se

que a velocidade vertical da imagem na parede, em

m/s, é a seguinte:

a) j 3v

b) j /33v

c) j /33v

d) j 3v

Page 115: curso de específica de física

115

Questão 10 - (Fac. de Ciências da Saúde de

Barretos SP/2013)

Um objeto O está parado entre dois espelhos planos, E1

e E2, a uma distância x do primeiro e y do segundo,

como mostra a ilustração.

Com o objeto mantido parado, aumentou-se de 2 m a

distância de cada espelho em relação ao objeto, de

modo que a distância entre as imagens dobrou. A

distância inicial em metros entre os espelhos é

a) 6.

b) 3.

c) 2.

d) 5.

e) 4.

Questão 11 - (UNESP/2010)

O fenômeno de retrorreflexão pode ser descrito como o

fato de um raio de luz emergente, após reflexão em dois

espelhos planos dispostos convenientemente, retornar

paralelo ao raio incidente. Esse fenômeno tem muitas

aplicações práticas.

No conjunto de dois espelhos planos mostrado na

figura, o raio emergente intersecta o raio incidente em

um ângulo . Da forma que os espelhos estão dispostos,

esse conjunto não constitui um retrorrefletor.

Determine o ângulo , em função do ângulo , para a

situação apresentada na figura e o valor que o ângulo

deve assumir, em radianos, para que o conjunto de

espelhos constitua um retrorrefletor.

Questão 12 - (UEG GO/2009)

Na figura abaixo, o logo do Núcleo de Seleção da UEG é

colocado em frente a dois espelhos planos (E1 e E2) que

formam um ângulo de 90º.

Qual alternativa corresponde às três imagens formadas

pelos espelhos?

a)

b)

c)

d)

Questão 13 - (UFSCar SP/2008)

Um dia, um cão, carregando um osso na boca, ia

atravessando uma ponte. Olhando para baixo, viu sua

própria imagem refletida na água. Pensando ver outro

cão, cobiçou-lhe logo o osso que este tinha na boca, e

pôs-se a latir. Mal, porém, abriu a boca, seu próprio osso

caiu na água e perdeu-se para sempre.

(Fábula de Esopo.)

Page 116: curso de específica de física

116

a) Copie a figura seguinte em seu caderno de

respostas.

Do ponto de vista de um observador que pudesse

enxergar os dois meios ópticos, ar e água, produza

um esquema de raios de luz que conduzem à

imagem do osso, destacando os raios incidentes e

refletidos, seus ângulos e as normais, que indicarão

a localização da imagem dos pontos A e B.

b) Admita 10,0 m/s2 o valor da aceleração da gravidade

e que a resistência do ar ao movimento de queda do

osso é desprezível. Se o osso largado pelo cachorro

atingiu a superfície da água em 0,4 s, determine a

distância que separava o cão ganancioso de sua

imagem, no momento em que se iniciou a queda do

osso.

Questão 14 - (FEI SP/2008)

Uma pessoa anda em direção a um espelho plano com

velocidade V. É correto afirmar que:

Adote g = 10 m/s2

a) Sua imagem se aproxima com velocidade igual a V.

b) Sua imagem se afasta com velocidade igual a V.

c) Sua imagem se afasta com uma velocidade maior

que V.

d) Sua imagem se aproxima com uma velocidade

maior que V.

e) A distância entre a pessoa e a imagem permanece

constante.

Questão 15 - (PUC GO/2014)

O Livro e a América

Talhado para as grandezas,

P’ra crescer, criar, subir,

O Novo Mundo nos músculos

Sente a seiva do porvir.

– Estatuário de colossos –

Cansado doutros esboços

Disse um dia Jeová:

“Vai, Colombo, abre a cortina

“Da minha eterna oficina...

“Tira a América de lá.”

Molhado inda do dilúvio,

Qual Tritão descomunal,

O continente desperta

No concerto universal.

Dos oceanos em tropa

Um – traz-lhe as artes da Europa,

Outro – as bagas de Ceilão...

E os Andes petrificados,

Como braços levantados,

Lhe apontam para a amplidão.

Olhando em torno então brada:

“Tudo marcha!... Ó grande Deus!

As cataratas – p’ra terra,

As estrelas – para os céus

Lá, do pólo sobre as plagas,

O seu rebanho de vagas

Vai o mar apascentar...

Eu quero marchar com os ventos,

Com os mundos... co’os firmamentos!!!”

E Deus responde – “Marchar!”

[...]

(ALVES, Castro. Melhores poemas de Castro Alves.

São Paulo: Global, 2003. p. 15-16.)

A poesia de Castro Alves caracteriza-se pela des-

crição de espaços amplos, como se percebe na última

estrofe do texto. Se observarmos o Sol no horizonte,

bem ao final de uma tarde de céu límpido, vemos que

sua cor aparenta-se “avermelhada”. A explicação para

esse fato é que:

a) A atmosfera da Terra espalha ondas com compri-

mentos menores, como o azul e o verde.

b) A atmosfera da Terra absorve ondas com compri-

mentos menores, como o azul e o verde.

c) A atmosfera da Terra reflete a luz que é emitida

pelo Sol.

d) A temperatura do Sol é menor ao final da tarde.

Questão 16 - (UEG GO/2013)

Explique, utilizando conceitos de ótica geométrica,

porque a impressão de tamanho de um objeto muda

quando um observador enxerga este objeto em duas

posições diferentes.

TEXTO: 1 - Comuns às questões: 17, 18

Foi René Descartes, em 1637, o primeiro a discutir

claramente a formação do arco-íris. Ele escreveu:

“Considerando que esse arco-íris aparece não apenas

no céu, mas também no ar perto de nós, sempre que

haja gotas de água iluminadas pelo Sol, como podemos

ver em certas fontes, eu imediatamente entendi que

isso acontece devido apenas ao caminho que os raios

de luz traçam nessas gotas e atingem nossos olhos.

Ainda mais, sabendo que as gotas são redondas, como

fora anteriormente provado e, mesmo que sejam

grandes ou pequenas, a aparência do arco-íris não

muda de forma nenhuma, tive a ideia de considerar

uma bem grande, para que pudesse examinar

melhor...”

Questão 17 - (PUC MG/2012)

Assinale os fenômenos ópticos responsáveis pela

formação do arco-íris:

a) refração e reflexão

b) difração e refração

c) reflexão e interferência

d) interferência e refração

Questão 18 - (PUC MG/2012)

Quanto ao fenômeno da dispersão das cores do arco-

íris, é CORRETO afirmar:

a) A dispersão da luz consiste em um fenômeno em

que a luz branca é decomposta em cores

fundamentais.

b) A refração da luz é maior para o vermelho que para

o violeta.

c) Quando a luz está se propagando no ar e atinge

uma gota de água, por exemplo, a velocidade da

luz muda para outro valor, maior do que quando

estava se propagando no ar.

Page 117: curso de específica de física

117

d) Outro fenômeno que pode ser explicado a partir da

dispersão é que, durante o dia, o céu se apresenta

na cor azul, mas no entardecer passa a ter

coloração avermelhada.

Questão 19 - (UFU MG/2010)

Ao olhar para um objeto (que não é uma fonte

luminosa), em um ambiente iluminado pela luz branca,

e constatar que ele apresenta a cor amarela, é correto

afirmar que:

a) O objeto absorve a radiação cujo comprimento de

onda corresponde ao amarelo.

b) O objeto refrata a radiação cujo comprimento de

onda corresponde ao amarelo.

c) O objeto difrata a radiação cujo comprimento de

onda corresponde ao amarelo.

d) O objeto reflete a radiação cujo comprimento de

onda corresponde ao amarelo.

Questão 20 - (UFSCar SP/2008)

A 1 metro da parte frontal de uma câmara escura de

orifício, uma vela de comprimento 20 cm projeta na

parede oposta da câmara uma imagem de 4 cm de

altura.

A câmara permite que a parede onde é projetada a

imagem seja movida, aproximando-se ou afastando-se

do orifício. Se o mesmo objeto for colocado a 50 cm do

orifício, para que a imagem obtida no fundo da câmara

tenha o mesmo tamanho da anterior, 4 cm, a distância

que deve ser deslocado o fundo da câmara,

relativamente à sua posição original, em cm, é de

a) 50.

b) 40.

c) 20.

d) 10.

e) 5.

GABARITO:

1) Gab: A 2) Gab: C 3) Gab: A

4) Gab: E 5) Gab: B 6) Gab: A

7) Gab: D 8) Gab: B 9) Gab: D

10) Gab: E

11) Gab:

= – 2

rad2

12) Gab: A

13) Gab:

a) Sendo A’ e B’ a imagem de A e B, respectivamente,

podemos ter:

b) 2d = 1,6 m

14) Gab: D

15) Gab: A

16) Gab:

Um objeto AB tem sua imagem real A’B’ projeta na

retina pelo sistema ótico. O ângulo visual do observador

muda quando ele muda de posição e isso faz com que

a imagem A’B’ mude de tamanho e, embora o tamanho

do objeto seja sempre o mesmo, o observador tem a

impressão de que ele muda de tamanho.

17) Gab: A 18) Gab: A 19) Gab: D

20) Gab: D 21) Gab: C 22) Gab: E

23) Gab: 10 24) Gab: A 25) Gab: D

26) Gab: E 27) Gab: A 28) Gab:

a) c

L2t

b)

c) s/m10.1,3c 8

29) Gab:

a) e b)

c) = 2º

d) AB = 330 cm

30) Gab: A 31) Gab: B 32) Gab: A

33) Gab: 06 34) Gab: C 35) Gab: A 36) Gab: D 37) Gab: 41 38) Gab: E 39) Gab:

a) D = 0,15 m b) S = 4,44 106W/m2

c) T = 500 K

40) Gab: d = 0,8 m e x = 0,15 m

NV2

1t

Page 118: curso de específica de física

118

LISTA 22 – REFLEXÃO DA LUZ ESPELHOS ESFÉRICOS

Espelhos Esféricos

Espelho Esférico

Com grande aplicação no dia a dia, o espelho esférico é uma calota esférica que possui uma de suas partes polida e com alto poder de reflexão. Esse espelho pode ser classificado de acordo com a superfície refletora. Se essa for interna, o espelho é côncavo; e se a superfície refletora é a externa, o espelho é convexo. Podemos representar essas duas classificações de espelhos esféricos da seguinte forma:

As imagens fornecidas por esses espelhos são sensivelmente distorcidas em relação aos objetos correspondentes. Essas distorções são chamadas de aberrações de esfericidade. De forma a minimizar essas aberrações, um físico e matemático alemão, chamado Karl Fredrich Gauss, em seus estudos, observou que se os raios luminosos incidissem paralelamente ou pouco inclinados e mais afastados do eixo principal, as aberrações de esfericidade ficariam sensivelmente minimizadas. Gauss fez uma série de trabalhos sobre a óptica, em especial envolvendo sistemas de várias lentes. Os espelhos esféricos, tanto côncavos quanto convexos, são muito utilizados em nosso cotidiano. Nos estojos de maquiagem, nos refletores atrás das lâmpadas de sistema de iluminação e projeção (lanternas e faróis, por exemplo), nas objetivas de telescópios, etc., são utilizados os espelhos esféricos côncavos. Já os espelhos esféricos convexos são utilizados, por exemplo, em retrovisores de automóveis.

Observe a gravura abaixo e conheça as características dos espelhos esféricos.

a) espelho côncavo b) espelho convexo C é o centro de curvatura; R é o raio da curvatura do espelho; CV é o eixo principal do espelho; F é o foco do espelho; V é o vértice do espelho. Como se percebe, as características descritas acima se aplicam tanto a espelhos côncavos quanto a espelhos convexos.

Focos dos espelhos esféricos

Para os espelhos côncavos de Gauss pode ser verificar que todos os raios luminosos que incidirem ao longo de uma direção paralela ao eixo secundário passam por (ou convergem para) um mesmo ponto F - o foco principal do espelho.

No caso dos espelhos convexos é a continuação do raio refletido é que passa pelo foco. Tudo se passa como se os raios refletidos se originassem do foco.

Page 119: curso de específica de física

119

Determinação de imagens

Analisando objetos diante de um espelho esférico, em posição perpendicular ao eixo principal do espelho podemos chegar a algumas conclusões importantes.

Um objeto pode ser real ou virtual. No caso dos espelhos, dizemos que o objeto é virtual se ele se encontra “atrás” do espelho. No caso de espelhos esféricos a imagem de um objeto pode ser maior, menor ou igual ao tamanho do objeto. A imagem pode ainda aparecer invertida em relação ao objeto. Se não houver sua inversão dizemos que ela é direita.

Equação fundamental dos espelhos esféricos

Dadas a distância focal e posição do objeto é possível determinar, analiticamente, a posição da imagem. Através da equação de Gauss, que é expressa por:

EXERCÍCIOS

Questão 01 - (Fac. Santa Marcelina SP/2014)

A fim de observar minuciosamente os dentes de um

paciente, um dentista utiliza um espelho esférico

côncavo, obtendo uma imagem virtual, direita e

ampliada do dente.

Para isso, o dentista posiciona o espelho próximo ao

dente, de tal forma que o dente permaneça

a) entre o vértice e o foco principal do espelho.

b) entre o foco principal e o centro de curvatura do

espelho.

c) sobre o foco principal do espelho.

d) sobre o centro de curvatura do espelho.

e) após o centro de curvatura do espelho.

Questão 02 - (FATEC SP/2014)

Como foi que um arranha-céus “derreteu” um

carro?

“É uma questão de reflexo. Se um prédio é curvilíneo e

tem várias janelas planas, que funcionam como

espelhos, os reflexos se convergem em um ponto” diz

Chris Shepherd, do Instituto de Física de Londres.

O edifício de 37 andares, ainda em construção, é de fato

um prédio curvilíneo e o carro, um Jaguar, estava

estacionado em uma rua próxima ao prédio,

exatamente no ponto atingido por luzes refletidas e não

foi o único que sofreu estrago.

O fenômeno é consequência da posição do Sol em um

determinado período do ano e permanece nessa posição

por duas horas por dia. Assim, seus raios incidem de

maneira oblíqua às janelas do edifício.

(bbc.co.uk/portuguese/noticias/2013/09/130904_como_luz

refletida_

derrete_carro_an.shtml Acesso em: 13.09.2013.

Adaptado. Foto: Original colorido)

Considerando o fato descrito e a figura da pessoa

observando o reflexo do Sol no edifício, na mesma

posição em que estava o carro quando do incidente,

podemos afirmar corretamente que o prédio se

assemelha a um espelho

a) plano e o carro posicionou-se em seu foco infinito.

b) convexo e o carro posicionou-se em seu foco

principal.

c) convexo e o carro posicionou-se em um foco

secundário.

d) côncavo e o carro posicionou-se em seu foco

principal.

e) côncavo e o carro posicionou-se em um foco

secundário.

Questão 03 - (Unicastelo SP/2014)

Na fotografia, pode-se ver as imagens de um mesmo

carro, circulado em vermelho, formadas por dois

espelhos, 1 e 2.

Page 120: curso de específica de física

120

(http://blog.brasilacademico.com. Adaptado.)

Comparando as características dessas imagens e

sabendo que o espelho 1 é esférico e o espelho 2 é

plano, é correto afirmar que o espelho 1 é

a) convexo e a imagem conjugada por ele é virtual.

b) côncavo e a imagem conjugada por ele é real.

c) côncavo e a imagem conjugada por ele é virtual.

d) convexo e a imagem conjugada por ele é

imprópria.

e) convexo e a imagem conjugada por ele é real.

Questão 04 - (PUC SP/2014)

Um estudante de física resolve brincar com espelhos

esféricos e faz uma montagem, utilizando um espelho

esférico côncavo de raio de curvatura igual a 80 cm e

outro espelho convexo de raio de curvatura cujo módulo

é igual a 40 cm. Os espelhos são cuidadosamente

alinhados de tal forma que foram montados

coaxialmente, com suas superfícies refletoras se

defrontando e com o vértice do espelho convexo

coincidindo com a posição do foco principal do espelho

côncavo. O aluno, então, colocou cuidadosamente um

pequeno objeto no ponto médio do segmento que une

os vértices desses dois espelhos. Determine, em relação

ao vértice do espelho convexo, a distância, em

centímetros, da imagem, formada por esse espelho ao

receber os raios luminosos que partiram do objeto e

foram refletidos pelo espelho côncavo, e classifique-a.

a) 16cm, virtual e direita

b) 16cm, virtual e invertida

c) 40cm, real e direita

d) 40cm, virtual e direita

e) 13,3cm, virtual e invertida

Questão 05 - (UFG GO/2013)

O sistema óptico encontrado no farol de um automóvel

é constituído por um espelho côncavo e uma lâmpada

posicionada sobre o seu eixo de simetria. Considerando-

se que o feixe de luz proveniente desse farol seja

divergente, a posição da lâmpada deve ser

a) sobre a posição focal.

b) entre o vértice e a posição focal.

c) entre a posição focal e o centro de curvatura.

d) após o centro de curvatura.

e) sobre a posição do centro de curvatura.

Questão 06 - (UFG GO/2012)

Alguns veículos possuem espelhos retrovisores

convexos no lado direito e, em alguns desses espelhos,

lê-se a seguinte frase:

“Objetos no espelho estão mais próximos do que parece.”

Isso ocorre porque o cérebro associa o tamanho da

imagem com o inverso da distância. Essa associação

deve-se ao fato de que quanto maior for a distância do

objeto ao observador menor será a imagem formada na

retina.

Um automóvel possui um retrovisor direito convexo

com raio de curvatura R. Seu motorista observa por

esse espelho, localizado a uma distância R do seu olho,

um automóvel de 2,0 metros de altura que se encontra

a 20R de distância do espelho.

Considerando o exposto, calcule:

a) o tamanho da imagem no espelho retrovisor;

b) a distância, como estimada pelo cérebro, do objeto

ao espelho retrovisor.

Questão 07 - (UNESP/2012)

Observe o adesivo plástico apresentado no espelho

côncavo de raio de curvatura igual a 1,0 m, na figura 1.

Essa informação indica que o espelho produz imagens

nítidas com dimensões até cinco vezes maiores do que

as de um objeto colocado diante dele.

Considerando válidas as condições de nitidez de Gauss

para esse espelho, calcule o aumento linear conseguido

quando o lápis estiver a 10 cm do vértice do espelho,

perpendicularmente ao seu eixo principal, e a distância

em que o lápis deveria estar do vértice do espelho, para

que sua imagem fosse direita e ampliada cinco vezes.

Questão 08 - (FCM MG/2012)

Dois espelhos esféricos côncavos de 40 cm de raio são

colocados a um metro de distância, um de frente para

o outro. Uma vela acesa é posicionada a 60 cm de um

Page 121: curso de específica de física

121

dos espelhos. A distância entre as duas primeiras

imagens formadas da vela é de:

a) 10 cm.

b) 20 cm.

c) 30 cm.

d) 40 cm.

Questão 09 - (MACK SP/2012)

Obedecendo às condições de Gauss, um espelho

esférico fornece, de um objeto retilíneo de altura y,

colocado perpendicularmente ao seu eixo principal, uma

imagem direita e de altura 2 y. A distância entre essa

imagem e o objeto é de 30 cm. O raio de curvatura

desse espelho mede

a) 10 cm

b) 20 cm

c) 30 cm

d) 40 cm

e) 50 cm

Questão 10 - (UEM PR/2012)

Com relação à reflexão e à formação de imagens em

espelhos planos e esféricos, assinale o que for correto.

01. Se a imagem de um objeto em um espelho

côncavo é virtual e maior do que o objeto, então o

objeto está localizado entre o centro de curvatura

e o foco do espelho.

02. Se um objeto de 20 cm de altura está a uma

distância de 10 cm de um espelho plano, então a

imagem do objeto está a uma distância maior do

que 10 cm do espelho.

04. Quanto mais próximo um objeto estiver do lado

refletor de um espelho convexo, maior é a sua

imagem.

08. Se um objeto de 30 cm de altura dista 60 cm do

lado refletor de um espelho plano, então sua

imagem é virtual e mede 30 cm de altura.

16. Um objeto de 2 cm de altura, colocado sobre o eixo

principal de um espelho côncavo de 40 cm de raio,

e com distância de 30 cm do vértice do espelho no

lado refletor, tem uma imagem que dista menos

de 35 cm do vértice do espelho.

Questão 11 - (UFG GO/2012)

Um objeto retangular é colocado diante de um espelho

côncavo, conforme representado na figura a seguir.

Para a situação apresentada, a imagem conjugada por

esse espelho é:

a)

b)

c)

d)

e)

Questão 12 - (UFU MG/2012)

Nos faróis dos veículos têm sido empregados dois

espelhos esféricos, de modo a se obter o máximo de

aproveitamento dos raios luminosos emitidos, como

exemplifica o esquema abaixo, no qual os espelhos são

representados pelas letras E1 e E2.

Para que haja o perfeito direcionamento dos raios de luz

para a frente do veículo, a lâmpada deve estar

posicionada

a) nos focos dos espelhos E1 e E2.

b) nos centros de curvatura de ambos os espelhos.

c) no foco do espelho E1 e no centro de curvatura de

E2.

d) no centro de curvatura do espelho E1 e entre o foco

e o vértice de E2.

Questão 13 - (UNIOESTE PR/2012)

As setas representam um objeto real e sua imagem,

também real e com o mesmo tamanho do objeto,

produzida por um único dispositivo ótico. Que

dispositivo é este?

a) Espelho plano.

b) Espelho convexo.

c) Espelho côncavo.

d) Lente divergente.

Page 122: curso de específica de física

122

e) Lente convergente.

Questão 14 - (UFRN/2011)

Os carros modernos usam diferentes tipos de espelhos

retrovisores, de modo que o motorista possa melhor

observar os veículos que se aproximam por trás dele.

As Fotos 1 e 2 abaixo mostram as imagens de um

veículo estacionado, quando observadas de dentro de

um carro, num mesmo instante, através de dois

espelhos: o espelho plano do retrovisor interno e o

espelho externo do retrovisor direito, respectivamente.

Foto 1

Foto 2

A partir da observação dessas imagens, é correto

concluir que o espelho externo do retrovisor direito do

carro é

a) convexo e a imagem formada é virtual.

b) côncavo e a imagem formada é virtual.

c) convexo e a imagem formada é real.

d) côncavo e a imagem formada é real.

Questão 15 - (PUC RJ/2011)

Para o espelho côncavo esférico da figura, onde R = 10

cm, s = 30 cm, determine a distância s’ em cm da

imagem ao vértice do espelho.

a) 3.

b) 5.

c) 6.

d) 10.

e) 12.

Questão 16 - (UEM PR/2011)

Sobre a formação de imagens de objetos pontuais e

extensos em espelhos planos e espelhos esféricos

estigmáticos, analise as alternativas e assinale o que for

correto.

01. Em um espelho plano, um ponto imagem virtual

pode ser definido pela interseção efetiva dos raios

de luz emergentes do espelho.

02. Nos espelhos côncavos, o foco principal é real,

enquanto nos espelhos convexos o foco principal é

virtual.

04. A imagem de um objeto extenso colocado entre o

foco e o vértice de um espelho esférico côncavo é

virtual, direita e maior.

08. Todo raio de luz que incide sobre o vértice de um

espelho esférico é refletido numa direção paralela

ao eixo principal do espelho.

16. Um espelho plano é um sistema óptico estigmático

que conjuga sempre um ponto objeto com um

ponto imagem.

Questão 17 - (UFAL/2010)

Um palito de fósforo, de 8 cm de comprimento, é

colocado a 80 cm de distância de um espelho esférico

convexo. A imagem do palito possui comprimento de 1,6

cm e a mesma orientação deste. Pode-se concluir que o

valor absoluto da distância focal do espelho vale:

a) 10 cm

b) 20 cm

c) 30 cm

d) 40 cm

e) 50 cm

Questão 18 - (UEFS BA/2013)

Disponível em:

<http://efisica.if.usp.br/otica/basico/espelhos_

esfericos/associacao/> Acesso em: 25 jun. 2013.

A figura representa o esquema simplificado de um

holofote, construído com dois espelhos esféricos

côncavos associados, para obter um feixe paralelo de

luz cilíndrico com alta eficiência no aproveitamento da

luz emitida por um pequeno filamento aquecido de uma

lâmpada.

Considerando-se que os espelhos obedecem às

condições de Gauss e sabendo-se que f1 e f2 são,

respectivamente, as distâncias focais dos espelhos 1 e

2, é correto afirmar:

a) A distância do filamento ao espelho E1 é igual a

2f1.

b) A distância entre os espelhos E1 e E2 é igual a f1 +

2f2.

c) A imagem da lâmpada conjugada pelo espelho E2

é virtual.

d) O filamento é colocado no centro de curvatura do

espelho E1.

Page 123: curso de específica de física

123

e) O espelho E2 deve ser posicionado de forma que o

seu foco coincida com a posição do filamento.

Questão 19 - (FMJ SP/2012)

Um brinquedo possui uma pequena lâmpada que deve

funcionar como uma lanterna de pouco alcance. Para

melhorar a qualidade do pincel de luz, um espelho

côncavo, feito em material plástico flexível, deve ser

encaixado em um orifício cônico. Após sua montagem

no orifício, o espelho, que possuía distância focal de 2

m, passa a ter distância focal de 1 m. A mudança

causada no espelho altera a posição de imagens que ele

poderia produzir. Considerando que um objeto real seja

colocado a menos de 1 m diante do espelho, antes e

depois de seu encaixe no orifício, e, em ambos os casos,

à mesma distância da superfície refletora, a nova

posição da imagem (p’’) obtida desse objeto, escrita em

termos da distância que se obteria se o espelho não

tivesse sido ainda colocado em seu lugar (p’), é dada

por

a) 2

'p''p

b) 'p

'p2''p

c) 2'p

'p2''p

d) 2'p

'p2''p

e) 'p4

'p2'p''p

Questão 20 - (UNESP/2007)

Um pesquisador decide utilizar a luz solar concentrada

em um feixe de raios luminosos para confeccionar um

bisturi para pequenas cirurgias. Para isso, construiu um

coletor com um espelho esférico, para concentrar o feixe

de raios luminosos, e um pequeno espelho plano, para

desviar o feixe em direção à extremidade de um cabo de

fibra óptica. Este cabo capta e conduz o feixe

concentrado para a sua outra extremidade, como

ilustrado na figura.

Em uma área de 1 mm2, iluminada pelo sol, a potência

disponível é 0,001 W/mm2. A potência do feixe

concentrado que sai do bisturi óptico, transportada pelo

cabo, cuja seção tem 0,5 mm de raio, é de 7,5 W. Assim,

a potência disponibilizada por unidade de área (utilize

3 ) aumentou por um fator de

a) 10 000.

b) 4 000.

c) 1 000.

d) 785.

e) 100.

GABARITO:

1) Gab: A 2) Gab: E 3) Gab: A

4) Gab: A 5) Gab: B

6) Gab:

a) i 0,05 cm.

b) 61 R

7) Gab:

A = 1,25

p = 40cm

8) Gab: C 9) Gab: D 10) Gab: 12

11) Gab: A 12) Gab: C 13) Gab: C

14) Gab: A 15) Gab: C 16) Gab: 22

17) Gab: B 18) Gab: B 19) Gab: C

20) Gab: A 21) Gab: E 22) Gab: D

23) Gab: C 24) Gab: A 25) Gab: C 26) Gab: D 27) Gab: 22 28) Gab: 20 29) Gab: A 30) Gab: B 31) Gab:

a) 0,5 m

b) 6

1

32) Gab: D 33) Gab: B 34) Gab: C 35) Gab: B 36) Gab: B 37) Gab: C 38) Gab: C 39) Gab: 03 40) Gab: D

Page 124: curso de específica de física

124

LISTA 23 – REFRAÇÃO DA LUZ LEI DE SNELL-DESCARTES

LEI DE SNELL-DESCARTES

Em alguns acontecimentos do nosso dia a dia, deparamo-nos com fenômenos físicos, mas nem nos damos conta de que eles estão inseridos em nosso cotidiano. Muitas vezes pensamos que só os conheceremos e os usaremos em sala de aula. Mas, ao contrário disso, como já dito, eles estão em diversos acontecimentos à nossa volta. Um desses fenômenos é o da foto acima, que se deve à refração da luz em cristais de gelo. Refração é o nome dado ao fenômeno que ocorre quando a luz, ao cruzar a fronteira entre dois meios, sofre uma variação em sua velocidade de propagação. No estudo da refração, levando em consideração a variação na velocidade de propagação da luz, define-se, para os meios homogêneos e transparentes, um número chamado de índice de refração. Podemos definir o índice de refração (n) de um meio como

sendo o quociente entre a velocidade de propagação da luz no vácuo (c) e sua velocidade de propagação no meio considerado (v).

Segunda Lei da Refração A Lei de Snell-Descartes também é conhecida comumente no meio físico como sendo a segunda lei da refração. Ela enuncia que: na refração, o produto do índice de refração do meio, no qual se encontra o raio pelo seno do ângulo que esse raio forma com a reta normal à interface no ponto de incidência, é constante. Analiticamente, podemos escrever o seguinte:

Na igualdade acima, se considerarmos que n2 > n1 (ou, o que é equivalente, v2 < v1), então sen r < sen i e r < i. Podemos, então, concluir que, quando a luz passa de um meio menos refringente para um meio mais refringente, a velocidade da luz diminui e o raio luminoso se aproxima da reta normal, isto é, o ângulo que o raio luminoso forma com a reta normal diminui. Veja a figura abaixo.

Ao passar de um meio mais refringente para um meio

menos refringente, a velocidade da luz diminui. EXERCÍCIOS

Questão 01 - (UNESP/2013)

Uma haste luminosa de 2,5 m de comprimento está

presa verticalmente a uma boia opaca circular de 2,26

m de raio, que flutua nas águas paradas e transparentes

de uma piscina, como mostra a figura. Devido à

presença da boia e ao fenômeno da reflexão total da

luz, apenas uma parte da haste pode ser vista por

observadores que estejam fora da água.

Considere que o índice de refração do ar seja 1,0, o da

água da piscina 3

4 , sen 48,6º = 0,75 e tg 48,6º = 1,13.

Um observador que esteja fora da água poderá ver, no

máximo, uma porcentagem do comprimento da haste

igual a

a) 70%.

b) 60%.

c) 50%.

d) 20%.

e) 40%.

Questão 02 - (UFT TO/2013)

Um arco-íris se forma quando o Sol, as gotas de chuva

e o observador estão dispostos em um determinado

ângulo. Os raios de sol penetram na gota de chuva e,

devido à forma da gota, emergem com a luz separada

nos seus vários comprimentos de onda, atingindo então

o olho do observador. O fenômeno que ocorre com a luz

na formação do arco-íris é chamado de:

a) Difração, interferência, dispersão.

Page 125: curso de específica de física

125

b) Interferência, reflexão, polarização.

c) Polarização, interferência, difração.

d) Reflexão, polarização, dispersão.

e) Refração, reflexão, dispersão.

Questão 03 - (UEM PR/2012)

Assinale o que for correto.

01. Um conjunto constituído de dois meios

homogêneos e transparentes à passagem da luz

visível, separados por uma superfície plana, é

chamado de dioptro plano.

02. Se o índice de refração da água contida em uma

piscina é maior que o do ar, a profundidade de

uma piscina contendo água, quando observada do

ar e da lateral da piscina, é sempre menor que sua

profundidade real.

04. A luz visível que atravessa uma lâmina de faces

paralelas, fazendo um ângulo de 30º com relação

a normal a essa superfície, é desviada

lateralmente em relação à sua direção de

incidência.

08. A luz visível polarizada não sofre refração ao

atravessar um dioptro plano.

16. A luz visível polarizada não obedece à lei de Snell

ao atravessar uma lâmina de faces paralelas.

Questão 04 - (PUC MG/2012)

Um bastão de vidro parece quebrado, quando colocado

inclinado dentro de um recipiente com água conforme o

diagrama a seguir.

A melhor explicação para esse fenômeno é:

a) a luz viaja mais rápido na água que no ar.

b) a luz é refletida na fronteira ar-água.

c) a luz é refratada na fronteira ar-água.

d) a luz é difratada na fronteira ar-água.

Questão 05 - (UCB DF/2012)

A respeito da refração da luz, julgue os itens a seguir,

assinalando (V) para os verdadeiros e (F) para os falsos.

00. O arco-íris é uma decorrência do fenômeno da

refração.

01. Quando um raio de luz passa do ar para a água, o

ângulo-limite forma-se na água.

02. Não pode ocorrer refração sem reflexão

simultânea.

03. É impossível impedir que a luz proveniente de uma

fonte luminosa na água propague-se no ar.

04. Considerando que o índice de refração do vidro é

da ordem de 1,5 e o da água de,

aproximadamente, 1,3, conclui-se que a

velocidade da luz no vidro é maior que na água.

Questão 06 - (UFPR/2013)

Ao ser emitida por uma fonte, uma luz monocromática,

cujo comprimento de onda no ar é 0, incide no olho de

uma pessoa. A luz faz o seguinte percurso até atingir a

retina: ar – córnea – humor aquoso – cristalino – humor

vítreo. Considerando que o índice de refração do ar é n0

= 1,00, da córnea é n1 = 1,38, do humor aquoso é n2 =

1,33, do cristalino é n3 = 1,40 e do humor vítreo é n4 =

1,34 e que 1, 2, 3 e 4 são os comprimentos de onda

da luz na córnea, no humor aquoso, no cristalino e no

humor vítreo, respectivamente, assinale a alternativa

correta.

a) 1 < 0.

b) 2 < 1.

c) 3 > 2.

d) 4 < 3.

e) 4 > 0.

Questão 07 - (UNICAMP SP/2011)

A radiação Cerenkov ocorre quando uma partícula

carregada atravessa um meio isolante com uma

velocidade maior do que a velocidade da luz nesse

meio. O estudo desse efeito rendeu a Pavel A. Cerenkov

e colaboradores o prêmio Nobel de Física de 1958. Um

exemplo desse fenômeno pode ser observado na água

usada para refrigerar reatores nucleares, em que ocorre

a emissão de luz azul devido às partículas de alta

energia que atravessam a água.

a) Sabendo-se que o índice de refração da água é n

= 1,3, calcule a velocidade máxima das partículas

na água para que não ocorra a radiação Cerenkov.

A velocidade da luz no vácuo é c = 3,0 108m/s.

b) A radiação Cerenkov emitida por uma partícula

tem a forma de um cone, como ilustrado na figura

abaixo, pois a sua velocidade, vp , é maior do que

a velocidade da luz no meio, vl . Sabendo que o

cone formado tem um ângulo θ = 50º e que a

radiação emitida percorreu uma distância d =

1,6m em t = 12ns, calcule vp.

Dados: cos50º = 0,64 e sen50º = 0,76.

Questão 08 - (UECE/2010)

Um raio de luz monocromático reduz sua velocidade em

50 % ao passar do meio I para o meio II. Podemos

afirmar que o índice de refração do meio II é maior que

o índice de refração do meio I:

Page 126: curso de específica de física

126

a) 1,3 vezes

b) 1,5 vezes

c) 2,0 vezes

d) 2,5 vezes

Questão 09 - (UEPG PR/2010)

A dispersão da luz consiste em um fenômeno no qual a

luz branca ao penetrar em um dioptro se separa em

várias cores, formando um espectro. Sobre esse

fenômeno, assinale o que for correto.

01. O índice de refração de um meio varia com a cor da

luz incidente sobre ele.

02. O arco íris é um espectro da luz solar, as gotículas

de chuva atuam como dioptros.

04. Toda refração é acompanhada de dispersão.

08. Se um objeto se apresenta branco, quando exposto

à luz solar, é porque ele absorve todas as cores.

16. A luz emitida por sólidos e líquidos incandescentes

fornece espectros contínuos.

Questão 10 - (UNIFICADO RJ/2013)

Um feixe de luz se propagando pelo ar incide em um

cubo de acrílico cuja aresta mede 6 cm, fazendo um

ângulo de 45º com a superfície desse cubo. O feixe de

luz atravessa o cubo e sai na face oposta a uma altura

h, acima da posição de incidência, como mostra a

Figura.

Calcule a distância h em cm.

a) 3,0

b) 3,4

c) 5,1

d) 6,0

e) 10,2

Questão 11 - (UNISA SP/2013)

Dois meios transparentes, sendo um deles o ar e o outro

mais refringente, estão separados por uma interface

plana. Sabese que o ângulo limite, a partir do qual há

reflexão total, é igual a 45°, conforme figura.

Considere os dados da tabela.

Em nova situação, o raio luminoso, proveniente do ar,

incide na interface formando com ela, novamente, um

ângulo de 45°. Nessa nova condição, o valor do ângulo

que o raio refratado forma com a interface é

a) 60°.

b) 30°.

c) 90°.

d) 45°.

e) 0°.

Questão 12 - (UFPB/2013)

Para estudar o comportamento da luz em meios

materiais, um grupo de alunos teve a idéia de incidir

uma luz emitida por um laser de hélio-neônio, cujo

comprimento de onda no ar é de 610–7m , sobre certo

meio aquoso. Com as ferramentas disponíveis, os

alunos mediram a velocidade da luz nesse meio e

constataram que era 2108m/s.

Considerando que no ar a velocidade da luz é de

3108m/s, os alunos concluíram que, nesse meio

aquoso, a luz incidente apresenta um comprimento de

onda, em m, de:

a) 110–7

b) 210–7

c) 310–7

d) 410–7

e) 510–7

Questão 13 - (UFU MG/2013)

O fenômeno da refração da luz está presente no nosso

dia a dia. Por exemplo, uma caneta, quando colocada

dentro de um copo de água, parece estar quebrada, ou,

quando olhamos uma piscina, temos a impressão de sua

profundidade ir diminuindo conforme nos afastamos do

extremo oposto da sua borda. Um estudante, sentado

próximo à janela de sua casa, observou que a luz do Sol

era refratada no vidro da janela, como mostra a figura

abaixo. Então, resolveu verificar se aprendeu os

conceitos sobre óptica ensinados pelo professor em sala

de aula.

Page 127: curso de específica de física

127

Considere o índice de refração do ar igual a 1 e do vidro

igual a 1,5.

Com base nas informações dadas, marque, para as

afirmativas abaixo, (V) Verdadeira, (F) Falsa ou (SO)

Sem Opção. (Dica: 1/2 2 /3).

1. O ângulo 2 é 45º.

2. A velocidade da luz no vidro é maior do que no ar.

3. O ângulo θ1 é aproximadamente 30º.

4. Parte da luz incidente ao atingir o vidro é refletida

por um ângulo de 45º.

Questão 14 - (UNIMONTES MG/2013)

Um feixe de luz é uma mistura de três cores: amarelo,

vermelho e violeta. Ele incide sobre um prisma de

material transparente, com índice de refração crescente

com a frequência, ou seja, quanto maior a frequência

relativa à cor, maior será o índice de refração do

material do prisma. Após atravessar o prisma, a luz

atinge um anteparo, formando três manchas coloridas,

nas posições 1, 2 e 3, indicadas na figura.

As cores das manchas formadas nas posições 1, 2 e 3

são, respectivamente:

a) vermelho, violeta e amarelo.

b) vermelho, amarelo e violeta.

c) violeta, vermelho e amarelo.

d) amarelo, violeta e vermelho.

TEXTO: 1 - Comum à questão: 15

Os Dez Mais Belos Experimentos da Física

A edição de setembro de 2002 da revista Physics World

apresentou o resultado de uma enquete realizada entre

seus leitores sobre o mais belo experimento da Física.

Na tabela abaixo são listados os dez experimentos mais

votados.

Foucault. de pêndulo

do aExperiênci )10

Young.por realizada

luz, da ciainterferên

da oExperiment )5

.Rutherford

de oExperiment 9)

Newton.por realizada

prisma, um comsolar

luz da ãoDecomposiç 4)

Galileu.por

realizado inclinado, plano

num corpos de movimento

o sobre oExperiment )8

Millikan.

por realizadaóleo, de

gota da oExperiment 3)

es.Erastósten

por realizada Terra, da

nciacircunferê da Medida 7)

Galileu.por realizada corpos, dos

queda da oExperiment 2)

Cavendish.por

realizada

torsão,de balança

a com oExperiment )6

elétrons. com realizado

Young, de fenda

dupla da oExperiment 1)

Questão 15 - (UEG GO/2011)

O experimento de decomposição (dispersão) da luz

solar, realizado por Newton, é extraordinariamente

simples, sendo necessário somente um prisma. Como

ilustra a figura abaixo, ao passar por um prisma, a luz

solar, que é branca, se decompõe nas cores do arco-

íris.

Com relação aos fenômenos da luz ao atravessar o

prisma, é CORRETO afirmar:

a) Na dispersão da luz, a luz monocromática de maior

frequência sofrerá o menor desvio.

b) Num prisma, a dispersão da luz branca é menos

acentuada que numa única superfície dióptrica.

c) A separação da luz branca nas cores do arco-íris é

possível porque cada cor tem um índice de

refração diferente.

d) Neste experimento, Newton demonstrou que,

combinando dois ou mais prismas, é possível

decompor a luz branca, porém a sua recomposição

não é possível.

Questão 16 - (UNIRG TO/2013)

Com a evolução da tecnologia, os meios de

comunicação substituíram os tradicionais cabos

metálicos por cabos de fibra óptica. A fibra óptica é um

filamento de vidro, o qual é formado basicamente por

um núcleo central de vidro, por onde ocorre a

transmissão da luz, que possui alto índice de refração,

e de uma casca envoltória, também feita de vidro,

porém com índice de refração menor em relação ao

núcleo. Conforme exposto, o princípio físico de

transmissão de informação em uma fibra óptica baseia-

se na

Page 128: curso de específica de física

128

a) polarização da luz.

b) transmissão da luz.

c) reflexão interna total.

d) refração da luz.

Questão 17 - (UNIOESTE PR/2012)

Um raio luminoso incide sobre um cubo de plástico

transparente formando um ângulo com uma das faces

e emerge na face seguinte também formando um

ângulo , como mostra a figura. O desvio sofrido pelo

raio, entre entrar e sair do cubo, é, também, .

Considerando estes dados e que o meio circundante

seja o ar, o valor do índice de refração do plástico de

que é feito o cubo vale

a) 1,5.

b) (3/2)1/2.

c) (4/3)1/2.

d) (5/2)1/2.

e) (5/3)1/2.

Questão 18 - (UFAC/2010)

Na figura abaixo é mostrado a propagação de um feixe

de luz (a) que incide sobre uma placa transparente de

faces paralelas. Como consequência dessa incidência são

originados outros feixes denominados (b), (c), (d), (e) e

(f).

ALONSO, M., FINN, E. Física, Volumen II: Campos y

Ondas, México, D. F.: Addison-Wesley Iberoamericana,

1985, p. 810. (com adaptações).

Análise as seguintes afirmações:

I. Os raios (a) e (c) são paralelos.

II. Os raios (f) e (e) não são paralelos.

III. Os raios (f) e (e) são paralelos.

IV. Os raios (a) e (c) não são paralelos.

V. Os raios (b) e (d) são simétricos em relação a um

eixo perpendicular à face inferior que passa pelo

ponto P.

a) I, III e V são corretas

b) II e IV são corretas.

c) III e IV são corretas.

d) I e II são corretas.

e) I e IV são incorretas.

Questão 19 - (FM Petrópolis RJ/2014)

A Figura acima ilustra um raio monocromático que se

propaga no ar e incide sobre uma lâmina de faces

paralelas, delgada e de espessura d com ângulo de

incidência igual a 60º. O raio sofre refração, se propaga

no interior da lâmina e, em seguida, volta a se propagar

no ar.

Se o índice de refração do ar é 1, então o índice de

refração do material da lâmina é

a) 3

6

b) 2

6

c) 3

2

d) 6

e) 3

Questão 20 - (UNEB/2014)

Cientistas da Stanford University lançaram dúvidas

sobre o fato de alimentos orgânicos serem realmente

mais nutritivos que os cultivados de maneira

convencional. O espinhoso segredo é que, sejam suas

maçãs e espinafres orgânicos ou não, os níveis de

nutrientes podem variar dramaticamente dependendo

das condições de cultivo, como tipo e qualidade do solo,

temperatura, e dias de sol ou chuva. Como consumidor,

não há meios para verificar, de maneira independente,

como escolher um lote de melhor qualidade. Mas um

scanner manual permite checar a densidade de

nutrientes.

A tecnologia básica existe há décadas. A espectroscopia

de infravermelho próximo, NIR, encontrou aplicações

na produção farmacêutica, na medicina, na agricultura

e na astronomia. O NIR funciona com o princípio de que

moléculas diferentes vibram de maneira levemente

diferente. Quando a luz infravermelha é espalhada em

certa amostra, ou refletida por ela, determinados

comprimentos de onda são absorvidos mais que outros

pelas ligações químicas em vibração. Ao medir a fração

de luz de infravermelho próximo absorvida em cada

comprimento de onda, cientistas podem obter um

registro distinto, característico da amostra. Os

resultados são precisos e rápidos.

O NIR tem uma grande limitação para um scanner de

supermercado: ele não dá leituras para compostos com

Page 129: curso de específica de física

129

uma concentração menor que 0,1%. Um vegetal médio

possui 92% de água. Depois disso, vêm os

macronutrientes, como carboidratos e proteínas, em

quantidades altas o bastante para detecção pelo NIR,

seguidos pelos micronutrientes, incluindo vitaminas,

minerais e antioxidantes, que, em geral, possuem

pequenas concentrações para serem detectadas.

(VITAMINAS e micronutrientes. 2013. p.12-13).

Disponível em:

<http://pt.wikipedia.org/wiki/Ficheiro:Aparato_de_

espectroscopia_IV_versao2.png>. Acesso em: 27 ago.

2013.

A figura representa um esquema simplificado de um

equipamento de espectroscopia de infravermelho.

Com base nos conhecimentos de óptica geométrica,

analise as afirmativas e marque V para as verdadeiras

e F, para as falsas.

( ) Os dois espelhos associados em forma de um V

formam entre si um ângulo de 45º.

( ) As propriedades físicas da luz do ponto de vista da

óptica geométrica envolvidas no processo de

espectroscopia são reflexão e refração.

( ) Os feixes de radiação infravermelha que incidem

sobre as superfícies dos espelhos sofrem reflexão

total.

( ) Um dos espelhos de um canto pode ser substituído

por um prisma óptico, de ângulo de abertura de

90º e de índice de refração 2 , com a base do

prisma, oposta ao ângulo de abertura, colocada

sobre o espelho.

A alternativa que indica a sequência correta, de cima

para baixo, é a

01. V F F V

02. V F V F

03. F V F V

04. V V F F

05. F V V F

GABARITO:

1) Gab: D 2) Gab: E 3) Gab: 07

4) Gab: C 5) Gab: VVVFF

6) Gab: A 7) Gab:

a) Vágua 2,3 108m/s

b) vp 2,08 108m/s

8) Gab: C 9) Gab: 23 10) Gab: B

11) Gab: A 12) Gab: D 13) Gab: VFVF

14) Gab: B 15) Gab: C 16) Gab: C

17) Gab: B 18) Gab: A 19) Gab: B

20) Gab: 03 21) Gab: E 22) Gab: A

23) Gab: 2,5º 24) Gab: 03 25) Gab: B

26) Gab: B

27) Gab: C

28) Gab:

a) d3

3

3

1d)(tgdd rA

b) 13

gd2v0

29) Gab:

a) O raio E representa a trajetória do raio de liz

quando o meio 2 é um metamaterial.

|n2| = 1,28

b) n = 1,5

30) Gab: E

31) Gab: 02 32) Gab: C 33) Gab: A 34) Gab: B 35) Gab: A 36) Gab: B 37) Gab: E 38) Gab: D 39) Gab: 03 40) Gab: A

Page 130: curso de específica de física

130

LISTA 24 – ESTUDO DAS LENTES ESFÉRICAS As lentes são dispositivos ópticos que funcionam por refração da luz e são muito utilizadas no nosso dia a dia, como nos óculos, nas lupas, nas câmeras fotográficas, nas filmadoras e em telescópios. O material que as constitui normalmente é o vidro, mas o plástico também pode ser utilizado. As principais características desses dispositivos são a transparência e a superfície esférica.

De acordo com a curvatura apresentada, as lentes esféricas podem ser classificadas como: Lentes convergentes, ou positivas: quando a parte do centro é mais espessa que as bordas. Elas podem ser de três tipos:

Lentes biconvexas: apresentam duas partes convexas;

Lentes plano-convexas: possuem um lado plano e

outro convexo;

Lentes côncavo-convexas: com um lado côncavo e o

outro convexo.

Lentes divergentes, ou negativas: se o centro é mais

fino que as bordas. Podem ser classificadas como:

Lentes bicôncavas: caso apresentem as duas faces

côncavas;

Lentes plano-côncavas: quando apresentam um lado

plano e o outro côncavo;

Lentes convexo-côncavas: com um lado convexo e

outro côncavo. A figura a seguir mostra o formato de cada um desses tipos de lente:

A figura apresenta os tipos de lentes convergentes e

divergentes Além do formato, as lentes também podem ser classificadas de acordo com o comportamento óptico dos raios de luz após atingi-las. Nesse caso, elas podem ser divergentes ou

convergentes.

Em uma lente divergente, quando os raios de luz incidem paralelos ao eixo principal, eles sofrem dupla refração e se espalham. Observe a figura:

Comportamento óptico dos raios de luz em uma lente

divergente Como o foco dessas lentes é formado pelo encontro de projeções dos raios de luz incidentes, ele é classificado como virtual. Nas lentes convergentes, os raios de luz incidem paralelos ao eixo principal e, após sofrerem refração, se concentram em um único ponto, este ponto é o foco.

Comportamento óptico dos raios de luz em uma lente

convergente O foco das lentes convergentes é classificado como foco real, pois é fruto do encontro dos raios de luz refratados. Elementos das lentes esféricas O que caracteriza uma lente esférica são os seus elementos geométricos, que são: C1 e C2: centros de curvatura das faces esféricas; R1 e R2: raios de curvatura das faces esféricas; Eixo principal da lente: onde estão contidos C1 e V1; e: espessura da lente; V1 e V2: Vértices da lente. Veja como é a disposição desses elementos nas lentes:

Disposição dos elementos nas lentes côncavas

Page 131: curso de específica de física

131

Disposição dos elementos nas lentes convexas

Quando a espessura da lente é muito menor do que o raio de suas faces, ela é denominada lente delgada. Nesse tipo de lente, os vértices V1 e V2 estão praticamente no mesmo ponto sobre o eixo principal e passam a ser chamados de centro óptico.

EXERCÍCIOS

Questão 01 - (UEPA/2013)

Um estudante percebeu que a menor distância na qual

ele conseguia enxergar com nitidez um objeto era 50

cm. Para corrigir esse defeito e enxergar nitidamente a

prova de Física do PRISE/PROSEL da UEPA, posicionada

a 25 cm de seus olhos, o estudante deveria utilizar

óculos cujas lentes possuem grau igual a:

a) 2,5

b) 2,0

c) 1,5

d) 1,0

e) 0,5

Questão 02 - (FGV/2012)

Uma estudante usou uma lupa para pesquisar a

formação de imagens de objetos reais. Num instante de

Sol a pino, ela conseguiu obter um ponto luminoso no

chão, colocando a lupa a 20 cm dele e paralelamente a

ele. A seguir, aproximando a lupa a 15 cm de seu

celular, obteve uma imagem do celular

a) real, invertida e ampliada.

b) real, invertida e reduzida.

c) virtual, direita e ampliada.

d) virtual, direita e reduzida.

e) virtual, invertida e ampliada.

Questão 03 - (PUCCAMP SP/2012)

Os projetores são aparelhos que ampliam e projetam

em anteparos as imagens de objetos gravados (filmes,

slides).

Em uma sala de projeção, a distância do projetor ao

anteparo é de 5,1 m e o filme, fortemente iluminado, é

colocado a 102 mm da lente do projetor. Sabendo que

a imagem do filme projetada no anteparo é nítida,

pode-se afirmar corretamente que a distância focal da

lente, em cm, e o aumento linear transversal valem,

respectivamente,

a) 10 e 100

b) 10 e –50

c) –10 e –100

d) –10 e 50

e) –20 e 25

Questão 04 - (UDESC/2012)

No olho humano a imagem é formada a partir da

convergência dos raios luminosos sobre a retina, após

atravessar o cristalino. Em alguns casos, o sistema

óptico humano pode sofrer ametropias, que englobam,

por exemplo, miopia e hipermetropia. Na miopia os

feixes de raios paralelos convergem em um ponto focal

anterior à retina. Uma maneira comum de corrigir tais

desvios é a utilização de lentes oculares, ajustadas a

fim de manterem a convergência sobre a retina. A

capacidade de uma lente de modificar o trajeto dos

raios luminosos é medida por uma grandeza chamada

dioptria, definida como o inverso da distância focal da

lente. Uma pessoa míope percebe que a maior distância

em que ela consegue ler um livro é 0,20 m.

A fim de ser capaz de lê-lo a uma distância muito

superior, assinale a alternativa que indica,

respectivamente, o tipo e a dioptria da lente que a

pessoa deve utilizar.

a) Lente convergente, com dioptria de 0,20 m–1.

b) Lente convergente, com dioptria de 5,0 m–1.

c) Lente divergente, com dioptria de 5,0 m–1.

d) Lente divergente, com dioptria de 0,20 m–1.

e) Lente divergente, com dioptria nula.

Questão 05 - (UEM PR/2012)

Com relação à formação de imagens de objetos

extensos, colocados no ar diante de lentes esféricas de

índices de refração maiores que o do ar, assinale o que

for correto.

01. A imagem formada por um objeto extenso

colocado sobre o centro de curvatura de uma lente

convergente é real, do mesmo tamanho que o

objeto e invertida.

02. A imagem formada por um objeto extenso

colocado sobre o foco de uma lente convergente é

virtual, maior que o objeto e direita.

04. A imagem formada por um objeto extenso

colocado sobre o foco de uma lente divergente é

virtual, do mesmo tamanho que o objeto e direita.

08. A imagem formada por um objeto extenso

colocado entre o foco e o centro de curvatura de

uma lente convergente é real, maior que o objeto

e invertida.

16. A imagem formada por um objeto extenso

colocado entre o foco e o centro de curvatura de

uma lente divergente é virtual, menor que o objeto

e invertida.

Questão 06 - (Emescam ES/2012)

Considere duas lentes de vidro, biconvexas, imersas em

dois recipientes, contendo respectivamente dois

líquidos diferentes com índices de refração nA e nB ,

como é mostrado na figura abaixo. Observando os

trajetos dos raios luminosos, podemos concluir

corretamente que:

Page 132: curso de específica de física

132

a) nA nvidro nB

b) nA nvidro nB

c) nA nvidro nB

d) nA = nvidro nB

e) nA nvidro nB

Questão 07 - (IFPE/2012)

Analisando os três raios notáveis de lentes esféricas

convergentes, dispostas pela figura abaixo, podemos

afirmar que:

a) Apenas um raio está correto.

b) Apenas dois raios são corretos.

c) Os três raios são corretos.

d) Os raios notáveis dependem da posição do objeto,

em relação ao eixo principal.

e) Os raios notáveis dependem da posição da lente,

em relação ao eixo principal.

Questão 08 - (ACAFE SC/2014)

Um médico oftalmologista realizou uma cirurgia no

globo ocular de dois pacientes (paciente A e paciente

B), a fim de corrigir dois defeitos da visão. Para tanto,

utiliza um método de cirurgia corretiva a Laser que

possui maior precisão e eficiência. No paciente A o

procedimento corrigiu o defeito e, com isso, o ponto

remoto do olho foi colocado para mais longe. No

paciente B houve a correção do defeito de tal modo

que o ponto próximo foi trazido para mais perto do

olho.

Nesse sentido, marque com V as afirmações

verdadeiras e com F as falsas.

( ) O paciente A pode ter corrigido o defeito da

hipermetropia.

( ) O paciente B utilizava uma lente convergente

para corrigir seu defeito visual antes da cirurgia.

( ) A cirurgia no paciente A fez com que a imagem

de um objeto, que se formava antes da retina, se

forme exatamente sobre a retina.

( ) Antes da cirurgia a imagem de um objeto se

formava atrás da retina no olho do paciente B.

( ) Uma das causas do defeito da visão do paciente

A poderia ser por que seu globo ocular é

achatado.

A sequência correta, de cima para baixo, é:

a) F - V - V - V - F

b) F - F - V - V - V

c) F - V - F - V - V

d) V - V - F - F - V

Questão 09 - (Fac. Santa Marcelina SP/2014)

O daltonismo é o resultado de um defeito na retina, que

altera a visão e dificulta a distinção entre cores. A retina

é a parte do globo ocular que

a) atua como um diafragma.

b) é sensível à luz.

c) impede a entrada de luz no olho.

d) simula uma lente biconvexa.

e) dá coloração ao olho.

Questão 10 - (UFG GO/2014)

Ao realizar exames oftamológicos, em um ambiente

pouco iluminado, os jovens J1 e J2 descobriram que o

diâmetro do corpo vítreo de seus olhos é de 18 mm.

Nesse exame, descobriu-se que, para J1, as imagens

dos objetos são formadas 13 mm após o cristalino,

enquanto, para J2, o diagnóstico atestou que ele não

visualiza nitidamente objetos a 25 cm do olho.

Conforme o exposto, quais são, respectivamente, os

tipos de lentes corretivas que J1 e J2 devem utilizar e

quais células responderam mais eficientemente ao

estímulo luminoso?

a) Divergente e convergente, e bastonetes na

córnea.

b) Divergente e divergente, e bastonetes na retina.

c) Convergente e divergente, e cones na córnea.

d) Divergente e divergente, e cones na retina.

e) Divergente e convergente, e bastonetes na retina.

Questão 11 - (Univag MT/2014)

O olho míope pode apresentar o globo ocular comprido

demais ou a córnea com curvatura exagerada não

conseguindo focalizar na retina objetos distantes, a

focalização ocorre antes da retina.

A miopia pode ser corrigida por meio de lentes

esféricas. Fazendo com que objetos localizados no

infinito sejam projetados na retina. Um oftalmologista

receita para um míope óculos de vergência – 2 di (2

graus). É correto afirmar que as lentes utilizadas e a

posição, em metros, do ponto remoto são,

respectivamente,

a) divergente e 0,75.

b) convergente e 0,5.

c) convergente e 0,75.

d) divergente e 1,0.

e) divergente e 0,5.

Questão 12 - (UFJF MG/2012)

O olho humano é uma estrutura biológica bem

complexa, mas seu funcionamento pode ser entendido

com base em conceitos de óptica geométrica. Em uma

Page 133: curso de específica de física

133

representação esquemática simplificada da estrutura do

olho, como mostrado na figura abaixo, é indicado que a

imagem forma-se sobre a retina a 15 mm do centro do

cristalino, que é considerado como uma lente

convergente. Considerando-se que um objeto encontra-

se a 25 cm de distância do olho, qual será a distância

focal da lente convergente do cristalino para que a

imagem se forme sobre a retina? A imagem será direita

ou invertida? A imagem será real ou virtual? Assinale a

afirmativa CORRETA.

Disponível em:

<http://www.sofisica.com.br/conteudos/Otica/

Instrumentosoticos/imagens/olho_reduzido.jpg>.

Acesso em: 20 abr. 2012.

a) A distância focal será de 12,2 mm. Será real e

invertida.

b) A distância focal será de 13,3 mm. Será virtual e

direita.

c) A distância focal será de 14,1 mm. Será real e

invertida.

d) A distância focal será de 15,0 mm. Será real e

direita.

e) A distância focal será de 15,0 mm. Será real e

invertida.

Questão 13 - (UEM PR/2012)

Assinale o que for correto.

01. Das frequências da luz visível que incide em um

corpo iluminado, a parte refletida é que promove

a sensação na retina do olho humano. Essa parte

corresponde à cor desse corpo.

02. A cor dos olhos humanos é o resultado da

dispersão da luz visível pela íris, local onde se

encontra a melanina.

04. A miopia é causada pelo alongamento do globo

ocular, que leva a uma excessiva curvatura da

córnea.

08. A hipermetropia é corrigida com o uso de lentes

divergentes, as quais direcionam os raios de luz

formadores da imagem para que sejam focalizados

no cristalino.

16. Os bastonetes da retina do olho humano são

células fotorreceptoras capazes de distinguir

cores.

Questão 14 - (FUVEST SP/2011)

O olho é o senhor da astronomia, autor da cosmografia,

conselheiro e corretor de todas as artes humanas (...).

É o príncipe das matemáticas; suas disciplinas são

intimamente certas; determinou as altitudes e

dimensões das estrelas; descobriu os elementos e seus

níveis; permitiu o anúncio de acontecimentos futuros,

graças ao curso dos astros; engendrou a arquitetura, a

perspectiva, a divina pintura (...). O engenho humano

lhe deve a descoberta do fogo, que oferece ao olhar o

que as trevas haviam roubado.

Leonardo da Vinci, Tratado da pintura.

Considere as afirmações abaixo:

I. O excerto de Leonardo da Vinci é um exemplo do

humanismo renascentista que valoriza o

racionalismo como instrumento de investigação

dos fenômenos naturais e a aplicação da

perspectiva em suas representações pictóricas.

II. Num olho humano com visão perfeita, o cristalino

focaliza exatamente sobre a retina um feixe de luz

vindo de um objeto. Quando o cristalino está em

sua forma mais alongada, é possível focalizar o

feixe de luz vindo de um objeto distante. Quando

o cristalino encontra-se em sua forma mais

arredondada, é possível a focalização de objetos

cada vez mais próximos do olho, até uma distância

mínima.

III. Um dos problemas de visão humana é a miopia.

No olho míope, a imagem de um objeto distante

formase depois da retina. Para corrigir tal defeito,

utilizase uma lente divergente.

Está correto o que se afirma em

a) I, apenas.

b) I e II, apenas.

c) I e III, apenas.

d) II e III, apenas.

e) I, II e III.

Questão 15 - (FCM MG/2014)

Três estudantes opinaram a respeito da formação de

imagens em espelhos e lentes.

Amanda afirmou que uma imagem virtual menor que

uma pessoa é obtida se a pessoa estiver em frente a

um espelho plano, distante dele.

Betina disse que uma imagem real e maior das letras

de um texto é obtida quando ela vê tais letras através

de uma lente biconvexa, posicionada próxima ao texto.

Camila acha que a imagem de uma ameba vista por um

microscópio simples, composto de duas lentes

convergentes, é virtual e invertida.

Do ponto de vista da Física, apenas

a) Betina fez uma afirmação correta.

b) Camila fez uma afirmação correta.

c) Amanda e Betina fizeram afirmações corretas.

d) Amanda e Camila fizeram afirmações corretas.

Questão 16 - (FGV/2014)

A lupa é um instrumento óptico constituído por uma

lente de aumento. Para cumprir sua função, ela deve

ser

a) divergente e estar posicionada a uma distância do

objeto analisado menor que sua distância focal.

b) divergente e estar posicionada a uma distância do

objeto analisado compreendida entre o foco e o

ponto antiprincipal da lente.

c) convergente e estar posicionada a uma distância

do objeto analisado menor que sua distância focal.

Page 134: curso de específica de física

134

d) convergente e estar posicionada a uma distância

do objeto analisado compreendida entre o foco e o

ponto antiprincipal.

e) convergente e estar posicionada a uma distância

do objeto analisado maior que a distância focal.

Questão 17 - (UEA AM/2014)

Considere a ilustração da bandeira do estado do

Amazonas:

(IBGE. Atlas geográfico escolar, 2009.)

A cor de um objeto iluminado é determinada pela

radiação luminosa que ele reflete. Assim, corpo verde

reflete apenas luz verde, corpo branco reflete luz de

qualquer cor que nele incide, enquanto corpo negro não

reflete luz alguma. Caso a bandeira do Amazonas venha

a ser iluminada apenas por luz monocromática

vermelha, as cores que ela mostrará serão somente

a) vermelha e branca.

b) vermelha, branca e preta.

c) vermelha e verde.

d) vermelha, branca e verde.

e) vermelha e preta.

Questão 18 - (UFPR/2014)

Um microscópio composto é constituído, em sua forma

mais simples, por duas lentes convergentes colocadas

em sequência, conforme esquematizado na figura

abaixo. A lente mais próxima ao objeto é chamada

objetiva e a lente mais próxima ao olho humano é

chamada ocular. A imagem formada pela objetiva é

real, maior e invertida, e serve como objeto para a

ocular, que forma uma imagem virtual, direita e maior

com relação à imagem formada pela objetiva. Suponha

que a distância focal da lente objetiva seja 1 cm, a

distância focal da lente ocular seja 4 cm e a distância

entre as lentes seja de 6 cm.

Com base nas informações acima e nos conceitos de

Óptica, identifique como verdadeiras (V) ou falsas (F)

as seguintes afirmativas:

( ) Para que a imagem formada pela objetiva tenha

as características especificadas no enunciado, o

objeto deve estar a uma distância maior que 2 cm

dessa lente.

( ) Supondo que o objeto esteja a uma distância de

1,5 cm da objetiva, a imagem formada por esta

lente estará a 3 cm dela.

( ) A imagem final formada por este microscópio é

virtual, invertida e maior em relação ao objeto.

( ) A imagem formada pela objetiva deve estar a uma

distância maior que 4 cm da ocular.

Assinale a alternativa que apresenta a sequência

correta, de cima para baixo.

a) V – F – F – V.

b) F – V – V – F.

c) V – V – F – F.

d) F – F – V – V.

e) F – V – V – V.

Questão 19 - (FUVEST SP/2014)

Um estudante construiu um microscópio ótico digital

usando uma webcam, da qual ele removeu a lente

original. Ele preparou um tubo adaptador e fixou uma

lente convergente, de distância focal f = 50 mm, a uma

distância d = 175 mm do sensor de imagem da

webcam, como visto na figura abaixo.

No manual da webcam, ele descobriu que seu sensor de

imagem tem dimensão total útil de 6 x 6 mm2, com 500

x 500 pixels. Com estas informações, determine

a) as dimensões do espaço ocupado por cada pixel;

b) a distância L entre a lente e um objeto, para que

este fique focalizado no sensor;

c) o diâmetro máximo D que uma pequena esfera

pode ter, para que esteja integralmente dentro do

campo visual do microscópio, quando focalizada.

Note e adote:

Pixel é a menor componente de uma imagem digital.

Para todos os cálculos, desconsidere a espessura da

lente.

Questão 20 - (UEM PR/2013)

Com relação ao funcionamento de instrumentos

ópticos, assinale o que for correto.

01. Uma lupa, que utiliza uma lente convergente de

grande convergência, conjuga uma imagem

virtual, direita e maior de um objeto real colocado

entre a lupa e seu plano focal.

02. Um telescópio refletor pode ser construído com

duas lentes divergentes de baixa divergência – a

objetiva e a ocular – e é utilizado para observar

objetos no infinito ou muito distantes.

04. Quando os focos da objetiva e da ocular de um

telescópio refrator coincidem, o aumento desse

telescópio é tanto maior quanto maior é a razão

entre as distâncias focais da objetiva e da ocular,

as quais são lentes convergentes.

08. Os projetores são construídos com lentes

divergentes de elevada divergência e fornecem

Page 135: curso de específica de física

135

uma imagem real, direita e maior, a qual pode ser

projetada em um anteparo.

16. Os instrumentos ópticos de observação são

compostos por lentes convergentes ou divergentes

e podem ser utilizados tanto para a aproximação

de objetos distantes quanto para a observação de

objetos muito pequenos.

GABARITO:

1) Gab: B 2) Gab: C 3) Gab: B

4) Gab: C 5) Gab: 09 6) Gab: A

7) Gab: C 8) Gab: A 9) Gab: B

10) Gab: E 11) Gab: E 12) Gab: C

13) Gab: 07 14) Gab: B 15) Gab: B

16) Gab: C 17) Gab: E 18) Gab: B

19) Gab:

a) 24 mm10.44,1A

b) mm70L

c) mm4,2D

20) Gab: 05 21) Gab: 01 22) Gab: 06

23) Gab: C 24) Gab: 02 25) Gab: E

26) Gab: C 27) Gab: 15 28) Gab: D

29) Gab: B

30) Gab:

a) p' = 0,3 cm

b) q = 1 pC

31) Gab: B 32) Gab: B 33) Gab: C 34) Gab: A 35) Gab: C 36) Gab:

a) Claramente as imagens terão ampliações diferentes, pois a ampliação por uma lente delgada é dada por A = –p’/p, onde p’ e p são, respectivamente, as distâncias entre imagem e objeto à lente. Na primeira posição da lente (P), em que ela está mais próxima do objeto, p’1 < p2 e, portanto, |A1| > 1 (ampliação do tamanho). Já na segunda posição da lente (P’), p’2 < p2 e, portanto, |A2|<1 (redução de tamanho).

b) p1 = 2,0 cm

c) De 11 pL

1

p

1

f

1

, obtemos:

cm

cm

0,10

)0,20,10(0,2

L

)pL(pf

211

f = 1,6

cm 37) Gab: A 38) Gab: A 39) Gab: 01 40) Gab:

A primeira imagem pedida está a 12 cm da lente e é real. Usando a lente como lupa, a imagem vista é ampliada é sua altura é 10 mm.

Page 136: curso de específica de física

136

LISTA 25 – MOVIMENTO HARMÔNICO SIMPLES E ONDULATÓRIA

Movimento periódico e oscilatório

Período do movimento

Frequência do movimento

Equivalência entre frequência e período

Funções horárias

Elongação

Velocidade

Aceleração

Page 137: curso de específica de física

137

Pulsação

Força no MHS

Força

Constante de força do MHS

Pulsação

Período do movimento

Frequência do movimento

Oscilador massa-mola

Força

Período

Pêndulo simples

Força

Page 138: curso de específica de física

138

Período

EXERCÍCIOS

Questão 01 - (Fac. de Ciências da Saúde de

Barretos SP/2014)

Em 1851, o francês Jean Bernard Foucault realizou uma

experiência simples e engenhosa que demonstrou a

rotação da Terra. No Panthéon de Paris, ele montou um

pêndulo que oscilava com período de aproximadamente

16 segundos.

Abandonado da posição mostrada na figura 1, um

pêndulo igual ao de Foucault passará pela terceira vez

pela posição mostrada na figura 2 após um intervalo de

tempo, em segundos, igual a

a) 12.

b) 24.

c) 28.

d) 16.

e) 20.

Questão 02 - (UEPG PR/2013)

Executando um movimento ideal, uma esfera é solta do

ponto A, atinge o ponto B, e retorna ao ponto A e assim

procede sucessivamente. Sua sombra é projetada sobre

um anteparo conforme é mostrado abaixo. Sobre o

movimento da esfera e sua sombra sobre o anteparo,

assinale o que for correto.

01. A frequência do movimento é o tempo entre duas

passagens consecutivas da sombra ou da esfera

pelo ponto de equilíbrio.

02. A energia mecânica do modelo apresentado é

variável e depende da altura e do grau de

inclinação da rampa que é solta.

04. O modelo apresentado se ajusta a um fenômeno

físico que tem como características peculiares

inter-valos de tempos e extensões, iguais e

sucessivas, caracterizando-o como um fenômeno

periódico.

08. Amplitude do movimento é a distância da posição

de equilíbrio ao ponto de inversão do movimento,

apresentada pela sombra da esfera sobre o

anteparo.

16. O módulo da aceleração da esfera é constante

apenas alternando sua ação, sendo o movimento

ora acelerado e ora retardado.

Questão 03 - (UEPG PR/2013)

Um pequeno bloco de massa m executa um movimento

harmônico simples (MHS) de amplitude A em relação a

um dado referencial. Sobre esse sistema físico, o

movimento descrito e suas relações com a energia

mecânica, assinale o que for correto.

01. Um MHS é caracterizado pela oscilação periódica

de um ponto material em torno de uma posição de

equilíbrio, sob a ação de uma força restauradora.

02. O período e a frequência de um MHS independem

da amplitude do movimento.

04. Quando o bloco se afasta da posição de equilíbrio,

sua energia cinética diminui enquanto sua energia

potencial aumenta.

08. No MHS quando o deslocamento é máximo, em

qualquer sentido, a velocidade é nula, o módulo da

aceleração é máximo, a energia cinética é nula e a

energia potencial é máxima.

Questão 04 - (UEM PR/2013)

Um ponto material oscila segundo um movimento

harmônico simples, com amplitude de 0,5 m e

frequência de 2 Hz. Considerando que, para o instante

t = 0 s, o ponto material estava com energia potencial

positiva e máxima, assinale o que for correto.

01. A frequência angular e a fase inicial do ponto

material são, respectivamente, 4 rad/s e 0 rad

(zero rad).

02. A função horária da posição do ponto material é

dada por x(t) = 0,5cos (4 t).

04. A energia cinética assume seu valor máximo duas

vezes a cada oscilação.

08. A aceleração do ponto material não varia com o

decorrer do tempo.

16. A energia mecânica nesse tipo de movimento

permanece constante.

Page 139: curso de específica de física

139

TEXTO: 1 - Comuns às questões: 5, 19

Adote os conceitos da Mecânica Newtoniana e as

seguintes convenções:

- O valor de = 3 ;

- A resistência do ar pode ser desconsiderada.

Questão 05 - (UFPB/2013)

Uma forma de se obter energia elétrica limpa é

aproveitar o movimento ondulatório das ondas

marítimas. Nesse sentido, considere que um objeto

flutuando sobre o mar realiza movimento harmônico

simples com sua energia potencial descrita pelo gráfico

abaixo.

Nessas condições, a variação da energia cinética desse

objeto em função do tempo está melhor representada

no gráfico:

a)

b)

c)

d)

e)

Questão 06 - (ACAFE SC/2014)

Na medicina os raios X são utilizados nas análises das

condições dos órgãos internos, pesquisas de fraturas,

tratamento de tumores, câncer (ou cancro), doenças

ósseas, etc. Raios X são basicamente o mesmo que os

raios de luz visíveis, ou seja, formado por ondas

eletromagnéticas que se propagam no vácuo à

velocidade da luz.

Em relação aos raios X, analise as afirmações a seguir.

I. Os raios X estão sujeitos ao fenômeno da

reflexão, refração, difração e interferência.

II. Sua penetrância nos materiais é relevante, pois

todas as substâncias são penetradas pelos raios

X em maior ou menor grau.

III. Por estar sujeito à reflexão, os raios X não pene

tram os ossos.

IV. A diferença entre raios X e raios de luz visível é

devida ao comprimento de onda diferente dos

mesmos.

Todas as afirmações corretas estão em:

a) I - II - III

b) I - II - IV

c) II - III

d) III - IV

Questão 07 - (UNEB/2014)

Os alvejantes para roupas coloridas, destinados à

limpeza e ao clareamento de manchas por meio de

lavagem, são produtos à base de oxigênio. Eles liberam

essa substância química na forma ativa, em processo

semelhante ao que ocorre quando se adiciona água

oxigenada sobre um ferimento, observando-se a

formação de “espuma” no local.

Diferentemente dos alvejantes para roupas brancas,

que são à base de cloro, eles têm uma ação mais suave

sobre as fibras e corantes do tecido, que não são

afetados pelo oxigênio ativo.

Funcionam, ainda, como bactericidas e fungicidas.

Nas roupas, o branqueamento ocorre pela destruição

dos corantes e compostos orgânicos presentes nos

tecidos, levando à reflexão da luz como um todo. Ou

seja, ao incidir luz branca, todas as cores são refletidas,

produzindo o branco. (NASCIMENTO. 2009. p. 6).

Page 140: curso de específica de física

140

Com base nas informações do texto e nos

conhecimentos da mecânica ondulatória, é correto

afirmar:

01. O feixe de luz que incide perpendicularmente

sobre o tecido não sofre reflexão nem refração.

02. Os tecidos submetidos ao processo de

branqueamento refletem integralmente toda

radiação incidente.

03. A frequência, o módulo da velocidade de

propagação e o comprimento de onda de um feixe

luminoso se mantêm constante no fenômeno de

reflexão.

04. As fibras dos tecidos submetidos ao processo de

branqueamento se comportam como uma

superfície espelhada ideal para possibilitar uma

reflexão regular da luz.

05. O comprimento de onda, , da luz refletida pelo

tecido branco é determinado pela igualdade v.f,

sendo v a velocidade de propagação e f, a

frequência da luz.

Questão 08 - (FMJ SP/2012)

Uma onda seguia da esquerda para a direita,

propagando-se em um fio considerado ideal, mantido

tenso. Decorrido algum tempo, observa-se a passagem

de uma outra onda, desta vez proveniente da direita

para a esquerda, idêntica à primeira, em amplitudes e

comprimento de onda.

Analise as seguintes afirmações sobre o que poderia ter

ocorrido para que se obtivesse o padrão de onda visto

momentos depois.

I. A onda sofreu reflexão total e, pelo formato de

onda do pulso recebido, a corda, ao lado direito,

está atada a um ponto que possui mobilidade

vertical.

II. À direita, a corda se encontra presa a uma

segunda corda, com o dobro da densidade da

primeira e, além do pulso refratado, também é

produzido um pulso refletido, igual ao original,

propagando-se em sentido oposto.

III. Uma outra onda, movendo-se da direita para a

esquerda, semelhante à primeira, com o mesmo

comprimento de onda, porém, o dobro das

amplitudes, interagiu com a primeira de modo

destrutivo, sendo o pulso recebido consequência

da interação entre as duas.

Do que foi levantado, com respeito às possibilidades

que geraram a onda da figura 2, está correto o contido

em

a) I, apenas.

b) II, apenas.

c) I e III, apenas.

d) II e III, apenas.

e) I, II e III.

Questão 09 - (UCS RS/2012)

Sem dúvida, a lua cheia é um dos mais belos fenômenos

naturais, inspirando poetas, romancistas e escritores de

histórias de terror. Mas essa imagem só é visível para

nós porque a superfície lunar apresenta o princípio físico

de

a) absorção e reflexão de ondas eletromagnéticas.

b) indução magnética.

c) conversão de energia mecânica em energia

luminosa.

d) fissão nuclear.

e) sublimação.

Questão 10 - (FCM MG/2014)

A figura abaixo mostra ondas correspondentes às

emitidas por cada nota de um piano.

Considerando as ondas emitidas por cada nota do

piano, V é a sua velocidade de propagação, f é sua

frequência, A é a sua amplitude e é o seu comprimento

de onda.

Ao compararem-se as características das ondas

especificadas na figura por P e Q, pode-se afirmar que

a) VP > VQ.

b) AP > AQ.

c) P > Q.

d) fP > fQ.

Questão 11 - (UFG GO/2014)

O princípio de funcionamento do forno de micro-ondas

é a excitação ressonante das vibrações das moléculas

de água contidas nos alimentos. Para evitar a fuga de

radiação através da porta de vidro, os fabricantes de

fornos de micro-ondas colocam na parte interna do

vidro uma grade metálica. Uma condição para que uma

onda eletromagnética seja especularmente refletida é

que seu comprimento de onda seja maior que o

tamanho das irregularidades da superfície refletora.

Considerando-se que a frequência de vibração da

molécula de água é aproximadamente 2,40 GHz e que

Page 141: curso de específica de física

141

o espaçamento da grade é da ordem de 1,0% do

comprimento de onda da micro-onda usada, conclui-se

que o espaçamento em mm é:

Dados: c = 3,00 x 108 m/s

a) 0,8

b) 1,25

c) 8

d) 80

e) 125

Questão 12 - (UNIRG TO/2014)

A figura a seguir mostra uma onda em dois instantes

distintos.

De acordo com a figura apresentada, a velocidade de

fase em cm/s, o comprimento de onda em cm e a

equação da onda são, respectivamente,

a) 2, 16 e sen[(/8)x + (/4)t]

b) 8, 16 e sen[(/4)x + (/8)t]

c) 2, 16 e cos[(/8)x – (/4)t]

d) 8, 16 e cos[(/4)x – (/8)t]

Questão 13 - (ACAFE SC/2013)

O estetoscópio é um instrumento utilizado para ouvir as

batidas do coração ou sons no interior do corpo

humano. Existe uma campânula metálica que encosta

no corpo e amplifica o som através de uma estrutura

interna que imita o ouvido humano. Depois o som

amplificado, segue por tubos de condução, passa pela

haste metálica e pelas olivas auriculares, chegando ao

ouvido do médico.

Fonte:

http://filhadosono.blogspot.com.br/2010_03_01_archi

ve.html.

Acesso em: 14/10/2012.

Considere uma consulta médica em que é utilizado um

estetoscópio para escutar os batimentos cardíacos do

paciente. Nesse processo, o médico escuta 21 batidas

em 0,25 min.

Considerando as informações dadas e os conhecimentos

de física, assinale a alternativa correta que representa

a frequência cardíaca do paciente.

a) 5,25 batidas/s.

b) 1,40 batidas/s.

c) 60 batidas/min.

d) 8,40 batidas/min.

Questão 14 - (UEL PR/2013)

Suponha que as ondas geradas pelo satélite

geoestacionário possuam uma frequência constante de

1,0108 Hz e demorem 1,110–1 s para percorrer a

distância de 3,3107 mentre o emissor e uma antena

receptora.

Com relação às ondas emitidas, considere as

afirmativas a seguir.

I. Sua velocidade é de 3,0108 m/s.

II. Sua velocidade é diretamente proporcional ao seu

comprimento de onda.

III. Sua velocidade é inversamente proporcional à sua

frequência.

IV. Seu comprimento de onda é de 3,010–3 m.

Assinale a alternativa correta.

a) Somente as afirmativas I e II são corretas.

b) Somente as afirmativas I e IV são corretas.

c) Somente as afirmativas III e IV são corretas.

d) Somente as afirmativas I, II e III são corretas.

e) Somente as afirmativas II, III e IV são corretas.

Questão 15 - (UFG GO/2013)

Nos jogos paraolímpicos de Londres, o sul-africano

biamputado Oscar Pistorius, após perder a medalha de

ouro para o brasileiro Alan Fonteles, indignado,

reclamou do tamanho das próteses de Fonteles. Antes

dos jogos, elas foram trocadas por um par 5,0 cm maior

que, no entanto, estavam dentro do limite estabelecido

pelo regulamento. Porém, mesmo com próteses mais

longas, as amplitudes de passada de Fonteles foram

menores do que as de Pistorius, conforme o quadro da

prova de 200 metros rasos apresentado a seguir.

Considere que Fonteles consiga aumentar a amplitude

média de sua passada em 1,0 cm, mantendo a mesma

frequência de passadas. Nessas circunstâncias, quantos

segundos, aproximadamente, será a nova vantagem de

Fonteles?

a) 0,05

b) 0,07

c) 0,10

d) 0,17

Page 142: curso de específica de física

142

e) 0,35

Questão 16 - (UFTM/2013)

Duas ondas, 1 e 2, propagam-se por cordas idênticas e

igualmente tracionadas. A figura representa parte

dessas cordas.

Sabendo que a frequência da onda 1 é igual a 8 Hz, é

correto afirmar que a frequência da onda 2, em hertz,

é igual a

a) 14.

b) 16.

c) 18.

d) 12.

e) 10.

Questão 17 - (UNIRG TO/2013)

Nos equipamentos domésticos atuais é comum se

encontrar lasers de cores azul, verde e vermelho. O uso

doméstico mais difundido do laser é para gravação de

dados em CD, DVD etc. A característica do laser que

possibilitou uma maior compactação de dados nos

dispositivos domésticos foi a obtenção de uma luz

visível de

a) alta potência.

b) baixa energia.

c) pequeno comprimento de onda.

d) baixa frequência.

TEXTO: 2 - Comum à questão: 18

A primeira usina de ondas da América Latina,

lançada oficialmente durante a Rio+20, funciona no

porto do Pecém, a 60 quilômetros de Fortaleza. Para os

pesquisadores, o local é um laboratório em escala real,

onde serão ampliados os horizontes da produção

energética limpa e renovável. O potencial é grande,

asseguram. Na prática, de acordo com especialistas da

Coppe, que desenvolve a tecnologia, é possível

converter cerca de 20% da energia das ondas do mar

em energia elétrica.

Dois enormes braços mecânicos foram instalados no

píer do porto do Pecém. Na ponta de cada um deles, em

contato com a água do mar, há uma boia circular.

Conforme as ondas batem, a estrutura sobe e desce. O

movimento contínuo dos flutuadores aciona bombas

hidráulicas, promovendo um ambiente de alta pressão.

A água, ejetada a altíssima pressão, faz a turbina girar.

Fazendo uma analogia com uma usina hidrelétrica, em

vez de termos uma queda d’água, temos isso de forma

concentrada em dispositivos relativamente pequenos,

onde a pressão simula cascatas extremas de

aproximadamente 400 metros, a depender da

intensidade das ondas. (PAÍS começa..., 2012).

PAÍS começa a explorar energia limpa das ondas.

Disponível em:

<http://oglobo.globo.com/ciencia/pais-comeca-explorar-

energia-limpa-das-ondas-

5122838#ixzz252MmZT4L>. Acesso em: 18 out. 2012.

(Adaptado)

Questão 18 - (UFBA/2013)

Considere que, por alguns instantes, uma onda senoidal

incide sobre uma boia que está acoplada ao braço da

usina de ondas, fazendo-a oscilar periodicamente. Essa

onda senoidal se propaga no mar com velocidade

aproximada de 20,0m/s e tem comprimento de onda de

1,0m, sendo responsável pela produção, ao final do

processo, de cerca de 100,0kJ de energia elétrica por

segundo.

Com base nessa informação e nos conhecimentos de

Física, e admitindo-se a densidade da água do mar

como sendo 1g/cm3 e o módulo da aceleração da

gravidade local igual a 10m/s2, determine

o período da oscilação gerada na boia devido à

passagem da onda;

a potência, aproximada, transportada pelas ondas

do mar;

a vazão volumétrica média de água em uma

hidroelétrica equivalente à usina de ondas,

desprezando as perdas.

TEXTO: 3 - Comuns às questões: 5, 19

Adote os conceitos da Mecânica Newtoniana e as

seguintes convenções:

- O valor de = 3 ;

- A resistência do ar pode ser desconsiderada.

Questão 19 - (UFPB/2013)

O robô Curiosity, que tem a missão de analisar o solo

do planeta Marte, comunica-se com sua base na Terra

por meio de ondas eletromagnéticas propagantes

através de dois canais diferentes. As funções de onda,

Page 143: curso de específica de física

143

associadas às ondas nos dois canais, são definidas por

Y1 e Y2, conforme equações abaixo:

Y1 = Asen(2 10–4 x – 6 104 t)

Y2 = Asen(1 10–4 x – 3 104t)

Nessas equações, x é a posição em metros ao longo de

uma linha reta unindo o robô e a Terra, e t é o tempo

em segundos.

Em relação às propriedades dessas ondas, considere as

afirmativas:

I. A frequência angular da onda Y1 é 1104 Hz.

II. O período da onda Y1 é o dobro do período da onda

Y2.

III. A onda Y2 tem comprimento de onda 6104m.

IV. As ondas Y1 e Y2 propagam-se com a mesma

velocidade de 3108m/s.

Estão corretas apenas as afirmativas:

a) I e II

b) II e IV

c) III e IV

d) I, III e IV

e) I, II e III

Questão 20 - (UECE/2014)

Uma massa m presa a uma mola de constante elástica

k oscila de modo que a coordenada posição da massa

seja dada por x = Xmaxsen t m/k e a velocidade

t m/kcosXm/kv max . Assim, pode-se afirmar

corretamente que

a) a energia cinética máxima é dada por 2maxX

m

k

2

1 .

b) a energia mecânica do sistema é dada por

2/kX2m ax

.

c) a energia potencial elástica máxima é dada por

m/ksenkX 22max

.

d) a energia cinética mínima é dada por

m/kcoskX2

1 22max

.

GABARITO:

1) Gab: E 2) Gab: 12 3) Gab: 15

4) Gab: 23 5) Gab: B 6) Gab: B

7) Gab: 03 8) Gab: A 9) Gab: A

10) Gab: D 11) Gab: B 12) Gab: A

13) Gab: B 14) Gab: A 15) Gab: D

16) Gab: D 17) Gab: C

18) Gab:

O período da onda incidente será T = 0,05s.

A onda excita a boia, fazendo-a oscilar com esse

mesmo período.

Pmar = 500 kW

0,025 m3/s

19) Gab: C 20) Gab: B 21) Gab: D

22) Gab: B 23) Gab: C

24) Gab: A energia não depende do tempo, assim a energia

será sempre ET = 0,125 J para qualquer instante de

tempo.

25) Gab:

a) k1 = 9,0 N/m

k2 = 50 N/m

b) T1 = 1,24 s, f1 = 0,8 Hz

T2 = 0,62 s, f2 = 1,6 Hz

c) madicional = 1,50 kg

26) Gab: B

27) Gab: C

28) Gab: A

29) Gab: C

30) Gab:

a) T = 0,628 s

b) Nada irá ocorrer com a marcação das horas,

porque o período do pêndulo independe de sua

massa.

c) A peça maciça deve ser colocada mais próxima à

engrenagem, ou diminuir a distância a que ela está

fixada da extremidade da haste. Assim, o período

do pêndulo irá diminuir, fazendo com que o relógio

passe a funcionar mais rapidamente, diminuindo o

atraso.

31) Gab: E 32) Gab: 14 33) Gab:

a) A força que age na mola tem o mesmo valor da força com que a mola age no corpo (3ª Lei de Newton).

Pode-se calcular seu valor aplicando-se a lei de Hooke. Fel = kx Fel = k (L – Lo) Fel = 300 (0,3 – 0,2) Fel = 30 N

b) Aplicando-se o princípio fundamental da dinâmica para o corpo, tem-se: m = 1kg Fel = 30N r = L = 0,3 m RC = Fel maC = Fel

m 2 r = Fel

12 0,3 = 30

s

rad10

c) No instante t, o corpo possui energia cinética e energia potencial elástica, portanto: v = 3 m/s x = 0,1 m m = 1 kg k = 300 N/m

elástica .potC.sist.mec

2

kx

2

mv 22.sist.mec

2

1,0300

2

31 22.sist.mec

J6.sist.mec

d) t

63,0

s20t

34) Gab: D 35) Gab: B 36) Gab: 29 37) Gab: 14 38) Gab: 03 39) Gab: D 40) Gab: 12 41) Gab: B

Page 144: curso de específica de física

144

42) Gab: B 43) Gab: 27 44) Gab: 14 45) Gab: C 46) Gab: 14 47) Gab: E 48) Gab: 12 49) Gab: A 50) Gab: D

Page 145: curso de específica de física

145

LISTA 26 – INTERFERÊNCIA E ACÚSTICA A área da Física que estuda o som é chamada de Acústica. Para entende-la melhor precisamos relembrar alguns conceitos: Onda: é a variação periódica de uma grandeza física. Uma onda é composta por: Crista: Pontos de maior intensidade, o topo da onda. Vale: Pontos de menor intensidade da onda. Nível Médio: Pontos entre o as Cristas e os Vales.

A distância entre a crista ou o vale e o nível médio é chamada amplitude (y). Já a distância entre duas cristas consecutivas ou dois vales consecutivos é chamada de comprimento de onda (λ).

Onde: λ –Comprimento y - Amplitude O tempo que uma oscilação leva para se repetir é chamado período (T), medido em segundos(s). A frequência (f) significa quantas vezes uma oscilação se repete por unidade de tempo, medida em Hertz (Hz). Dessa forma:

f = 1/t O Som é uma onda mecânica que possui a intensidade e frequência necessárias para ser percebida pelo ser humano. Entendemos como onda mecânica uma onda que precisa de meios materiais, como o ar ou o solo, para se propagar. As frequências audíveis pelo ouvido humano ficam entre 16 Hz e 20000Hz (20kHz). Dentro desta faixa a encontram-se a voz humana, instrumentos, musicais, alto-falantes, etc. Abaixo de 16Hz temos os infrassons, produzidos por vibrações da água em grandes reservatórios, batidas do coração, etc. Acima de 20kHz estão os ultrassons emitidos por alguns animais e insetos (morcegos, grilos, gafanhotos...), sonares, aparelhos médicos e industriais. Os dispositivos que produzem ondas sonoras são chamados de fontes sonoras. Entre os que mais se destacam estão aqueles compostos por:

Cordas vibrantes como violão o piano, as cordas vocais etc.

Tubos sonoros como órgão flauta, clarineta.

Membranas e placas vibrantes tal como o tambor

Hastes vibrantes como o diapasão, triangulo, etc.

Podemos caracterizar os sons a partir de sua intensidade, altura ou timbre. A intensidade está ligada à quantidade de energia transportada pelo som. Desta forma, conforme a intensidade do som dizemos que ele é mais forte (a onda possui maior amplitude) ou mais fraca (a onda possui menor amplitude). A altura está relacionada com a frequência do som. Assim distinguimos os sons mais altos como os de maior frequência (mais agudos) e os mais baixos como os de menor frequência (mais graves). As notas musicais buscam agrupar diferentes frequências sonoras produzidas por um instrumento. O timbre corresponde ao conjunto de ondas sonoras que formam um som. O timbre permite diferenciar diferentes

fontes sonoras, por exemplo é fácil perceber que o som de uma guitarra e de uma flauta são completamente diferentes. A velocidade do som no ar é de 340 m/s. A fórmula que relaciona velocidade, amplitude e frequência sonora é:

V = λ . f

EXERCÍCIOS

Questão 01 - (UECE/2014)

Uma onda sonora de 170 Hz se propaga no sentido

norte-sul, com uma velocidade de 340 m/s. Nessa

mesma região de propagação, há uma onda

eletromagnética com comprimento de onda 2106 m

viajando em sentido contrário. Assim, é correto afirmar-

se que as duas ondas têm

a) mesmo comprimento de onda, e pode haver

interferência construtiva.

b) mesmo comprimento de onda, e pode haver

interferência destrutiva.

c) mesmo comprimento de onda, e não pode haver

interferência.

d) diferentes comprimentos de onda, e não pode

haver interferência.

Questão 02 - (Fac. Santa Marcelina SP/2013)

O Supremo Tribunal de Justiça determinou que a

prescrição do pedido de indenização por erro médico

inicia-se na data em que o paciente toma conhecimento

da lesão, e não na data em que o profissional comete o

ilícito. Submetida anteriormente a uma cesariana, uma

paciente realizou, após queda acidental, exames

radiográficos para avaliar possível deslocamento dos

rins. Durante os exames, foi detectada em seu

abdômen uma agulha cirúrgica que, até então, ela

afirmava não sentir.

(http://extra.globo.com)

Nesse caso, o diagnóstico ocorreu devido ao fato de a

paciente ser submetida à radiação ionizante, de

natureza

a) eletromagnética e de grande capacidade de

penetração, denominada partícula alfa.

b) mecânica e de baixa capacidade de penetração,

denominada raio X.

c) eletromagnética e de baixa capacidade de

penetração, denominada raio gama.

Page 146: curso de específica de física

146

d) mecânica e de grande capacidade de penetração,

denominada raio gama.

e) eletromagnética e de grande capacidade de

penetração, denominada raio X.

Questão 03 - (UEM PR/2013)

Com relação à produção, à caracterização e ao

comportamento de ondas, analise as alternativas

abaixo e assinale o que for correto.

01. Uma onda progressiva transversal ou longitudinal

pode ser considerada como uma perturbação que

se propaga em um meio, transportando energia de

um ponto a outro desse meio sem transportar

matéria.

02. Quando uma frente de ondas transversais

luminosas atravessa de um meio a outro (tendo

esses meios índices de refração diferentes), ocorre

mudança na direção de propagação da frente de

ondas, sem alteração na frequência de oscilação

dessas ondas.

04. O comprimento de onda de uma onda mecânica

transversal independe do meio de propagação

dessa onda.

08. Quanto maior a frequência de uma onda

eletromagnética, maior é a energia que essa onda

transporta, e menor é o seu comprimento de onda.

16. O fenômeno da interferência, que pode ser

construtiva ou destrutiva, ocorre quando há

superposição de ondas que se propagam em um

meio.

Questão 04 - (UEMA/2012)

Técnicos em acústica utilizam o caráter ondulatório do

som para eliminação, total ou parcial, de ruídos

indesejáveis. Para isso, microfones captam o ruído do

ambiente e o enviam a um computador, programado

para analisá-lo e para emitir um sinal ondulatório que

anule o ruído original indesejável. Em qual fenômeno

ondulatório se fundamenta essa tecnologia?

a) Interferência.

b) Polarização.

c) Difração.

d) Reverberação.

e) Reflexão.

Questão 05 - (UDESC/2012)

De acordo com o princípio da superposição, duas ondas

luminosas podem se somar (interferência construtiva)

ou se subtrair (interferência destrutiva) quando

atingem simultaneamente o mesmo ponto do espaço.

Analise as proposições em relação ao exposto.

I. Luz mais luz pode resultar em escuridão.

II. Luz mais escuridão pode resultar em escuridão.

III. Luz mais escuridão pode resultar em luz.

IV. Escuridão mais escuridão pode resultar em luz.

Assinale a alternativa correta.

a) Somente as afirmativas II e IV são verdadeiras.

b) Somente as afirmativas I e III são verdadeiras.

c) Somente as afirmativas II e III são verdadeiras.

d) Somente as afirmativas I e IV são verdadeiras.

e) Todas as afirmativas são verdadeiras.

Questão 06 - (UDESC/2013)

Analise as proposições em relação ao efeito de

polarização das ondas eletromagnéticas.

I. A polarização é uma característica das ondas

transversais.

II. A polarização é uma característica das ondas

longitudinais.

III. Os óculos de sol são exemplos de filtros

polarizadores e aumentam a intensidade da

radiação incidente.

IV. Os óculos de sol são exemplos de filtros

polarizadores e reduzem a intensidade da radiação

incidente.

Assinale a alternativa correta.

a) Somente as afirmativas I e III são verdadeiras.

b) Somente as afirmativas I e IV são verdadeiras.

c) Somente as afirmativas II e III são verdadeiras.

d) Somente as afirmativas II e IV são verdadeiras.

e) Somente a afirmativa III é verdadeira.

Questão 07 - (UEPG PR/2012)

Onda pode ser definida como uma perturbação em um

meio que tem como efeito o transporte de energia sem

o arraste de matéria. Com relação a ondas, assinale o

que for correto.

01. Tanto as ondas mecânicas quanto as ondas

eletromagnéticas atingem suas maiores

velocidades nos meios sólidos.

02. As frequências de ondas de infrasom e do raio X

estão abaixo dos órgãos sensitivos, por isso não

atingem a audição e nem a visão do ser humano.

04. Não há necessidade de um meio material para que

a energia liberada pela vibração de cargas

elétricas se propague.

08. Reflexão, refração, difração e interferência são

propriedades das ondas e podem ser observadas

tanto nas ondas mecânicas quanto nas ondas

eletromagnéticas.

Questão 08 - (UFAL/2011)

Alex encontra-se dentro de uma sala, cujas paredes

laterais e superior possuem isolamento acústico. A

porta da sala para o exterior está aberta. Alex chama

Bruno, que está fora da sala (ver figura). Pode-se

afirmar que Bruno escuta Alex porque, ao passar pela

porta, a onda sonora emitida por este sofre:

a) polarização.

b) regularização.

c) fissão.

Page 147: curso de específica de física

147

d) refração.

e) difração.

Questão 09 - (UFG GO/2010)

Uma estação de rádio emite ondas médias na faixa de 1

MHz com comprimento de onda de 300 m. Essa radiação

contorna facilmente obstáculos como casas, carros,

árvores etc. devido ao fenômeno físico da

a) difração.

b) refração.

c) reflexão.

d) interferência.

e) difusão.

Questão 10 - (UDESC/2014)

Assinale a alternativa incorreta a respeito dos

fenômenos ondulatórios.

a) O som é uma onda mecânica longitudinal.

b) Se uma das extremidades de uma corda

tensionada passar a vibrar verticalmente,

produzirá ondas transversais.

c) Uma onda eletromagnética propaga-se no ar com

velocidade aproximadamente igual à da luz no

vácuo.

d) O eco é um fenômeno causado pela reflexão do

som em um obstáculo.

e) Cada modo de oscilação de uma onda estacionária,

que se forma em uma corda esticada, pode ser

considerado uma consequência da interferência de

duas ondas senoidais idênticas que se propagam

no mesmo sentido.

Questão 11 - (ESCS DF/2013)

Criados em 1816 por René Laennec, os primeiros

estetoscópios consistiam em simples tubos de madeira.

A função do estetoscópio é permitir a auscultação —

escuta dos sons internos do corpo. Com esse aparelho,

podem ser examinados os sistemas circulatório,

respiratório e até o digestivo, de forma rápida e não

invasiva.

As figuras acima ilustram um estetoscópio cujo modelo

é o de um tubo de duas extremidades abertas

preenchidas com ar. Com base nesses dados e em

aspectos relacionados à propagação do som, assinale a

opção correta.

a) Na ressonância, não há transferência de energia

para o tubo.

b) Para que ocorra ressonância sonora, é

imprescindível que ambas as extremidades do

tubo estejam fechadas.

c) Caso se preencha completamente o tubo com um

fluido de maior densidade que o ar, será menor a

velocidade de propagação da onda sonora nesse

tubo.

d) Na situação de ressonância sonora, o comprimento

de onda do primeiro harmônico é igual ao dobro

do valor do comprimento L do tubo.

e) A frequência fundamental é inversamente

proporcional à velocidade de propagação da onda

no tubo.

Questão 12 - (Anhembi Morumbi SP/2014)

A figura mostra uma onda estacionária em uma corda

de comprimento 80 cm no instante t0 = 0 s, no qual o

deslocamento vertical dos ventres é máximo.

Se a velocidade de propagação das ondas nessa corda

é de 2,0 m/s, a forma da corda no instante t1 = 0,05 s

será

a)

b)

c)

d)

e)

Questão 13 - (UNESP/2014)

Page 148: curso de específica de física

148

Duas ondas mecânicas transversais e idênticas, I e II,

propagam-se em sentidos opostos por uma corda

elástica tracionada. A figura 1 representa as

deformações que a onda I, que se propaga para direita,

provocaria em um trecho da corda nos instantes t = 0

e 4

Tt , em que T é o período de oscilação das duas

ondas. A figura 2 representa as deformações que a onda

II, que se propaga para esquerda, provocaria no mesmo

trecho da corda, nos mesmos instantes relacionados na

figura 1. Ao se cruzarem, essas ondas produzem uma

figura de interferência e, devido a esse fenômeno,

estabelece-se uma onda estacionária na corda. A figura

3 representa a configuração da corda resultante da

interferência dessas duas ondas, nos mesmos instantes

t = 0 e 4

Tt .

A figura que melhor representa a configuração da corda

nesse mesmo trecho devido à formação da onda

estacionária, no instante 4

T3 , está representada na

alternativa

a)

b)

c)

d)

e)

Questão 14 - (PUC RJ/2013)

Uma corda é fixa em uma das extremidades, enquanto

a outra é vibrada por um menino. Depois de algum

tempo vibrando a corda, o menino observa um padrão

de ondas estacionário. Ele verifica que a distância

entre dois nós consecutivos deste padrão é de 0,50 m.

Determine em metros o comprimento de onda da

vibração imposta à corda.

a) 0,25

b) 0,50

c) 1,00

d) 1,25

e) 1,50

Questão 15 - (UEG GO/2013)

Ondas estacionárias são produzidas em um tubo

sonoro, de comprimento L, aberto nas duas

extremidades. Para o primeiro harmônico, o

comprimento de onda relaciona-se com o

comprimento do tubo pela equação 1=2L. A equação

que relaciona essas duas grandezas, comprimento de

onda e o comprimento do tubo, para o oitavo

harmônico, é:

a) 8L

b) L/4

c) L/2

d) 4L

Questão 16 - (UEPG PR/2011)

A luz natural apresenta várias propriedades, entre elas

a polarização. Sobre polarização da luz, assinale o que

for correto.

01. Se a luz não polarizada tornar-se polarizada, a

intensidade luminosa é reduzida pela metade.

02. Todo ponto de uma luz polarizada corresponde a

um mesmo plano de vibração, em qualquer

instante.

04. A luz natural pode ser polarizada por reflexão ou

por refração.

08. Só é possível obter-se a luz polarizada por meio de

uma lâmpada especial.

16. O olho humano não consegue distinguir se um

feixe luminoso é ou não polarizado.

Questão 17 - (UEM PR/2011)

Considere um experimento de interferência e difração

de luz visível em uma fenda dupla (experimento de

Young), no qual as frentes de onda plana satisfazem o

Princípio de Huygens, analise as alternativas e assinale

o que for correto.

01. Tal experimento pode comprovar a natureza

corpuscular da luz.

02. As chamadas franjas de interferência podem ser

observadas em tal experimento.

Page 149: curso de específica de física

149

04. Quando ocorre interferência destrutiva entre as

ondas emergentes das fendas, as frequências de

oscilação e os comprimentos de onda dessas

ondas se cancelam.

08. A diferença de caminho óptico entre as ondas

emergentes das fendas é que determina se pode

haver ou não interferência construtiva entre tais

ondas.

16. A experiência de Young permite determinar o

comprimento de onda da onda eletromagnética

utilizada no experimento.

Questão 18 - (UEPG PR/2010)

Quando uma pedra é jogada na água é possível observar

que a perturbação que ela produz se propaga em toda a

superfície livre da água por meio de ondas. O movimento

ondulatório apresenta fenômenos, tais como reflexão,

refração, difração, polarização, entre outros.

Sobre esses fenômenos ondulatórios, assinale o que for

correto.

01. Uma onda quando muda de velocidade ao passar de

um meio para outro meio pode sofrer reflexão e

refração.

02. Ondas sonoras não sofrem o fenômeno de

polarização.

04. A difração, através de uma fenda, somente é

observada quando a fenda é menor ou da mesma

ordem de grandeza do comprimento de onda.

08. Numa onda polarizada todas as partículas do meio

vibram numa única direção perpendicular à direção

de propagação da onda.

16. O fenômeno de difração ocorre quando uma onda

contorna um obstáculo que, parcialmente, a

interrompe.

Questão 19 - (UEPG PR/2010)

Sobre o fenômeno de difração, assinale o que for

correto.

01. A difração só é observável quando as dimensões

do obstáculo ou da fenda são da mesma ordem de

grandeza do comprimento de onda da onda

incidente.

02. A ocorrência da difração da luz visível está restrita

a fendas estreitas.

04. As ondas mecânicas não sofrem difração.

08. O grau de difração de uma onda independe da sua

frequência.

16. O fenômeno da difração ocorre somente para

alguns tipos de ondas.

Questão 20 - (UEPG PR/2010)

Uma onda mecânica capaz de provocar a sensação

auditiva é uma onda sonora. Sobre esse assunto,

assinale o que for correto.

01. Se uma onda sonora se propaga em uma

determinada direção, o ar não se movimenta,

apenas a sua pressão aumenta e diminui

sucessivamente.

02. A velocidade de uma onda sonora em um meio

sólido é maior do que em um meio líquido, que por

sua vez, é maior do que em um meio gasoso.

04. Com as ondas sonoras ocorrem os fenômenos de

reflexão, refração, difração e interferência.

08. Alguns aparelhos usados para a realização de

exames médicos emitem uma onda sonora, de alta

frequência, imperceptível aos nossos ouvidos.

GABARITO:

1) Gab: C 2) Gab: E 3) Gab: 25

4) Gab: A 5) Gab: B 6) Gab: B

7) Gab: 12 8) Gab: E 9) Gab: A

10) Gab: E 11) Gab: D 12) Gab: E

13) Gab: D 14) Gab: C 15) Gab: B

16) Gab: 23 17) Gab: 26 18) Gab: 31

19) Gab: 03 20) Gab: 15 21) Gab: C

22) Gab: D

23) Gab:

a) 9,5510–2 m/ s

b) 445 Hz

24) Gab: 13 25) Gab: C 26) Gab: C

27) Gab: C 28) Gab: A 29) Gab: C

30) Gab: A 31) Gab: C 32) Gab: 04

33) Gab: B 34) Gab: B 35) Gab:

a) O item trata-se de uma onda mecânica que se propaga no ar dentro de um tubo com uma das extremidades fechada. O resultado, segundo a figura, é devido à formação de ondas

estacionárias, onde a matéria (talco) acumula-se na região dos nós, uma vez que não há deslocamento de matéria nesses pontos.

b) Através da equação da velocidade de propagação da onda:

fv (1)

onde v é a velocidade do som no ar, é

comprimento de onda e f é a frequência do som emitido, pode-se obter a frequência do som emitido pelo alto-falante. Ainda, segundo a figura, o comprimento de onda (dobro da distância entre dois nós consecutivos) é 2 x 0,1 m = 0,2 m. Assim, substituindo os valores da velocidade do som e do comprimento de onda na Equação (1), determina-se o valor da frequência do som emitido:

Hz 17002,0

340f

Portanto, a frequência do som emitido pelo alto-falante é 1700 Hz

36) Gab: C 37) Gab: E 38) Gab:

a) ID = 1 W/m2 b) De acordo com o enunciado, como o tempo de

exposição caiu pela metade, de 8h para 4h, o nível de intensidade sonora deverá sofrer um acréscimo de 5 dB, apresentando o valor máximo de 90 dB.

c) I = 10–2 W/m2 P = 7,5 10–7 W

39) Gab: 03 40) Gab: 14

Page 150: curso de específica de física

150

LISTA 27 – MAGNETISMO E FONTES DE CAMPO MAGNÉTICO O ímã é um minério que tem a propriedade de atrair pedaços de ferro. A esse minério foi dado o nome de magnetita, por ser encontrado numa região chamada Magnésia, localizada na atual Turquia. Os ímãs possuem dois polos que são denominados de polo norte e polo sul, se tivermos dois imãs próximos, observamos que polos de mesmo nome se repelem e que polos de nomes diferentes se atraem, quer dizer: polo norte repele polo norte, polo sul repele polo sul, e polo norte e polo sul se atraem. Veja abaixo:

Representação da interação entre polos magnéticos

Um determinado ímã cria no espaço em sua volta um campo magnético que podemos representar pelas linhas de indução magnética, essas linhas de indução atravessam de um polo a outro do ímã. É por esse motivo, inclusive, que mesmo que um ímã seja partido ao meio, separando os polos: norte e sul, ele sempre se reorganizará de maneira a formar dois polos. Em outras palavras, podemos afirmar que não existe monopolo magnético. Se pegarmos pequenas bússolas e colocarmos sobre as linhas de indução magnética, a agulha da bússola sempre apontará na mesma direção do vetor indução magnética B, e o norte da agulha no mesmo sentido de B, ou seja, B estará apontando para o polo sul do ímã. Assim, podemos dizer que em cada ponto em torno do ímã, o vetor B se afasta do polo norte e se aproxima do polo sul. Na região dos polos vemos que as linhas de indução magnética estão mais próximas umas das outras, sendo assim, consideramos que próximos aos polos o campo magnético é mais intenso. É possível visualizar as linhas de indução magnética, espalhando limalhas de ferro em torno de um ímã. Os traços formados pela limalha representam as linhas de indução magnética. Observe a figura abaixo:

Representação das linhas de indução através de limalhas de

ferro Campo magnético da Terra Nosso planeta se comporta como um gigantesco ímã. Sendo assim, ele cria à sua volta um campo magnético. Quando observamos uma bússola, vemos que o polo norte da agulha da bússola se orienta na direção do norte geográfico. Como polos de nomes diferentes se atraem, concluímos que o polo norte da agulha da bússola está sendo atraída pelo polo sul da Terra. Então o polo sul magnético coincide com o polo norte geográfico e o polo norte magnético coincide com o polo sul geográfico. Os polos magnéticos da Terra têm uma pequena inclinação em relação ao seu eixo de rotação, essa inclinação é de aproximadamente 11°. Cientistas acreditam que essa propriedade magnética deve-se ao movimento circular de correntes elétricas no núcleo de ferro fundido do planeta. Pesquisas mostram que as posições dos polos magnéticos mudam com o passar dos anos, chegando a inverter sua polaridade, os polos norte e sul trocam sua posição.

Campo magnético de um fio retilíneo

Quando um fio retilíneo é percorrido com uma corrente

elétrica i, ele gera ao seu redor um campo magnético, cujas

as linhas do campo são circunferências concêntricas

pertencentes ao plano perpendicular ao fio e com centro

comum em um ponto dele.

Para sabermos qual o sentido do campo magnético deste fio

utilizamos a regra da mão direita. Coloca-se polegar direito

no mesmo sentido que a corrente, assim a direção que os

demais dedos curvados nos mostrarão será o sentido do

campo, como mostra a figura abaixo:

Page 151: curso de específica de física

151

A lei de Ampére nos permitiu determinar o módulo do

campo magnético. Ela nos diz que “o vetor campo

magnético é tangente as linhas do campo magnética”.

Assim a tangente as linhas do campo magnética será a

direção dele, e a intensidade do campo será dado pela

equação:

Onde R é a distância do fio até um ponto da linha do

campo, e m0 é a constante de permeabilidade magnética do

vácuo que vale m0 = 4π . 10-7 T.m/A.

EXERCÍCIOS

Questão 01 - (ESCS DF/2014)

As bactérias magnéticas são seres uni ou

pluricelulares que usam o campo magnético da Terra

para se orientar, porque elas produzem e mantêm,

dentro de si, cristais nanométricos chamados de

magnetossomos, que funcionam como ímãs comuns do

mineral magnetita. Essas bactérias não crescem em

ambientes com muito oxigênio e consomem carbono e

nitrogênio; preferem as águas calmas encontradas em

baías, lagos e lagoas. Esses microrganismos têm

despertado o interesse dos pesquisadores,

principalmente da área de biomedicina. No genoma

dessas bactérias, existe a informação para a produção

de proteínas que interferem na síntese dos

magnetossomos.

Internet: <www.revista.fapesp.br> (com adaptações).

Tendo como referência o texto acima, assinale a opção

correta.

a) Em geral, as bactérias são autótrofas, ou seja,

incapazes de sintetizar seus próprios alimentos.

b) Infere-se do texto que as bactérias citadas

crescem em ambientes de pouca profundidade.

c) O norte magnético dessas bactérias aponta para o

norte geográfico da Terra, dado que elas se

comportam como ímãs.

d) A interação do momento magnético celular dessas

bactérias com o campo magnético terrestre não

interfere no movimento celular com relação às

linhas de campo, como ocorre com as bússolas.

Questão 02 - (FAMECA SP/2014)

Dispõe-se de cinco discos magnetizados que possuem

um furo central e que têm os polos magnéticos

distribuídos conforme indicado na figura 1.

Os discos são encaixados sem travamento em um longo

bastão plástico disposto verticalmente como

representado na figura 2. Nessa situação, os cinco

discos permanecem em equilíbrio, sendo que os

numerados por 2 e 3 são mantidos unidos graças à

utilização de cola, já que, por suas naturezas, ficariam

afastados.

Desconsiderando o atrito entre os discos e o bastão, e

sabendo que na situação de equilíbrio a parte superior

do ímã 4 abriga um polo norte magnético, é correto

afirmar que os polos magnéticos das faces voltadas

para baixo dos ímãs 1, 2, 3, 4 e 5 são, respectivamente,

a) S, N, S, N e S.

b) N, N, S, N e S.

c) S, S, N, S e S.

d) S, N, S, S e N.

e) N, S, N, S e N.

Questão 03 - (UEM PR/2013)

Em relação ao campo magnético terrestre, assinale o

que for correto.

01. A bússola é um instrumento que utiliza o campo

magnético terrestre para indicar a direção

aproximada dos polos terrestres.

02. O norte geográfico coincide com o norte magnético

terrestre.

04. A declinação magnética é definida como o ângulo

formado entre os meridianos magnéticos e

geográficos.

08. A declinação magnética no Estado do Paraná é

maior do que 30º.

16. O campo magnético terrestre é uma consequência

somente do movimento de rotação da Terra.

Questão 04 - (UEG GO/2013)

Page 152: curso de específica de física

152

A Terra assemelha-se a um grande imã, apresentando

um polo norte magnético e um polo sul magnético,

exercendo atração de ligas de ferro natural, como a

Magnetita. Sobre o campo magnético terrestre, verifica-

se que

a) a Terra tem propriedades magnéticas mais

acentuadas nas regiões próximas às

extremidades.

b) a posição dos polos magnéticos é fixa, se situando

diametralmente oposta no planeta.

c) o equador é desprovido de eletromagnetismo por

ser equidistante dos dois polos.

d) o polo norte magnético se localiza sobre polo norte

geográfico do planeta.

Questão 05 - (UEPA/2013)

Em um experimento de eletromagnetismo, um

professor revestiu a parede interna de um copo de

plástico de 50 ml com papel alumínio. Um segundo

copo, com volume igual a 250 ml, foi totalmente

preenchido com água, depois o copo menor foi posto a

flutuar sobre a superfície dessa água (Figura 1). Um

pequeno ímã foi então preso a um fio e posicionado no

vão livre situado na parte interna do copo de 50 ml, mas

sem tocá-lo (Figura 2). Observou-se que, quando o fio

era torcido e em seguida solto, tanto o ímã quanto o

copo menor giravam em torno do eixo do fio. Para

garantir que o fenômeno não foi causado pelo vento o

professor repetiu o experimento, substituindo o ímã por

um objeto de plástico com as mesmas dimensões do

ímã, observando, nesse caso, que o copo menor não se

movia.

Fonte: Física na Escola, v.11, n.2, 2010.

Para essa situação são feitas as seguintes afirmações:

I. Quando o ímã gira no sentido horário, o copo

menor gira no sentido horário.

II. O papel alumínio é atraído pelo polo norte e

repelido pelo polo sul do ímã.

III. O campo magnético do ímã atrai o campo elétrico

induzido do papel alumínio.

IV. Uma parte da energia do sistema é dissipada por

efeito Joule.

A alternativa que contém todas as afirmativas corretas

é:

a) I e III

b) I e IV

c) II e III

d) II e IV

e) III e IV

Questão 06 - (UFU MG/2012)

Domínios magnéticos são aglomerados de bilhões de

átomos organizados de forma alinhada. Num pedaço de

ferro, por exemplo, os domínios não estão alinhados

entre si; no entanto, alinhando-os, o material adquire

características magnéticas, podendo tornar-se um ímã

permanente.

Para conseguir tal alinhamento nos domínios

magnéticos, deve-se

a) colocar pedaços de ferro sujeitos a um campo

elétrico uniforme de alta intensidade, única

direção e sentido.

b) colocar pedaços de ferro sujeitos a um campo

magnético intenso ou esfregá-los em um ímã

permanente.

c) ligar os pedaços de ferro a uma corrente contínua,

de modo que a ddp estabelecida alinhe os

domínios.

d) atritar dois pedaços de ferro, entre si, sempre na

mesma direção e com a mesma velocidade.

Questão 07 - (UEPG PR/2010)

O eletromagnetismo estuda tanto as interações elétricas

como as magnéticas. Sobre o eletromagnetismo,

assinale o que for correto.

01. Se um imã for partido em duas partes, o pólo sul se

conserva enquanto o pólo norte desaparece.

02. A Terra pode ser considerada como um grande imã,

cujos pólos norte e sul magnéticos se localizam

aproximadamente nos pólos sul e norte geográficos,

respectivamente.

04. A atração que ocorre quando aproximamos certos

minérios de um pedaço de ferro é uma manifestação

de natureza elétrica.

08. Um fio condutor percorrido por uma corrente

elétrica produz deflexões em uma agulha imantada.

Questão 08 - (UEM PR/2008)

O diagrama abaixo representa as linhas de um campo

magnético uniforme.

Assinale a alternativa que melhor representa a posição

da agulha de uma bússola colocada em um ponto P, no

mesmo plano do campo magnético.

Page 153: curso de específica de física

153

Questão 09 - (UFF RJ/2007)

O disco rígido de um computador é um meio magnético

utilizado para armazenar informação em forma digital.

Sua superfície é dividida em trechos retangulares, muito

pequenos, que funcionam como ímãs microscópicos e

podem ser orientados em dois sentidos opostos

, respectivamente.

Um modelo simplificado do processo de leitura da

informação gravada no disco rígido envolve um conjunto

de bússolas I, II e III representado na figura. Se o pólo

norte da bússola aponta para cima, sua orientação é

representada pelo dígito 1, se aponta para baixo, é

representada pelo dígito 0.

Assinale a opção que representa a orientação das

bússolas na situação da figura.

a) 1 0 1

b) 0 1 0

c) 1 0 0

d) 0 1 1

e) 0 0 1

Questão 10 - (Fac. Santa Marcelina SP/2014)

Pesquisadores verificaram a alta incidência de leucemia

em crianças que moram perto de cabos de alta-tensão

e também de linfomas e outros tipos de neoplasias

malignas em adultos que trabalham com

transformadores, fios de alta-tensão e em estações de

radar.

Nesses casos, é correto concluir que essas doenças são

provocadas devido à intensa exposição de pessoas a

campos de origem

a) eletromagnética.

b) mecânica.

c) termodinâmica.

d) inercial.

e) gravitacional.

Questão 11 - (UDESC/2014)

Analise as proposições relacionadas às linhas de campo

elétrico e às de campo magnético.

I. As linhas de força do campo elétrico se estendem

apontando para fora de uma carga pontual

positiva e para dentro de uma carga pontual

negativa.

II. As linhas de campo magnético não nascem nem

morrem nos ímãs, apenas atravessam-nos, ao

contrário do que ocorre com os corpos condutores

eletrizados que originam os campos elétricos.

III. A concentração das linhas de força do campo

elétrico ou das linhas de campo magnético indica,

qualitativamente, onde a intensidade do

respectivo campo é maior.

Assinale a alternativa correta.

a) Somente as afirmativas I e III são verdadeiras.

b) Somente a afirmativa II é verdadeira.

c) Somente as afirmativas II e III são verdadeiras.

d) Somente as afirmativas I e II são verdadeiras.

e) Todas as afirmativas são verdadeiras.

Questão 12 - (UNESP/2014)

A figura é o esquema simplificado de um disjuntor

termomagnético utilizado para a proteção de

instalações elétricas residenciais. O circuito é formado

por um resistor de baixa resistência R; uma lâmina

bimetálica L, composta pelos metais X e Y; um

eletroímã E; e um par de contatos C. Esse par de

contatos tende a abrir pela ação da mola M2, mas o

braço atuador A impede, com ajuda da mola M1. O

eletroímã E é dimensionado para atrair a extremidade

do atuador A somente em caso de corrente muito alta

(curto circuito) e, nessa situação, A gira no sentido

indicado, liberando a abertura do par de contatos C pela

ação de M2.

De forma similar, R e L são dimensionados para que

esta última não toque a extremidade de A quando o

circuito é percorrido por uma corrente até o valor

nominal do disjuntor. Acima desta, o aquecimento leva

o bimetal a tocar o atuador A, interrompendo o circuito

de forma idêntica à do eletroímã.

(www.mspc.eng.br. Adaptado.)

Na condição de uma corrente elevada percorrer o

disjuntor no sentido indicado na figura, sendo X e Y os

coeficientes de dilatação linear dos metais X e Y, para

que o contato C seja desfeito, deve valer a relação

________ e, nesse caso, o vetor que representa o

campo magnético criado ao longo do eixo do eletroímã

apontará para a ________.

Os termos que preenchem as lacunas estão indicados

correta e respectivamente na alternativa

a) X > Y … esquerda.

b) X < Y … esquerda.

c) X > Y … direita.

d) X = Y … direita.

e) X < Y … direita.

Page 154: curso de específica de física

154

Questão 13 - (Univag MT/2014)

O circuito da figura é constituído por uma bateria ideal

de 12 V, resistores ôhmicos de resistência elétrica 6

cada e fios de ligação de resistência elétrica desprezível.

A intensidade de corrente elétrica que percorre o fio

longo AB gera no ponto P um campo magnético B de

direção perpendicular ao plano da folha.

Considerando o circuito imerso em um meio cuja

permeabilidade magnética é = 6 10–7 T m/A e as

informações contidas na figura, é correto afirmar que a

intensidade do campo magnético B, em tesla, e seu

sentido são, respectivamente,

a) 3 10–6 e entrando no plano da folha.

b) 2 10–7 e entrando no plano da folha.

c) 3 10–4 e saindo do plano da folha.

d) 3 10–7 e entrando no plano da folha.

e) 3 10–6 e saindo do plano da folha.

Questão 14 - (UEFS BA/2014)

Disponível em:

<http://osfundamentosdafisica.blogspot.com.br/2013/10/c

ursos-

do-blog-eletricidade.html>. Acesso em: 17 out. 2013.

Um estudante, com o intuito de repetir a experiência de

Orested, utilizou um circuito elétrico constituído por

uma bateria de força eletromotriz, , igual a 12,0V e

resistência interna, r, igual a 5,0 e um resistor de

resistência elétrica também de 5,0. A agulha de uma

bússola foi colocada a 2,0cm abaixo do fio e sofreu

deflexão quando o gerador lançou potencia máxima no

circuito, conforme a figura.

Sabendo-se que a permeabilidade magnética do meio,

, é igual a 410–7T.m/A, e correto afirmar que a

intensidade do campo magnético que causou deflexão

na agulha da bússola é igual, em T, a

a) 1,210-5

b) 1,510-5

c) 7,510-5

d) 3,010-7

e) 8,610-7

Questão 15 - (PUC SP/2013)

Na figura abaixo temos a representação de dois

condutores retos, extensos e paralelos. A intensidade

da corrente elétrica em cada condutor é de A220 nos

sentidos indicados. O módulo do vetor indução

magnética resultante no ponto P, sua direção e sentido

estão mais bem representados em

Adote 0 = 4 10–7 Tm/A

a) T1024 4 e

b) T1028 4 e

c) T108 4 e

d) T104 4 e

e) T1024 7 e

Questão 16 - (UEL PR/2013)

Com o objetivo de estudar a estrutura da matéria, foi

projetado e construído no CERN (Centro Europeu de

Pesquisas Nucleares) um grande acelerador (LHC) para

fazer colidir dois feixes de prótons, ou íons pesados.

Nele, através de um conjunto de ímãs, os feixes de

prótons são mantidos em órbita circular, com

velocidades muito próximas à velocidade da luz c no

vácuo. Os feixes percorrem longos tubos, que juntos

formam um anel de 27 km de perímetro, onde é feito

vácuo. Um desses feixes contém N = 2,01014 prótons

distribuídos uniformemente ao longo dos tubos. Os

prótons são mantidos nas órbitas circulares por horas,

estabelecendo, dessa forma, uma corrente elétrica no

anel.

a) Calcule a corrente elétrica i, considerando o tubo

uma espira circular de corrente.

b) Calcule a intensidade do campo magnético gerado

por essa corrente no centro do eixo de simetria do

anel do acelerador LHC (adote = 3).

Apresente os cálculos realizados na resolução deste

item.

Questão 17 - (UEM PR/2013)

As imagens obtidas por ressonância magnética, como

aquelas oriundas de exames de ressonância nuclear

magnética, têm sido cada vez mais empregadas na

análise de tecidos humanos. A respeito dos conceitos

relacionados a ressonância, campo magnético e

histologia, assinale o que for correto.

01. O fenômeno da ressonância é observado quando

afrequência da fonte externa coincide com a

frequência natural de oscilação do sistema que

entra em ressonância.

Page 155: curso de específica de física

155

02. Cargas elétricas estáticas geram campos

magnéticos estáticos, que interagem com campos

magnéticos uniformes.

04. O tecido muscular é constituído por células

alongadas que são altamente contráteis devido ao

encurtamento de filamentos proteicos

citoplasmáticos dispostos ao longo de seu

comprimento.

08. No Sistema Internacional de Unidades, a unidade

de intensidade do vetor campo magnético, B ,

denomina-se tesla.

16. O tecido nervoso forma os diversos componentes

do sistema nervoso central, constituído pelo

encéfalo e pela medula espinhal, e do sistema

nervoso periférico, constituído pelos nervos e

gânglios nervosos.

Questão 18 - (UEM PR/2013)

Um detector de metais é constituído de uma bobina

chata e circular composta por 50 enrolamentos de 20

cm de raio, que são percorridos por uma corrente

elétrica de 100 x 10–3 A, quando esse detector está

ligado e em pleno funcionamento. Com base nessas

informações e considerando que o detector é utilizado

no vácuo e que a permeabilidade magnética do vácuo é

de 4 x 10–7 T.m/A, analise as alternativas abaixo e

assinale o que for correto.

01. O vetor indução magnética resultante no centro da

bobina do detector de metais está direcionado

perpendicularmente ao plano da bobina e sua

intensidade pode ser determinada a partir da lei

de Biot-Savart.

02. A intensidade do vetor indução magnética, gerado

no centro da bobina por somente um de seus

enrolamentos, é de x 10–7 T.

04. As linhas de campo do campo magnético gerado

pela bobina do detector de metais estão contidas

no plano da bobina, e sua densidade diminui com

o aumento da quantidade de espiras na bobina.

08. A intensidade do vetor indução magnética,

determinada no centro da bobina do detector de

metais, é dada na forma R2

iNB 0 , sendo N o

número de enrolamentos da bobina, 0 a

permeabilidade magnética do vácuo, i a corrente

que flui nos enrolamentos da bobina e R o raio

desses enrolamentos.

16. Ao inverter-se o sentido da corrente elétrica que

flui na bobina, a direção e o sentido da força

magnética e do vetor indução magnética no centro

da bobina são invertidos.

Questão 19 - (UDESC/2012)

Dois fios retilíneos e de tamanho infinito, que conduzem

correntes elétricas i1 e i2 em sentidos opostos, são

dispostos paralelamente um ao outro, como mostra a

Figura 5. A intensidade de i1 e a metade da intensidade

de i2 e a distancia entre os dois fios ao longo da linha

ox e d.

Considere as seguintes proposições sobre os campos

magnéticos produzidos pelas correntes i1 e i2 nos

pontos localizados ao longo da linha ox:

I. À esquerda do fio 1 não existe ponto no qual o

campo magnético resultante seja nulo.

II. Nos pontos localizados entre o fio 1 e o fio 2, os

campos magnéticos produzidos por ambas as

correntes tem o mesmo sentido.

III. À direita do fio 2 existe um ponto no qual o campo

magnético resultante é nulo.

IV. O campo magnético resultante é nulo no ponto que

fica à distancia 3d/4 à esquerda do fio 2.

Assinale a alternativa correta.

a) Somente as afirmativas II e III são verdadeiras.

b) Somente as afirmativas I e II são verdadeiras.

c) Somente a afirmativa III e verdadeira.

d) Somente a afirmativa II e verdadeira.

e) Somente a afirmativa IV e verdadeira.

Questão 20 - (UFTM/2012)

Um suporte feito de plástico na forma de U pode girar

livremente paralelo ao plano xy, sobre o eixo E, vertical

e paralelo ao eixo z. Por meio de uma haste fina,

paralela ao plano xy, presa ao suporte, um ímã, em

forma de barra, gira livremente ao redor do seu eixo de

simetria A, coaxial à haste. Paralelamente ao plano xz

e na coordenada y = 0, uma espira circular de raio r,

cujo centro está contido no eixo A de simetria do ímã,

é percorrida no sentido horário pela corrente elétrica i,

conforme indica a figura.

Com a passagem da corrente elétrica i na espira, a

direção do eixo M, que passa pelos polos N e S do ímã,

se tornará paralela ao eixo

a) x, com o norte do ímã apontando para o sentido

positivo desse eixo.

b) x, com o norte do ímã apontando para o sentido

negativo desse eixo.

c) y, com o norte do ímã apontando para o sentido

positivo desse eixo.

d) y, com o norte do ímã apontando para o sentido

negativo desse eixo.

e) z, com o norte do ímã apontando para o sentido

positivo desse eixo.

Page 156: curso de específica de física

156

GABARITO:

1) Gab: C 2) Gab: E 3) Gab: 05

4) Gab: A 5) Gab: B 6) Gab: B

7) Gab: 10 8) Gab: B 9) Gab: A

10) Gab: A 11) Gab: E 12) Gab: C

13) Gab: C 14) Gab: A 15) Gab: C

16) Gab:

a) i = 0,36 A

b) A intensidade do campo magnético B gerado no

centro de um anel condutor de raio r é

Observação: Este campo é muito pequeno quando

comparado aos campos que aparecem em

aparelhos eletrônicos que utilizamos no dia a dia.

Por exemplo, a intensidade do campo magnético

da terra varia entre 2,510–5 T e 6, 510–5 T e o de

um refrigerador é de 10–2 T.

17) Gab: 29 18) Gab: 11 19) Gab: D

20) Gab: D 21) Gab: 2 x 10–5 T.

22) Gab: B

23) Gab:

a) B = 4T

b) J102,4 3m

F

24) Gab: B

25) Gab: 13

26) Gab: 10

27) Gab: A

28) Gab: C

29) Gab: B

30) Gab:

a) h4

i 3B 0

i

b) 03

hB4i

31) Gab: 11 32) Gab: C 33) Gab: A 34) Gab: D 35) Gab: B 36) Gab: C 37) Gab: D 38) Gab: A

39) Gab: A 40) Gab: A

Page 157: curso de específica de física

157

LISTA 28 – FORÇA MAGNÉTICA E INDUÇÃO MAGNÉTICA

Força magnética sobre cargas

Um campo magnético não atua sobre cargas elétricas em

repouso, mas se pegarmos esta carga e lançarmos com

uma velocidade v em direção a uma área onde há um

campo magnético B pode aparecer uma força F atuando

sobre esta carga, denominada força magnética. As

características desta força magnética foram determinadas

pelo físico Hendrick Antoon Lorentz (1853-1920).

A intensidade da força magnética pode ser obtida por:

F = q . B . v . sen(a)

Onde a é o ângulo entre os vetores v e B. No SI a unidade

de intensidade do campo magnético é o tesla representado

pelo símbolo T.

A força magnética que age sobre a carga móvel é sempre

perpendicular ao plano formado pelos vetores v e B.

Observando a equação acima veremos que quando a=0 ou

a=180º a força magnética será nula, portanto quando o

lançamento for paralelo ao campo não teremos a força

magnética atuando sobre esta carga, assim descrevendo

um movimento retilíneo uniforme.

O sentido da força é dada pela regra da mão esquerda,

como mostra a figura abaixo:

http://educacao.uol.com.br/fisica/forca-magnetica-carga-

em-movimento.jhtm

ou seja, o dedo indicador no sentido do campo e o dedo

médio no sentido da velocidade, dando no dedo polegar o

sentido da força magnética.

Essa regra é válida para cargas positivas, se a carga for

elétricamente negativa basta utilizar a mão direita.

Força magnética sobre um fio condutor Sempre que uma carga é posta sobre influência de um campo magnético, esta sofre uma interação que pode alterar seu movimento. Se o campo magnético em questão for uniforme, vimos que haverá uma força agindo sobre a

carga com intensidade , onde é o ângulo formado no plano entre os vetores velocidade e

campo magnético. A direção e sentido do vetor serão

dadas pela regra da mão direita espalmada. Se imaginarmos um fio condutor percorrido por corrente, haverá elétrons livres se movimentando por sua secção

transversal com uma velocidade . No entanto, o sentido

adotado para o vetor velocidade, neste caso, é o sentido

real da corrente ( tem o mesmo sentido da corrente).

Para facilitar a compreensão pode-se imaginar que os elétrons livres são cargas positivas. Como todos os elétrons livres têm carga (que pela suposição adotada se comporta como se esta fosse positiva), quando o fio condutor é exposto a um campo magnético uniforme, cada elétron sofrerá ação de uma força magnética.

Mas se considerarmos um pequeno pedaço do fio ao invés de apenas um elétron, podemos dizer que a interação

continuará sendo regida por , onde Q

é a carga total no segmento do fio, mas como temos um comprimento percorrido por cada elétron em um determinado intervalo de tempo, então podemos escrever a velocidade como:

Ao substituirmos este valor em teremos a força

magnética no segmento, expressa pela notação :

Mas sabemos que indica a intensidade de corrente no fio, então:

Sendo esta expressão chamada de Lei Elementar de Laplace.

A direção e o sentido do vetor são perpendicular ao

plano determinado pelos vetores e , e pode ser

determinada pela regra da mão direita espalmada, apontando-se o polegar no sentido da corrente e os demais

dedos no sentido do vetor .

Indução eletromagnética

O professor de Física dinamarquês, Hans Christian Oesterd, descobriu com os seus estudos, em 1820, que cargas elétricas em movimento davam origem a um campo magnético. A comprovação dessa teoria levou vários cientistas a acreditar que o contrário também deveria ocorrer, ou seja: a variação de campo magnético deveria gerar uma corrente elétrica.

Page 158: curso de específica de física

158

A comprovação dessa proposição foi feita em 1931 pelo professor de física dinamarquês Michael Faraday. Esse fenômeno passou então a ser conhecido como indução eletromagnética e foi descrito teoricamente e matematicamente pela Lei de Faraday. Entre os experimentos utilizados para demonstrar essas teorias, Faraday realizou o descrito a seguir:

Inicialmente se conecta uma espira condutora,

desconectada de uma fonte de tensão, a um amperímetro (aparelho usado para medir corrente elétrica). Em seguida, aproxima-se dessa espira um imã em forma de barra. O amperímetro, nesse momento, indica a passagem de uma corrente elétrica pela espira. Quando o imã é afastado da espira, a corrente aparece novamente, porém, em sentido contrário. Observe na figura:

Na experiência de Faraday, as linhas de campo magnético

do imã geram uma corrente induzida na espira A partir desse experimento, é possível chegar às seguintes conclusões:

1. A corrente elétrica somente é observada no caso de haver movimento entre a espira e o imã. Se o imã ficar em repouso, não surge corrente;

2. Quanto mais rápido o movimento do imã, maior a corrente; 3. O sentido da corrente elétrica dependerá do polo magnético

e da direção do imã que está interagindo com a espira. Se o polo norte do imã é aproximado da espira, a corrente tem sentido horário; mas quando o polo norte afasta-se da espira, a corrente tem sentido anti-horário. Se o polo sul do imã é aproximado da espira, a corrente tem sentido anti-horário; caso o polo sul esteja se afastando do imã, a corrente tem sentido horário.

Denominamos a corrente gerada na espira de corrente induzida. O trabalho executado por unidade de carga para produzir essa corrente é chamado de força eletromotriz induzida, e o processo utilizado para produzir a corrente e a força eletromotriz recebe o nome de indução. O desenvolvimento da teoria da indução eletromagnética foi responsável pelo surgimento de instrumentos importantíssimos, como o motor elétrico, que serve como base para o funcionamento de vários aparelhos (liquidificadores, bombeadores de água, ventiladores, motores de geladeira, entre outros).

EXERCÍCIOS

Questão 01 - (FPS PE/2014)

Uma partícula carregada com carga elétrica q = 0.06

Coulomb propaga-se com velocidade constante, cujo

módulo vale v = 100 m/s. A partícula está num local

onde existe um campo magnético uniforme e

perpendicular à direção de propagação da partícula

carregada. O módulo do campo magnético é B = 0.8

Tesla. A força magnética (em módulo) sentida pela

partícula será:

a) 1,8 N

b) 5,8 N

c) 3,8 N

d) 4,8 N

e) 2,8 N

Questão 02 - (UNEB/2014)

Atualmente, a comunidade científica admite que certos

animais detectam e respondem a campos magnéticos,

e que para muitos deles essa capacidade é útil para a

sobrevivência. Um sentido magnético tem sido, de fato,

bem documentado em muitas espécies — desde

migrantes sazonais, como tordos e borboletas-

monarcas, até mestres navegadores, como pombos-

correios e tartarugas marinhas; desde invertebrados,

como lagostas, abelhas e formigas, a mamíferos, como

toupeiras e focas-elefante; e de minúsculas bactérias a

corpulentas baleias.

Nos anos 70, pesquisadores demonstraram que certas

bactérias contêm filamentos de partículas

microscópicas de magnetitas — uma forma fortemente

magnética de óxido de ferro que orienta o organismo

inteiro. (CASTELVECCHI. 2012. p. 29-33).

Tratando-se de fenômenos físicos oriundos de um ímã

natural, a magnetita, como encontrado em certas

bactérias, é correto afirmar:

01. As linhas de indução magnética e as linhas de força

são linhas contínuas e fechadas que formam

círculos concêntricos em torno de magnetita.

02. Os elétrons e prótons em repouso, ao serem

expostos a campos magnéticos, serão submetidos

a uma força magnética.

03. Um campo magnético pode ser usado como

acelerador de partículas porque esse campo

aumenta o módulo da velocidade dessas

partículas.

04. Uma bobina chata percorrida por uma corrente

elétrica forma no seu eixo uma região de campo

magnético com as propriedades idênticas ao de

um ímã natural.

05. As partículas eletrizadas, ao serem lançadas

paralelamente às linhas de indução magnéticas

com velocidade constante, interagem com o

campo magnético, submetidas às forças

magnéticas atrativas ou repulsivas.

TEXTO: 1 - Comuns às questões: 3, 22

Dados:

Aceleração da gravidade: 10 m/s2

sen(37°) = 0,60; cos(37°) = 0,80

sen(60°) = 0,86; cos(60°) = 0,50

Questão 03 - (UFPE/2014)

Uma partícula carregada eletricamente penetra em uma

região do espaço, no vácuo, onde há um campo elétrico

uniforme e constante. O vetor campo elétrico Eé

perpendicular à velocidade inicial da partícula. Despreze

os efeitos da força gravitacional. Analise as afirmações

seguintes.

00. Embora a partícula esteja carregada, não há força

sobre ela, pois não há campo magnético na região

considerada, somente campo elétrico.

11. Embora não haja um campo magnético, há uma

força sobre a partícula porque ela está carregada

e na presença de um campo elétrico.

22. Embora haja uma força sobre a partícula, ela não

a acelera, pois a força é perpendicular à trajetória

da partícula.

Page 159: curso de específica de física

159

33. Embora haja uma força sobre a partícula, não há

trabalho realizado por esta força ao longo da

trajetória.

44. A energia cinética da partícula cresce à medida

que ela se desloca.

Questão 04 - (UDESC/2013)

Um campo elétrico de 1,5kV/m, vertical para cima, e

um campo magnético de 0,4T atuam sobre um elétron

em movimento horizontal para a direita, de modo que

a trajetória do elétron não é alterada. Lembrando que

e representam, respectivamente, campo

magnético saindo desta folha e campo magnético

entrando nesta folha. Assinale a alternativa que

apresenta a velocidade do elétron e a direção do campo

magnético, na sequência:

a) 3750 m/s ;

b) 3,750 m/s ;

c) 37,50 m/s ;

d) 3750 m/s ;

e) 3,750 m/s ;

Questão 05 - (UEM PR/2013)

Sobre os conceitos relativos à formação de campos

magnéticos e à atuação de forças magnéticas, analise

as alternativas abaixo e assinale o que for correto.

01. Um ímã, ou um condutor metálico percorrido por

uma corrente elétrica, origina um campo

magnético na região do espaço que o envolve.

02. O campo magnético no interior de um solenoide é

diretamente proporcional à intensidade da

corrente elétrica que flui no solenoide e ao número

de espiras desse solenoide.

04. A força magnética que surge em um fio condutor

percorrido por uma corrente elétrica é

perpendicular à direção de propagação das cargas

elétricas nesse condutor.

08. Condutores elétricos paralelos percorridos por

correntes elétricas de mesmo sentido se repelem.

16. O vetor campo magnético, em cada ponto do

espaço onde existe um campo magnético, é

tangente às linhas do campo magnético que

passam por esse ponto.

TEXTO: 2 - Comum à questão: 6

Dados:

Aceleração da gravidade: 10 m/s2

Densidade do mercúrio: 13,6 g/cm3

Pressão atmosférica: 1,0x105 N/m2

Constante eletrostática: k0 = 1/40 = 9,0x109

N.m2/C2

Questão 06 - (UFPE/2012)

Uma partícula carregada eletricamente penetra em uma

região do espaço, no vácuo, onde há um campo

magnético uniforme e constante. O vetor campo

magnético B é perpendicular a velocidade inicial da

partícula. Neste contexto, podemos afirmar que:

00. Embora a partícula esteja carregada, não há força

sobre a mesma pois não há campo elétrico na

região considerada, somente campo magnético;

01. Embora não haja um campo elétrico, há uma força

sobre a partícula porque ela está carregada e se

move na presença de um campo magnético;

02. Embora haja uma força sobre a partícula, ela não

a acelera, pois a força é perpendicular a trajetória

da partícula;

03. Embora haja uma força sobre a partícula, não há

trabalho realizado por esta força;

04. A energia mecânica da partícula cresce à medida

que ela se desloca.

Questão 07 - (UNIFOR CE/2012)

Os cientistas que estudam a física das partículas

necessitam estudar o comportamento e as propriedades

do núcleo atômico. Para estudar os componentes dos

prótons no maior acelerador do mundo, recentemente

inaugurado na Suíça “LHC (Large Hadron Collider)”,

prótons de massa ‘m’ e carga positiva ‘q’ são

disparados em colisão frontal, com velocidades

perpendiculares a Campos Magnéticos Uniformes,

sofrendo ação de forças magnéticas. Os Campos

Magnéticos utilizados são uniformes e atuam

perpendicularmente à velocidade destas partículas.

Podemos afirmar que estas forças magnéticas:

a) Mantêm as velocidades escalares dos prótons constantes, mas os colocam em trajetórias circulares.

b) Mantêm as velocidades escalares dos prótons constantes, mas os colocam em trajetórias helicoidais.

c) Aumentam as velocidades escalares dos prótons e mantêm suas trajetórias retilíneas.

d) Diminuem as velocidades escalares dos prótons e mantêm suas trajetórias retilíneas.

e) Não alteram as velocidades escalares dos prótons

nem alteram as suas trajetórias.

Questão 08 - (UEM PR/2012)

Uma partícula, de massa M e carga elétrica Q positiva,

é lançada horizontalmente com velocidade v, da direita

para a esquerda, em uma região do espaço onde

existem vácuo e um campo elétrico uniforme E , que

está direcionado de cima para baixo. Nessa região do

espaço, também existem um campo magnético

uniforme H , orientado perpendicularmente para

dentro do plano da página. De posse dessas

informações, desconsiderando ação da gravidade,

assinale o que for correto.

01. O módulo da força resultante que atua sobre a

partícula é Q(vH + E).

Page 160: curso de específica de física

160

02. Para que a trajetória da partícula se mantenha

retilínea, é necessário que o módulo da sua

velocidade de lançamento seja v0 = H

E.

04. Na situação descrita no enunciado, a trajetória da

partícula é sempre desviada para baixo.

08. As linhas de força do campo magnético formam

superfícies fechadas.

16. A passagem da partícula carregada na região dos

campos elétrico e magnético altera as

características físicas desses campos.

Questão 09 - (FMJ SP/2014)

A figura mostra um dispositivo em que um pedaço de

fio de cobre (fio condutor) tem sua base AB imersa em

um campo magnético produzido por um ímã em forma

de ferradura. Inicialmente, o sistema está desconectado

da pilha.

Quando o fio da direita for conectado ao terminal

negativo da pilha, a corrente elétrica através do fio fará

com que a base AB sofra uma força dirigida para

a) dentro do ímã.

b) o ponto A do fio.

c) o polo sul (S) do ímã.

d) o polo norte (N) do ímã.

e) fora do ímã.

Questão 10 - (PUC GO/2014)

O Homem e sua Hora

(Fragmentos)

“De pios templos, cujos sacerdotes

Repetirão a cada aurora (hrodo,

Hrododáktulos Eos, brododáktulos!)

Que Santo, Santo, Santo é o Ser Humano

– Flecha partindo atrás de flecha eterna –

Agora e sempre, sempre, nunc et semper...”

No texto é citada a palavra aurora, que entra na no-

meação de um fenômeno que ocorre nas regiões

polares devido à força magnética exercida pelo campo

magnético terrestre sobre partículas em movimento.

Nas auroras boreal e austral, temos um lindo espetáculo

formado por uma “cortina de luz” na atmosfera

terrestre. Considere um segmento de fio horizontal

percorrido por uma corrente elétrica de 30 A para a

direita. Num determinado instante, uma partícula com

carga positiva de 8 10–6 C está se movendo a uma

velocidade de 100 m/s para a direita, num ponto 5 cm

acima do segmento de fio. Considerando-se a constante

de permeabilidade magnética = 4 10–7 Tm/A,

analise as afirmativas a seguir:

I. No ponto em que se encontra a carga (5 cm acima

do fio), o campo magnético gerado pelo segmento

de fio tem um valor de 1,2 10–4 T.

II. O campo magnético citado no item anterior (item

I) tem direção horizontal e sentido para a direita.

III. No instante considerado, a força magnética exerci-

da pelo segmento de fio sobre a carga elétrica vale

9,6 10–8 N.

IV. A força magnética citada no item anterior (item

III) tem um sentido de afastamento (vertical para

cima) do segmento de fio.

Em relação às afirmativas analisadas, assinale a única

cujos itens estão todos corretos:

a) I, III

b) I, IV

c) I, III, IV

d) II, III, IV

Questão 11 - (UCS RS/2013)

Os motores elétricos são importantes instrumentos na

vida moderna, pois elevadores, liquidificadores,

aspiradores de pó e vários outros equipamentos de uso

cotidiano dependem deles. O princípio de

funcionamento desses motores é baseado na interação

entre corrente elétrica e campo magnético. Considere

um fio reto de 0,2 m de comprimento, no qual circula

uma corrente elétrica de 2 A. Esse fio está submetido a

um campo magnético de 0,09 T, cujo sentido faz 30º

com o sentido da corrente. Qual é o módulo da força

magnética sobre o fio? Considere cos30º = 0,87 e

sen30º = 0,5.

a) 0,018 N

b) 0,028 N

c) 0,038 N

d) 0,110 N

e) 0,509 N

Questão 12 - (UDESC/2013)

Um fio retilíneo e horizontal, com 15g de massa e 1,0m

de comprimento, é percorrido por uma corrente elétrica

de intensidade i. O fio está a uma altura h do chão e há

um campo magnético uniforme B=0,50T entrando no

plano desta página, como mostra a Figura 3.

Assinale a alternativa que apresenta, respectivamente,

o valor e o sentido da corrente elétrica, para que o fio

flutue permanecendo em repouso.

a) 0,3A, para a direita

Page 161: curso de específica de física

161

b) 0,3A, para a esquerda

c) 300A, para a direita

d) 300A, para a esquerda

e) 30A, para a direita

Questão 13 - (IFGO/2013)

Em um ambiente onde é feito vácuo, dois fios retos,

longos e paralelos são dispostos, a uma distância de

10,0 cm um do outro, e são percorridos por correntes

de intensidades 2,0 A e 4,0 A em sentidos opostos. É

correto afirmar que cada metro linear desses fios

sofrerá:

Dado: o = 4 10–7 TmA–1.

a) Atração de intensidade 1,6 10–6N.

b) Atração de intensidade 1,6 106N.

c) Atração de intensidade 3,2 10–5N.

d) Repulsão de intensidade 1,6 10–5N.

e) Repulsão de intensidade 1,6 105N.

Questão 14 - (PUC MG/2013)

PQ e RS são dois fios retos e paralelos separados por

certa distância; os fios conduzem as correntes de 2A e

1A respectivamente. M é um ponto na metade da

distância entre os fios. O campo magnético total em M

vale B. Se a corrente de 2A é desligada, o campo em M

valerá agora:

a) B/2

b) 3B

c) B

d) 2B

Questão 15 - (UFG GO/2013)

Uma haste metálica fixa está conectada a uma bateria

que estabelece uma corrente i. Conectada a ela,

encontra-se uma haste condutora móvel de

comprimento L, que está conectada à haste fixa por

dois fios condutores, conforme a figura a seguir.

Aplica-se um campo magnético uniforme ao longo de

um dos eixos do sistema e, como resultado, observa-se

um deslocamento da haste, de um ângulo com a

vertical, permanecendo em equilíbrio conforme a figura.

Considerando-se o exposto, determine:

a) o diagrama de forças sobre a haste e a direção e

o sentido do campo magnético aplicado, conforme

o sistema de eixos da figura apresentada;

b) a intensidade do campo magnético aplicado.

Questão 16 - (UFPB/2013)

Um estudante investigou o comportamento de forças

magnéticas entre dois fios condutores, a e b, retos,

longos e paralelos. Primeiro, fez variar apenas as

correntes ia e ib que percorrem os fios. Depois, fez

variar apenas a separação espacial entre eles. No final

do experimento, o estudante entregou um relatório ao

professor com as seguintes conclusões:

I. O módulo das forças entre os fios depende,

linearmente, das intensidades das correntes ia e

ib.

II. Quanto menor a separação entre os fios maior

será o módulo das forças entre eles.

III. Quando ib for nula, a força que o fio a faz sobre o

fio b induzirá uma corrente i no fio b.

IV. O fato das forças entre os fios ser de atração ou

de repulsão só depende dos sentidos relativos das

correntes nesses fios.

Estão corretas apenas as conclusões:

a) I, II e IV

b) I, II e III

c) III e IV

d) I e IV

e) II e III

Questão 17 - (UEPG PR/2012)

Em 1819, H. C. Oersted descobriu que uma corrente

elétrica é uma fonte de campo magnético. A figura

abaixo mostra dois extensos condutores retilíneos de

comprimento L, dispostos paralelamente entre si e

perpendicularmente ao plano da folha de questões. Os

dois condutores são percorridos por correntes elétricas

de mesma intensidade e sentidos contrários. Com

relação a esse evento físico, assinale o que for correto.

01. As linhas de indução dos campos magnéticos

provocados pelas correntes elétricas que

percorrem os condutores formam circunferências

concêntricas com os condutores, situadas em

planos perpendiculares a eles.

02. O campo magnético resultante no ponto médio

entre os dois condutores está dirigido para baixo e

tem intensidade igual a B = 2d

i0

.

04. O campo magnético resultante a uma distância d

à esquerda do condutor 1 está dirigido para cima

e tem intensidade igual a B = d

i

4

0

.

Page 162: curso de específica de física

162

08. O campo magnético resultante a uma distância d

à direita do condutor 2 está dirigido para cima e

tem intensidade igual a B = d

i

4

0

.

16. Os dois condutores se repelem com uma força cuja

intensidade é igual a F = Ld

ii

2

210

.

Questão 18 - (FUVEST SP/2014)

Partículas com carga elétrica positiva penetram em uma

câmara em vácuo, onde há, em todo seu interior, um

campo elétrico de módulo E e um campo magnético de

módulo B, ambos uniformes e constantes,

perpendiculares entre si, nas direções e sentidos

indicados na figura. As partículas entram na câmara

com velocidades perpendiculares aos campos e de

módulos v1 (grupo 1), v2 (grupo 2) e v3 (grupo 3). As

partículas do grupo 1 têm sua trajetória encurvada em

um sentido, as do grupo 2, em sentido oposto, e as do

grupo 3 não têm sua trajetória desviada. A situação

está ilustrada na figura abaixo.

Considere as seguintes afirmações sobre as velocidades

das partículas de cada grupo:

I. v1 > v2 e v1 > E/B

II. v1 < v2 e v1 < E/B

III. v3 = E/B

Está correto apenas o que se afirma em

a) I.

b) II.

c) III.

d) I e III.

e) II e III.

Note e adote:

Os módulos das forças elétrica (FE) e magnética (FM)

são:

FE = qE

FM = qvB

Questão 19 - (UDESC/2014)

Uma partícula, de massa m = 5,010–18 kg e carga q =

8,010–6 C, penetra perpendicularmente em um campo

magnético uniforme, com velocidade constante de

módulo v = 4,0106 m/s, passando a descrever uma

órbita circular de raio r = 5,0103 cm, desprezando o

efeito do campo gravitacional. O módulo do campo

magnético a que a partícula está submetida é igual a:

a) 4,010–4 T

b) 0,510–8 T

c) 2,010–6 T

d) 5,010–8 T

e) 5,010–7 T

Questão 20 - (UFPR/2014)

O espectrômetro de massa é um equipamento utilizado

para se estudar a composição de um material. A figura

ao lado ilustra diferentes partículas de uma mesma

amostra sendo injetadas por uma abertura no ponto O

de uma câmara a vácuo. Essas partículas possuem

mesma velocidade inicial v, paralela ao plano da página

e com o sentido indicado no desenho. No interior desta

câmara há um campo magnético uniforme B

perpendicular à velocidade v, cujas linhas de campo

são perpendiculares ao plano da página e saindo desta,

conforme representado no desenho com o símbolo .

As partículas descrevem então trajetórias circulares

identificadas por I, II, III e IV.

Considerando as informações acima e os conceitos de

eletricidade e magnetismo, identifique como

verdadeiras (V) ou falsas (F) as seguintes afirmativas:

( ) A partícula da trajetória II possui carga positiva e

a da trajetória IV possui carga negativa.

( ) Supondo que todas as partículas tenham mesma

carga, a da trajetória II tem maior massa que a da

trajetória I.

( ) Supondo que todas as partículas tenham mesma

massa, a da trajetória III tem maior carga que a

da trajetória II.

( ) Se o módulo do campo magnético B fosse

aumentado, todas as trajetórias teriam um raio

maior.

Assinale a alternativa que apresenta a sequência

correta, de cima para baixo.

a) V – V – V – F.

b) F – V – F – V.

c) V – F – V – V.

d) V – V – F – F.

e) F – F – V – V.

GABARITO:

1) Gab: D

2) Gab: 04

3) Gab: FVFFV

4) Gab: A

5) Gab: 23

6) Gab: FVFVF

7) Gab: A

8) Gab: 13

9) Gab: E

10) Gab: A

11) Gab: A

12) Gab: A

Page 163: curso de específica de física

163

13) Gab: D

14) Gab: C

15) Gab:

a) Para o sistema estar em equilíbrio, a força

magnética deve estar direcionada ao longo do eixo

y positivo, e portanto o campo magnético dever

estar direcionado ao logo do eixo z no sentido

negativo.

b) Do diagrama de forças tem-se:

2Tcos = P

2Tsen = iLB

Logo,

tgiL

PB

P

iLBtg

16) Gab: A

17) Gab: 31

18) Gab: E

19) Gab: D

20) Gab: D

21) Gab: 05

22) Gab: 10

23) Gab: 15

24) Gab: D

25) Gab: E

26) Gab: E

27) Gab: 31

28) Gab: 02

29) Gab: C

30) Gab: 05

31) Gab: D 32) Gab:

a) Usando a regra da mão esquerda (figura 1),

determinamos que a força magnética está orientada para direita (figura 2), pelo qual o sistema gira no sentido horário.

Fig.1. Representação da regra da mão esquerda

Fig. 2. Resultado da aplicação da regra da mão esquerda à espira (fig. 1).

b) Como F = iLBsen

e, considerando que não podem ser alterados os componentes e a estrutura do disco rígido, deve-se aumentar a corrente que passa pela espira para que a força seja maior, fazendo o braço girar mais rapidamente.

33) Gab: A 34) Gab: E 35) Gab: 35 36) Gab: B 37) Gab: 15 38) Gab: D 39) Gab: C 40) Gab: D

Page 164: curso de específica de física

164

LISTA 29 – FÍSICA MODERNA A Física do final do século XIX

Pouco antes do princípio do século XX, o final do chamado período clássico caracterizado pelo ponto de vista newtoniano, a Física encontrava-se em uma posição única em relação às outras ciências e a outros diversos campos do saber.

A situação era única porque os físicos, em seu intento de estudar os fenômenos naturais, aparentemente haviam tido um êxito completo, já que praticamente tudo o que se observava podia ser explicado em termos de conceitos e leis conhecidas, como as leis de Newton do movimento, etc.; haviam sido tão cuidadosos nas suas conclusões que os físicos pareciam ter a situação completamente nas mãos.

De uma maneira muito menos milagrosa, as novas observações se haviam ajustado nos lugares aparentemente dispostos para elas nas teorias e hipóteses válidas por mais de um século.

Os físicos teóricos perceberam que a Física estava se aproximando a um estado de um tema clássico já morto e tão completamente conhecido em detalhe que tinha poucas coisas interessantes, se é que tinha, para as futuras gerações investigarem. Nessa época os mais ilustres físicos declaravam que todas as descobertas importantes da Física já haviam sido feitas e que grande parte da aventura havia desaparecido; tudo o que se esperava das gerações futuras era a tarefa incolor de aumentar a precisão das medidas das constantes físicas e de suas relações.

Este ponto de vista mostrou-se totalmente equivocado e presunçoso. O propósito deste curso é considerar as idéias e conceitos da Física Moderna .

1. A Teoria Quântica

Segundo o físico alemão Max PLANCK (1900):

“A energia radiante de freqüência f, só pode ser emitida ou absorvida em quantidades discretas (quantum), múltiplos inteiros de hf, sendo h a constante universal de Planck (6,6 x 10-34 J.s)”

Observações:

* Um fóton é um quantum (partícula) de energia eletromagnética.

* Os fótons não têm todos a mesma a energia. Os “quanta” de luz azul são de maior energia que os de luz vermelha, pois têm menor comprimento de onda e portanto, maior freqüência.

Page 165: curso de específica de física

165

* Duas fontes luminosas de mesma freqüência (isto é, de mesma cor) emitem fótons de igual energia “hf” .

* Uma fonte “brilhante” (grande intensidade luminosa) emite MAIS fótons por segundo do que uma fonte “tênue” (pequena intensidade luminosa) da mesma cor, porém os fótons de ambas as fontes têm a mesma ENERGIA.

2. O Efeito Fotoelétrico

Experimentalmente se verifica que:

* Quando a luz incide sobre a superfície de um metal, elétrons podem ser emitidos por ela.

* Quando a luz de certa freqüência (f) arranca elétrons do metal, eles não saem todos com a mesma energia. Suas energias distribuem-se entre um valor mínimo e um máximo.

* É necessária uma energia mínima, para arrancar um elétron de determinado metal, é chamada FUNÇÃO TRABALHO (W).

* Para explicar o efeito fotoelétrico o físico alemão Albert EINSTEIN (1905) afirmou que:

* A taxa de emissão de fotoelétrons é diretamente proporcional à intensidade da luz incidente.

* Para que um elétron escape da superfície de um metal, deve-se fazer um trabalho contra as forças que o fixam aí, ou seja, os fotoelétrons devem adquirir energia suficiente para serem ejetados.

Page 166: curso de específica de física

166

* A energia absorvida em excesso aparece na forma de energia cinética:

* A energia cinética dos fotoelétrons é independente da intensidade da luz incidente.

3. O Efeito Compton

“Os fótons apresentam propriedades corpusculares (de partículas) quando se chocam com um elétron. Nessas circunstâncias, o fóton perde energia para o elétron, diminuindo sua freqüência e aumentando o seu comprimento de onda.”

4. Dualidade onda-partícula

“A Luz apresenta propriedades ondulatórias (reflexão, refração, difração, interferência e efeito Dopler) e corpusculares (efeito fotoelétrico e efeito Compton)”.

Segundo o físico francês Louis DE BROGLIE as partículas subatômicas (elétrons, prótons, etc.) também possuem características ondulatórias. Esse fato foi comprovado por Clinton DAVISSON, Lester GERMER e G. P. THOMSON (filho de J. J. Thomson) .

FÍSICA ATÔMICA E NUCLEAR

1. MODELOS ATÔMICOS

O MODELO DE BOHR PARA O ÁTOMO DE HIDROGÊNIO

Para interpretar o espectro do átomo de hidrogênio, o físico dinamarquês Niels BOHR desenvolveu uma teoria baseada nos seguintes postulados.

1) O elétron pode permanecer em CERTAS órbitas sem irradiar, apesar da aceleração centrípeta.

2) As órbitas permitidas aos elétrons são tais que o momento angular e a energia do sistema atômico seja múltiplo de uma constante (a energia do elétron é “quantizada”).

Page 167: curso de específica de física

167

3) Se um elétron passa de uma órbita de energia E2 para outra de menor energia E1 , é emitido um fóton

de energia:

4) A teoria de Bohr foi generalizada e aplicada posteriormente a outros átomos mais complexos. Na Mecânica Ondulatória (De Broglie, Schrödinger e outros), abandonou-se o conceito de órbitas eletrônicas definidas ao redor do núcleo, falando-se hoje em dia de “níveis de energia do átomo”.

2. COMPOSIÇÃO DO NÚCLEO ATÔMICO

3. ÁTOMOS IMPORTANTES NA FÍSICA NUCLEAR

ISÓTOPOS DE HIDROGÊNIO: : prótio : deutério : trítio

(Obs.: o deutério é importante devido à água pesada , D20, que é usada como moderador em reatores nucleares.)

ISÓTOPOS DE URÂNIO: : 0,72% : 99,28%

HÉLIO:

PLUTÔNIO:

4. OS RAIOS X (RAIOS RÖNTGEN)

São ondas eletromagnéticas de alta freqüência, ou seja, alta energia – 10 keV (dentários) à 1000 keV (aceleradores lineares) - , obtidas quando elétrons rápidos colidem sobre uma placa metálica (wolfrânio ou tungstênio).

5. DECAIMENTOS RADIOATIVOS

Muitos núcleos atômicos são instáveis e para atingir a estabilidade emitem radiação. As três principais formas de radiação nuclear são:

ALFA (a)

· são núcleos de Hélio, isto é, íons duplamente positivos

Page 168: curso de específica de física

168

BETA (b)

· é formada por elétrons rápidos, de velocidades próximas à velocidade da luz.

· surgem na desintegração de nêutrons em prótons.

GAMA (g)

· é formada por ondas eletromagnéticas de altíssima freqüência.

· é usada em tratamentos de radioterapia (cobalto-60 ou césio-137 são boas fontes desse tipo de radiação).

PODER DE PENETRAÇÃO

Quando um núcleo emite radiação ele sofre o que chamamos de decaimento:

DECAIMENTO ALFA

Exemplo:

T1/2 = 4,47 BILHÕES DE ANOS

DECAIMENTO BETA

Exemplo:

T1/2= 5570 ANOS

Observações

Page 169: curso de específica de física

169

· Em muitos decaimentos alfa ou beta ocorre também a emissão de fótons gama.

· T1/2 (meia-vida) é o tempo necessário para que se reduza à metade, por desintegração, a massa de uma amostra de um núcleo radioativo.

Ex:

T1/2 do césio-137 é de aproximadamente 30 anos.

2001 è128 g de césio-137

2031 è 64 g de césio-137

2061 è 32 g de césio-137

2091 è 16 g de césio-137

6. REAÇÕES NUCLEARES

FISSÃO NUCLEAR

“Reação nuclear, espontânea ou provocada, em que um núcleo atômico, geralmente pesado se divide em duas partes de massas comparáveis, emitindo nêutrons e liberando grande quantidade de energia ( equação de EinsteinèE=mc2, onde m é a massa e c é a velocidade da luz no vácuo – aproximadamente 300.000km/s).”

Observações:

· bomba A (Hiroshima e Nagazaki)

· usina de Angra dos Reis

· submarinos nucleares

· deixa “lixo atômico”

Exemplo:

Page 170: curso de específica de física

170

Também são fissionáveis

e

Se os nêutrons filhos não forem controlados poderemos ter uma reação em cadeia perigosa (explosão).

FUSÃO NUCLEAR

“Reação nuclear em que núcleos leves reagem para formar outro mais pesado, com grande desprendimento de energia. Na reação, parte da massa dos núcleos reagentes se transforma em energia ( equação de EinsteinèE=mc2), é por isso a massa do núcleo resultante é menor do que a soma dos reagentes. “

Observações:

· é a fonte e de energia das estrelas e do Sol, obviamente.

· é o processo usado na bomba H.

· é a fonte de energia dos TOKAMAKS (reatores de fusão nuclear inventados no Japão e usados em pesquisas universitárias).

· não deixa “lixo atômico”.

Exemplo:

Informação adicional

Espécie de Matéria Processo Por quanto tempo a energia liberada pode manter em operação uma lâmpada de 100 W?

1kg de Água Queda d’água de 50 m

5 s

1kg de Carvão Combustão 8 h

1kg de U-235 Fissão completa 30000 anos

1kg de Deutério Fusão completa 30000 anos

1kg de Matéria e Antimatéria

Aniquilamento completo

30.000.000 anos

EXERCÍCIOS

Questão 01 - (UDESC/2014)

O enunciado “Em um mesmo átomo, não podem existir

dois elétrons com o mesmo conjunto de números

quânticos” refere-se a(ao):

a) Princípio da Exclusão de Pauli.

b) Princípio da Conservação de Energia.

c) modelo atômico de Thomson.

d) modelo atômico de Rutherford.

e) um dos Princípios da Teoria da Relatividade

Restrita.

Questão 02 - (UEL PR/2014)

Uma das contribuições da Física para o bem-estar e a

segurança nas cidades é o constante avanço tecnológico

aplicado à iluminação pública. Parte das luminárias do

século XIX era acesa manualmente por várias pessoas

ao entardecer. Hoje, o acionamento das lâmpadas

tornou-se automático devido à aplicação dos

conhecimentos sobre o efeito fotoelétrico (descrito por

Page 171: curso de específica de física

171

Albert Einstein, em 1905) e ao desenvolvimento das

células fotoelétricas instaladas nos postes de iluminação

pública, capazes de detectar a presença de luz natural.

Sobre o efeito fotoelétrico, considere as afirmativas a

seguir.

I. Consiste na emissão de elétrons de uma superfície

metálica quando esta é iluminada com luz de

determinada frequência.

II. Ocorre independentemente da frequência da luz

incidente na superfície do metal, mas é

dependente de sua intensidade.

III. Os elétrons ejetados de uma superfície metálica,

devido ao efeito fotoelétrico, possuem energia

cinética igual à energia do fóton incidente.

IV. Por mais intensa que seja a luz incidente, não

haverá ejeção de elétrons enquanto sua

frequência for menor que a frequência limite (ou

de corte) do metal.

Assinale a alternativa correta.

a) Somente as afirmativas I e II são corretas.

b) Somente as afirmativas I e IV são corretas.

c) Somente as afirmativas III e IV são corretas.

d) Somente as afirmativas I, II e III são corretas.

e) Somente as afirmativas II, III e IV são corretas.

Questão 03 - (UFG GO/2014)

Em 1913, há cem anos, Niels Bohr, para resolver o

problema da emissão de radiação por partículas

carregadas que movem-se em uma órbita circular,

formulou a hipótese de que o momento angular do

elétron no átomo de hidrogênio era quantizado, ou seja,

de que nmvr com n = 1,2,3,…. Essa hipótese foi

necessária, pois, de acordo com a física clássica, o

elétron colapsaria no núcleo, o que seria explicado

a) pela perda discreta de energia potencial e

diminuição do raio da órbita por saltos quânticos.

b) pela conservação da energia mecânica com perda

de energia potencial e ganho de energia cinética.

c) pela perda contínua de energia cinética e de

quantidade de movimento.

d) pela conservação do momento angular e

diminuição do raio da órbita.

e) pelo aumento da força centrípeta e aumento da

velocidade.

Questão 04 - (UFG GO/2014)

Um projeto de propulsão para espaçonaves de longas

distâncias é baseado no fenômeno de liberação de

energia na aniquilação de um elétron pela sua

antipartícula, o pósitron. Sabendo que a massa de cada

uma destas duas partículas é igual a 9,0910–31 kg, e

desconsiderando quaisquer interações gravitacionais,

determine:

Dados:

velocidade da luz no vácuo: c = 3 108 m/s

g = 10 m/s2

a) o número de pares elétron-pósitron necessários

para gerar energia suficiente para se acelerar uma

espaçonave de massa 181,8 toneladas do repouso

à velocidade de 60.000 m/s, havendo uma

eficiência de 50% no processo inteiro;

b) a aceleração constante necessária, em unidades

de g, para que se atinja uma velocidade de 0,01%

da velocidade da luz no vácuo em 6 min 40 s.

Questão 05 - (UEG GO/2014)

Em 1947 César Lattes, físico brasileiro, utilizando um

equipamento conhecido como câmara de Wilson

(aparelho que registrava a passagem de partículas

carregadas através de uma emulsão), conseguiu

comprovar a existência de uma partícula mediadora da

força nuclear. Essa partícula, idealizada por Hideki

Yukawa, um físico japonês, é responsável pela ligação

entre: prótons com prótons, nêutrons com nêutrons e

prótons com nêutrons. A essa partícula, Hideki Yukawa

denominou de

a) méson PI

b) beta

c) próton

d) neutrino

Questão 06 - (Fac. de Ciências da Saúde de

Barretos SP/2014)

Radioisótopo muito utilizado na medicina nuclear, o

tecnécio-99-metaestável decai por emissão de um fóton

com energia 2,21 10–14 J. De acordo com Max Planck,

essa energia é dada por EF = h f, sendo h uma

constante de valor 6,63 10–34 J s e f a frequência da

radiação. Considerando a velocidade das ondas

eletromagnéticas no vácuo igual a 3,0 108 m/s, o

comprimento de onda, em metros, da radiação emitida

por esse radioisótopo é, aproximadamente,

a) 9,0 10–14.

b) 6,0 10–14.

c) 9,0 10–12.

d) 1,0 10–10.

e) 6,0 10–12.

Questão 07 - (UEPA/2014)

A célula fotovoltaica e a célula fotoelétrica são

dispositivos muito utilizados na eletrônica, seja em

circuitos de controle de portas automáticas, postes de

luz e alarmes de incêndio, seja como fonte alternativa

de energia elétrica, caso em que são chamados de

células solares. A figura abaixo mostra um esquema

simplificado de uma célula fotoelétrica cujo anodo é

feito de um material com função trabalho igual a 3,0

eV. O comprimento de onda da luz incidente na placa é

155 nm. Nessas condições, a diferença de potencial

mínima que deve ser aplicada para que a corrente

observada no amperímetro seja nula, em V, é igual a:

Dados: hc = 1240 eV.nm

1 nm = 10– 9 m

Page 172: curso de específica de física

172

a) 3

b) 5

c) 7

d) 9

e) 11

Questão 08 - (UEPA/2014)

O tratamento cirúrgico dos cálculos de rim e ureter,

recentemente em nosso meio, adicionou uma

importante e efetiva forma de fragmentação dos

cálculos, com o advento do raio laser.

Fonte:

http://www.minhavida.com.br/saude/materias/3762-

laser-em-calculos-renais-e-ureterais.

Neste tipo de cirurgia, pulsos de laser de 16 ms, com

potência de 20 W e comprimento de onda de 620 nm,

incidem diretamente no cálculo, provocando sua

fragmentação para tamanhos microscópicos. Sob essas

condições, afirma-se que o número de fótons emitidos

em cada pulso é igual a:

Dados: hc = 1240 eV.nm;

1 nm = 10–9 m;

1 eV = 1,6 10–19 J.

a) 106

b) 109

c) 1012

d) 1015

e) 1018

Questão 09 - (ACAFE SC/2013)

Em regiões afastadas, as torres de telefonia celular

podem ser abastecidas com energia fotovoltaica. Esse

modo de geração de energia está baseado no efeito

fotoelétrico.

Em relação a esse efeito, analise as afirmações a seguir.

I. A emissão de elétrons por uma superfície metálica

atingida por uma onda eletromagnética

caracteriza o efeito fotoelétrico.

II. A emissão de fotoelétrons em uma superfície

metálica fotossensível ocorre quando a frequência

de luz incidente nessa superfície apresenta um

valor mínimo, que depende do material.

III. O efeito fotoelétrico só ocorre com a utilização de

uma onda eletromagnética na faixa de frequência

da luz visível.

IV. A explicação do efeito fotoelétrico está baseada

em um modelo corpuscular da luz.

Todas as afirmações corretas estão em:

a) III - IV

b) I - II - III

c) II - III - IV

d) I - II - IV

TEXTO: 1 - Comuns às questões: 10, 11

O 99mTc (tecnécio-99 metaestável) é um emissor gama

com meia vida de 6 horas amplamente utilizado em

procedimentos de Medicina Nuclear. É produzido pelo

decaimento do 99Mo (molibdênio-99), que tem uma

meia-vida de 66 horas. O gerador de tecnécio consiste

em um recipiente com pequenas esferas de alumina

sobre as quais o 99Mo (molibdênio- 99), produzido em

um reator nuclear, liga-se firmemente. Quando decai

em tecnécio este, por ser quimicamente diferente,

desliga-se da alumina, e pode ser “lavado” do recipiente

por uma solução salina. A foto mostra a coluna de

alumina no centro de uma espessa blindagem para a

radiação. Acima, à esquerda, o local onde é conectado

o frasco da solução salina; à direita, o local onde é

colocado o frasco evacuado que “suga” a solução,

fazendo-a passar pela coluna de alumina.

(Nelson Canzian da Silva. www.fsc.ufsc.br, abril de

2008.)

Questão 10 - (Fac. Santa Marcelina SP/2013)

Comparando-se os números atômicos do molibdênio e

do tecnécio, pode-se afirmar que o decaimento do

molibdênio-99 em tecnécio-99 metaestável ocorre com

a emissão radioativa constituída por

a) elétrons (partículas –).

b) prótons.

c) pósitrons (partículas +).

d) núcleos de hélio (partículas ).

e) nêutrons.

Questão 11 - (Fac. Santa Marcelina SP/2013)

Grande parte do molibdênio-99 utilizado em clínicas e

hospitais do mundo todo é obtida em um reator nuclear

do Canadá, fato que implica na necessidade de

transporte aéreo para os demais países. Caso haja uma

greve que acarrete um atraso de 132 horas em um voo

que transportará o gerador de tecnécio, a atividade

radioativa inicial do molibdênio-99, durante este

período de greve, terá caído, em relação à inicial a

a) 12,5%.

Page 173: curso de específica de física

173

b) 3,12%.

c) 50%.

d) 6,25%.

e) 25%.

Questão 12 - (UEG GO/2013)

Grande parte dos avanços que ocorreram na medicina

nas últimas décadas foram relacionados à física.

Algumas técnicas muito utilizadas estão baseadas nos

conceitos fundamentais dessa ciência. Sobre elas,

destaca-se:

a) a radiografia é uma técnica que se baseia na

emissão de raios gama, sendo bastante utilizada

na ortopedia.

b) na cirurgia a laser, um feixe de luz ultravioleta

invisível retira camadas de tecidos com precisão

microscópica.

c) na tomografia, forma-se um modelo

computadorizado completo de um órgão do corpo

humano, em uma única dimensão.

d) nos aparelhos de ultrassom, uma fonte de ondas

eletromagnéticas emite um pulso que é refletido

na forma de imagem.

Questão 13 - (UEG GO/2013)

Em 1905, Albert Einstein publicou vários artigos. Um

deles, rendeu-lhe o prêmio Nobel de Física em 1914, o

que tratava do efeito fotoelétrico. Com base nesta

informação, explique em que consiste o efeito

fotoelétrico.

Questão 14 - (UEM PR/2013)

Analise as alternativas abaixo e assinale o que for

correto.

01. O princípio da constância da velocidade da luz

estabelece que a velocidade da luz no vácuo tem

o mesmo valor para todos os observadores,

qualquer que seja seu movimento ou o movimento

da fonte de luz.

02. O princípio da incerteza de Heisenberg estabelece

que quanto maior a precisão na determinação da

posição de um corpo, menor é a precisão na

determinação da velocidade desse corpo.

04. No modelo atômico de Bohr, os elétrons

descrevem órbitas elípticas em torno do núcleo

atômico, com energias diretamente proporcionais

à distância desses elétrons ao centro do núcleo

atômico.

08. Quando radiação ultravioleta incide sobre a

superfície polida de um metal de transição,

elétrons podem ser arrancados dessa superfície

em resposta ao efeito Compton relativo à

interação dessa radiação com os elétrons de

valência do metal.

16. A radioatividade consiste na emissão de partículas

e radiações eletromagnéticas por núcleos

atômicos instáveis que, após a emissão,

transformam-se em núcleos mais estáveis.

Questão 15 - (UEPG PR/2013)

A partir da metade do século XVII, surgiu uma longa

controvérsia científica sobre a natureza da luz que

durou quase dois séculos. Sobre a luz, assinale o que

for correto.

01. As radiações luminosas se originam de oscilações

eletromagnéticas ou das oscilações de cargas

elétricas.

02. Além de considerar a luz como uma onda

eletromagnética, pode-se também considerá-la

como um fluxo de partículas energéticas

desprovidas de massa, os fótons.

04. O espectro luminoso é o conjunto das frequências

que compõem a radiação luminosa e sua

velocidade é a mesma de todas as ondas que

compõem o espectro eletromagnético.

08. O espectro luminoso, ondas de rádio, raio X,

radiações , e são algumas das radiações que

fazem parte do espectro eletromagnético. O que

difere é o modo de obtê-las, o espectro luminoso

é visível e as demais não.

16. Os olhos são sensores de ondas eletromagnéticas,

portanto, pode-se dizer que a luz é a modalidade

de energia radiante percebida por um observador

mediante sensações visuais oriundas da retina.

Questão 16 - (UFG GO/2013)

Em 1964, o físico britânico Peter Higgs propôs a

existência de um campo, o qual, ao interagir com uma

partícula, conferia a ela a sua massa. A unidade básica

desse campo foi chamada de bóson de Higgs. Em julho

de 2012, os cientistas do CERN (Centro Europeu de

Pesquisas Nucleares) anunciaram terem identificado o

bóson de Higgs, com uma massa de 125 GeV

(gigaelétronvolt). O valor dessa massa, em kg, é de:

Dados:

1 eV = 1,6 10–19 J

c = 3,0 108 m/s

a) 4,50 10+24

b) 6,66 10–18

c) 2,22 10–25

d) 6,66 10–27

e) 2,22 10–34

Questão 17 - (UFPE/2013)

A respeito do modelo atômico de Rutherford, podemos

afirmar que:

00. em seu modelo atômico, Rutherford propôs que os

elétrons se moviam em órbitas elípticas

quantizadas ao redor do núcleo.

01. Rutherford teve dificuldades em explicar a

estabilidade atômica com o seu modelo.

02. de acordo com o modelo de Rutherford, o

Hidrogênio era o único elemento químico a ter as

linhas do seu espectro calculadas

quantitativamente.

03. Rutherford elaborou o seu modelo a partir da

observação de que partículas alfa incidindo em

uma fina folha metálica jamais eram espalhadas

em ângulos maiores que 90º.

04. de acordo com o modelo de Rutherford, o elétron

deveria espiralar até o núcleo em um tempo

superior ao tempo de vida do Universo.

Questão 18 - (UFRN/2013)

Page 174: curso de específica de física

174

Quando olhamos para o céu noturno, vemos uma

grande quantidade de estrelas, muitas das quais se

encontram a dezenas e até a centenas de anos-luz de

distância da Terra. Na verdade, estamos observando as

estrelas como elas eram há dezenas, centenas ou até

milhares de anos, e algumas delas podem nem mais

existir atualmente.

Esse fato ocorre porque

a) a velocidade da luz no vácuo é infinita e não

depende do movimento relativo entre fontes e

observadores.

b) a velocidade da luz no vácuo, apesar de ser muito

grande, é finita e depende do movimento relativo

entre fontes e observadores.

c) a velocidade da luz no vácuo, apesar de ser muito

grande, é finita e não depende do movimento

relativo entre fontes e observadores.

d) a velocidade da luz no vácuo é infinita e depende

do movimento relativo entre fontes e

observadores.

Questão 19 - (UFRN/2013)

O Diodo Emissor de Luz (LED) é um dispositivo

eletrônico capaz de emitir luz visível e tem sido utilizado

nas mais variadas aplicações. A mais recente é sua

utilização na iluminação de ambientes devido ao seu

baixo consumo de energia e à sua grande durabilidade.

Atualmente, dispomos de tecnologia capaz de produzir

tais dispositivos para emissão de luz em diversas cores,

como, por exemplo, a cor vermelha de comprimento de

onda, V, igual a 629 nm, e a cor azul, de comprimento

de onda, A, igual a 469 nm.

A energia, E, dos fótons emitidos por cada um dos LEDs

é determinada a partir da equação de Einstein E = hf

onde h é a constante de Planck, e f é a frequência do

fóton emitido.

Sabendo ainda que c = f, onde c é a velocidade da luz

no vácuo e , o comprimento de onda do fóton, é

correto afirmar que

a) o fóton correspondente à cor vermelha tem menos

energia que o fóton correspondente à cor azul,

pois sua frequência é menor que a do fóton de cor

azul.

b) o fóton correspondente à cor vermelha tem mais

energia que o fóton correspondente à cor azul,

pois sua frequência é maior que a do fóton de cor

azul.

c) o fóton correspondente à cor azul tem menos

energia que o fóton correspondente à cor

vermelha, pois seu comprimento de onda é maior

que o do fóton de cor vermelha.

d) o fóton correspondente à cor vermelha tem mais

energia que o fóton correspondente à cor azul,

pois seu comprimento de onda é menor que a do

fóton de cor azul.

Questão 20 - (UFSC/2013)

Em um experimento semelhante aos realizados por

Hertz, esquematizado na figura abaixo, um estudante

de Física obteve o seguinte gráfico da energia cinética

(E) máxima dos elétrons ejetados de uma amostra de

potássio em função da frequência (f) da luz incidente.

Com base nas características do fenômeno observado e

no gráfico, assinale a(s) proposição(ões) CORRETA(S).

01. O valor da constante de Planck obtida a partir

do gráfico é de aproximadamente 4,43 x 10–15

eVs.

02. A função trabalho do potássio é maior que 2,17

eV.

04. Para frequências menores que 5,0 x 1014 Hz, os

elétrons não são ejetados do potássio.

08. O potencial de corte para uma luz incidente de

6,0 x 1014 Hz é de aproximadamente 0,44 eV.

16. Materiais que possuam curvas de E (em eV) em

função de f (em Hz) paralelas e à direita da

apresentada no gráfico possuem função trabalho

maior que a do potássio.

32. A energia cinética máxima dos elétrons emitidos

na frequência de 6,5 x 1014 Hz pode ser

aumentada, aumentando-se a intensidade da luz

incidente.

GABARITO:

1) Gab: A

2) Gab: B

3) Gab: C

4) Gab:

a) 41027 pares

b) 5,7g

a

5) Gab: A

6) Gab: C

7) Gab: B

8) Gsb: E

9) Gab: D

10) Gab: A

11) Gab: E

12) Gab: B

13) Gab:

O efeito fotoelétrico é o fenômeno pelo qual uma luz

consegue arrancar elétrons de uma placa metálica. Isso

ocorre porque os elétrons da placa absorvem os fótons

de luz, com isso recebem energia dos fótons ganhando

velocidade. A energia recebida pelos elétrons é a

Page 175: curso de específica de física

175

energia do fóton cuja definição é E = h.f, onde E é a

energia, h é a constante de Planck e f é a frequência do

fóton. A energia cinética do elétron ejetado será a

diferença positiva entre a energia do fóton e a função

trabalho, w, definida para o material considerado. w é

a energia necessária para remover um elétron da

superfície do material (varia de material para material).

O efeito fotoelétrico evidencia o caráter corpuscular da

radiação.

14) Gab: 19

15) Gab: 23

16) Gab: C

17) Gab: FVFFF

18) Gab: C

19) Gab: A

20) Gab: 23

21) Gab:

a) 1,2 1025 m

b) 4 1031 kg

22) Gab: A

23) Gab: B

24) Gab: C

25) Gab: D

26) Gab: E

27) Gab: A

28) Gab: C

29) Gab: E

30) Gab: 30

31) Gab: B 32) Gab: B 33) Gab: 13 34) Gab: D 35) Gab: E 36) Gab: E 37) Gab: 09 38) Gab:

a) O comprimento de onda )( da luz desse laser

pode ser determinado por meio da equação fundamental da ondulatória:

v = f 3 108 = 0,6 1015 = 5

10–7 m b) Aplicando-se a expressão da definição de potência,

tem-se:

15

15

1030

E10

tP

E = 30 J c) De acordo com a expressão da potência:

t

303

tP

s 10t

d) Inicialmente, é possível determinar a energia de um fóton por meio da expressão apresentada: E = h f = 6 10–34 0,6 1015

E = 3,6 10–19 J

Em seguida, pode-se determinar a quantidade de fótons (n) em cada pulso por meio da relação: 1 fóton –––– 3,6 10–19 J

n –––– 30 J n = 8,3 1019 fótons

39) Gab: B 40) Gab: D

Page 176: curso de específica de física

176

PROVAS DE VESTIBULARES

UEL 1ª FASE 2017 1 - Uel 2017 1ª. Fase. Atualmente, os combustíveis mais utilizados para o abastecimento dos carros de passeio, no Brasil, são o etanol e a gasolina. Essa utilização somente é possível porque os motores desses automóveis funcionam em ciclos termodinâmicos, recebendo combustível e convertendo-o em trabalho útil. Com base nos conhecimentos sobre ciclos termodinâmicos, assinale a alternativa que apresenta corretamente o diagrama da pressão (P) versus volume (V)

de um motor a gasolina.

2 - Uel 2017 1ª. Fase. Considere que as lâmpadas descritas na charge emitem luz amarela que incide na superfície de uma placa metálica colocada próxima a elas. Com base nos conhecimentos

sobre o efeito fotoelétrico, assinale a alternativa correta. a) A quantidade de energia absorvida por um elétron que escapa da superfície metálica é denominada de fótons e tem o mesmo valor para qualquer metal. b) Se a intensidade luminosa for alta e a frequência da luz incidente for menor que a frequência-limite, ou de corte, o efeito fotoelétrico deve ocorrer na placa metálica. c) Se a frequência da luz incidente for menor do que a frequência-limite, ou de corte, nenhum elétron da superfície metálica será emitido. d) Quando a luz incide sobre a superfície metálica, os núcleos atômicos próximos da superfície absorvem energia suficiente e escapam para o espaço. e) Quanto maior for a função trabalho da superfície metálica, menor deverá ser a frequência-limite, ou de corte, necessária para a emissão de elétrons. 3 - Uel 2017 1ª. Fase.

Com base na charge e nos conceitos da termodinâmica, é correto afirmar que as luvas de amianto são utilizadas porque a condutividade térmica a) da cuia de cristal é menor que a do líquido. b) da cuia de cristal e a do amianto são iguais. c) do amianto é menor que a da cuia de cristal. d) do amianto é maior que a da cuia de cristal. e) do amianto é maior que a do líquido. 4 - Uel 2017 1ª. Fase.

Com base no diálogo entre Jon e Garfield, expresso na tirinha, e nas Leis de Newton para a gravitação universal, assinale a alternativa correta. a) Jon quis dizer que Garfield precisa perder massa e não peso, ou seja, Jon tem a mesma ideia de um comerciante que usa uma balança comum b) Jon sabe que, quando Garfield sobe em uma balança, ela mede exatamente sua massa com intensidade definida em

quilograma-força. c) Jon percebeu a intenção de Garfield, mas sabe que, devido à constante de gravitação universal “g”, o peso do gato será o mesmo em qualquer planeta. d) Quando Garfield sobe em uma balança, ela mede exatamente seu peso aparente, visto que o ar funciona como um fluido hidrostático. e) Garfield sabe que, se ele for a um planeta cuja gravidade seja menor, o peso será menor, pois nesse planeta a massa aferida será menor. 5 - Uel 2017 1ª. Fase. A questão do tempo sempre foi abordada por filósofos, como Kant. Na física, os resultados obtidos por Einstein sobre a ideia da “dilatação do tempo” explicam situações cotidianas, como, por exemplo, o uso de GPS. Com base nos conhecimentos sobre a Teoria da Relatividade de Einstein, assinale a alternativa correta. a) O intervalo de tempo medido em um referencial em que se empregam dois cronômetros e dois observadores é menor do que o intervalo de tempo próprio no referencial em que a medida é feita por um único observador com um único cronômetro. b) Considerando uma nave que se movimenta próximo à velocidade da luz, o tripulante verifica que, chegando ao seu destino, o seu relógio está adiantado em relação ao relógio da estação espacial da qual ele partiu. c) As leis da Física são diferentes para dois observadores posicionados em sistemas de referência inerciais, que se deslocam com velocidade média constante.

Page 177: curso de específica de física

177

d) A dilatação do tempo é uma consequência direta do princípio da constância da velocidade da luz e da cinemática elementar. e) A velocidade da luz no vácuo tem valores diferentes para observadores em referenciais privilegiados. GABARITO

1 – B 2 – C 3 – C 4 – A 5 – D

UEL 2ª FASE 2017 1 - UEL 2ª. FASE 2017.

No modelo padrão da física das partículas elementares, o

próton e o nêutron são partículas compostas constituídas

pelas combinações de partículas menores chamadas de

quarks u (up) e d (down). Nesse modelo, o próton (p) e o

nêutron (n) são compostos, cada um, de três quarks, porém

com diferentes combinações, sendo representados por p =

(u, u, d) e n = (u, d, d). Os prótons e os nêutrons

comportam-se, na presença de um campo magnético, como

se fossem minúsculos ímãs, cujas intensidades são

denominadas de momento magnético e medidas em

magnetons nucleares (mn). Para o próton, o momento

magnético é dado por

2 - UEL 2ª. FASE 2017.

Nos Jogos Olímpicos Rio 2016, o corredor dos 100 metros

rasos Usain Bolt venceu a prova com o tempo de 9 segundos

e 81 centésimos de segundo. Um radar foi usado para medir

a velocidade de cada atleta e os valores foram registrados

em curtos intervalos de tempo, gerando gráficos de

velocidade em função do tempo. O gráfico do vencedor é

apresentado a seguir.

Considerando o gráfico de V versus t, responda aos itens a

seguir.

a) Calcule a quantidade de metros que Bolt percorreu desde

o instante 2,5 s até o instante 4,5 s, trecho no qual a

velocidade pode ser considerada aproximadamente

constante.

b) Calcule o valor aproximado da aceleração de Usain Bolt

nos instantes finais da prova, ou seja, a partir de 9 s.

3 - UEL 2ª. FASE 2017.

O LED (Light Emitting Diode) é um diodo semicondutor que

emite luz quando polarizado eletricamente. A curva

característica de um LED está indicada na Figura 1.

Percebe-se que, na região de condução elétrica do LED, um

aumento pequeno na diferença de potencial U leva a um

aumento considerável na corrente elétrica i que passa pelo

LED. Por isso, no circuito elétrico de polarização, é

geralmente necessário conectar um resistor R em série com

o LED, como esquematizado na Figura 2, de maneira a

limitar a corrente elétrica que passa pelo diodo.

Pode-se observar, pelo gráfico da Figura 1, que, se a

corrente elétrica no circuito for de 100 mA, a diferença de

potencial Ud aplicada sobre o LED será de 2 V. A partir

dessas informações, responda aos itens a seguir.

a) Sabe-se que, ao longo de um circuito fechado, como o da

Figura 2, a soma das diferenças de potencial (ddp) e das

quedas de tensão sobre cada componente do circuito é zero.

Considerando que a ddp da fonte vale +12 V e que as

quedas de tensão do resistor e do LED são,

respectivamente, −R · i e −Ud, determine o valor de R para

que a corrente elétrica no circuito seja de 100 mA.

Page 178: curso de específica de física

178

b) Considere o LED como uma fonte puntiforme de luz, cuja

emissão tem um ângulo de divergência total de 60. Uma

lente delgada convergente, de 6 cm de diâmetro, é colocada

a uma distância o do LED, de maneira que a luz emitida pelo

LED ilumine toda a superfície da lente, conforme

esquematizado na Figura 3.

Deseja-se que os raios luminosos que emergem da lente

sejam perfeitamente paralelos (ou seja, que a distância

imagem seja infinita). Nesse caso, determine a distância

focal da lente.

Considere tg(30) = 0, 6

4 - UEL 2ª. FASE 2017.

Considere o diagrama pV da Figura 4 a seguir.

O ciclo fechado ao longo do percurso abcda é denominado

ciclo Otto e representa o modelo idealizado dos processos

termodinâmicos que ocorrem durante o funcionamento de

um motor a gasolina. O calor recebido pelo motor, dado por

Q1, é fornecido pela queima da gasolina no interior do

motor. W representa o trabalho realizado pelo motor em

cada ciclo de operação, e Q2 é o calor rejeitado pelo motor,

por meio da liberação dos gases de exaustão pelo

escapamento e também via sistema de arrefecimento.

Considerando um motor que recebe 2500 J de calor e que

realiza 875 J de trabalho em cada ciclo de operação,

responda aos itens a seguir.

a) Sabendo que o calor latente de vaporização da gasolina

vale 5 × 104 J/g , determine a massa de gasolina utilizada

em cada ciclo de operação do motor.

b) Sabendo que, em um ciclo termodinâmico fechado, a

soma das quantidades de calor envolvidas no processo é

igual ao trabalho realizado no ciclo, determine a quantidade

de calor rejeitada durante cada ciclo de operação do motor.

UEM 1º DIA 2016 13 - UEM 2016 1º DIA

No Reino Animália, há um grupo que possui como

característica marcante a capacidade de voo. Os animais

deste grupo possuem corpo aerodinâmico, que diminui a

resistência do ar durante o voo, e ossos porosos e menos

densos. Considerando os representantes deste grupo

animal, nos aspectos físicos e biológicos, assinale o que for

correto.

01) Numericamente, a força de resistência do ar é igual para

todos os animais voadores do planeta.

02) A força gravitacional diminui drasticamente à medida

que o animal atinge maiores altitudes durante o voo.

04) Para alçar voo, estes animais usam músculos peitorais

que movimentam as asas e geram força, mas só decolam

quando o módulo resultante da força gerada for maior que

o peso do animal.

08) Os animais do grupo citado no comando da questão

pertencem ao Filo Chordata.

16) Como são pecilotérmicos, estes animais não podem voar

em grandes altitudes por tempos prolongados, em virtude

da baixa temperatura nestas altitudes.

27 - UEM 2016 1º DIA

Três fontes luminosas pontuais A, B e C formam um

segmento de reta de extremidades A e C em que B é o ponto

médio deste segmento AC . Suponhamos que a reta AC

esteja posicionada a 40 cm de um plano π e que P ∈ π é um

ponto de incidência de luz. Suponhamos que a reta AC é

perpendicular à reta PB e que o meio é homogêneo e

transparente. Assinale o que for correto.

01) O raio de luz proveniente do ponto A, e que sofre

reflexão em P, atinge o ponto B.

02) Existe ao menos um ponto Q ∈ AC com uma fonte

luminosa pontual, cujo raio de luz incide em P, que teria sua

reflexão incidindo em Q´∈ AC , de maneira que o triângulo

QPQ´seja escaleno.

04) Se o raio de luz proveniente de A sofre reflexão em P e

atinge A´ ∈ AC, e se o triângulo APA´ é um triângulo

retângulo, então a distância de A até A´ é de 80 cm.

Page 179: curso de específica de física

179

08) Considere duas fontes luminosas, pontuais e distintas,

M , N ∈ AB distintas de B, emitindo um raio de luz em P

cujas reflexões M´ e N´ estão na reta AC , respectivamente.

Assim definidos, os triângulos MPM´ e NPN´ são

semelhantes.

16) Sendo a reflexão do raio de luz difusa, a trajetória da

luz refletida será parabólica.

28 - UEM 2016 1º DIA

Um cubo de 100 g de certa substância, inicialmente em

estado sólido, recebe calor de uma fonte térmica numa taxa

constante de 5 cal s (desconsidere perdas de calor para o

ambiente). Durante os primeiros 180 s, a temperatura da

substância cresce linearmente com o tempo, de 20 ºC para

60 ºC. Durante os próximos 240 s, a temperatura

permanece constante e igual a 60 ºC A temperatura da

substância começa a subir novamente, linearmente com o

tempo, por mais 120 s até chegar a 90 ºC. O

comportamento da temperatura T (em ºC) da substância,

em função do tempo t (em s) de exposição à fonte térmica,

pode ser representado pelas seguintes funções: T = at + b

, no intervalo compreendido entre 0 s e 180 s; T = e, de

180 s a 420 s; e T = ct + d, entre 420 s e 540 s; com

a,b,c,d,e constantes. Sobre este sistema e as funções

descritas acima, assinale o que for correto.

01) O calor específico desta substância no estado sólido é

maior que seu calor específico no estado líquido.

02) O calor latente de fusão desta substância é maior que

20 cal/g.

04) No instante t = 45 s, a temperatura da substância é o T

= 34 oC.

08) No instante t =480 s, a temperatura da substância é o

T = 75 oC.

16) b + d > 0.

29 - UEM 2016 1º DIA

Em um evento esportivo, um atleta faz um lançamento de

um disco de metal de 2 kg. Assim que o disco deixa a mão

do atleta, sua trajetória no plano xy pode ser representada

pela função 682

3 2xxy definida no intervalo entre x =

0 e x = A. Nesta expressão, y representa a altura em metros

do disco em relação ao solo, x representa a distância

horizontal em metros do disco em relação ao ponto de

lançamento, e A representa o alcance horizontal máximo em

metros. Sobre esse lançamento, assinale o que for correto.

Dado: A equação da trajetória num lançamento oblíquo

pode ser escrita como

2

22)( xox

v

gxox

v

vyoy

oxox

oy em que xo e yo são

as coordenadas iniciais, vox e voy são as projeções do vetor

velocidade inicial nas direções x e y, respectivamente, e g

= 10 m/s2 é a aceleração gravitacional.

01) No ponto mais alto da trajetória, y > 17 m.

02) No ponto mais alto da trajetória, x = 35 m.

04) A > 68 m.

08) O módulo da velocidade inicial é sm/680 .

16) O vetor velocidade inicial faz um ângulo de 42º com a

horizontal.

30 - UEM 2016 1º DIA

Um sistema termicamente isolado é composto por dois

cilindros A e B conectados por uma válvula. Numa situação

inicial, o cilindro A contém um gás ideal e o cilindro B está

completamente vazio. Abrindo-se a válvula, o gás contido

no cilindro A se expande e parte dele flui espontaneamente

para o cilindro B até atingir um equilíbrio quanto ao

preenchimento dos volumes dos dois cilindros. Suponha que

não há troca de calor entre o gás, os cilindros e a válvula.

Sobre esse sistema, assinale o que for correto.

01) A transformação ocorrida é adiabática.

02) Não há realização de trabalho visto que não houve

resistências contra a expansão do gás.

04) Não há variação de energia interna do gás.

08) A temperatura do gás diminui, pois a distância média

entre as moléculas desse gás diminui com a expansão.

16) Ao expandir-se, o gás sofre uma transformação

reversível.

31 - UEM 2016 1º DIA

Em 1800, Alessandro Volta publicou um artigo em que

apresentou um dispositivo ao qual denominou “órgão

elétrico artificial”. Ao empilhar uma série de discos de dois

metais diferentes intercalados por um pedaço de tecido

embebido em salmoura, o cientista italiano construiu o que

costumamos chamar de pilha elétrica. Em relação a pilhas

em geral, assinale o que for correto.

01) É possível sentir uma pequena dor ao encostar um

talher de alumínio em uma obturação metálica, constituída

por outros metais, devido ao choque provocado por uma

corrente elétrica gerada pela pilha constituída pelo talher,

pela obturação e pela saliva como eletrólito.

02) A corrente elétrica gerada nas pilhas de Volta pode ser

contínua ou alternada, dependendo do tipo de eletrólito que

se use.

04) Pilhas podem ser usadas no estudo de várias situações

que envolvem diferença de potencial constante.

08) Existem pilhas que não apresentam resistência interna.

16) Pilhas não podem ser empregadas para gerar campos

elétricos ou magnéticos constantes.

32 - UEM 2016 1º DIA

Um gás monoatômico ideal opera segundo um ciclo

termodinâmico circular em um diagrama de Clapeyron.

Nesse diagrama, o menor valor para o volume V (eixo das

abscissas) é 2V0, e o maior valor é 4V0. A pressão P (eixo

Page 180: curso de específica de física

180

das ordenadas) varia de 3P0 a 5P0. A partir destas

informações, assinale a(s) alternativa(s) correta(s) .

01) No ciclo, a temperatura do gás, na posição em que P é

máxima, é maior que na posição em que V é máximo.

02) Ao completar-se o ciclo, a variação da energia interna é

nula.

04) O trabalho realizado, quando o ciclo é completado no

sentido horário do círculo, é numericamente igual ao valor

de ∏.V02 .

08) Ao completar-se o ciclo no sentido horário do círculo, o

calor é convertido em trabalho.

16) A energia interna do gás, quando a pressão é igual a

3P0, é dada por U = (27/2).Po.Vo.

GABARITO

13 – 4+8 27 – 4

28 – 1+8 29 – 1+4+8

30 – 1+2+4 31 – 1+4

32 – 2+4+8+16